0% found this document useful (0 votes)
49 views375 pages

Math Workbook

Uploaded by

Brian Fiddes
Copyright
© © All Rights Reserved
We take content rights seriously. If you suspect this is your content, claim it here.
Available Formats
Download as PDF, TXT or read online on Scribd
0% found this document useful (0 votes)
49 views375 pages

Math Workbook

Uploaded by

Brian Fiddes
Copyright
© © All Rights Reserved
We take content rights seriously. If you suspect this is your content, claim it here.
Available Formats
Download as PDF, TXT or read online on Scribd
You are on page 1/ 375

Calculus I

For Management and Economics

Exercise Book

ˆ Practice Problems
ˆ Concept Summaries
ˆ Algebra Review
ˆ Very Detailed Solution To Each Problem

by Paul Tsopméné

Last Update: December 1, 2022

Calculus I for Management and Economics Exercise Book by Paul Tsopméné is


licensed under a Creative Commons Attribution-NonCommercial-ShareAlike 4.0 In-
ternational (CC BY-NC-SA 4.0).
ii

About the Author

ˆ Title: Assistant Professor of Teaching


ˆ Since: September 2020
ˆ Institution: University of British Columbia Okanagan
ˆ Department: CMPS (Computer Science, Mathematics, Physics
and Statistics)
ˆ Email: [email protected]

Source File. To get the source file, please get in touch with the
author at the following email address: [email protected]
iii

For Instructors
Dear instructors,

Thank you for your interest in this open exercise book. I hope you find it a
valuable addition to your existing teaching material.

I would love to hear your feedback and adoption decision for your course.
Your feedback will be highly appreciated and will be used for future im-
provement of the book.

Please take a few minutes to fill in the survey, which can be accessed in the
following way:

Survey Link

Thank you for your time and support on this open exercise book develop-
ment.

Best Regards,

Paul Tsopméné

This page and survey is adapted from Introduction to Engineering Thermodynamics by


Dr. Claire Yan which is licensed under a Creative Commons Attribution-NonCommercial-
ShareAlike 4.0 International License, except where otherwise noted.
iv
Contents

Preface ix

I Exercises 1
1 Linear Functions 3
1.1 Slopes and Equations of Lines . . . . . . . . . . . . . . . . . . . . . . . . . 3
1.2 Linear Functions and Applications . . . . . . . . . . . . . . . . . . . . . . . 3

2 Quadratic, Exponential, and Logarithmic Functions 7


2.1 Quadratic Functions . . . . . . . . . . . . . . . . . . . . . . . . . . . . . . 7
2.2 Exponential Functions and Compound Interest . . . . . . . . . . . . . . . . 8
2.3 Logarithmic Functions . . . . . . . . . . . . . . . . . . . . . . . . . . . . . 9

3 Limits and Derivatives 11


3.1 Finding Limits Using Tables and Graphical Limits . . . . . . . . . . . . . . 11
3.2 Finding Limits Using Rules and One-Sided Limits . . . . . . . . . . . . . . 12
3.3 Infinite Limits and Vertical Asymptotes . . . . . . . . . . . . . . . . . . . . 15
3.4 Limits at Infinity and Horizontal Asymptotes . . . . . . . . . . . . . . . . 15
3.5 Continuity . . . . . . . . . . . . . . . . . . . . . . . . . . . . . . . . . . . . 16
3.6 Rates of Change . . . . . . . . . . . . . . . . . . . . . . . . . . . . . . . . . 19
3.7 Definition of the Derivative . . . . . . . . . . . . . . . . . . . . . . . . . . 21
3.8 Graphical Differentiation . . . . . . . . . . . . . . . . . . . . . . . . . . . . 23

4 Differentiation Rules 31
4.1 Basic Rules of Differentiation . . . . . . . . . . . . . . . . . . . . . . . . . 31
4.2 The Product and Quotient Rules . . . . . . . . . . . . . . . . . . . . . . . 32
4.3 The Chain Rule . . . . . . . . . . . . . . . . . . . . . . . . . . . . . . . . . 33
4.4 Derivatives of Exponential Functions . . . . . . . . . . . . . . . . . . . . . 34
4.5 Derivatives of Logarithmic Functions . . . . . . . . . . . . . . . . . . . . . 35
4.6 Derivatives of Trigonometric Functions . . . . . . . . . . . . . . . . . . . . 36
4.7 Exponential Growth and Decay . . . . . . . . . . . . . . . . . . . . . . . . 37

5 Applications of the Derivative I 41


5.1 Increasing and Decreasing Functions . . . . . . . . . . . . . . . . . . . . . 41
5.2 Relative (or Local) Extrema . . . . . . . . . . . . . . . . . . . . . . . . . . 42

v
vi CONTENTS

5.3 Concavity . . . . . . . . . . . . . . . . . . . . . . . . . . . . . . . . . . . . 43
5.4 Curve Sketching . . . . . . . . . . . . . . . . . . . . . . . . . . . . . . . . . 44

6 Applications of the Derivative II 45


6.1 Absolute Extrema . . . . . . . . . . . . . . . . . . . . . . . . . . . . . . . . 45
6.2 Optimization Problems . . . . . . . . . . . . . . . . . . . . . . . . . . . . . 46
6.3 Implicit Differentiation and Related Rates . . . . . . . . . . . . . . . . . . 46
6.4 Elasticity of Demand . . . . . . . . . . . . . . . . . . . . . . . . . . . . . . 48
6.5 Differentials and Linear Approximations . . . . . . . . . . . . . . . . . . . 49
6.6 Newton’s Method . . . . . . . . . . . . . . . . . . . . . . . . . . . . . . . . 50

II Solution to Exercises 53
1 Linear Functions 55
1.1 Slopes and Equations of Lines . . . . . . . . . . . . . . . . . . . . . . . . . 55
1.2 Linear Functions and Applications . . . . . . . . . . . . . . . . . . . . . . . 60

2 Quadratic, Exponential, and Logarithmic Functions 67


2.1 Quadratic Functions . . . . . . . . . . . . . . . . . . . . . . . . . . . . . . 67
2.2 Exponential Functions and Compound Interest . . . . . . . . . . . . . . . . 76
2.3 Logarithmic Functions . . . . . . . . . . . . . . . . . . . . . . . . . . . . . 82

3 Limits and Derivatives 93


3.1 Finding Limits Using Tables and Graphical Limits . . . . . . . . . . . . . . 93
3.2 Finding Limits Using Rules and One-Sided Limits . . . . . . . . . . . . . . 96
3.3 Infinite Limits and Vertical Asymptotes . . . . . . . . . . . . . . . . . . . . 112
3.4 Limits at Infinity and Horizontal Asymptotes . . . . . . . . . . . . . . . . 119
3.5 Continuity . . . . . . . . . . . . . . . . . . . . . . . . . . . . . . . . . . . . 127
3.6 Rates of Change . . . . . . . . . . . . . . . . . . . . . . . . . . . . . . . . . 140
3.7 Definition of the Derivative . . . . . . . . . . . . . . . . . . . . . . . . . . 148
3.8 Graphical Differentiation . . . . . . . . . . . . . . . . . . . . . . . . . . . . 164

4 Differentiation Rules 183


4.1 Basic Rules of Differentiation . . . . . . . . . . . . . . . . . . . . . . . . . 183
4.2 The Product and Quotient Rules . . . . . . . . . . . . . . . . . . . . . . . 193
4.3 The Chain Rule . . . . . . . . . . . . . . . . . . . . . . . . . . . . . . . . . 203
4.4 Derivatives of Exponential Functions . . . . . . . . . . . . . . . . . . . . . 214
4.5 Derivatives of Logarithmic Functions . . . . . . . . . . . . . . . . . . . . . 222
4.6 Derivatives of Trigonometric Functions . . . . . . . . . . . . . . . . . . . . 233
4.7 Exponential Growth and Decay . . . . . . . . . . . . . . . . . . . . . . . . 246

5 Applications of the Derivative I 259


5.1 Increasing and Decreasing Functions . . . . . . . . . . . . . . . . . . . . . 259
5.2 Relative (or Local) Extrema . . . . . . . . . . . . . . . . . . . . . . . . . . 269
5.3 Concavity . . . . . . . . . . . . . . . . . . . . . . . . . . . . . . . . . . . . 278
5.4 Curve Sketching . . . . . . . . . . . . . . . . . . . . . . . . . . . . . . . . . 289
vii

6 Applications of the Derivative II 303


6.1 Absolute Extrema . . . . . . . . . . . . . . . . . . . . . . . . . . . . . . . . 303
6.2 Optimization Problems . . . . . . . . . . . . . . . . . . . . . . . . . . . . . 310
6.3 Implicit Differentiation and Related Rates . . . . . . . . . . . . . . . . . . 316
6.4 Elasticity of Demand . . . . . . . . . . . . . . . . . . . . . . . . . . . . . . 331
6.5 Differentials and Linear Approximations . . . . . . . . . . . . . . . . . . . 338
6.6 Newton’s Method . . . . . . . . . . . . . . . . . . . . . . . . . . . . . . . . 351
viii
Preface

The author wrote this exercise book when teaching MATH 116 (Calculus I for Management
and Economics) at the University of British Columbia Okanagan. The book is primarily
for students taking a first-year differential calculus course (with business and economics
applications). Students taking a general Calculus I course can also use it to get down basic
content in differential calculus.
The most important way to learn calculus is through problem-solving. While going
through the solution to a problem, students are often faced with several issues. They may
not see the connection between the concept taught in class and the solution. Others may
not understand the solution because a step is missing or there are not enough explanations.
Or because they have weak algebra skills. I think that nearly everybody agrees that the
latter issue is the main reason many students struggle with calculus.
My main goal in this book is to address these issues to help students learn the material
more efficiently and get better results. To that end, I have included the following features.

ˆ Concept Summaries Boxes: They allow students to review the material and make
the book self-contained. Each concept involved in a problem is summarized in a box
right before the solution. The relevant information needed to solve the problem is
given. And the solution clearly shows how that information is used.

ˆ Very Detailed Solutions: They provide an “easy-to-understand” approach. Each


problem has a complete step-by-step solution with careful explanations for better
comprehension. Alternate solutions (to some problems) are also given.

ˆ “Help Texts”: They are located to the right of the solution sequence and help
students recognize which rule, property, or procedure is being applied.

ˆ Algebra Review: They help students understand the solution, especially those with
weaker algebra skills. Basic algebra rules and procedures (exponent rules, adding
fractions, factoring, etc.) are listed next to the steps within solutions. Also, not only
the calculus steps are shown in the solution, but also the algebra steps. The idea is
that students with a poor algebra background can just jump in and learn as they go.

The book is divided into two parts. The first part includes a variety of problems as well
as applications in business and economics. The idea is not to have as many problems as
in a traditional textbook but a fair amount of problems that cover essential/basic skills in
differential calculus. The second part includes full solutions to every problem. Hyperlinks

ix
x Preface

are added to ease the navigation of the book (they are mainly used to go back and forth
between the two parts).
To the Student: If you try to solve a problem and have no idea what to do, you can
look at the solution and try to understand it (if there is a box, you should first try to
read and understand its content). Try to solve the subsequent few problems yourself before
checking the solution. It’s not a big deal if you make a mistake; you will learn better from
that mistake. If your final answer does not match the one in the solution, try to see if it
is equivalent (since there are often several different

ways of expressing the answer). For
x
example, if the answer given in the book is x and you obtain √1x , you are right because

x
rationalizing √1 will give you .
x x
PartI

Exercises

1
Chapter1

Linear Functions

1.1 Slopes and Equations of Lines


1. Find the slope of the line through each pair of points. [Solution on page 55]

(a) (1, 2) and (4, 11) (e) (−2, −3) and (0, 6)
(b) (0, 1) and (2, 9) (f ) (−15, −14) and (−7, −2)
(c) (5, 6) and (6, 4) (g) (1, 1) and ( 31 , 34 )
(d) (10, 13) and (7, 19) (h) ( 21 , 0) and (− 34 , 45 )

2. Find the value of x such that the slope of the line through (−1, 1) and (2, 4x) is 2.
[Solution on page 57]
3. Find an equation of the line through the given point with slope m. [Solution on
page 57]
1
(a) (2, 3) and m = 4 (b) (0, 1) and m = 3

4. Find an equation of the line through the given points. [Solution on page 58]

(a) (1, 2) and (4, 5) (e) (3, 3) and (1, 4)


(b) (2, −7) and (3, 1)
(f ) ( 12 , 1) and ( 52 , 10)
(c) (0, 2) and (−1, 6)
(d) (−1, 2) and (−3, 6) (g) (2, − 12 ) and ( 23 , 14 )

5. Graph each equation. [Solution on page 59]

(a) x = 2 (c) y = 2x − 1
(b) y = 1 (d) y = −x + 4

1.2 Linear Functions and Applications


1. The sales of a small company were $17,000 in its fourth year of operation and $25,000
in its sixth year. Let y represent sales in the xth year of operation. Assume that the
data can be approximated by a straight line. [Solution on page 60]

3
4 Chapter 1. Linear Functions

(a) Find the slope of the sales line, and give an equation for the line in the form
y = mx + b.
(b) Use your answer from part (a) to find out how many years must pass before the
sales surpass $48,500.

2. Consider the following data collected from a population of students.

Number of hours studied 1 2 3 4 5


Grades on exam 10 14 16 18 20

Let x represent the number of hours studied and let y represent the grades on exam.
[Solution on page 61]

(a) Plot the date. Are the data exactly linear? Could the data be approximated by
a linear equation?
(b) Use the points (1, 10) and (5, 20) to determine a linear equation (of the form
y = mx + b) that models the data. What does the slope of the graph of the line
indicate?
(c) Use the equation in part (b) to estimate the grade of a student who studied 6
hours.

3. Determine whether each function is linear or not. [Solution on page 62]

(a) f (x) = 9x − 6 (c) f (q) = 1 − q + q 2



(b) g(t) = 1 − ( 3)t

4. Let f (x) = 5 − 3x and g(x) = 34 x + 7. Find the following. [Solution on page 62]
√ 
(a) f (2) (c) f 2
(b) g 13


5. [Solution on page 62]


The demand function for a certain type of shoe is given by

p = D(q) = 10 − 0.5q

where p is the price expressed in dollars and q is the quantity demanded (in thou-
sands).

(a) Find the price for each quantity. (i) q = 0 shoe; (ii) q = 5000 shoes.
(b) Find q for each price. (i) p = $4; (ii) p = $9.
(c) Suppose the supply function is given by p = S(q) = 0.25q + 1. Graph the supply
and demand functions on the same axes.
(d) Find the equilibrium quantity and price.
1.2. Linear Functions and Applications 5

6. [Solution on page 64]


Suppose the supply and demand functions for a product are given by
4 3
p = S(q) = q and p = D(q) = 105 − q
5 5
Find the equilibrium quantity and the equilibrium price.

7. [Solution on page 64]


A company has a fixed cost of $37, 000 and spends $0.24 to produce a pencil. Find
the cost function assuming that it is linear.

8. [Solution on page 64]


Suppose the fixed cost of a company is $80, 000, and the production costs $7 for each
unit produced. Suppose further that each unit is sold at $10.

(a) Find the cost function assuming that it is linear.


(b) Find the revenue function.
(c) What is the profit corresponding to the production level of 35000 units?

9. [Solution on page 65]


The cost of producing x units of T-shirt by a manufacturer is C(x) = 3x + 85, and
the revenue function is R(x) = 20x, where C(x) and R(x) are expressed in dollars.

(a) Find the break-even quantity.


(b) What is the profit if 10 units are sold?
(c) How many units need to be sold to get a profit of $11, 815?
6 Chapter 1. Linear Functions
Chapter2

Quadratic, Exponential, and


Logarithmic Functions

2.1 Quadratic Functions


1. Find the coefficients of each quadratic function. [Solution on page 67]

(a) f (x) = 3x2 + 6x + 8 (d) u(x) = 16 − 3x2


(b) f (x) = x2 − x − 5
√ q2
(c) g(t) = −t2 + 11 (e) h(q) = 2q − 7

2. Factor each expression. [Solution on page 67]

(a) x2 − 5x (d) −9x2 + 49


(b) −6x2 − 9x + 4x + 6 (e) x2 + x − 2
(c) x2 − 25 (f ) 2x2 − 7x + 3

3. Solve each equation. [Solution on page 68]

(a) 4x2 − 9 = 0 (e) 4x2 + 12x = −9


(b) x2 − 3x = 0 (f ) x2 − 3x + 4 = 0
(c) x2 + 3x − 4 = 0 (g) −12x2 + 17x = 20
(d) −3x2 + x + 2 = 0 (h) 0.1x2 = 4.5x − 12.6

4. Graph each function. [Solution on page 71]

(a) f (x) = x2 − 4x + 3 (c) f (x) = x2 + 4x + 4


(b) f (x) = −2x2 + 7x + 4 (d) f (x) = −3x2 + 2x − 1

5. The revenue from producing x units of a product is given by R(x) = −x2 + 26x, while
the cost in dollars is given by C(x) = 7x + 48 (revenue and cost are in thousands of
dollars). [Solution on page 74]
(a) Find the minimum break-even quantity.

7
8 Chapter 2. Quadratic, Exponential, and Logarithmic Functions

(b) Find the maximum revenue.


(c) Find the maximum profit.

6. Suppose the revenue and cost functions (in thousands of dollars) are given by

x2 35
R(x) = − + 5x and C(x) = x +
3 3
[Solution on page 75]

(a) Find the minimum break even quantity.


(b) Find the maximum profit.

2.2 Exponential Functions and Compound Interest


1. Let a be a real number such that a > 0 and a 6= 1. Decide whether the statement is
true or false. [Solution on page 76]

(a) The exponential function f (x) = ax is defined for every real number x.
(b) The function f (x) = ax is increasing when 0 < a < 1.
(c) The function f (x) = ax is decreasing when a > 1.
(d) The graph of f (x) = ax intersects the x-axis.
(e) The y-intercept of f (x) = ax is 1.
(f ) It is possible to find x such that ax ≤ 0.

2. Write each expression as a single exponential. [Solution on page 78]


32x 3−2 5x (7x 7−5x )2
(a) 3x−1
(b) √ 3x (c) √
5( 1
) 71−x 3 (7x )2
52

3. Solve each equation. [Solution on page 78]


2
(a) 4x = 410 (e) 36x = 216 (i) 5x 5x = 56
2
(b) 5x = 59 (f ) 27x−1 = 813x+2 7x
2
(j) 73x−2
=1
(c) 32x = 3x (g) 2x = 1
2 2
(d) 25x = 58 (h) 34x − 336 = 0 (k) (3x )x = 3x

4. Investing $2000 at a rate of 12% compounded annually, how much would you have
after 5 years? [Solution on page 79]

5. Investing $7000 at a rate of 4% compounded quarterly, how much would you have
after 3 years? [Solution on page 80]

6. Suppose you invest $12000 in an account earning 5% compounded semiannually for


6 years. [Solution on page 80]
2.3. Logarithmic Functions 9

(a) Find the compound amount.


(b) How much interest will you earn?

7. [Solution on page 81]


Samuel invests $5000 at 7% annual interest compounded monthly for 9 years. How
much interest will he earn?

8. Investing $15000 at a rate of 5% compounded continuously, how much would you


have after 12 years? [Solution on page 81]

9. Suppose you invest $8000 at a rate of 3% annual interest compounded continuously


for 10 years. How much interest will you earn? [Solution on page 81]

2.3 Logarithmic Functions


1. Evaluate each expression without using a calculator. [Solution on page 82]

(a) log2 (64) (d) log10 (0.0001)


(b) log3 (81) (e) log2 (0.25)
q 
1 (f ) log2 3 18

(c) log5 125

2. Decide whether the statement is true or false. [Solution on page 82]

(a) The logarithm of a negative number is undefined.


(b) Given a base a > 0, a 6= 1, loga (0) = 1.
(c) The graph of f (x) = loga (x) is decreasing when a > 1 and increasing when
0 < a < 1.
(d) The x-intercept of the function f (x) = loga (x) is 1 for any base a > 0, a 6= 1.
(e) The function f (x) = loga (x) has a y-intercept.
(f ) The logarithm of a number can be negative.

3. Evaluate each expression without using a calculator. [Solution on page 84]

(a) log3 (3) + log3 (27) (c) log5 (0.043 )


(b) log2 (40) − log2 (5)

4. Simplify the following expressions. [Solution on page 84]


3
(a) log2 x2020 (d) ln(x8 e−x )

(b) log5 (x2 y −7 ) (e) ln 3 x2 − 3x
 4 x2

(f ) ln (3x−1)
8
(c) log( xy 3 ) (2x+1)5

5. Express each quantity as a single logarithm. [Solution on page 86]


10 Chapter 2. Quadratic, Exponential, and Logarithmic Functions

(a) 3 ln x + 2 ln(x − 1) (c) log(x + 2) − 4 log(x) + 4


1
(b) 3 ln x − ln(5x) + 7
2

6. If x is a positive number (x > 0), show that log9 x = 21 log3 x. [Solution on page 86]

7. Solve each equation. [Solution on page 87]

(a) logx 16 = 4. (d) log2 x − log2 (x − 2) = 1


(b) logx 27

8
=3
1
(c) log5 x = 2 (e) log6 x + log6 (x − 1) = 1

8. Solve each equation. [Solution on page 87]

(a) ln(2x + 1) = ln(x + 5) (d) ln(3x + 1) = 7


(b) ln(x2 + x) = ln(2x + 6)
(c) ln x = 2 (e) ln(2x) + ln(5x) = −1

9. Solve each equation. [Solution on page 89]

(a) 2x = 5 (d) 2e−3x = 9


(b) 32x−1 = 7
(c) 36x = 4x+1 (e) 2e−0.1x + 3 = 15

10. Write each expression using base e. [Solution on page 90]

(a) 2−x (b) 35x−2

11. Suppose you invest $1000 at a rate of 4%. After t years your amount on deposit is
the double of the initial amount. Find t in each case. [Solution on page 90]

(a) The interest is compounded annually.


(b) The interest is compounded quarterly.
(c) The interest is compounded continuously.

12. Suppose you invest a certain amount of money. After t years your amount on deposit
is A(t) = 400 ln(t + 4), where A is in millions of dollars. [Solution on page 91]

(a) What is the value of the account when t = 0?


(b) How long does it take for the account to double its initial value?
Chapter3

Limits and Derivatives

3.1 Finding Limits Using Tables and Graphical Limits


1. Use a table of values to guess the following limits. [Solution on page 93]

(a) lim x2 + 3 x2 − 4 (c) lim ex −1


x→1 (b) lim x→0 x
x→−2 x+2

2. Let f be the function whose graph is shown in Figure 3.1. Use the graph to state the
value of each quantity, if it exists. If it does not exist, explain why. [Solution on
page 94]

(a) f (2) (c) f (4) (e) f (3)


(b) lim f (x) (d) lim f (x) (f ) lim f (x)
x→2 x→4 x→3

−2 2 4 6 x

−2

Figure 3.1

11
12 Chapter 3. Limits and Derivatives

3. Let f be the function whose graph is shown in Figure 3.2. Use the graph to state the
value of each quantity, if it exists. If it does not exist, explain why. [Solution on
page 94]

(a) lim− f (x) (c) lim f (x) (e) lim f (x) (g) lim f (x)
x→2 x→2 x→6 x→−1+

(b) lim+ f (x) (d) f (2) (f ) f (6) (h) lim+ f (x)


x→2 x→7

−2 2 4 6 8 x
−1

−2

Figure 3.2

4. Let f be the function whose graph is shown in Figure 3.3. Use the graph to state the
value of each quantity, if it exists. If it does not exist, explain why. [Solution on
page 95]

(a) lim f (x) (c) lim f (x) (e) lim f (x) (g) lim− f (x)
x→−1− x→−1 x→3 x→6

(b) lim f (x) (d) f (−1) (f ) f (3) (h) f (6)


x→−1+

5. Let f be the function whose graph is shown in Figure 3.4. State the equations of the
vertical asymptotes. [Solution on page 95]

3.2 Finding Limits Using Rules and One-Sided Limits


1. Find each limit (using the rules). [Solution on page 96]

(a) lim x3 + x (c) lim − x2 + 5 x
x→4 x→9

9x2 −4
(b) lim x5 − 4x3 − 7x + 4 (d) lim
x→2 x→1 x+1
3.2. Finding Limits Using Rules and One-Sided Limits 13

−4 −2 2 4 6 x

Figure 3.3

−1 1

Figure 3.4

x4√
−3x2 +1 3 +x
(e) lim x+98
(h) lim e−2x
x→2 x→0
2

3
(f ) lim (x −3x−8)
x−1
2x−3
x→5

 
x2 +x
(g) lim x2 − 16 (i) lim1 ln 2x+3
x→3 x→ 2

2. Evaluate each limit. [Solution on page 99]


14 Chapter 3. Limits and Derivatives

x2 −16 3x2 +8x−3


(a) lim x−4
(f ) lim x2 +3x
x→4 x→−3

x2 +2x−3 8x2 +2x−3


(b) lim (g) lim1 2x2 +3x−2
x→1 5x−5 x→ 2
 
x2 −2x−3 x2 +14x−15
(c) lim x2 −9 (h) lim log2
x→3 x→1 x2 −1

x2 +3x−10 x2 −x
(d) lim 2 (i) lim 3 x
x→−5 x +6x+5 x→0
1
2x2 −x−6 − 18
(e) lim 2 (j) lim x
x→2 x +2x−8 x→8 x−8

3. Evaluate each limit. [Solution on page 102]



√2x x2 +16−4
(a) lim+ 2x
(f ) lim x2
x→0 x→0

(b) lim √x−4 (g) lim 1− x1−x
2

x→4 x−2
x→0
√ √
(c) lim x−5 (h) x−2
lim −3x+11
x→25 x−25 x→4
√ √ √
(d) lim x3− x
2 −81 (i) lim 2x+5− x+7
x→9 x→2 x−2

(e) lim x4−2 x
2 −x−12 (j) lim √ x−3
x2 −5x+10−2
x→4 x→3

4. Consider the functions


 
1 + x3 if x 6= 2 −4
√ if x = −3
f (x) = and g(x) = 2
5 if x = 2 x + 16 if x 6= −3

[Solution on page 108]

(a) Find f (2) (c) Find g(−3)


(b) Find lim f (x) (d) Find lim g(x)
x→2 x→−3

5. Let
 
x2 + x + 1 if x ≤ 1 −3 − x if x < −2
f (x) = and g(x) =
x+2 if x > 1 x2 − 10 if x ≥ −2

[Solution on page 108]

(a) Find f (−2) (c) Find f (2) (e) Find lim f (x)
x→1

(b) Find f (1) (d) Find g(0) (f ) Find lim g(x)


x→−2

6. Let 
 x − 5 if x < −2
f (x) = 0 if −2 ≤ x < 4
 x2
2
− 8 if x ≥ 4
[Solution on page 109]
3.3. Infinite Limits and Vertical Asymptotes 15

(a) Find f (−2) (c) Find lim f (x)


x→−2

(b) Find f (4) (d) Find lim f (x)


x→4

7. Find each limit. [Solution on page 110]


x2 +2x−3
(a) lim |x − 2| (c) lim |x−1|
x→2 x→1
(b) lim 3x + |x + 6|
x→−6

3.3 Infinite Limits and Vertical Asymptotes


x+1
1. Find lim [Solution on page 112]
x→3 −3x+9

4−x
2. Find lim 2 [Solution on page 114]
x→2 x −4x+4

−5
3. Find lim 2 [Solution on page 115]
x→−2 (x+2)

1
4. Find lim 2 [Solution on page 116]
x→0 x

x2 +3x−4
5. Find lim 2 [Solution on page 116]
x→1 x −2x+1

7x
6. Let f (x) = (x−5)3
. [Solution on page 117]

(a) Find lim f (x)


x→5

(b) Find the vertical asymptotes of f .


x2 +x−6
7. Find the vertical asymptotes of f (x) = x2 −x−2
. [Solution on page 117]

3.4 Limits at Infinity and Horizontal Asymptotes


1. Find each limit. [Solution on page 119]

(a) lim x4 (f ) lim 3x2


x→∞ x→∞

(b) lim x5 (g) lim − 2x10


x→∞ x→∞

(c) lim x8 (h) lim − 5x10


x→−∞ x→−∞

(d) lim x7 (i) lim 5x9


x→−∞ x→−∞

(e) lim 3 x (j) lim − 6x31
x→∞ x→−∞

2. Find each limit. [Solution on page 120]


16 Chapter 3. Limits and Derivatives

(a) lim x5 + x3 + 1 (d) lim 1 − x2 + x2020


x→∞ x→−∞
3 5
(b) lim 2x − x + 7x − 1 (e) lim 1 − 8x5 + 2x2
x→∞ x→−∞
(c) lim − 4x3 + x − 1
x→∞

3. Find each limit. [Solution on page 120]


1 −3x+5
(a) lim 4 (f ) lim 2
x→−∞ x x→−∞ x −x+1

(b) lim 5 3x2 −2x


9 (g) lim 3 2
x→−∞ 3x x→∞ −5x +x −x

√1 2x3 −x2 +3
(c) lim (h) lim 2 +3x−1
x→∞ x x→∞ −4x

x2 +1 x7 −x3 +1
(d) lim 2 (i) lim 4 2
x→∞ 3x +x+1 x→−∞ −x +3x +8

7x3 x+x2
(e) lim 3 (j) lim 2x−x
x→−∞ −4
x 2
x→∞

4. Evaluate each limit. [Solution on page 124]



(a) lim x2 + 3 − x
x→∞

(b) lim x2 + x + 1 − x
x→∞

5. In each case, find the horizontal asymptotes (if any). [Solution on page 126]
4x3 +1 x3
(a) f (x) = x3 −x
(c) f (x) = x2 +1
x
(b) f (x) = x2 +1

6. [Solution on page 127]


A manufacturer has found that the cost (in dollars) of producing x items of a partic-
ular commodity is given by
C(x) = 13x + 8000
Find the limit of the average cost C(x) as x goes to infinity, and interpret your answer.

3.5 Continuity
1. Let f be the function whose graph is shown in Figure 3.5. Find all the points of
discontinuity. Verify that each point does not satisfy the definition of continuity.
[Solution on page 127]

2. Consider the following functions. [Solution on page 128]


 x2 −4  x2 −4
x2 − 4 x−2
if x 6= 2 x−2
if x 6= 2
f (x) = g(x) = h(x) =
x−2 3 if x = 2 4 if x = 2

Are these continuous at 2?


3.5. Continuity 17

y
6

−8 −6 −4 −2 2 4 6 8 x

−1

−2

Figure 3.5

3. In each case, determine if the function is continuous at the given point. [Solution on
page 129]


x + 2 if x < 0
(a) f (x) = point: x = 0
x2 if x > 0


2x − 1 if x ≤ 1
(b) f (x) = x2 point: x = 1
x+1
if x > 1


x√2 + 2x if x < −2
(c) f (x) = point: x = −2
x + 3 if x ≥ −2


e2x
 if x < 0
(d) f (x) = 2 if x = 0 point: x = 0
 3
x + 1 if x > 0
18 Chapter 3. Limits and Derivatives

x2 −2x−3

−2x+6
if x < 3
(e) f (x) = point: x = 3
−2 + ln(x − 2) if x ≥ 3

ex−1

 2
if x ≤ 1
(f ) f (x) = √
point: x = 1
 x−1
x−1
if x > 1

4. Let f be the function defined by



 log3 (7 − x) if x < 4
f (x) = 0 if x = 4
 2
x − 4x + 1 if x > 4

[Solution on page 132]

(a) Explain why f is discontinuous at x = 4.


(b) If you could change the value of f (4), what should it be to make the function
continuous at 4.

5. In each case, find all constants k such that the function is continuous at the given
point. [Solution on page 132]

kx2 − 6 if x ≤ 2
(a) f (x) = point: x = 2
3x + k if x > 2

2x2 −3x−5

if x 6= −1
(b) f (x) = x+1 point: x = −1
kx + 5 if x = −1


 −2x3 − 4x if x < 1
(c) f (x) = k 2 − 5k if x = 1 point: x = 1
−6x if x > 1

6. Let f be the function whose graph is shown in Figure 3.5. Determine if f is continuous
on the following intervals: [Solution on page 134]

(−7, −5), (−6, −4), [0, 2], [5, 6], [4, 6], and [3, 4].

7. Show that the function f (x) = 9 − x2 is continuous on the interval [−3, 3]. [Solution
on page 135]

8. Let f be the function defined by



7x − 2 if x ≤ 1
f (x) =
x2 − 3x + 8 if x > 1

Determine if f is continuous on the interval [1, 4]. [Solution on page 136]

9. Show that the function f (x) = |x| is continuous for all x. [Solution on page 136]
3.6. Rates of Change 19


x + 2 if x ≤ k
10. Let f (x) = Find all constants k that make f continuous on
x2 if x > k.
R = (−∞, ∞). [Solution on page 136]

11. Find the intervals where each function is continuous. [Solution on page 137]

(a) f (x) = x5 − 3x4 + 7x − 1 (c) h(x) = 2x − 6
2x
(d) k(x) = e
x2
(b) g(x) = 2 (e) l(x) = ln(−3x + 12)
x − 16

12. Find all values x = c where the function is discontinuous. [Solution on page 138]
2x + 1 |x − 6|
(a) f (x) = (c) f (x) =
(2x − 3)(x + 7) x−6
x3
(b) f (x) = (d) f (x) = x4 − x + 1
x2 − 3x + 2

13. Let f be the function defined by



 x+5 if x<0
2
f (x) = x − 2x + 4 if 0 ≤ x < 3
10 − x if x≥3

Find all values x where f is discontinuous. [Solution on page 138]

14. Show that the equation x3 + x − 1 = 0 has a solution between 0 and 1. [Solution on
page 139]

15. Show that the equation x5 − 2x2 = 7 has a solution between −1 and 2. [Solution on
page 139]

16. [Solution on page 140]


A firm has determined that they spend $15/unit to produce 280 units or less of a
certain product, and $12/unit if they want to produce more than 280 units. Let C(x)
be the cost of producing x units. Find the cost to make the following number of
units.

(a) 250 units (c) 300 units


(b) 280 units (d) 500 units

(e) For what values of x is the function C discontinuous?

3.6 Rates of Change


1. In each case, find the average rate of change of the given function over the given
interval. [Solution on page 140]

(a) f (x) = 10 over the interval from x = 1 to x = 2


20 Chapter 3. Limits and Derivatives

(b) f (x) = −π 2 over the interval from x = −3 to x = 5


(c) f (x) = 4x + 5 over the interval from x = 0 to x = 7
(d) f (x) = −x2 + 3x + 8 over the interval from x = −2 to x = 8
(e) f (x) = 6 + x − x3 over the interval from x = −11 to x = −1

(f ) f (x) = 2x + 9 over the interval from x = −3 to x = 0
1 1
(g) f (x) = e2x over the interval from x = 4
to x =2
1 1
(h) f (x) = ln(1 + x2 ) over the interval from x = −3 to x = 5

2. In each case, find the instantaneous rate of change for f (x) when x = a. [Solution
on page 142]

(a) f (x) = 3x at x = 1 (c) f (x) = −5x2 at x = −1


(b) f (x) = 4x2 at x = 3 (d) f (x) = −x2 + 7 at x = −6

3. A car is moving along a straight road. Suppose its position (in kilometers) is given
by s(t) = 25t2 , where t is measured in hours. [Solution on page 143]

(a) Find the average velocity of the car between t = 0 and t = 2.


(b) Find the instantaneous velocity of the car at t = 2.

4. An object moves along a straight line. Its position in feet at time t is given by
1
s(t) = t2 − 5t + 19,
3
where t is measured in seconds. [Solution on page 144]

(a) Find the instantaneous velocity when t = 9.


(b) Find the instantaneous velocity when t = 13.

5. Suppose that the cost in dollars of producing x items of a certain commodity is given
by
C(x) = 200 + 21x − x2 (0 ≤ x ≤ 10)
[Solution on page 146]

(a) Find the average rate of change of cost if the number of items produced changes
from 2 to 3.
(b) Suppose the production level is x = 2 items. Find the cost of producing one
additional item. Compare your answer to the one found in part (a).
(c) Find the instantaneous rate of change of cost when 2 items are produced. Com-
pare your answer to the one found in part (b).

6. Suppose that the profit (in thousands of dollars) from selling x items of a certain
product is given by
P (x) = 3x2 − 9x + 8
[Solution on page 147]
3.7. Definition of the Derivative 21

(a) Find the average rate of change of profit when the number of items sold changes
from 1 to 2.
(b) Find the average rate of change of profit when the number of items sold changes
from 2 to 5.
(c) Find the instantaneous rate of change of profit when x = 1. Interpret your
answer.
(d) Find and interpret the instantaneous rate of change of profit when x = 5.

3.7 Definition of the Derivative


1. Let f be the function whose graph is shown in Figure 3.6. [Solution on page 148]

−2 −1 1 2 3 4 x

−2

P1 −4

−6

Figure 3.6

(a) The tangent line to the curve at P1 (−1, −4) passes through the points (−1, −4)
and (0, 5). Find the derivative f 0 (−1).
(b) Find the derivative f 0 (1) and the equation of the tangent line at (1, −2).
(c) Find the points where the derivative is zero.
(d) Is the derivative f 0 (3) negative? Justify your answer.

2. Let f be the function whose graph is shown in Figure 3.7. Find f 0 (1). [Solution on
page 150]
22 Chapter 3. Limits and Derivatives

1 x

−1

Figure 3.7

3. Find an equation of the tangent line to f (x) = −x2 at (1, −1). [Solution on
page 151]

4. Find an equation of the tangent line to f (x) = x at (4, 2). [Solution on page 152]

5. Let f (x) = −3x2 + 4x. Find the derivative of f at a = 2 using the limit definition of
derivative. [Solution on page 153]

6. Let f (x) = x3 − x. Find the derivative of f at a = −1 using the limit definition of


derivative. [Solution on page 154]
x
7. Let f (x) = x+1
. Find the derivative of f at a = 3 using the limit definition of
derivative. [Solution on page 154]

8. Give the derivative of each function just by considering the slope of the tangent line.
[Solution on page 155]

(a) f (x) = 10 (c) f (x) = − 23


5−4x
(b) f (x) = −2x + 3 (d) f (x) = 6

9. In each case, find f 0 (x) by using the definition of the derivative. [Solution on
page 156]

(a) f (x) = k, where k is a (c) f (x) = x2 (e) f (x) = x
constant.
1
(b) f (x) = x (d) f (x) = x3 (f ) f (x) =
x

10. In each case, find f 0 (x) by using the definition of the derivative. Then find f 0 (−3), f 0 (0),
and f 0 (2) (if the derivative exists). [Solution on page 158]
3.8. Graphical Differentiation 23

(a) f (x) = −4x + 1 2x − 1


(c) f (x) =
3x + 7

(b) f (x) = 3x2 − 5x (d) f (x) = 3x

11. Show that the function f (x) = |x − 1| is not differentiable at x = 1. [Solution on


page 161]

12. Consider the function f whose graph is shown in Figure 3.8. [Solution on page 162]

y
7

−6 −5 −4 −3 −2 −1 1 2 3 4 5 6 x

−1

−2

Figure 3.8

(a) Determine if f is continuous and differentiable at the following points:

−6, −4, −3, −1, 0, 1, 3, 4, 5.

(b) Find f 0 (−5), f 0 (−2), and f 0 (2).

3.8 Graphical Differentiation


1. Let f be the function whose graph is shown below. Sketch the graph of the derivative
of f . [Solution on page 164]
24 Chapter 3. Limits and Derivatives

−3 3 x

2. Let f be the function whose graph is shown below. Sketch the graph of the derivative
of f . [Solution on page 165]

−3 −2 −1 1 2 3 x
−1

3. Sketch the graph of the derivative of the following function. [Solution on page 165]

y
3
2
1

−3 −2 −1 1 2 3 x
−1
−2
−3

4. Sketch the graph of the derivative of the following function. [Solution on page 166]
3.8. Graphical Differentiation 25

4
3
2
1

−1 1 2 3 4 5 x
−1

5. Sketch the graph of the derivative of the following function. [Solution on page 170]

3
2
1

−3 −2 −1 1 2 x
−1
−2

6. Sketch the graph of the derivative of the following function. [Solution on page 171]

y
4
3
2
1

−1 1 2 3 4 x
−1
−2

7. Sketch the graph of the derivative of the following function. [Solution on page 172]
26 Chapter 3. Limits and Derivatives

y
4

−4 −2 2 4 x
−2

−4

8. Sketch the graph of the derivative of the following function. [Solution on page 173]

y
4

−2 2 4x

−2

9. Sketch the graph of the derivative of the following function. [Solution on page 174]

y
4

−6 −4 −2 2 x

10. Sketch the graph of the derivative of the following function. [Solution on page 175]
3.8. Graphical Differentiation 27

y
3

−5 5 x

11. Let f be the function whose graph is shown below. [Solution on page 176]

y
4

−2 2 4 6 x

(a) Which is bigger, f 0 (0) or f 0 (2)?


(b) Which is bigger, f 0 (1.5) or f 0 (4)?
(c) Find the instantaneous rate of change of f when x = 0.5.
(d) Sketch the graph of the derivative of f .

12. Let f be the function whose graph is shown below. [Solution on page 178]

−4 −2 2 4x
−2

−4

(a) Which is bigger, f 0 (−0.5) or f 0 (2)?


28 Chapter 3. Limits and Derivatives

(b) Sketch the graph of the derivative of f .

13. Shown is the graph of a function f and its derivative f 0 . Decide which graph is that
of f and which is the graph of f 0 . Justify your answer. [Solution on page 178]

(b) 10 (a)

−2 2 4 6x

−5

14. Shown is the graph of a function f and its derivative f 0 . Decide which graph is that
of f and which is the graph of f 0 . Justify your answer. [Solution on page 179]

y
(a) 5 (b)

−4 −2 2 x

−5

−10

−15

15. Consider the graphs in (a)–(d). Match each graph with the graph of its derivative in
(i)–(iv). [Solution on page 179]

y y
(a) (b)

x x
3.8. Graphical Differentiation 29

y y
(c) (d)

x x

y y
(i) (ii)

x x

y y
(iii) (iv)

x x

16. Consider the graphs in (a)–(d). Match each graph with the graph of its derivative in
(i)–(iv). [Solution on page 180]

y y
(a) (b)

x x

y y
(c) (d)

x x
30 Chapter 3. Limits and Derivatives

y y
(i) (ii)

x x

y y
(iii) (iv)

x x

17. Let P (x) be the profit in thousands of dollars from selling x × 100 units of a certain
product. Shown is the graph of P . [Solution on page 180]

1 2 3 4 x

(a) Find the interval where the rate of change of profit (also called the marginal
profit) is positive.
(b) Find the rate of change of profit at the level of production of 3 × 100 (x = 3)
units.
(c) Sketch the graph of the marginal profit. What does the graph tell you?
Chapter4

Differentiation Rules

4.1 Basic Rules of Differentiation


1. In each case, find the derivative of f . [Solution on page 183]

(a) f (x) = 20 (i) f (x) = x3
(b) f (x) = 34 (j) f (x) = 2
x3
(c) f (x) = π 2
1
(k) f (x) = √
(d) f (x) = x6 3x

(e) f (t) = t100 .


p
(l) f (p) = 3
p5
(f ) f (x) = 3x−2 √
8 x
(g) f (x) = x3
2 (m) f (x) = 2
4 e2
(h) f (t) = 12 t 3 (n) f (q) = q

2. Find the derivative of each function. [Solution on page 186]


2x3 −3x2
(a) f (x) = x + 1 (g) f (x) = 4
5 2
(b) f (x) = 3x2 + 11x (h) f (x) = x 3 − x 3
1
(c) f (x) = x − 5 (i) f (x) = x2 − x
2z
(d) f (z) = z 2 − 5
(j) f (x) = 1.4x5 − 2.5x2 + 3.8

t− t (k) f (x) = √x
(e) f (t) = 2 x

x+x
(f ) f (x) = −3x4 − 2x3 + x2 − 1 (l) f (x) = x2

3. Find an equation of the tangent line to the curve y = 2x3 −x2 +2 at the point P (1, 3).
[Solution on page 189]
4. Find the points on the curve y = x3 − 3x + 1 where the tangent line is horizontal.
[Solution on page 190]
5. The cost (in dollars) of producing x units of a certain commodity is
C(x) = 3x2 + 75x + 400
[Solution on page 190]

31
32 Chapter 4. Differentiation Rules

(a) Find the marginal cost function C 0 (x).


(b) Find the marginal cost for the production level x = 4 units. Interpret your
answer.
(c) Find the marginal cost for the production level x = 35 units. Interpret your
answer.

6. [Solution on page 192]


Suppose the cost (in dollars) of producing q units of a certain commodity is given by

C(q) = 3500 − 15q + 0.02q 2

Also suppose that the demand function is given by p = −0.01q + 60 (where p is the
price per unit).

(a) Find the marginal revenue function R0 (q).


(b) Find the marginal profit for the following values of q: (i) q = 400 units (ii)
q = 935 units. Interpret your answers.
(c) Find the value of q for which the marginal profit is equal to 0.
(d) When the marginal is zero, what is the profit?

4.2 The Product and Quotient Rules


1. Find the derivative of each function. [Solution on page 193]

(a) f (x) = (5x + 7)(6x2 ) (d) f (x) = (x3 + 1)(2x2 − 4x − 1)


(b) f (x) = (3x + 4)(x − 5)

(c) f (x) = (5x2 − 2)(x3 + 3x) (e) f (x) = (2 + x)(x2 − 3x)

2. Find the derivative of each function. [Solution on page 195]



5x+1 x
(a) f (x) = 5x−1
(f ) f (x) = 3+x
1+2x 2x5 +x4 −6x
(b) f (x) = 3−4x (g) f (x) = x
x2 +1 x2 +4x+3
(c) f (x) = x3 −1
(h) f (x) = √
x
x
(d) f (x) = x3 +3x (i) f (x) = x+ x1
x2 −4x+3
1 (6x+1)(3x2 −4)
(e) f (x) = x3 +2x2 −1
(j) f (x) = 7x−2

x3 −x
3. Find an equation of the tangent line to the curve y = x2 +1
at the point P (1, 0).
[Solution on page 200]

4. Suppose the cost (in dollars) of producing x units of a certain product is given by
C(x) = 4000 + 3x. [Solution on page 201]

(a) Find the average cost for each production level. (i) 8 units, (ii) 20 units.
4.3. The Chain Rule 33

(b) Find the marginal average cost function.


(c) Find the rate of change of the average cost when x = 15 units.

5. Suppose the cost and revenue functions are given by

4x2 + 100
C(x) = and R(x) = 7x + 200.
3x + 2
[Solution on page 202]

(a) Find the marginal average cost function.


(b) Find the marginal average revenue function.
(c) Find the marginal average profit function.

4.3 The Chain Rule


1. Let f (x) = x2 − 3x and g(x) = 5x + 4. [Solution on page 203]

(a) Find

(i) g(f (0)) (iii) f (g(−2))


(ii) g(f (1)) (iv) f (g(3)).

(b) Find g(f (x)).


(c) Find f (g(x)).

2. In each case, write h(x) as the composition of two functions. (There may be more
than one way to do this.) [Solution on page 204]

(a) h(x) = (x2 + 3)5 (c) h(x) = − 3 10 + 7x
√ 1 3
(b) h(x) = 7x + 4 (d) h(x) = (x2 + x) 4 − 3(x2 + x) 4 + 11.

dy
3. In each case, find dx
. [Solution on page 205]

(a) y = (2x)5 (e) y = 3(5x − 1)7


(b) y = (−3x + 4)8 (f ) y = −2(5x6 − 2x)4
3 √
(c) y = (x2 − 1) 2 (g) y = x2 − x
1
(d) y = (−x3 + 2x + 1)15 (h) y = √
3 2
x −1

4. Suppose that the revenue (in dollars) from the sale of x items of a certain product is
given by p
R(x) = 16 3 (x2 + x)2 .
[Solution on page 208]

(a) Find the marginal revenue when (i) x = 100 items, (ii) x = 250 items.
34 Chapter 4. Differentiation Rules

(b) Find the average revenue from the sale of x items.


(c) Find the marginal average revenue.

5. [Solution on page 209]


The cost (in dollars) of producing q items of a particular product is given by

C(q) = 1500q + 4000

The quantity (q) and price per item (p) are related by the following equation:
p
q = 16000 − 2.5p

(a) Find the marginal profit in terms of q.


(b) What is the marginal profit when p = $6000?
dq
(c) Find dp . (This is the rate of change in demand for the product per unit change
in price.)
dy
6. In each case, find dx
. [Solution on page 210]
2
(a) y = (−5x + 4)(x3 + 1)6 (e) y = √x
√ x3 +1
(b) y = x 2 − x2 p x
(f ) y = x2 +3
2 3 2 6
(c) y = (x + 1) (x + 2)  5
(x+1)5 x4 +1
(d) y = x5 +1
(g) y = x2 +1


7. Find an equation of the tangent line to the curve y = x2 x3 − 2 at P (3, 45). [Solution
on page 213]
x
8. Find the points on the curve y = (x2 +28)4
where the tangent line is horizontal. [Solu-
tion on page 213]

4.4 Derivatives of Exponential Functions


1. In each case, find f 0 (x). [Solution on page 214]

(a) f (x) = 2ex (c) f (x) = x3 ex


√ x2
(b) f (x) = 1 + 2x − ex (d) f (x) = ex +3

2. Find the derivative of each function. [Solution on page 215]


2 +1

(a) y = e3x (f ) y = 7e−x (k) y = ex 2x+1

(b) y = e−5x (g) y = e2x + e−2x xex


2

2 +5x+1 3 (l) y = x+e−x


(c) y = ex (h) y = xex
(d) y = −e−x (i) y = x2 e−4x (m) y = (1 + xe4x )5
e−2x 2 √
(e) y = −4e−1.5x (j) y = x2 +5
(n) y = ex +1 2x + 3
4.5. Derivatives of Logarithmic Functions 35

3. Find the derivative of each function. [Solution on page 219]


2 √
(a) y = 5x (b) y = 3x (c) y = 2 · 5 x

2 −6x
4. Find the points on the curve y = ex where the tangent line is horizontal. [Solution
on page 221]

5. The cost in dollars to produce x items of a certain commodity can be approximated


by [Solution on page 221]

C(x) = 600 − 500 · 1.2−x .

Find the marginal cost when (i) x = 0; (ii) x = 15.

4.5 Derivatives of Logarithmic Functions


1. In each case, find f 0 (x). [Solution on page 222]

(a) f (x) = 5 ln x (c) f (x) = x2 ln x


x
(b) f (x) = x3 − ln x (d) f (x) = 1+ln x

2. Find the derivative of each function. [Solution on page 223]

(a) y = ln(3x) (h) y = x3 ln(4x2 − 6x + 3)


(b) y = ln(−5x + 4) 2
(i) y = ex ln(3x)
(c) y = ln(x2 + 9) √ 
(j) y = ln 6x + 7
(d) y = ln(x − ex ) √
(k) y = ln(xe x + 8)
(e) y = 4 ln(−2x) √
(f ) y = ln(4x) + ln(−4x) (l) y = e−x + ln 2x
xe−x
(g) y = x ln(x2 ) (m) y = 1+ln(5x)

3. Find the derivative of each function. [Solution on page 227]

(a) y = log3 x (c) y = log2 (−2x3 + x2 )


5
(b) y = 7 log8 x (d) y = log(x2 − x) 2

4. Find the derivative of each function. [Solution on page 229]


p
(a) y = ln |x| (c) y = ln |6 − x|
(b) y = ln | ln x| (d) y = x2 log5 |x3 + 1|

5. Find the derivative of each function. [Solution on page 230]


36 Chapter 4. Differentiation Rules

2 +7
(a) y = x2x (c) y = |1 + e2x |x
(b) y = (x2 + 1)3x

6. Find the points on the curve y = xx where the tangent line is horizontal. [Solution
on page 232]

7. The cost in dollars to produce x items of a certain product can be approximated by

C(x) = 3 log2 x + 10

Find the marginal average cost function. [Solution on page 232]

4.6 Derivatives of Trigonometric Functions


1. Find the derivative of each function. [Solution on page 233]
cos x
(a) f (x) = −5 cos x (j) f (x) = x sin x + x

sin x 9
(b) f (x) = 4
− x3 (k) f (x) = x9 + x
+ tan x
2
4
(c) f (x) = π + 3 tan x (l) f (x) = x2 cos x − 2 tan x + 3
(d) f (x) = 3 sin x − 8 cos x + 2 tan x (m) f (x) = x cos x + x2 sin x
(e) f (x) = 7 sec x − csc x + cot x sin x
(n) f (x) = 1+tan x
1 csc x √
(f ) f (x) = x cot x (o) f (x) = cos x
+ 3
− 7
x
(g) f (x) = sin x cos x (p) f (x) = x cos x sin x
sin x
(h) f (x) = x (q) f (x) = 5 sin4 x
sec x
(i) f (x) = csc x
(r) f (x) = x(1 + tan x)8

2. Find the derivative of each function. [Solution on page 238]

(a) y = sin(2x) (i) y = x sin x1


(b) y = cos(−3x + 1) (j) y = sec4 (x3 + 1)
(c) y = tan(x3 − x2 ) (k) y = sin (3e5x )
2
(d) y = x sin(x + 1)
2
(l) y = ln | cos2 x|
(e) y = 3x cos(πx − 1)
(m) y = 3cot(−x)
(f ) y = sin(cos x)
(g) y = cos(sec 4x) (n) y = ecsc(log x)
√ x5 csc(3x)
(h) y = sin 1 + x2 (o) y = 1+4 cot x2

3. Find an equation of the tangent line to the curve y = cos(3x) at the point π9 , 12 .


[Solution on page 243]

4. Find an equation of the tangent line to the curve y = sin(sin x) at (π, 0). [Solution
on page 244]
4.7. Exponential Growth and Decay 37

5. Find the second derivative of each function. [Solution on page 244]

(a) f (x) = x cos(4x)


(b) f (x) = sin6 (x)

6. [Solution on page 245]


The revenue in dollars from selling a certain brand of snow tires at a time t (in
months) is given by π 
R(t) = 250 sin t + 0.1 + 700,
8
where t = 0 corresponds to September.

(a) Find the derivative of R with respect to t.


(b) Find the instantaneous rate of change of the revenue when t = 3.
(c) Determine R0 (4) and interpret your answer.

4.7 Exponential Growth and Decay


1. Let y be a quantity that depends on time t. We say that y grows or decays exponen-
tially if it is of the form
y(t) = y0 ekt
where y0 = y(0) is the initial quantity and k is a constant. If k > 0, we say that y
grows exponentially and in this case k is called the growth constant. If k < 0, then y
decays exponentially and in this case k is called the decay constant. [Solution on
page 246]

(a) Show that if y is of the form y = y0 ekt , then dy


dt
= ky. (This means that if y
grows or decays exponentially, then the rate of change of y is proportional to y.)
(b) In each case, find the initial quantity and the growth or decay constant.

i. y = 10e7t iii. y = 110e−0.04t


ii. y = 53e0.3t iv. y = 284e2−5t

2. One can prove that the converse to the statement “if y grows or decays exponentially,
then the rate of change of y is proportional to y” is true. Specifically, one can prove
that if the rate of change of a quantity y = y(t) is proportional to y, say dy dt
= ky,
kt
then y has the form y = y0 e , where y0 is the quantity present at time t = 0. Use
this to find an expression for y in each case. And state whether y grows or decays
exponentially. [Solution on page 246]

(a) The initial quantity is 17 and the rate of change of y is 4y.


(b) The quantity present at time t = 0 is 300 and the rate of change of y is −0.05y.

3. In each case, find an expression for y = y(t), assuming exponential growth or decay.
[Solution on page 246]
38 Chapter 4. Differentiation Rules

(a) y(0) = 75, y(2) = 105 (c) y(2) = 265.46, y(7) = 308.42
(b) y(0) = 10000, y(5) = 7046.88 (d) y(3) = 1182.80, y(7) = 558.72

4. A bacteria culture initially contains 2000 bacteria and grows exponentially. After 4
hours the culture contains 3502 bacteria. Let y be the number of bacteria after t
hours. [Solution on page 248]

(a) Find an expression for y.


(b) Find the number of bacteria after 7 hours.
(c) Find the rate of growth after 7 hours.
(d) How long will it be until there are 10000 bacteria?

5. A culture initially contains 33000 bacteria and grows at a rate proportional to its
size. After 7 hours the culture contains 54000 bacteria. [Solution on page 249]

(a) Determine a function that expresses the number of bacteria in terms of t.


(b) Find the number of bacteria after 8 hours.
(c) Find the rate of growth after 8 hours.
(d) How long will it take for a culture of 33000 bacteria to double?

6. A radioactive sample decays exponentially. Initially, the sample weighs 10 g. After 6


years, the sample weighs 7 g. [Solution on page 250]

(a) Find a formula for the mass remaining after t years.


(b) Find the mass of the sample after 13 years.
(c) Find the rate of change of the mass of the sample after 13 years.
(d) In how many years will half of the weight of the sample remain?

7. Suppose a radioactive substance decays according to the law

m(t) = m0 e−0.02476t

where m0 is the initial quantity and t is measured in days. What is the half-life of
this substance? [Solution on page 251]

8. The half-life of Polonium-210 is 140 days. [Solution on page 252]

(a) A sample has a mass of 35 mg initially. Find the mass remaining after 200 days.
(b) How many days does it take a sample weighting 35 mg to decay to a mass of 6
mg?

9. The world population in 2000 was approximately 6.143 billion. Assume that the pop-
ulation grows at the rate of approximately 1.31% per year. Suppose t = 0 corresponds
to 2000. [Solution on page 252]

(a) Find a formula for the world population after t years.


4.7. Exponential Growth and Decay 39

(b) Estimate the population in 2021. How about 2040?


(c) Find the rate of change in 2025.
(d) If the population continues to grow at the rate of approximately 1.31% per year,
find the time required for the world population to double in size.

10. Suppose you invest P dollars. After t years your amount on deposit is A dollars. In
each case, find the interest rate if the money is compounded continuously. [Solution
on page 254]

(a) P = 4000, t = 7, A = 9000. (b) P = 11000, t = 5, A = 15700.

11. Suppose you invest $25500 in an account paying interest compounded continuously.
Also suppose that after 1 year your amount on deposit is $27076.83. Let A = A(t)
be the amount on deposit after t years. [Solution on page 254]

(a) Find an expression for A(t).


(b) Find the compound amount after 5 years.
(c) What is the rate of growth of A after 5 years?

12. Alex deposits $7500 in an account paying interest compounded continuously. After 2
years the amount on deposit is $8247.44. Find the amount on deposit after 8 years.
[Solution on page 255]

13. Find the present value of A = $14000 in each case. [Solution on page 255]

(a) Interest is 5% compounded semiannually for 9 years.


(b) Interest is 3% compounded quarterly for 6 years.

14. Find the present value of A = $78500 if interest is 2.78% compounded continuously
for 4 years. [Solution on page 256]

15. Suppose you have to make a car payment of $40000 in 5 years. What is the present
value of the payment if it includes annual interest of 3.9% compounded continuously.
[Solution on page 257]
40 Chapter 4. Differentiation Rules
Chapter5

Applications of the Derivative I

5.1 Increasing and Decreasing Functions


1. Find the critical numbers of each of the following functions. [Solution on page 259]

1
(a) f (x) = −2x2 + 8x − 3 (f ) f (x) = x
(b) f (x) = x3 + x2 + 5 2x+1
(g) f (x) = x−3
4 3 2
(c) f (x) = 3x + 8x − 18x
√ (h) f (x) = x ln(3x)
(d) f (x) = 3 x

(e) f (x) = 1 − x2 (i) f (x) = x2 · 3−x

2. Find the intervals where each function is increasing and where it is decreasing. [So-
lution on page 262]

2x+1
(a) f (x) = −2x2 + 8x − 3 (e) f (x) = x−3
x
(b) f (x) = x3 + x2 + 5 (f ) f (x) = x2 +1
3
(c) f (x) = 3x4 + 8x3 − 18x2 (g) f (x) = xe−x
 2 
2
(d) f (x) = (x − 1) 3 (h) f (x) = ln 7x +6
x2 +3

3. [Solution on page 268]


The cost of producing x items of a certain commodity is

C(x) = 0.28x2 − 0.00006x3 , 0 ≤ x ≤ 4000.

And the revenue function is

R(x) = 0.936x2 − 0.0004x3 , 0 ≤ x ≤ 4000.

(a) Find the interval where the profit is increasing.


(b) Determine the interval where the profit is decreasing.

41
42 Chapter 5. Applications of the Derivative I

y
4

−2 −1 1 2 3 4 x
−2

−4

Figure 5.1

5.2 Relative (or Local) Extrema


1. Let f be the function whose graph is shown in Figure 5.1. Find the relative extrema
of f . [Solution on page 269]

2. In each case find the relative extrema of f . [Solution on page 270]


x
(a) f (x) = −2x2 + 8x − 3 (f ) f (x) = x2 +1

(b) f (x) = x3 + x2 + 5 (g) f (x) = 4x ln(5x)


3
(c) f (x) = 3x4 + 8x3 − 18x2 (h) f (x) = xe−x
(d) f (x) = (x − 1) 3
2
(i) f (x) = (e2x − 3)5
 2 
(e) f (x) = 2x+1
x−3
(j) f (x) = ln 7x +6
x2 +3

3. [Solution on page 275]


Suppose the cost and demand functions are given by

C(q) = 8q + 100 and p = −3q + 80

(a) Find the number of units that maximizes the profit.


(b) Determine the price corresponding to the maximum profit.
(c) What is the maximum profit?

4. [Solution on page 276]


Repeat Exercise 3 with the following functions.

C(q) = 120 + 50qe−0.02q and p = 100e−0.02q

5. [Solution on page 277]


The cost of producing x items of a certain commodity is

C(x) = 0.28x2 − 0.00006x3 , 0 ≤ x ≤ 4000.


5.3. Concavity 43

And the revenue function is

R(x) = 0.936x2 − 0.0004x3 , 0 ≤ x ≤ 4000.

(a) Determine the number of units to sell to maximize profit.


(b) Find the maximum profit.

5.3 Concavity
1. Let f be the function whose graph is shown in Figure 5.2. Find the intervals where
the function is concave upward or concave downward. Find any inflection points.
[Solution on page 278]

y
4

−2 −1 1 2 3 4 x
−2

−4

Figure 5.2

2. Find the second derivative of f . [Solution on page 278]

(a) f (x) = 3x4 − 2x3 + 7x − 11 (d) f (x) = ln(x2 + 1)


(b) f (x) = −5x + 6 (e) f (x) = x2 e−3x
3
(c) f (x) = e−x (f ) f (x) = log(x + e−x )

3. Find the third and fourth derivative of each function. [Solution on page 280]

(a) f (x) = −x4 + 2x3 − 5x + 6.


(b) f (x) = e2x .

4. In each case, find the intervals where f (x) is concave upward or concave downward.
Find any inflection points. [Solution on page 281]
x
(a) f (x) = x2 − 6x + 3 (e) f (x) = x2 +1
−4x
(b) f (x) = x3 − 3x2 + 5 (f ) f (x) = xe
4 3
(c) f (x) = x − 4x (g) f (x) = ln(x2 + 4)
−3
(d) f (x) = x+1
(h) f (x) = (x − 1)4
44 Chapter 5. Applications of the Derivative I

5. In each case find the critical numbers of f (x). Then use the second derivative test to
decide whether the critical numbers lead to local extrema. If f 00 (c) = 0 or f 00 (c) does
not exist for a critical number c; then the second derivative test gives no information.
In this case, use an alternative method instead. [Solution on page 287]

(a) f (x) = 1 − 8x − x2
(b) f (x) = −x3 + 6x2 + 7
(c) f (x) = (x + 1)4

6. In each case find the Point of Diminishing Returns for the revenue function R(x).
[Solution on page 288]

(a) R(x) = −x3 + 12x2 + 100, 0 ≤ x ≤ 10


(b) R(x) = xe−x , 0 ≤ x ≤ 15

5.4 Curve Sketching


In each case, determine the following features of f : domain, intercepts, asymptotes (show
the relevant limits), intervals where f is increasing or decreasing, local extrema, intervals
where f is concave upward or concave downward, inflection points. Use all those features
to sketch the graph of f .

1. f (x) = −x3 + 3x2 − 1 [Solution on page 289]

2. f (x) = x4 − 4x3 + 21 [Solution on page 292]


−2x2
3. f (x) = [Solution on page 294]
x2 − 1
6
4. f (x) = [Solution on page 297]
x2 −x−6
Chapter6

Applications of the Derivative II

6.1 Absolute Extrema


1. Let f be the function whose graph is shown in Figure 6.1. Find the absolute extrema
of f . [Solution on page 303]

−2 −1 1 2 3 4 x

−2

Figure 6.1

2. In each case, find the absolute extrema, as well as all values of x where they occur.
[Solution on page 304]

(a) f (x) = 12 + 4x − x2 on [0, 5] (e) f (x) = (x2 − 4)3 on [−2, 3]


(b) f (x) = −x3 + 3x2 + 1 on [−1, 2] x
(f ) f (x) = x2 −x+1
on [0, 3].
(c) f (x) = 2x3 − 3x2 − 12x + 1 on [−2, 1]
2 8 2
(d) f (x) = xx2 −4
+4
on [−4, 4] (g) f (x) = 3x 3 − 108x 3 on [−2, 8].

3. Suppose the cost (in dollars) of producing x units of a certain product is given by

C(x) = x3 − 3x + 100, 0 ≤ x ≤ 20.

[Solution on page 309]

(a) Find the value of x that minimizes the cost.


(b) Find the minimum cost.

45
46 Chapter 6. Applications of the Derivative II

6.2 Optimization Problems


1. [Solution on page 310]
A firm has found that its revenue (in dollars) from selling x items of a commodity
can be modelled by
R(x) = −x3 + 450x2 + 52500x
What production level x will give a maximum revenue?

2. [Solution on page 310]


A rancher has 2000 ft of fencing and wants to enclose a rectangular field. No fence is
needed along one side. What dimensions of the field should be used to maximize the
area?

3. [Solution on page 312]


Jack wants to fence a rectangular garden of 1536 m2 , and no fence is needed along one
side. Suppose his landlord will pay half the cost of one of the sides perpendicular to
the side with no fence. Suppose further that Jack buys the fencing at $27/m. What
dimensions of the garden will minimize the cost?

4. [Solution on page 313]


Let h and r be the height and radius of a cylinder of volume 2500 cm3 . What are the
values of h and r that minimize the total surface area of the cylinder?

5. [Solution on page 314]


Suppose you want to make a cardboard box with a square base and open top. What is
the maximum possible volume of the box you can make with 1875 cm2 of cardboard?

6. [Solution on page 315]


Sylvie is building a closed box with a square base. The material for the bottom and
the sides costs 6 cents/cm2 , while the material for the top costs 12 cents/cm2 . Let x
be the length of the base, and let h be the height.

(a) Find a formula for the cost of this box in terms of x and h.
(b) Express the volume of the box in terms of x and h.
(c) If the volume must be 768 cm3 , find h in terms of x.
(d) If the volume of the box must be 768 cm3 , what value of x should be used for
the cheapest box?

6.3 Implicit Differentiation and Related Rates


dy
1. In each case, find dx
. [Solution on page 316]
6.3. Implicit Differentiation and Related Rates 47

(a) x2 + y 2 = 25 (h) −5x2 + xy − y 3 = 1


(b) x3 + y 3 = 1 (i) x3 y 2 + y 4 = x
x+y
(c) 2x2 − y 2 = x (j) x−y
=1
(d) x4 + 3y 3 = 5y (k) ey = xy

(e) x − y = y 2 + 3 (l) yex − y = x
3
(f ) xy = 5 (m) exy = 2x + 3y + 1
(g) 3x2 + 2xy + y 2 = 2 (n) x + ln y = x3 y 3

2. Use implicit differentiation to find an equation of the tangent line to the curve at the
given point. [Solution on page 323]
(a) x2 − xy − y 2 = 1 at (2, 1)
(b) (x2 + y 2 )2 = 2(x3 + y 2 ) at (1, 1)
d2 y
 dy 
3. If x2 −y 2 = 4, find the second derivative y 00 = dx2
= d
dx dx
by implicit differentiation.
[Solution on page 324]
4. [Solution on page 325]
Suppose the cost function C is given as follows.

C 2 − 40 = x2 + 80 x
dC
Find the marginal cost dx
at x = 4, and interpret your answer.
5. [Solution on page 326]
Suppose the cost and revenue functions are given by
R = 80x − 0.25x2 and C = 7x + 20
If the rate of change of production is 5 units per hour, find the rate of change of each
quantity when x = 100 units.

(a) Revenue (b) Cost (c) Profit

6. [Solution on page 327]


A firm has found that the cost (in dollars) of producing x units per week of a certain
product can be modeled by C = 0.3x2 + 7000. If the production is increasing at the
rate of 10 units per week, find the rate at which the cost is changing when x = 95
units.
7. [Solution on page 327]
Suppose the cost C and revenue R (in dollars) are related by the equation
R2
C= + 8000
300000
If the rate of change of the cost is $10 per week, how fast is the revenue changing
when the weekly revenue is $20000?
48 Chapter 6. Applications of the Derivative II

8. [Solution on page 328]


Suppose the quantity q and unit price p are related by the equation
1
3p − q 2 = 62
3

9. [Solution on page 328]


dS
Suppose the surface S of a balloon with radius r changes with time t. Express dt
in
terms of dr
dt
.

10. [Solution on page 328]


Suppose the radius of a sphere is increasing at a rate of 0.5 cm/s. How fast is the
volume increasing when the diameter is 6 cm?

11. [Solution on page 329]


Suppose you are pumping air into a spherical balloon so that its volume increases at
a rate of 180 cm3 /s. How fast is the diameter of the balloon growing when the radius
is R = 75 cm?

12. [Solution on page 330]


3
Oil is added to a cylindrical tank at a rate of 4 m /min. If the radius of the tank is
7 m, find the rate of change of the height of the oil.

13. [Solution on page 330]


A drone is flying horizontally at 90 mi/h, 0.06 mi above the ground when it passes
over a person on the ground. How fast is the distance between the drone and the
person increasing when the drone is 0.1 mi away from the person?

6.4 Elasticity of Demand


1. In each case, calculate and interpret the elasticity of demand for the given price p.
[Solution on page 331]

(a) q = −0.05p + 8.5 when p = $102.


(b) q = 10000 − 30p2 when p = $10.
(c) q = 100e−0.1p when p = $10.

2. In each case, calculate and interpret the elasticity of demand. [Solution on page 332]
2
(a) q = p

(b) q = e5−0.3 ln p
(c) q = 15000p−1.68

3. In each case, calculate and interpret the elasticity of demand for the given demand
q. [Solution on page 334]
6.5. Differentials and Linear Approximations 49

(a) p = 0.604q 2 − 20.16q + 263.067 when q = 11.


−0.6q
(b) p = 900e when q = 8.

4. [Solution on page 335]


Suppose the demand q and price p (in dollars) for a commodity are related by the
equation q = 108 − p2 .

(a) Express the elasticity of demand E in terms of p.


(b) What interval should the price be for demand to be (i) elastic and (ii) inelastic?
(c) What price yields the maximum revenue? Find the maximum revenue.

5. [Solution on page 337]


Repeat Exercise 4, but with q = 16 − 4 ln p.

6.5 Differentials and Linear Approximations


1. Find the differential of each function. [Solution on page 338]

(a) y = x2 (c) y = 4x3 − 5x2 + x − 8


5 1
(b) y = x3 + 1 (d) y = 3x 4 + x 4

2. Find the differential of each function y and evaluate it for the given values of x and
dx = ∆x. [Solution on page 340]

(a) y = x + 3, x = 1, dx = 0.1 (c) y = 15x+6
−x+2
, x = −2, dx = 0.03
√ x
(b) y = 4x2 + 13, x = 3, dx = 0.04 (d) y = e 8 , x = 0, dx = −0.02

3. In each case, use the differential to approximate the change in y as x changes from
x1 to x2 . [Solution on page 341]
ln(x−2)
(a) y = −x3 − x2 + 150, x1 = 1, x2 = 2 (c) y = x+1
, x1 = 3, x2 = 2.9
−2x
(b) y = xe , x1 = 0, x2 = 0.2

4. Find the linear approximation of each function at a. [Solution on page 344]

(a) f (x) = x2 , a = 4 (c) f (x) = 73x+1 , a = − 31



(b) f (x) = x, a = 25 (d) f (x) = log5 (6x + 7), a = 3

5. Use a linear approximation (or differentials) to approximate each number. [Solution


on page 345]
√ √
(a) 5 (c) 3 28 (e) ln(0.99)

(b) 47 (d) e0.02 (f ) (−2.001)5
50 Chapter 6. Applications of the Derivative II

6. The cost (in dollars) of producing x items of a certain product is given by

C(x) = x3 − 2x2 + 4x + 2500

Use the differential to approximate the change in cost in each case. [Solution on
page 348]

(a) x changes from 8 to 9 (b) x changes from 29 to 30

7. Suppose the revenue and cost functions are given by

R(x) = 25000 ln(0.03x + 2) and C(x) = 478 + 89x

[Solution on page 349]

(a) Use the differential to approximate the change in profit if x increases from 500
to 501.
(b) At the level of production of 500 units, find the marginal profit. And compare
your answer to the one found in part (a).

8. [Solution on page 350]


The demand function for a certain product is given by
4000
p = D(q) =
0.05q 2 + 3

where q is the quantity demanded (in hundreds of units) and p is the price in dollars.
Use differentials to estimate the change in price when q changes from 300 to 370 units.

9. [Solution on page 350]


Suppose you are pumping air into a spherical balloon. Use differentials to find the
approximate change in volume if the diameter increases from 10 cm to 10.3 cm.

10. [Solution on page 351]


The side of a square field is x = 225 ft. If x has been measured with a possible error
of ±0.005 ft, use differentials to estimate the error that might occur when the area
of the field is calculated.

6.6 Newton’s Method


1. Approximate a solution, to the nearest hundredth, for the equation [Solution on
page 351]
x2 + x − 1 = 0
in the interval [0, 1].

2. In each case, approximate a solution, to the nearest hundredth, for the equation in
the given intervals. [Solution on page 354]
6.6. Newton’s Method 51

(a) 3x2 − 8x − 2 = 0; [2, 3]


3 2
(b) x + x − 5 = 0; [1, 2]
(c) −2x3 + 6x2 − 2x + 13 = 0; [3, 4]
(d) x4 + 3x3 − 5x2 − 9 = 0; [−5, −4], [1, 2]
1
(e) 3x − 2x4 + 7 = 0;
5 [1, 3]
(f ) x3 e−2x + x2 + 1 = 0; [−1, 1]
(g) 3 ln x + x − 5 = 0; [2, 5]

3. In each case, use Newton’s method to approximate the given number correct to four
decimal places. [Solution on page 356]
√ √
(a) 5 (c) 3 50
√ √
(b) 23 (d) 7 139

4. Explain why Newton’s method doesn’t work for the equation x2 + 3x + 4 = 0 in the
interval [0, 1]. [Solution on page 358]

5. Consider the equation 1 + ln(x − 2) = 0. [Solution on page 358]

(a) Find the exact solution. Then find an approximate solution (round your answer
to two decimal places).
(b) If the initial guess is c1 = 2.1, use Newton’s method to approximate the solution
to the nearest hundredth. Compare your answer to the one found in part (a).
(c) Explain why Newton’s method does not work if the initial guess is c1 = 3.

6. Let f (x) = x4 + x3 − 2x + 1. [Solution on page 359]

(a) Use Newton’s method to approximate the critical number for f (x) to the nearest
hundredth.
(b) Decide whether the critical number found in part (a) leads to a relative minimum
or a relative maximum.

7. Repeat Exercise 6, but with f (x) = (1 − x)ex + x. [Solution on page 360]

8. Suppose the revenue and cost functions are given by


1
R(x) = 15(x + 12) 3 and C(x) = 3x + 29

Approximate the break-even quantity to the nearest hundredth. [Solution on


page 362]

9. [Solution on page 363]


Suppose the savings and costs in dollars of a firm are given by

S(t) = t2 + 55t + 30 and C(t) = t3 + 7t2 + 11

where t is time in years.


52 Chapter 6. Applications of the Derivative II

(a) Sketch the graphs of S and C on the same axes and show that the equation
S(t) = C(t) has only one solution t ≥ 0.
(b) Find the number of years, to the nearest hundredth, that the firm will realize
savings.

10. Suppose the total profit (in thousands of dollars) from selling x kilograms of a certain
product is given by
1
P (x) = − x5 + 2x3 + 3x2 + 5x − 4
7
It turns out that the function P has only one critical number c > 0. [Solution on
page 364]

(a) Find the number of kilograms, to the nearest hundredth, that should be sold in
order to maximize the total profit.
(b) What is the maximum profit?
PartII

Solution to Exercises

53
Chapter1

Linear Functions

1.1 Slopes and Equations of Lines


1. [Exercise on page 3]

(a) Solution.

Vertical Lines
Given two points P1 (x1 , y1 ) and P2 (x2 , y2 ),

ˆ if x1 6= x2 the line through P1 and P2 is called nonvertical.

ˆ If x1 = x2 , the line through P1 and P2 is called vertical.

Slope of a Line

ˆ The slope of a nonvertical line passing through the points P1 (x1 , y1 )


and P2 (x2 , y2 ) is given by the formula
y2 − y1
m= . (1.1.1)
x2 − x 1
The difference y2 − y1 is called change in y, and is denoted ∆y. The
difference x2 − x1 is called change in x, and is denoted ∆x. So the
formula for the slope can rewritten as
change in y ∆y
m= = .
change in x ∆x

ˆ The slope of a vertical line is undefined.

We now want to find the slope of the line through the points (1, 2) and (4, 11).
Let P1 = (1, 2) and P2 = (4, 11). We have x1 = 1 and y1 = 2. We also have
x2 = 4 and y2 = 11. Using the formula (1.1.1), we get
y2 − y1 11 − 2 9
m= = = = 3.
x2 − x1 4−1 3

55
56 Chapter 1. Linear Functions

So the slope of the line through (1, 2) and (4, 11) is m = 3.


Note. If one uses P1 = (4, 11) and P2 = (1, 2) instead, one will end up with
the same answer. Indeed, let P1 = (4, 11) and P2 = (1, 2). Then x1 = 4, y1 =
11, x2 = 1, y2 = 2, and m = 2−11
1−4
= −9
−3
= 3.
(b) Solution. Here x1 = 0, y1 = 1, x2 = 2, and y2 = 9. Using the formula (1.1.1) of
slope, we get
9−1 8
m= = = 4.
2−0 2
(c) Solution. The slope is
4−6 −2
m= = = −2.
6−5 1

(d) Solution. The slope is


19 − 13 6
m= = = −2.
7 − 10 −3

(e) Solution. The slope is

6 − (−3) 6+3 9
m= = = .
0 − (−2) 0+2 2

(f ) Solution. The slope is

−2 − (−14) −2 + 14 12 3
m= = = = .
−7 − (−15) −7 + 15 8 2

(g) Solution.

Fractions
We recall the sum, difference, multiplication, and quotient of two fractions.
Let ab and dc be fractions.

ˆ Sum: a
b
+ c
d
= ad+bc
bd
.

ˆ Subtraction: a
b
− c
d
= ad−bc
bd
.

ˆ Multiplication: a
b
× c
d
= ac
bd
.
a
ˆ Quotient: b
c = a
b
× d
c
= ad
bc
.
d

For the sum and difference one could also use the method of common
denominators that we won’t recall.

The slope of the line through the points (1, 1) and ( 13 , 34 ) is


3 3 1 3−4 −1
−1 −
  
4 4 1 4 4 −1 3 −3 3
m= 1 = 1 1 = 1−3 = −2 = = = .
3
−1 3
− 1 3 3
4 −2 −8 8
1.1. Slopes and Equations of Lines 57

(h) Solution. The slope is


4 4 4
−0
  
5 5 5 4 8 32 16
m= −3 = −6−4 = −10 = = =− .
4
− 12 8 8
5 −10 −50 25

2. [Exercise on page 3]
Solution.

ˆ The slope of the line through (−1, 1) and (2, 4x) is m = 4x−1
2−(−1)
= 4x−1
2+1
= 4x−1
3
.
ˆ Since the slope is 2, we must have 4x−1
3
= 2. Multiplying both sides of the latter
equation by 3, we get 4x − 1 = 6. Adding 1 to both sides, we get 4x = 7.
Dividing both sides by 4, we get x = 74 .

3. [Exercise on page 3]

(a) Solution.

Point-Slope Form

An equation of the line through the point (x1 , y1 ) with slope m is

Point-Slope Form: y − y1 = m(x − x1 ).

Slope-Intercept Form
ˆ The y-intercept of a line is the point of intersection between the
line and the y-axis. This is obtained by substituting x by 0 into
the equation of the line. For example, the y-intercept of the line
y = 2x + 3 is 2(0) + 3 = 0 + 3 = 3.

ˆ An equation of the line with slope m and y-intercept b is

Slope-Intercept Form: y = mx + b.

This is the most common form for writing the equation of a line. We
will always use that form.

We now want to find an equation of the line through (2, 3) with slope m = 4.
Let x1 = 2 and y1 = 3. Using the point-slope form, we get the equation y − 3 =
4(x − 2). Distributing the right hand side, we get y − 3 = 4x − 8. Adding 3 to
both sides, we get y = 4x − 5. So the equation of the line through (2, 3) with
slope m = 4 is y = 4x − 5.

Alternate Solution (using directly the slope-intercept form).

We have y = mx + b and we need to find m and b.

ˆ From the problem, m = 4. So y = 4x + b. It remains to find b.


58 Chapter 1. Linear Functions

ˆ Since the line passes through (2, 3), we have 3 = 4(2) + b, that is, 3
= 8 +b. Subtracting 8 from both sides, we get −5 = b. So b = −5.
Therefore, the required equation is y = 4x − 5

(b) Solution. The required equation is


1
y − 1 = (x − 0)
3
1
y−1= x
3
1
y = x+1
3
4. [Exercise on page 3]

(a) Solution. Two steps.


ˆ First, we need to find the slope:
5−2 3
m= = = 1.
4−1 3
ˆ Using the point (1, 2), the point-slope form is y − 2 = 1(x − 1). Solving this
for y, we get y = x + 1 which is the required equation.
Note. Here we used the point (1, 2). By using (4, 5) instead, we will end up
with the same answer. Indeed, the point-slope form would be y − 5 = 1(x − 4)
or y − 5 = x − 4. Adding 5 to both sides, we get y = x + 1. No matter which
point we use, we will obtain the same answer.
(b) Solution.
ˆ The slope is m = 1−(−7)
3−2
= 1+7
3−2
= 81 = 8.
ˆ Using the point (2, −7), the point-slope form is y − (−7) = 8(x − 2), that
is, y + 7 = 8x − 16. Subtracting 7 from both sides, we get y = 8x − 23.
(c) Solution.
ˆ The slope is m = −1−0
6−2 4
= −1 = −4.
ˆ Using the point (0, 2), the point-slope form is y − 2 = −4(x − 0), that is,
y − 2 = −4x. Adding 2 to both sides, we get y = −4x + 2.
(d) Solution.
ˆ The slope is m = 6−2
−3−(−1)
= 4
−3+1
= 4
−2
= −2.
ˆ Using the point (−1, 2), the point-slope form is y − 2 = −2(x − (−1)), that
is, y − 2 = −2(x + 1) or y − 2 = −2x − 2. Adding 2 to both sides, we get
y = −2x.
(e) Solution.
ˆ The slope is m = 4−3
1−3
= 1
−2
= − 12 .
ˆ Using the point (3, 3), we get y − 3 = − 12 (x − 3), that is, y − 3 = − 12 x + 32 .
Solving this for y, we get y = − 12 x + 92 .
1.1. Slopes and Equations of Lines 59

(f ) Solution.
ˆ The slope is m = 10−1
5
− 21
= 94 = 92 .
2 2

ˆ Using the point ( 21 , 1), we


get y − 1 = 29 (x − 12 ), that is, y − 1 = 92 x − 94 .
Adding 1 to both sides, we get y = 92 x − 54 .
(g) Solution.
ˆ The slope is
1
+ 12 3   
4 4 3 3 9
m= 2 = −4 = =− .
3
−2 3
4 −4 16
ˆ Using the point (2, − 12 ), the point-slope form is y + 12 = − 16
9
(x − 2), that is,
9
1 9 9 1
y + 2 = − 16 x + 8 . Subtracting 2 from both sides, we get y = − 16 x + 58 .

5. [Exercise on page 3]

(a) Solution.

Graph of a Vertical Line


Let k be a number. The line with equation x = k is the vertical line that
intersects the x-axis precisely at k.

−2 −1 1 2 3 x
−1

−2

Figure 1.1: Graph of x = 2

(b) Solution.

Graph of a Horizontal Line


Let b be a number. The line with equation y = b is the horizontal line
that intersects the y-axis precisely at b.

The graph of y = 1 is shown in Figure 1.2.


(c) Solution. To sketch the graph of a line y = mx + b, we need to find at least
two points that lie on the line. To get one point, choose a value for x. Then
substitute that value into the equation to get the corresponding value for y. For
the equation y = 2x − 1, we have the following.
60 Chapter 1. Linear Functions

−2 −1 1 2 x

−1

Figure 1.2: Graph of y = 1

ˆ If x = 0, then y = 2(0) − 1 = −1. So the line passes through the point


(0, −1).
ˆ If x = 1, then y = 2(1) − 1 = 2 − 1 = 1. So the line passes through the
point (1, 1).
Thus, the graph of y = 2x − 1 is the line through the points (0, −1) and (1, 1)
as shown in Figure 1.3.

1 (1, 1)

−2 −1 1 2 3 x
−1 (0, −1)

−2

Figure 1.3: Graph of y = 2x − 1

(d) Solution. We need to find two points on the line.


ˆ If x = 0, then y = −(0) + 4 = 4. So the line passes through (0, 4).
ˆ If x = 1, then y = −1 + 4 = 3. So the line passes through (1, 3).
Thus the line y = −x + 4 passes through the points (0, 4) and (1, 3) as shown in
Figure 1.4.

1.2 Linear Functions and Applications


1. [Exercise on page 3]
1.2. Linear Functions and Applications 61

4 (0, 4)

3 (1, 3)

2
1

1 2 3 4 5 x

Figure 1.4: Graph of y = −x + 4

(a) Solution.
The sentence “The sales of a small company were $17,000 in its fourth year”
means that the sales line passes through the point (4, 17000). And the sentence
“$25,000 in its sixth year” means that the sales line passes through (6, 25000).
Thus, we are looking for the line that passes through the points (4, 17000) and
(6, 25000).
ˆ The slope is
25000 − 17000 8000
m= = = 4000.
6−4 2
ˆ The required equation is

y − 17000 =4000(x − 4)
y − 17000 =4000x − 16000
y =4000x + 1000

(b) Solution. The sales surpass $48,500 if 4000x + 1000 > 48500. Subtracting
1000 from both sides, we get 4000x > 47500. Dividing both sides by 4000, we
get x > 47500
4000
= 11.875 ≈ 12. So 12 years must pass before the sales surpass
$48,500.

2. [Exercise on page 4]

(a) Solution. Plotting the points (1, 10), (2, 14), (3, 16), (4, 18), and (5, 20), we get
Figure 1.5. According to the plot, the data are not exactly linear (as they don’t
lie along a straight line). But the plot is approximately linear, so it could be
approximated by a linear equation.
(b) Solution.
ˆ The slope is m = 20−10
5−1
= 10
4
= 2.5.
ˆ The equation is y − 10 = 2.5(x − 1), that is, y − 10 = 2.5x − 2.5. So
y = 2.5x + 7.5 is the equation of the line that models the data.
ˆ The slope indicates that the grade on exam increases by about 2.5 when the
number of hours studied increases by 1.
62 Chapter 1. Linear Functions

y
20

15

10

1 2 3 4 5 x

Figure 1.5

(c) Solution. To estimate the grade of a student who studied 6 hours, all we have
to do is to substitute x by 6 into the equation y = 2.5x + 7.5. We obtain
y = 2.5(6) + 7.5 = 15 + 7.5 = 22.5.

3. [Exercise on page 4]

(a) Solution.

Linear Functions
A linear function is a function of the form f (x) = mx + b, where m and b
are both real numbers.

According to the definition, f (x) = 9x − 6 is a linear function with m = 9


and b = −6.

(b) Solution. One
√ can rewrite this as g(t) = − 3t + 1. So it is a linear function
with m = − 3 and b = 1.
(c) Solution. This is not a linear function because it is not of the form mx + b.
Actually, it is a quadratic function as we will see in Chapter 2.

4. [Exercise on page 4]

(a) Solution. Substituting x with 2 into f (x), we have

f (2) = 5 − 3(2) = 5 − 6 = −1.

(b) Solution. Substituting x with 31 into g(x), we have


   
1 3 1 1 1 7 1 + 28 29
g = +7= +7= + = = .
3 4 3 4 4 1 4 4
√  √ √
(c) Solution. We have f 2 = 5 − 3( 2) = 5 − 3 2.

5. [Exercise on page 4]
1.2. Linear Functions and Applications 63

(a) Solution.
(i) For 0 shoe, q = 0, and therefore the corresponding price is p = D(0) =
10 − 0.5(0) = 10 − 0 = $10.
(ii) For 5000 shoes, q = 5 (remember the quantity is in thousands), and therefore
the corresponding price is p = D(5) = 10 − 0.5(5) = 10 − 2.5 = $7.5.
(b) Solution.
(i) If the price is p = 4, then 10 − 0.5q = 4. Subtracting 10 from both sides, we
−6
get −0.5q = −6. Dividing both sides by −0.5, we get q = −0.5 = 12. Thus,
at a price of $4, the quantity demanded is 12, 000 shoes.
(ii) If the price is p = 9, then 10 − 0.5q = 9. Solving this for q, we get q =
9−10
−0.5
−1
= −0.5 = 2. Thus, at a price of $9, the quantity demanded is 2, 000
shoes.
(c) Solution. The graphs of D(q) and S(q) are shown in Figure 1.6.
p
D(q) = 10 − 0.5q
S(q) = 0.25q + 1
10

4 Equilibrium point

5 10 15 20 25 q

Figure 1.6

(d) Solution.

Equilibrium Quantity and Equilibrium Price


ˆ The equilibrium quantity is the quantity where the supply is equal
to the quantity demanded.

ˆ The equilibrium price is the price where the quantity demanded is


equal to the quantity supplied.

ˆ The equilibrium point is the point (q0 , p0 ) where q0 is the equilibrium


quantity and p0 is the equilibrium price. Geometrically, it is the
point of intersection between the graph of the supply function and
that of the demand function.

To find the equilibrium point, one needs to solve the equation S(q) = D(q).
64 Chapter 1. Linear Functions

If q0 is the solution to this latter equation, then p0 = S(q0 ) = D(q0 ).

ˆ Finding the equilibrium quantity. Set S(q) = D(q), that is, 0.25q + 1 =
10−0.5q. Gathering the q’s on the left hand side, we get 0.75q = 9. Dividing
9
both sides by 0.75, we get q = 0.75 = 12. So the equilibrium quantity is
12, 000 shoes.
ˆ The equilibrium price is p = S(12) = 0.25(12) + 1 = 3 + 1 = $4.

6. [Exercise on page 5]
Solution.

ˆ Set S(q) = D(q), that is,

4 3
q = 105 − q
5 5
7 3
q = 105 Add q to both sides
5 5
7q = 525 Multiply both sides by 5
525
q= = 75
7
So the equilibrium quantity is q = 75.
ˆ The equilibrium price is p = S(75) = 45 (75) = 60.

7. [Exercise on page 5]
Solution.

Linear Cost Function


The cost function, C(x), is linear if it is of the form C(x) = mx + b, where m
and b are real numbers. In the linear cost function,

ˆ x is the number of items.

ˆ C(x) is the cost of producing x items.

ˆ m is the cost of producing one additional item. The slope, m, is called


marginal cost.

ˆ b is the fixed cost (the cost of producing 0 item).

The company spends $0.24 to produce a pencil. So, the cost of producing x pencils
is 0.24x. Since the fixed cost is $37000, the required linear cost function is C(x) =
0.24x + 37000.

8. [Solution on page 5]
1.2. Linear Functions and Applications 65

(a) Solution. Since the company spends $7 to produce one unit, the cost of pro-
ducing x units is 7x. Taking into account the fixed cost, $80000, we have

C(x) = 7x + 80000

(b) Solution.

The Revenue Function


Suppose the price for selling one unit is p. If x units are sold, the revenue
is given by the formula R(x) = px.

Since each unit is sold at $10, the revenue for selling x units is R(x) = 10x.
(c) Solution.

The Profit Function


Let C(x) and R(x) be the cost and revenue function. Then the profit
function is given by P (x) = R(x) − C(x).

Let P (x) be the profit corresponding to the production of x units. Then

P (x) = R(x) − C(x)


= 10x − (7x + 80000)
= 10x − 7x − 80000
= 3x − 80000

For x = 35000 units, the profit is

P (35000) = 3(35000) − 80000 = 105000 − 80000 = $25000

9. [Exercise on page 5]

(a) Solution.

Break-Even Quantity

Let C(x), R(x), and P (x) be the cost, revenue, and profit functions respec-
tively. The break-even quantity is the value of x where revenue equals cost
(R(x) = C(x)). This can also be found by solving the equation P (x) = 0.

Set R(x) = C(x). Then 20x = 3x + 85. Solving this for x, we get 17x = 85, so
that x = 85
17
= 5. Thus, the break-even quantity is x = 5.
(b) Solution.
ˆ First, we need to find the profit function:

P (x) = R(x) − C(x) = 20x − (3x + 85) = 20x − 3x − 85 = 17x − 85.


66 Chapter 1. Linear Functions

ˆ If 10 items are sold, then x = 10. And therefore, the profit is

P (10) = 17(10) − 85 = 170 − 85 = $85.

(c) Solution. We need to solve the equation P (x) = 11815 for x. We have 17x −
85 = 11815. Adding 85 to both sides, we get 17x = 11900. Dividing both sides
by 17, we get x = 11900
17
= 700. So 700 units will produce a profit of $11815.
Chapter2

Quadratic, Exponential, and


Logarithmic Functions

2.1 Quadratic Functions


1. Solution. [Exercise on page 7]
(a)
Quadratic Function

A quadratic function is a function of the form f (x) = ax2 + bx + c, where


a, b, and c are real numbers with a 6= 0. The numbers a, b, and c are called
coefficients.

For the function f (x) = 3x2 + 6x + 8, the coefficients are a = 3, b = 6, and


c = 8.
(b) One can rewrite this as f (x) = (1)x2 + (−1)x + (−5). So the coefficients are
a = 1, b = −1, and c = −5.
(c) Here the variable is not x, but t instead. Rewriting the function, g(t) = −t2 +
0t + 11, one can see that the coefficients are a = −1, b = 0, and c = 11.
(d) Rewriting the function, u(x) = −3x2 + 0x + 16, we have a = −3, b = 0, and
c = 16.

(e) Rewriting the function, h(q) = − 17 q 2 + 2q + 0, one can see that a = − 17 , b =

2, and c = 0.

2. [Exercise on page 7]

(a) Solution. One can see that x is a common factor. Taking that out, we get
x2 − 5x = x(x − 5).
(b) Solution. One can factor this by grouping:

−6x2 − 9x + 4x + 6 = −6x2 − 9x + |4x{z


+ 6}
| {z }
= −3x(2x + 3) + 2(2x + 3) = (2x + 3)(−3x + 2).

67
68 Chapter 2. Quadratic, Exponential, and Logarithmic Functions

(c) Solution. First we recall the difference of squares which states that for any
numbers a and b, one has a2 − b2 = (a − b)(a + b). Applying this, we get
x2 − 25 = x2 − 52 = (x − 5)(x + 5).
(d) Solution. We have

−9x2 + 49 = 49 − 9x2 = 72 − (3x)2 = (7 − 3x)(7 + 3x).


Warning. A common mistake is to write −9x2 + 49 = (−3x − 7)(−3x + 7).
This is wrong because the expansion of the right hand side gives 9x2 − 49, and
this latter is not equal to the original expression.
(e) Solution. There are several common methods to factor polynomials of the
form ax2 + bx + c, with a 6= 0 including the quadratic formula, the ac-method,
the trial-and-error method, and so on. Let us focus on the ac-method whose
procedure is as follows.
Step 1. Find two integers whose product is ac and whose sum is b. Here
a = 1, b = 1, and c = −2. The product ac is equal to −2. One
can see that the numbers we are looking for are −1 and 2 since
−1 × 2 = −2 = ac and −1 + 2 = 1 = b.
Step 2. Rewrite the middle term bx as the sum of the terms whose coeffi-
cients are the integers found in Step 1. Here the middle term is x,
and this can be rewritten as x = (−1)x + 2x = −x + 2x.
Step 3. Factor by grouping. We have
x2 + x − 2 = x2 − x + 2x − 2
| {z } | {z }
= x(x − 1) + 2(x − 1) = (x − 1)(x + 2).

(f ) Solution. First, one has a = 2, b = −7, and c = 3. We are looking for two
integers whose product is ac = 6 and whose sum is b = −7. After trying a
number of combinations, we get that the relevant numbers are −1 and −6. So

2x2 − 7x + 3 = 2x2 − x − 6x +3
| {z } | {z }
= x(2x − 1) − 3(2x − 1) = (2x − 1)(x − 3).

3. [Exercise on page 7]

(a) Solution.

Zero Product Property


The zero product property says that if ab = 0, then either a or b equal 0.
In other words,

ab = 0 implies that a = 0 or b = 0

We want to solve the equation 4x2 − 9 = 0.


4x2 − 9 = 0
2.1. Quadratic Functions 69

(2x − 3)(2x + 3) = 0 a2 − b2 = (a − b)(a + b)


2x − 3 = 0 or 2x + 3 = 0 Zero Product Property
3 3
x= or x = −
2 2
3
So the solutions are 2
and − 32 .

Alternate Solution
An alternate way to solve the equation 4x2 − 9 = 0 is to use the following
fact: √
x2 = a implies that x = ± a,
where a is a nonnegative number.

4x2 − 9 = 0
4x2 = 9 Add 9 to both sides
9
x2 = Divide both sides by 4
4
r
9 3
x=± =±
4 2
3
So the solutions are 2
and − 32 .

(b) Solution.

x2 − 3x = 0
x(x − 3) = 0 Factor
x = 0 or x − 3 = 0
x = 0 or x = 3

So the solutions are 0 and 3.


(c) Solution. Factoring the lefthand side of the equation x2 + 3x − 4 = 0, we get
(x − 1)(x + 4) = 0. This implies that x = 1 or x = −4.
(d) Solution. Factoring the lefthand side of the equation −3x2 + x + 2 = 0, we get
(3x + 2)(−x + 1) = 0. This implies that

3x + 2 = 0 or −x + 1 = 0
3x = −2 or −x = −1
2
x=− or x=1
3
70 Chapter 2. Quadratic, Exponential, and Logarithmic Functions

Alternate Solution
Another way to solve the equation −3x2 +x+2 = 0 is to use the quadratic
formula.

Quadratic Formula

Consider the equation ax2 +bx+c = 0, where a, b, and c are numbers


with a 6= 0. The quadratic formula is

−b ± b2 − 4ac
x=
2a
ˆ If b2 − 4ac > 0, the equation ax2 + bx + c = 0 has two solutions
x1 and x2 given by
√ √
−b + b2 − 4ac −b − b2 − 4ac
x1 = and x2 =
2a 2a

ˆ If b2 − 4ac = 0, the equation ax2 + bx + c = 0 has one solution


given by
−b
x=
2a
ˆ If b2 − 4ac < 0, there is no (real) solutions.

For the equation −3x2 + x + 2 = 0, a = −3, b = 1, and c = 2. Since

b2 − 4ac = (1)2 − 4(−3)(2) = 1 + 24 = 25

is positive, there are two solutions given by


√ √
−b + b2 − 4ac −b − b2 − 4ac
x1 = and x2 =
2a 2a
√ √
−(1) + 25 −(1) − 25
x1 = and x2 =
2(−3) 2(−3)
−1 + 5 −1 − 5
x1 = and x2 =
−6 −6
2
x1 = − and x2 = 1
3
So the solutions are − 23 and 1.

(e) Solution. The equation 4x2 + 12x = −9 is equivalent to 4x2 + 12x + 9 = 0. So


a = 4, b = 12, and c = 9. Using the quadratic formula, we find
p √
−(12) ± 122 − 4(4)(9) −12 ± 0 12 3
x= = =− =−
2(4) 8 8 2
2.1. Quadratic Functions 71

(f ) Solution. For the equation x2 − 3x + 4 = 0, a = 1, b = −3, c = 4. Using the


quadratic formula, we get
p √ √
−(−3) ± (−3)2 − 4(1)(4) 3 ± 9 − 16 3 ± −7
x= = =
2(1) 2 2
Since the square root of a negative number is undefined, it follows that there is
no solution.
(g) Solution. We want to solve −12x2 + 17x = 20. Subtracting 20 from both sides,
we get −12x2 + 17x − 20 = 0. Using the quadratic formula, we get
p √ √
−(17) ± 172 − 4(−12)(−20) −17 ± 289 − 960 −17 ± −671
x= = =
2(−12) −24 −24

Since −671 is undefined, there is no solution.
(h) Solution. The equation 0.1x2 = 4.5x−12.6 is equivalent to 0.1x2 −4.5x+12.6 =
0. Using the quadratic formula, we get
p √
−(−4.5) ± (−4.5)2 − 4(0.1)(12.6) 4.5 ± 20.25 − 5.04 4.5 ± 3.9
x= = =
2(0.1) 0.2 0.2
Written separately, this becomes
4.5 + 3.9 4.5 − 3.9
x= or x=
0.2 0.2
x = 42 or x=3
So the solutions are 42 and 3.

4. [Exercise on page 7]

(a) Solution.

Graphing a Quadratic Function

To graph a quadratic function f (x) = ax2 + bx + c, one can go through


the following steps.
(i) Find the sign of a. If a > 0, the parabola opens upward. If a < 0,
the parabola opens downward.
(ii) Find the y-intercept, which is the point where the graph intersects
the y-axis. It is given by (0, f (0)). For a quadratic function of the
form f (x) = ax2 + bx + c, f (0) = c, and the y-intercept is (0, c).
(iii) Find the x-intercepts by solving the equation ax2 + bx + c = 0 for x.
b b

(iv) Find the vertex of the parabola, which is given by − 2a , f − 2a .
(v) Sketch the graph. First plot the points (y-intercept, x-intercepts (if
they exist), and the vertex). Then join these by a curve.

Let us graph the function f (x) = x2 − 4x + 3.


72 Chapter 2. Quadratic, Exponential, and Logarithmic Functions

ˆ The coefficients are a = 1, b = −4, and c = 3. Since a > 0, the parabola


opens upward.
ˆ The y-intercept is (0, 3).
ˆ Finding the x-intercepts. First, we have b2 − 4ac = (−4)2 − 4(1)(3) =
16 − 12 = 4. Since b2 − 4ac is positive, the equation x2 − 4x + 3 = 0 has
two solutions given by
√ √
−b ± b2 − 4ac −(−4) ± 4 4±2
= = = 2 ± 1.
2a 2(1) 2

So the x-intercepts are 2 + 1 = 3 and 2 − 1 = 1.

Alternate way to find the x-intercepts

Factoring x2 − 4x + 3, we get (x − 1)(x − 3). Setting (x − 1)(x − 3) = 0,


we get x = 1 or x = 3, which are the x-intercepts.

ˆ Finding the vertex. First, we need to calculate − 2a


b
. Then we will find
b
f (− 2a ).
b
– We have − 2a = − (−4)
2(1)
= 4
2
= 2.
– Now f (2) = 22 − 4(2) + 3 = 4 − 8 + 3 = −1
Thus, the vertex is the point (2, −1).
ˆ The graph of f (x) = x2 − 4x + 3 is shown in Figure 2.1.

4
x-intercepts

2
y-intercept

2 4 6 x
−1
vertex

Figure 2.1: Graph of f (x) = x2 − 4x + 3

(b) Solution.
ˆ The coefficients are a = −2, b = 7, and c = 4. Since a is negative, the
parabola opens downward.
ˆ The y-intercept is (0, 4).
2.1. Quadratic Functions 73

ˆ Factoring f (x), we get f (x) = (−2x − 1)(x − 4). And solving the equation
f (x) = 0, we get −2x − 1 = 0 or x − 4 = 0, that is, x = − 12 or x = 4. These
are the x-intercepts.
ˆ For the vertex, we have − 2ab 7
= − 2(−2) = 74 . And
   2    
7 7 7 49 49
f = −2 +7 + 4 = −2 + +4
4 4 4 16 4
98 196 64 81
=− + + = .
16 16 16 8

So the vertex is 47 , 81

8
.
ˆ The graph of f (x) = −2x2 + 7x + 4 is shown in Figure 2.2.

y
10

2 4 x

Figure 2.2: Graph of f (x) = −2x2 + 7x + 4

(c) Solution.
ˆ The coefficients are a = 1, b = 4, and c = 4. Since a > 0, the parabola
opens upward.
ˆ The y-intercept is (0, 4).
ˆ Setting x2 + 4x + 4 = 0, we get (x + 2)2 = 0, which implies that x = −2.
So there is only one x-intercept, namely −2.
ˆ We have
b 4
− =− = −2 and f (−2) = (−2)2 + 4(−2) + 4 = 4 − 8 + 4 = 0.
2a 2(1)

So the vertex is (−2, 0).


ˆ The graph of f (x) = x2 + 4x + 4 is shown in Figure 2.3
(d) Solution.
ˆ The coefficients are a = −3, b = 2, and c = −1. Since a < 0, the parabola
opens downward.
ˆ The y-intercept is (0, −1).
74 Chapter 2. Quadratic, Exponential, and Logarithmic Functions

−4 −3 −2 −1 1 x

Figure 2.3: Graph of f (x) = x2 + 4x + 4

ˆ Set −3x2 + 2x − 1 = 0. Since b2 − 4ac = 22 − 4(−3)(−1) = 4 − 12 = −8 is


negative, the equation −3x2 + 2x − 1 = 0 has no solutions. So there is no
x-intercepts.
ˆ We have − 2a
b 2
= − 2(−3) = 1
3
and
   2    
1 1 1 1 2
f =−3 +2 − 1 = −3 + −1
3 3 3 9 3
3 6 9 6 2
=− + − =− =− .
9 9 9 9 3

So the vertex is 13 , − 23 .


ˆ The graph of f (x) = −3x2 + 2x − 1 is shown in Figure 2.4

y
1

−2 −1 1 2 x
−1

−2

−3

Figure 2.4: Graph of f (x) = −3x2 + 2x − 1

5. [Exercise on page 7]

(a) Solution. We need to solve the equation R(x) = C(x) for x. This is amount
to solving −x2 + 26x = 7x + 48 or −x2 + 19x − 48 = 0. Using the quadratic
2.1. Quadratic Functions 75

formula, we get
p √
−19 ±(19)2 − 4(−1)(−48) −19 ± 361 − 192
x= =
2(−1) −2

−19 ± 169 −19 ± 13
= = .
−2 −2

So
−19 + 13 −6 −19 − 13 −32
x= = = 3 or x = = = 16.
−2 −2 −2 −2
We then have two break-even quantities: 3 and 16. The minimum one is 3.
(b) Solution.
ˆ From the problem, the revenue function is R(x) = −x2 + 26x.
ˆ The graph of the revenue function is a parabola, which is opens downward
because a = −1 is negative. So the vertex of the parabola will give us the
maximum revenue.
ˆ Finding the vertex. We have − 2a b 26
= − 2(−1) = 13, and f (13) = −(13)2 +
26(13) = 169. So the vertex is (13, 169), and therefore
the maximum revenue is $169000. (Remember that the profit is in thou-
sands of dollars.)
(c) Solution.
ˆ The profit function is

P (x) = R(x) − C(x) = (−x2 + 26x) − (7x + 48)


= −x2 + 26x − 7x − 48 = −x2 + 19x − 48.

ˆ We need to find the vertex of the profit function. We have

b 19 19
− =− = = 9.5.
2a 2(−1) 2

And

P (9.5) = −(9.5)2 + 19(9.5) − 48 = −90.25 + 180.5 − 48 = 42.25.

So the maximum profit is $42250. (Remember that the profit is in thou-


sands of dollars.)

6. [Exercise on page 8]

(a) Solution. To find the break-even quantity, we need to solve the equation R(x) =
C(x).

x2 35
− + 5x = x +
3 3
−x2 + 15x = 3x + 35 Multiply both sides by 3
76 Chapter 2. Quadratic, Exponential, and Logarithmic Functions

−x2 + 12x − 35 = 0 Subtract 3x + 35 from both side

Using the quadratic formula, we get


p √
−12 ± 122 − 4(−1)(−35) −12 ± 4 −12 ± 2
x= = =
2(−1) −2 −2

Written separately, this becomes

−12 + 2 −12 − 2
x= or x=
−2 −2
x=5 or x=7

So the break-even quantities are 5 and 7. And the minimum one is 5.


(b) Solution. To find the maximum profit, first we need to find the profit function.
 2
x2
  
x 35 35
P (x)R(x) − C(x) = − + 5x − x + = − + 5x − x −
3 3 3 3
x2 35
=− + 4x −
3 3

This is a quadratic function with a = − 13 , b = 4, and c = − 35


3
. So the graph of P
is a parabola. Since a is negative, the parabola opens downward and therefore
the vertex gives the maximum profit. All we have to do now is to find the vertex.
First,
 
b 4 4 3 a ac
− =− 1
 = − 2 = −4 − =6 b
=
2a 2 −3 −3 2 c
b

Substituting x with 6 into P (x), we get

1 35 35 35
P (6) = − (6)2 + 4(6) − = −12 + 24 − = 12 −
3 3 3 3
36 − 35 b ac − b
= a− =
3 c c
1
=
3

So the vertex is 6, 13 . And the maximum value of P (x) is 31 ≈ 0.3333. Since




the profit is in thousands of dollars, the maximum profit is $333.33.

2.2 Exponential Functions and Compound Interest


1. [Exercise on page 8]

(a) Solution.
2.2. Exponential Functions and Compound Interest 77

Exponential Functions
Let a be a real number such that a > 0 and a 6= 1. The function defined
by f (x) = ax is called the exponential function of base a. Some facts about
exponential functions include the following.

ˆ The function f (x) = ax is defined for every real number x.

ˆ The y-intercept of f (x) = ax is 1. That is, f (0) = a0 = 1.

ˆ The graph of y = ax does not intersect the x-axis. So there is no


x-intercept.

ˆ The graph of y = ax is above the x-axis. In other words,

ax > 0 for all x.

ˆ The graph of f (x) = ax is increasing when a > 1 and decreasing


when 0 < a < 1. (For example, see Figure 2.5.)

The statement “the exponential function f (x) = ax is defined for every real
number x” is True.

4 y = 2x

1
y = (0.5)x
−4 −3 −2 −1 1 2 3 4 x

Figure 2.5

(b) Solution. False. The function f (x) = ax is increasing when a > 1.


(c) Solution. False. The function f (x) = ax is decreasing when 0 < a < 1.
(d) Solution. False. The graph of f (x) = ax gets closer and closer to the x-axis,
but never touches it.
(e) Solution. True.
(f ) Solution. False. Indeed, the graph of y = ax is above the x-axis.
78 Chapter 2. Quadratic, Exponential, and Logarithmic Functions

2. [Exercise on page 8]
32x 3−2
(a)
3x−1
Solution.

Properties of Exponential Functions


If a > 0, b > 0, and x, y are real numbers, then
ax
(i) ax ay = ax+y (ii) y = ax−y
a

(iii) (ax )y = axy (iv) (ab)x = ax bx .

Using these properties, we get

32x 3−2 32x−2


x−1
= x−1 = 32x−2−(x−1) = 32x−2−x+1 = 3x−1 .
3 3
√ 1
(b) Solution. Remembering that a = a 2 , we have
5x 5x 5x 5x 1 1
√  = = = = 5x− 2 +6x = 57x− 2 .
1 3x 1 1 1
5 52
5 2 (5−2 )3x 5 5−6x
2 5 2
−6x

√ 1
(c) Solution. Remembering that 3
a = a 3 , we have

(7x 7−5x )2 (7−4x )2 7−8x


p = 1 = 2x
71−x 3 (7x )2 71−x (72x ) 3 71−x 7 3
7−8x 7−8x −8x−1+ x3
= 2x = x = 7
71−x+ 3 71− 3
−23x
−1
=7 3

3. [Exercise on page 8]

(a) Solution. We will use the following property.

Property of Exponential Functions (continued)

Let a be a positive number (a > 0) different from 1 (a 6= 1). Then

ax = ay if and only if x = y.

Note. Before using this, we have to make sure that we have the same
base on both sides of the equation ax = ay .

Using this, the equation 4x = 410 is equivalent to x = 10.


2
(b) Solution. The equation 5x = 59 is equivalent to x2 = 9 or x = ±3.
2.2. Exponential Functions and Compound Interest 79

(c) Solution. The equation 32x = 3x is equivalent to 2x = x. Subtracting x from


both sides, we get x = 0.
(d) Solution. The base of the left hand side of the equation 25x = 58 is 25 while
the base of the right hand side is 5. To get the same base, we can use the fact
that 25 = 52 . The equation becomes

(52 )x = 58
52x = 58
2x = 8
x=4 Divide both sides by 2.

(e) Solution. The equation 36x = 216 is equivalent to (62 )x = 63 or 62x = 63 . This
latter equation implies that 2x = 3. Dividing both sides by 2, we get x = 23 .
(f ) Solution.
ˆ Since 27 = 33 and 81 = 34 , the equation 27x−1 = 813x+2 becomes (33 )x−1 =
(34 )3x+2 , that is, 33x−3 = 312x+8 .
ˆ This latter equation is equivalent to 3x − 3 = 12x + 8. Subtracting 12x from
both sides, we get −9x − 3 = 8. Adding 3 to both sides, we get −9x = 11.
Dividing both sides by −9, we get x = − 11 9
.
(g) Solution. The equation 2x = 1 is equivalent to 2x = 20 . So x = 0.
(h) Solution.
2 2
ˆ Adding 336 to both sides of the equation 34x − 336 = 0, we get 34x = 336 .
ˆ This implies that 4x2 = 36. Dividing both sides by 4, we get x2 = 9. So
x = ±3.
2 2
(i) Solution. The equation 5x 5x = 56 is equivalent to 5x +x = 56 . So x2 + x = 6
and therefore x2 +x−6 = 0. Factoring the left hand side, we get (x−2)(x+3) = 0.
So x = 2 or x = −3.
x2 2
(j) Solution. The equation 73x−2 7
= 1 is equivalent to 7x −(3x−2) = 1, that is,
2
7x −3x+2 = 70 . This implies that x2 − 3x + 2 = 0. Factoring the left hand side,
we get (x − 1)(x − 2) = 0, which implies that x = 1 or x = 2.
2 3
(k) Solution. The equation (3x )x = 3x is equivalent to 3x = 3x , that is, x3 = x.
Moving x to the left hand side, we have x3 − x = 0. Factoring this, we get
x(x2 − 1) = 0. Factoring further, we get x(x − 1)(x + 1). This implies that
x = 0 or x = 1 or x = −1.
4. [Exercise on page 8]
Solution.

Compound Amount
The formula for the compound amount is
 r tm
A=P 1+ ,
m
80 Chapter 2. Quadratic, Exponential, and Logarithmic Functions

where

ˆ P represents the principal amount or amount invested or initial amount.

ˆ r represents the interest rate.

ˆ m is the number of times interest is compounded in one year.

ˆ t represents the amount of time in years.

ˆ A represents the compound amount or the amount of money after t years.

Some values for m


If the interest is compounded annually, then m = 1. If it is compounded
semiannually, then m = 2, etc.

Annually Semiannually Quarterly Monthly


m= 1 2 4 12

From the problem, P = 2000, r = 12% = 0.12, t = 5. Since the interest is com-
pounded annually, m = 1. Substituting these into the formula, we get
 5(1)
0.12
A = 2000 1 + = 2000(1.12)5 = 2000(1.762341) ≈ 3524.68.
1

So A ≈ $3524.68.

5. [Exercise on page 8]
Solution. Here P = 7000, r = 4% = 0.04, t = 3. Since the interest is compounded
quarterly, we have m = 4. Using the formula, we get
 3(4)
0.04
A = 7000 1 + = 7000(1 + 0.01)12 = 7000(1.01)12 ≈ 7887.77.
4

So A ≈ $7887.77.

6. [Exercise on page 8]
Solution.

(a) We have P = 12000, r = 5% = 0.05, t = 6, and m = 2 (because the interest is


compounded semiannually). Using the formula, we get
 6(2)
0.05
A = 12000 1 + = 12000(1 + 0.025)12 ≈ 16138.67.
2

(b) The interest earned is the final amount minus the initial amount: 16138.67 −
12000 = 4138.67.
2.2. Exponential Functions and Compound Interest 81

7. [Exercise on page 9]
Solution.
ˆ First, we need to find the compound amount. We have P = 5000, r = 7%, t = 9,
and m = 12 (because the interest is compounded monthly). Using the formula,
we get
 9(12)  108
0.07 0.07
A = 5000 1 + = 5000 1 + ≈ 9370.88.
12 12
ˆ The interest earned is 9370.88 − 5000 = $4370.88.

8. [Exercise on page 9]
Solution.

Continuous Compounding
ˆ If the number of times interest is compounded in a year goes to infinity,
we say that the interest is compounded continuously.

ˆ When the interest is compounded continuously, the compound amount is


given by the following formula.

A = P ert ,

where

– P represents the principal amount or amount invested or initial


amount.
– r represents the interest rate.
– t represents the amount of time in years.
– A represents the compound amount or the amount of money after t
years.
– e is the Euler number. An approximate value of e is e ≈ 2.718281828.

From the problem, we have P = 15000, r = 5% = 0.05, and t = 12. Substituting


these into the formula, we get
A = 15000e0.05(12) = 15000e0.6 = 15000(1.8221188) ≈ 27331.78.
So the compound amount is A ≈ $27331.78.
9. [Exercise on page 9]
Solution.
ˆ First, we need to find the compound amount. We have P = 8000, r = 3% = 0.03,
and t = 10. Using the formula, we get
A = 8000e0.03(10) = 8000e0.3 = 8000(1.3498588) ≈ 10798.87
82 Chapter 2. Quadratic, Exponential, and Logarithmic Functions

So the compound amount is A = $10, 798.87.


ˆ The interest earned is equal to the final amount minus the initial amount. That
is, Interest = 10798.87 − 8000 = $2798.87.

2.3 Logarithmic Functions


1. [Exercise on page 9]

(a) Solution.

Logarithms–Definition

Let a be a positive number (a > 0) not equal to 1 (a 6= 1). Let x > 0 be


a positive real number. The logarithm of x to the base a, denoted loga x,
is the number such that aloga x = x. In other words,

y = loga x if and only if ay = x.

For example, what is log2 8? To find log2 8, we need to find the exponent
y of 2 such that 2y = 8. Since 23 = 8, we have that log2 8 = 3. Similarly,
log2 32 = 5 because 25 = 32.

One way to find loga x is to write x as a power of a.

To find log2 (64), we need to write 64 as a power of 2. Since 64 = 26 , we have


that log2 (64) = log2 (26 ) = 6.
(b) Solution. Since 81 = 34 , it follows that log3 (81) = log3 (34 ) = 4.
1
= log5 513 = log5 (5−3 ) = −3.
 
(c) Solution. We have log5 125
(d) Solution.
We have log10 (0.0001) = log10 (10−4 ) = −4.
(e) Solution. We have
   
1 1
log2 (0.25) = log2 = log2 = log2 (2−2 ) = −2.
4 22
(f ) Solution.
r !   r
3 1 1 3 1 1 1
log2 = log2 = √
3
=
8 2 8 8 2
= log2 (2−1 )
=−1

2. [Exercise on page 9]
(a) Solution.
2.3. Logarithmic Functions 83

Logarithmic Functions
Let a > 0, a 6= 1. The function defined by

f (x) = loga (x) x>0

is called the logarithmic function of base a. Some facts about the loga-
rithmic function include the following.

ˆ The function f (x) = loga (x) is defined only for positive number. So
loga (0) is undefined, loga (−4) is undefined, etc.

ˆ The x-intercept of f (x) = loga (x) is 1 because loga (1) = loga (a0 ) =
0.

ˆ The graph of f (x) = loga (x) has no y-intercept.

ˆ The graph of f (x) = loga (x) is increasing when a > 0 and decreasing
when 0 < a < 1. (For example, see Figure 2.6.)

y = log2 x
2

1 2 3 4 x

−2
y = log0.5 x

−4

Figure 2.6

The statement “The logarithm of a negative number is undefined.” is True by


definition of the logarithmic function.
(b) Solution. False because loga (0) is undefined.
(c) Solution. False. It is the other way around.
(d) Solution. True.
(e) Solution. False because the graph does not intersect the y-axis. Actually, the
graph gets closer and closer to the y-axis but never touches it.
(f ) Solution. True. For example, the logarithm of 2−3 to the base 2 is equal to
−3, that is, log2 (2−3 ) = −3.
84 Chapter 2. Quadratic, Exponential, and Logarithmic Functions

3. [Exercise on page 9]

(a) Solution. We have

log3 (3) + log3 (27) = log3 (31 ) + log3 (33 ) = 1 + 3 = 4.

Alternate Solution
Another way to compute this is to use the following property (i).

Some Properties of Logarithms


Suppose x and y are positive real numbers and n is a real number.
Let a be a real number such that a > 0 and a 6= 1. Then we have
the following properties.

(i) loga (xy) = loga x + loga y


 
(ii) loga xy = loga x − loga y

(iii) loga (xn ) = n loga x

Using property (i), we get

log3 (3) + log3 (27) = log3 (3 × 9) = log3 (81) = log3 (34 ) = 4.

(b) Solution.
 
40
log2 (40) − log2 (5) = log2 Property (ii)
5
= log2 (8)
= log2 (23 )
=3

(c) Solution.

log5 (0.043 ) = 3 log5 (0.04) Property (iii)


 
1 4 1
= 3 log5 0.04 = =
25 100 25
 
1
= 3 log5
52
= 3 log5 (5−2 )
= 3(−2) = −6

4. [Exercise on page 9]

(a) Solution. Using Property (iii), we get log2 x2020 = 2020 log2 x .
2.3. Logarithmic Functions 85

(b) Solution.

log5 (x2 y −7 ) = log5 x2 + log5 y −7 Property (i)


=2log5 x − 7log5 y Property (iii).
 8
y
(c) log x3
Solution.

Definition of log x and ln x


ˆ When the base is a = 10, log10 x is simply written log x, and called
common logarithm. That is,

log x = log10 x.

ˆ When the base is a = e, loge x is simply written ln x, and called


natural logarithm. That is,

ln x = loge x.

The function f (x) = ln x is called natural logarithmic function.

We have
y8
 
log = log y 8 − log x3 = 8 log y − 3 log x.
x3
(d) Solution.

Some Useful Facts About Logarithms


Using the definition of logarithm, one has the following.

ˆ Let a > 0, a 6= 1. Then for every positive real number x (x > 0),
one has
aloga x = x

ˆ And for every real number y, one has

loga (ay ) = y

If the base a is equal to e (a = e), these become:

eln(x) = x for every x > 0

and
ln(ey ) = y for every real number y. (2.3.1)
In particular, ln(e) = 1.
86 Chapter 2. Quadratic, Exponential, and Logarithmic Functions

3 3
ln(x8 e−x ) = ln x8 + ln e−x
3
= 8 ln x − x3 ln e−x = −x3 by (2.3.1)

(e) Solution.
√ 1 1
x2 − 3x = ln(x2 − 3x) 3 = ln (x2 − 3x).
3
ln
3
(f ) Solution.

(3x − 1)4 x2
 
ln = ln((3x − 1)4 x2 ) − ln(2x + 1)5
(2x + 1)5
= ln(3x − 1)4 + ln x2 − ln(2x + 1)5
= 4 ln (3x − 1) + 2 ln x − 5 ln (2x + 1).

5. [Exercise on page 9]

(a) Solution. Using the properties of ln(x) = loge (x), we get

3 ln x + 2 ln(x − 1) = ln(x3 ) + ln(x − 1)2 = ln x3 (x − 1)2 .




(b) Solution.

1 1
3 ln x − ln(5x) + 7 = 3 ln x − ln(5x) + ln(e7 ) ln(e7 ) = 7 by (2.3.1)
2 2
1
= ln(x ) − ln((5x) 2 ) + ln(e7 )
3
 3 
x
= ln √ + ln(e7 )
5x
 3 7
x e
= ln √ .
5x

(c) Solution. Using the fact that 4 = log(104 ), the expression becomes

log(x + 2) − 4 log(x) + 4 = log(x + 2) − 4 log(x) + log(104 )


= log(x + 2) − log(x4 ) + log(10000)
 
x+2
= log + log(10000)
x4
 
(x + 2)10000
= log .
x4

6. [Exercise on page 10]


Solution.
2.3. Logarithmic Functions 87

Change-of-Base Theorem for Logarithms


Let x be a positive real number and let a and b be positive real numbers such
that a 6= 1 and b 6= 1. Then
logb x
loga x = . (2.3.2)
logb a

When b = e, this formula becomes


ln x
loga x = .
ln a
This says that any logarithm can be expressed using the natural logarithm.

Using (2.3.2) with a = 9 and b = 3, we get

log3 x log3 x log3 x 1


log9 x = = 2
= = log3 x.
log3 9 log3 (3 ) 2 2

7. [Exercise on page 10]

(a) Solution. By definition of logarithms,


√ logx 16 = 4 if and only if x4 = 16. This
4
latter equation implies that x = 16 = 2. So the solution is x = 2.
(b) Solution. Again by definition of logarithms, logx 27 = 3 if and only if x3 = 27

8 8
.
q
This implies that x = 3 278
= 32 . So the solution is x = 32 .
1 1
(c) Solution. Again by definition of logarithms, log5 x = if and only if 5 2 = x.
√ 2
So x = 5.
x

(d) Solution. The equation log2 x − log2 (x − 2) = 1 is equivalent to log2 x−2 =
x
1. By definition of logarithms, this latter equation is equivalent to 21 = x−2 .
Multiplying both sides by x − 2, we get x = 2(x − 2), which is equivalent to
x = 2x − 4. Solving this for x, we get x = 4.
(e) Solution.
ˆ The equation log6 x + log6 (x − 1) = 1 is equivalent to log6 (x(x − 1)) = 1,
which is equivalent to 61 = x(x − 1), that is, 6 = x2 − x. Subtracting 6 from
both sides, the equation becomes x2 − x − 6 = 0.
ˆ Factoring the left hand side, we get (x − 3)(x + 2) = 0. So x = 3 or x = −2.
ˆ The solution x = −2 is to be rejected because it is not a valid value in the
original equation (note that the logarithm of a negative number is unde-
fined). So the solution to the original equation is x = 3.

8. [Exercise on page 10]

(a) Solution.
88 Chapter 2. Quadratic, Exponential, and Logarithmic Functions

Equality of logarithms
Let a > 0, a 6= 1, and let x, y be positive numbers.

loga x = loga y if and only if x = y.

If a = e, this becomes

ln x = ln y if and only if x = y. (2.3.3)

Using (2.3.3), the equation ln(2x + 1) = ln(x + 5) is equivalent to 2x + 1 = x + 5.


Subtracting x from both sides, we get x + 1 = 5. Subtracting 1 from both sides,
we get x = 4.
(b) Solution.
ˆ The equation ln(x2 + x) = ln(2x + 6) is equivalent to x2 + x = 2x + 6.
Subtracting 2x + 6 from both sides, we get x2 − x − 6 = 0. Factoring this,
we get (x − 3)(x + 2) = 0. So x = 3 or x = −2.
ˆ We need to check if the values 3 and −2 satisfy the original equation.
– For x = 3, the left hand side is ln(x2 + x) = ln(32 + 3) = ln(12), and the
right hand side is ln(2x + 6) = ln(2(3) + 6) = ln 12. So x = 3 works.
– For x = −2, the left hand side of the original equation becomes

ln(x2 + x) = ln((−2)2 − 2) = ln(4 − 2) = ln(2),

and the right hand side becomes

ln(2x + 6) = ln(2(−2) + 6) = ln(−4 + 6) = ln 2.

So x = −2 is a valid solution.
Thus, the solutions are −2 and 3.
(c) Solution. To isolate x, we need to take the natural exponential of both sides.
This gives eln x = e2 . Since eln x = x, the latter equation becomes x = e2 , which
is the solution to the original equation.

Alternate Solution
Since ln x = loge x, the equation becomes loge x = 2. By definition of
logarithms, loge x = 2 if and only if e2 = x. So x = e2 .

(d) Solution. Taking the natural exponential on both sides, we get 3x + 1 = e7 .


Subtracting 1 from both sides, we get 3x = e7 − 1. Dividing both sides by 3, we
7
get x = e 3−1 .
(e) Solution. The equation ln(2x) + ln(5x) = −1 is equivalent to ln((2x)(5x)) =
−1, that is, ln(10x2 ) = −1. Taking the natural exponential on both sides,
q we
−1 e−1 −1
get 10x = e . Dividing both sides by 10, we get x = 10 . So x = e10 or
2 2
2.3. Logarithmic Functions 89

q
−1
x = − e10 . The negative value is to be rejected because when we substitute it
into the original equation, we have the logarithm of a negative
q number, which
e−1
is undefined. So the solution to the original equation is x = 10
.

9. [Exercise on page 10]

(a) Solution. Taking the natural logarithm on both sides of the equation 2x = 5,
we get ln(2x ) = ln 5. This is equivalent to x ln 2 = ln 5. Dividing both sides by
ln 5
ln 2, we get x = ln 2
.

Alternate Solution
The equation 2x = 5 suggests that the base is 2. So by definition of
logarithms, 2x = 5 if and only if x = log2 5. This solution is the same as
the one we found using the natural logarithm. Indeed, using the Change-
of-Base Theorem for Logarithms, we get log2 5 = ln 5
ln 2
.

(b) Solution. Taking natural logarithms on both sides of the equation 32x−1 = 7,
the equation becomes ln(32x−1 ) = ln 7. Using the property ln(xr ) = r ln x, the
latter equation becomes (2x − 1) ln 3 = ln 7. Dividing both sides by ln 3, we
get 2x − 1 = ln 7
ln 3
. Adding 1 to both sides, we get 2x = ln 7
ln 3
+ 1 = ln 7+ln
ln 3
3
. So
ln 7+ln 3
x = 2 ln 3 .

Alternate Solution
Using the definition of logarithms, we have 32x−1 = 7 if and only if 2x−1 =
log3 7. Solving this for x, we get x = 1+log
2
37
. This is the same as ln27+ln
ln 3
3
.
Indeed,

1 + log3 7 1 + ln 7
ln 3 Change-of-Base Theorem for Log-
=
2 2 arithms (loga x = ln x
ln a
)
ln 3+ln 7
ln 3
=
2
ln 3 + ln 7
=
2 ln 3

(c) Solution. Taking the natural logarithm on both sides of the equation 36x =
4x+1 , we get 6x ln 3 = (x + 1) ln 4. Distributing the right hand side, we get
6x ln 3 = x ln 4+ln 4. Subtracting x ln 4 from both sides, we get 6x ln 3−x ln 4 =
ln 4. Factoring the left hand side, we get x(6 ln 3 − ln 4) = ln 4. This implies
that x = 6 lnln 4
3−ln 4
.
(d) Solution. Dividing both sides of the equation 2e−3x = 9 by 2, we get e−3x = 92 .
9

Taking the natural logarithm on both sides, we get −3x ln
 e = ln 2 . (Remem-
9
ber ln e = 1.) The latter equation becomes −3x = ln 2 . Dividing both sides
ln( 9 )
by −3, we get x = −32 .
90 Chapter 2. Quadratic, Exponential, and Logarithmic Functions

(e) Solution. Subtracting 3 from both sides of the equation 2e−0.1x + 3 = 15, we
get 2e−0.1x = 12. Dividing both sides by 2, we get e−0.1x = 6. Taking the natural
logarithm on both sides, we get −0.1x = ln 6. Dividing both sides by −0.1, we
get x = −10 ln 6.

10. [Exercise on page 10]

(a) Solution.

Change-of-Base Theorem for Exponentials


Let a > 0 be a positive real number, and let x be any real number. Then

ax = ex ln a .

This says that any exponential function can be written as an exponential


function with base e.

Using this theorem, we have 2−x = e−x ln 2 .


(b) Solution. Using the Change-of-Base Theorem for Exponentials, we have 35x−2 =
e(5x−2) ln 3 .

11. [Exercise on page 10]

(a) Solution.
ˆ First, we have P = 1000, r = 4% = 0.04, and m = 1 (because the interest
is compounded annually). We also have A = 2000 because the amount has
double.
ˆ Substituting these into the formula for the compound amount, we get 2000 =
1000(1 + 0.04
1
)t(1) . This is equivalent to 2000 = 1000(1.04)t .
ˆ We solve the latter equation for t. Dividing both sides by 1000, we get
(1.04)t = 2. Taking the natural logarithm on both sides, we get t ln(1.04) =
ln 2. This implies that
ln 2 0.69314718056
t= = ≈ 17.67.
ln(1.04) 0.03922071315

(b) Solution.
ˆ If the interest is compounded quarterly, m = 4, and the formula for the
compound amount becomes 2000 = 1000 1 + 0.04 4
)t(4) . This is equivalent
to 2000 = 1000(1 + 0.01) , that is, 2000 = 1000(1.01)4t . So (1.01)4t = 2.
4t

ˆ We solve this latter equation for t. Taking the natural logarithm on both
sides, we get 4t ln(1.01) = ln 2. This implies that

ln 2
t= ≈ 17.41.
4 ln(1.01)

(c) Solution.
2.3. Logarithmic Functions 91

ˆ Because the interest is compounded continuously, the formula for the com-
pound amount is A = P ert . Substituting A with 2000, P with 1000, and r
with 0.04, we get 2000 = 1000e0.04t .
ˆ We solve this latter equation for t. Dividing both sides by 1000, we get
e0.04t = 2. Taking the natural logarithm on both sides, we get 0.04t = ln 2.
Dividing both sides by 0.04, we get
ln 2
t= ≈ 17.33.
0.04
12. [Exercise on page 10]

(a) Solution. The value of the account when t = 0 is

A(0) = 400 ln(0 + 4) = 400 ln(4) ≈ 554.517744448

Since A is in millions of dollars, the value of the account when t = 0 is


$554, 517, 744.45.
(b) Solution. We need to solve the equation A(t) = 2A(0) for t. This is equivalent
to 400 ln(t + 4) = 2A(0). Dividing both sides by 400, we get

A(0) 2(400 ln 4)
ln(t + 4) = 2 = = 2 ln 4
400 400
So ln(t + 4) = 2 ln 4. Taking the natural exponential on both sides, we get
t + 4 = e2 ln 4 . Subtracting 4 from both sides, we have
2
t = e2 ln 4 − 4 = eln(4 ) − 4 = eln(16) − 4 = 16 − 4 = 12

Thus, it takes 12 years for the account to double its initial value.
92 Chapter 2. Quadratic, Exponential, and Logarithmic Functions
Chapter3

Limits and Derivatives

3.1 Finding Limits Using Tables and Graphical Limits


1. [Exercise on page 11]

(a) Solution.

Limit – Intuitive Definition


ˆ Let a be a number. If a variable x gets closer and closer to a with-
out ever reaching a, we write x → a. And we simply say that x
approaches a. So

x → a means that x gets closer to a and x 6= a

ˆ Let y = f (x) be a function and let a be a number. If the output


f (x) gets closer and closer to a number L as x approaches a, we say
that the limit of f (x) as x approaches a equals L, and we write

lim f (x) = L
x→a

Let f (x) = x2 + 3. Consider the following table.

x 0.5 0.9 0.99 0.999 1 1.001 1.01 1.1 1.5


f (x) 3.25 3.81 3.9801 3.998 4.002 4.0201 4.21 5.25

The table tells us that f (x) gets closer and closer to 4 as x approaches 1. We
can then make the guess that lim (x2 + 3) = 4.
x→1
x2 −4
(b) Solution. Let f (x) = x+2
. Consider the following table.

x −2.5 −2.1 −2.01 −2.001 −2 −1.999 −1.99 −1.9 −1.5


f (x) −4.5 −4.1 −4.01 −4.001 −3.999 −3.99 −3.9 −3.5

93
94 Chapter 3. Limits and Derivatives

x2 −4
From the table, we can make the guess that lim = −4.
x→−2 x+2
ex −1
(c) Solution. Let f (x) = x
. Consider the following table.

x −0.5 −0.1 −0.01 −0.001 0 0.001 0.01 0.1 0.5


f (x) 0.787 0.95 0.995 0.9995 1.0005 1.005 1.05 1.297

ex −1
From the table, we can make the guess that lim x
= 1.
x→0

2. [Exercise on page 11]


Solution. Consider the graph from Figure 3.1.

(a) f (2) does not exist because the function is not defined at 2 (as there is a hole
on the graph).

(b) We have lim f (x) = 3 because when x approaches 2 from either side, f (x) gets
x→2
closer and closer to 3.
(c) f (4) = 6.
(d) lim f (x) = 5 because when x approaches 4 from either side, f (x) is gets closer
x→4
and closer to 5.
(e) f (3) = 4
(f ) lim f (x) = 4.
x→3

3. [Exercise on page 12]


Solution.

One-Sided Limits
ˆ Left-Sided: When x gets closer and closer to a and x is less than a
(x < a), we say that x approaches a from the left and we write x → a− .
The limit of f (x) from the left is written lim− f (x).
x→a

ˆ Right-Sided: Similarly, x approaches a from the right if x gets closer


and closer to a and x > a. We write x → a+ . The limit of f (x) from the
right is written lim+ f (x).
x→a

ˆ A Two-Sided limit lim f (x) exists if and only if both one-sided limits
x→a
exist and are the same. More precisely,
   
lim f (x) = L if and only if lim− f (x) = L and lim+ f (x) = L .
x→a x→a x→a

So, if the limit from the left is not the same as the limit from the right,
then lim f (x) does not exist.
x→a
3.1. Finding Limits Using Tables and Graphical Limits 95

Consider the graph from Figure 3.2.

(a) lim− f (x) = 1


x→2

(b) lim+ f (x) = −1


x→2

(c) lim f (x) does not exist because the limit from the left is not equal to the
x→2
limit from the right.
(d) f (2) = 1
(e) lim f (x) = 3 because the limit from the left ( lim− f (x) = 3) is equal to the
x→6 x→6
limit from the right ( lim+ f (x) = 3).
x→6

(f ) f (6) does not exist because f (x) is not defined at x = 6.


(g) lim f (x) = −2
x→−1+

(h) lim+ f (x) does not exist because the function is not defined for values of x
x→7
bigger than 7.

4. [Exercise on page 12]


Solution. Consider the graph from Figure 3.3.

(a) lim f (x) = 4.


x→−1−

(b) lim f (x) = 1


x→−1+

(c) lim f (x) does not exist because the limit from the left is not the same as
x→−1
the limit from the right.
(d) f (−1) = 2.
(e) lim f (x) does not exist because the limit from the left ( lim− f (x) = 4) is not
x→3 x→3
the same as the limit from the right ( lim+ f (x) = 6).
x→3

(f ) f (3) = 6.
(g) lim− f (x) = 4.
x→6

(h) f (6) does not exist because the function f (x) is not defined at x = 6.

5. [Exercise on page 12]


Solution.

Vertical Asymptotes

Let f (x) be a function, and let a be a number. The line x = a is called vertical
asymptote of the curve y = f (x) if at least one of the following holds.
96 Chapter 3. Limits and Derivatives

(i) lim f (x) = ∞ (ii) lim+ f (x) = ∞ (iii) lim− f (x) = ∞


x→a x→a x→a

(iv) lim f (x) = −∞ (v) lim+ f (x) = −∞ (vi) lim− f (x) = −∞


x→a x→a x→a

Consider the graph from Figure 3.4.


ˆ From Figure 3.4, we have that lim − f (x) = ∞. So the statement (ii) is true,
x→−1
and therefore the line x = −1 is a vertical asymptote.
ˆ Again from Figure 3.4, we have that lim− f (x) = −∞. So the statement (v) is
x→1
true, and therefore the line x = 1 is a vertical asymptote.

3.2 Finding Limits Using Rules and One-Sided Limits


1. [Exercise on page 12]
(a) Solution. We will use the following rules for limits.

Rules for Limits


Let f and g be functions, and let a be a real number. Let L and M be
real numbers such that
lim f (x) = L and lim g(x) = M.
x→a x→a

Then we have the following rules.


1. If k is a real number (also called a constant), then
lim k = k and lim (kf (x)) = klim f (x)
x→a x→a x→a

2. ˆ lim (f (x) + g(x)) = lim f (x) + lim g(x)


x→a x→a x→a
ˆ lim (f (x) − g(x)) = lim f (x) − lim g(x)
x→a x→a x→a
  
3. lim (f (x)g(x)) = lim f (x) lim g(x)
x→a x→a x→a

f (x) lim f (x) L


4. lim = x→a = provided that M 6= 0.
x→a g(x) lim g(x) M
x→a

5. If P (x) is a polynomial, then lim P (x) = P (a)


x→a

6. For any real number n,


n
n
lim (f (x)) = lim f (x) = Ak
x→a x→a

provided that An exists. (This limit does not exist, for example,
when A < 0 and n = 12 .)
3.2. Finding Limits Using Rules and One-Sided Limits 97

7. If f (x) = g(x) for all x 6= a, then lim f (x) = lim g(x)


x→a x→a

8. For any positive real number b,


lim f (x)
lim bf (x) = bx→a = bL
x→a

9. For any positive real number b such that b 6= 1,


 
lim (logb f (x)) = logb lim f (x) = logb L
x→a x→a

provided that L > 0.

Polynomials
ˆ A term is an expression of the form axn , where a is a real number,
called coefficient, x is the variable, and n is the exponent. Examples
1
of terms include 2x3 , −3x6 , 13 x 2 .

ˆ A monomial is a term where the exponent is a nonnegative integer


1
(0, 1, 2, 3, 4, · · · ). For example, 2x3 , −3x6 are monomials. But 5x 2 is
not a monomial because the exponent, 21 , is not an integer.

ˆ A polynomial is a finite sum of monomials. Examples of polynomials


include x3 , 3x5 − 7x2 + 5, and 12 y 4 − 23 y 3 .

Consider the limit lim x3 + x. The function P (x) = x3 + x is a polynomial.


x→4
Therefore, by using Rule 5, we have lim x3 + x = P (4). But P (4) = 43 + 4 =
x→4
64 + 4 = 68. So lim x3 + x = 68.
x→4
(b) Solution.
lim x5 − 4x3 − 7x + 4 = (2)5 − 4(2)3 − 7(2) + 4 Rule 5
x→2
= 32 − 4(8) − 7(2) + 4
= 32 − 32 − 14 + 4 = −10

(c) Solution. The function f (x) = −x2 + 5 x is not a polynomial because the
√ 1
exponent of x = x 2 is not an integer. So we cannot use Rule 5. Instead, we
are going to use Rule 2 to break the limit into two limits.
√ √
lim − x2 + 5 x = lim (−x2 ) + lim (5 x) Rule 2
x→9 x→9 x→9

= −(9)2 + lim (5 x) Rule 5
x→9
1
= −81 + 5lim (x) 2 Rule 1
x→9
  12
= −81 + 5 lim x Rule 6
x→9
98 Chapter 3. Limits and Derivatives

1
= −81 + 5(9) 2 Rule 5
= −81 + 5(3)
= −81 + 15 = −66

(d) Solution.

9x2 − 4 lim 9x2 − 4


x→1
lim = Rule 4
x→1 x + 1 lim x + 1
x→1

9(1)2 − 4
= Rule 5
1+1
9−4 5
= =
2 2
(e) Solution.

x4 − 3x2 + 1 lim x4 − 3x2 + 1


x→2
lim √ = √ Rule 4
x→2 x + 98 lim x + 98
x→2

(2)4 − 3(2)2 + 1
= √ Rule 5 and Rule 6
2 + 98
16 − 12 + 1
= √
100
5 1
= =
10 2
(f ) Solution.
√ √ 
(x2 − 3x − 8) 3 2x − 3 lim (x2 − 3x − 8) 3 2x − 3
lim = x→5 Rule 4
x→5 x−1 lim (x − 1)
x→5
  √ 
lim (x2 − 3x − 8) lim 3 2x − 3
= x→5 x→5
Rule 3
lim (x − 1)
x→5
p
(52 − 3(5) − 8) 3 2(5) − 3
= Rule 5 and Rule 6
5−1

(25 − 15 − 8) 3 10 − 3
=
4
2√ 3 1√ 3
= 7= 7
4 2

(g) Solution. The limit lim x2 − 16 does not exist because the limit
x→3

lim (x2 − 16) = 9 − 16 = −7


x→3

is a negative number and the square root of a negative number is undefined. is


a negative number and the square root of a negative number is undefined.
3.2. Finding Limits Using Rules and One-Sided Limits 99

(h) Solution.

3 +x lim −2x3 +x
lim e−2x = ex→0 Rule 8
x→0
3
= e−2(0) +0 Rule 5
= e0 = 1

(i) Solution.
!
x2 + x x2 + x
 
lim ln = ln lim1 Rule 9
x→ 21 2x + 3 x→ 2 2x + 3
1 1
+
= ln 4 2 Rule 4 and Rule 5
1+3
3  
4
3
= ln = ln
4 16

2. [Exercise on page 13]

(a) Solution. Here we cannot use Rule 4 because lim (x − 4) = 4 − 4 = 0. Using


x→4
x2 −16 (x−4)(x+4)
algebra, we can rewrite the function as x−4
= x−4
= x + 4. The latter
equality holds for x 6= 4. So

x2 − 16
=x+4 for all x 6= 4.
x−4
x2 −16
Using Rule 7, we get lim x−4
= lim (x + 4). This latter limit is equal to
x→4 x→4
x2 −16
4 + 4 = 8 by Rule 5. Thus, lim x−4
= 8.
x→4

0
Indeterminate Form 0

P (x)
Let f (x) = Q(x) be a function where P (x) and Q(x) are both polynomials
(such function is called a rational function). Let a be a number such that
P (a)
P (a) = 0 and Q(a) = 0. Then the quotient Q(a) = 00 is undefined and is
P (x)
called an indeterminate form. To find lim , one can proceed as follows.
x→a Q(x)

i. Factor the numerator and denominator. The common factor on top


and bottom is x − a (or a power of x − a).

ii. Simplify.

iii. Substitute x with a into the simplified function. The result is the
P (x)
limit lim Q(x) .
x→a
100 Chapter 3. Limits and Derivatives

2
(b) Solution. Substituting x with 1 into the function f (x) = x 5x−5
+2x−3
, we get 00 ,
which is undefined. Since the numerator and denominator are both polynomials,
we can factor and simplify.

x2 + 2x − 3 (x − 1)(x + 3) Factor the numerator


lim = lim
x→1 5x − 5 x→1 5(x − 1) and denominator
x+3
= lim Simplify and use Rule 7
x→1 5
1+3
= Substitute x with 1
5
4
=
5

2
(c) Solution. Substituting x with 3 into x x−2x−3
2 −9 , we get the indeterminate form 00 .
Since the numerator and denominator are both polynomials, we need to factor
them and simplify.

x2 − 2x − 3 (x − 3)(x + 1) Factor the numerator


lim 2
= lim
x→3 x −9 x→3 (x − 3)(x + 3) and denominator
x+1
= lim Simplify and use Rule 7
x+3
x→3

3+1
= Substitute x with 3
3+3
4 2
= =
6 3

x2 +3x−10
(d) Solution. Substituting x with −5 into x2 +6x+5
, we get the indeterminate form
0
0
.

x2 + 3x − 10 (x + 5)(x − 2)
lim = lim Factor
x→−5 x2 + 6x + 5 x→−5 (x + 5)(x + 1)

x−2
= lim Simplify
x→−5 x + 1

−5 − 2
= Substitute x with −5
−5 + 1
−7 7
= =
−4 4

2x2 −x−6
(e) Solution. Substituting x with 2 into x2 +2x−8
, we get 00 , which is an indetermi-
3.2. Finding Limits Using Rules and One-Sided Limits 101

nate form.

2x2 − x − 6 (x − 2)(2x + 3)
lim 2
= lim Factor
x→2 x + 2x − 8 x→2 (x − 2)(x + 4)
2x + 3
= lim Simplify
x→2 x + 4

2(2) + 3
= Substitute x with 2
2+4
7
=
6

3x2 +8x−3
(f ) Solution. Substituting x with −3 into x2 +3x
, we get the indeterminate form
0
0
.

3x2 + 8x − 3 (x + 3)(3x − 1)
lim 2
= lim Factor
x→−3 x + 3x x→−3 (x + 3)x
3x − 1
= lim Simplify
x→−3 x
3(−3) − 1
= Substitute x with −3
−3
−10 10
= =
−3 3

1 8x2 +2x−3
(g) Solution. Substituting x with 2
into 2x2 +3x−2
, we get the indeterminate form
0
0
.

x − 12 (8x + 6)

8x2 + 2x − 3
lim 2
= lim1 1
 Factor
x→ 12 2x + 3x − 2 x→ 2 x − 2 (2x + 4)

8x + 6
= lim1 Simplify
x→ 2 2x + 4

8 12 + 6

1
= Substitute x with
2 21 + 4

2
4+6 10
= = =2
1+4 5
102 Chapter 3. Limits and Derivatives

(h) Solution.
 2
x2 + 14x − 15
  
x + 14x − 15
lim log2 = log2 lim Rule 9
x→1 x2 − 1 x→1 x2 − 1
 
(x − 1)(x + 15)
= log2 lim Factor
x→1 (x − 1)(x + 1)
 
x + 15
= log2 lim Simplify
x→1 x + 1
 
1 + 15 Substitute
= log2
1+1 x with 1
 
16
= log2 = log2 (8) = log2 (23 ) = 3
2

(i) Solution.
x2 −x x2 −x
lim x
lim 3 x = 3x→0 Rule 8
x→0
(x−1)x
lim x
= 3x→0 Factor
lim (x−1)
= 3x→0 Simplify
= 30−1 Substitute x with 0
1
= 3−1 =
3
(j) Solution.
1
x
− 18 8−x
a c ad − bc
lim = lim 8x − =
x→8 x − 8 x→8 x − 8 b d bd
a
8−x b a
= lim =
x→8 8x(x − 8) c bc
−(x − 8)
= lim Factor the numerator
x→8 8x(x − 8)
−1
= lim Simplify
x→8 8x

1
=− Substitute x with 8
8(8)
1
=−
64
3. [Exercise on page 14]

(a) Solution. If we substitute x with 0 into √2x2x , we get 00 , which is an indeterminate


form. To solve this, we need to rationalize the denominator because there is a
radical there. The following is a brief review on rationalizing.
3.2. Finding Limits Using Rules and One-Sided Limits 103

Rationalizing (with one term)

The idea of rationalizing is to get rid of the radical in the denominator or


numerator.

ˆ Rationalizing the Denominator (with one term). To ratio-


nalize an expression of the form z√x y , we multiply the numerator and

the denominator by y:
√ √
x x y x y
√ = √ √ = .
z y z( y y) zy

√2 2 3
For example, 3
= 3
.

ˆ Rationalizing the Numerator√


(with one term). To rationalize
z x
an expression of the form y , we multiply the numerator and the

denominator by x:
√ √ √
z x z( x)( x) zx
= √ = √
y y x y x
√ √ √
5 2 5( 2)( 2) 5(2) 10
For example, 3
= √
3 2
= √
3 2
= √ .
3 2

Rationalizing (with two terms)

ˆ The following identity is useful while rationalizing the numerator or


denominator with two terms:

(a − b)(a + b) = a2 − b2 . (3.2.1)

ˆ Rationalizing the Denominator (with two terms). If x and


y are two numbers, the conjugate of x + y is x − y and vice versa.
To rationalize the denominator with two terms, we multiply the nu-
merator and the denominator by the conjugate of the denominator:
√ √ √
x x(y − z) x(y − z) x(y − z)
√ = √ √ = √ =
y+ z (y + z)(y − z) (y)2 − ( z)2 y2 − z
√ √
x x(y + z) x(y + z)
√ = √ √ =
y− z (y − z)(y + z) y2 − z
Suppose we want to rationalize 3−25√7 . We need to multiply the nu-
merator and the
√ denominator by the conjugate of the denominator,
which is 3 + 2 7:
√ √
5 5(3 + 2 7) 5(3 + 2 7)
√ = √ √ = √
3−2 7 (3 − 2 7)(3 + 2 7) 32 − (2 7)2
104 Chapter 3. Limits and Derivatives

√ √
5(3 + 2 7) 5(3 + 2 7)
= =− .
9 − 28 19

ˆ Rationalizing the Numerator (with two terms). To rationalize


the numerator with two terms, we multiply the numerator and the
denominator by the conjugate of the numerator. For example,
√ √ √ √
x−2 ( x − 2)( x + 2) ( x)2 − (2)2 x−4
= √ = √ = √ .
x x( x + 2) x( x + 2) x( x + 2)

We come back to the limit lim+ √2x . We have


x→0 2x


2x 2x 2x
lim √ = lim+ √ √ Rationalize the denominator
x→0+ 2x x→0 ( 2x)( 2x)

2x 2x
= lim+
x→0 2x

= lim+ 2x Simplify 2x
x→0
p
= 2(0) Substitute x with 0
=0

(b) Solution.


x−4 (x − 4)( x + 2)
lim √ = lim √ √ Rationalize the denominator
x→4 x − 2 x→4 ( x − 2)( x + 2)

(x − 4)( x + 2) Distribute the denominator
= lim √
x→4 ( x)2 − (2)2 using (3.2.1)

(x − 4)( x + 2) √ √ √
= lim ( x)2 = ( x)( x) = x
x→4 x−4

= lim x + 2 Simplify
x→4

= 4+2 Substitute x with 4
3.2. Finding Limits Using Rules and One-Sided Limits 105

Alternate Solution
√ √
Using the fact that x − 4 = ( x − 2)( x + 2), we get
√ √
x−4 ( x − 2)( x + 2)
lim √ = lim √
x→4 x − 2 x→4 x−2

= lim x + 2 Simplify
x→4

= 4+2 Substitute x with 4
=2+2=4

(c) Solution.
√ √ √
x−5 ( x − 5)( x + 5)
lim = lim √ Rationalize the numerator
x→25 x − 25 x→25 (x − 25)( x + 5)

( x)2 − (5)2 Distribute the numerator
= lim √
x→25 (x − 25)( x + 5) using (3.2.1)
x − 25
= lim √
x→25 (x − 25)( x + 5)
1
= lim √ Simplify
x→25 x+5
1
=√ Substitute x with 25
25 + 5
1 1
= =
5+5 10

(d) Solution.
√ √ √
3− x (3 − x)(3 + x)
lim = lim 2 √ Rationalize the numerator
x→9 x2 − 81 x→9 (x − 81)(3 + x)
9−x Distribute the numerator
= lim √
x→9 (x2 − 81)(3 + x) using (3.2.1)
9−x
= lim √ Factor x2 − 81
x→9 (x − 9)(x + 9)(3 + x)
−(x − 9)
= lim √ Factor the numerator
x→9 (x − 9)(x + 9)(3 + x)
−1
= lim √ Simplify
x→9 (x + 9)(3 + x)
1
=− √ Substitute x with 9
(9 + 9)(3 + 9)
1 1
=− =−
18(6) 108
106 Chapter 3. Limits and Derivatives

(e) Solution.

√ √ √
4−2 x (4 − 2 x)(4 + 2 x) Rationalize the
lim = lim 2 √
x→4 x2 − x − 12 x→4 (x − x − 12)(4 + 2 x) numerator

(42 ) − (2 x)2 Distribute the numer-
= lim 2 √
x→4 (x − x − 12)(4 + 2 x) ator using (3.2.1)
16 − 4x
= lim √
x→4 (x2 − x − 12)(4 + 2 x)
−4(x − 4) Factor the numerator
= lim √
x→4 (x − 4)(x + 3)(4 + 2 x) and denominator
−4
= lim √ Simplify
x→4 (x + 3)(4 + 2 x)
−4
= √ Substitute x with 4
(4 + 3)(4 + 2 4)
−4 1
= =−
7(8) 14

(f ) Solution.

√ √ √
x2 + 16 − 4 ( x2 + 16 − 4)( x2 + 16 + 4) Rationalize the
lim = lim √
x→0 x2 x→0 x2 ( x2 + 16 + 4) numerator
(x2 + 16) − 16 Distribute the numer-
= lim √
x→0 x2 ( x2 + 16 + 4) ator using (3.2.1)
x2
= lim √
x→0 x2 ( x2 + 16 + 4)
1
= lim √ Simplify
x→0 x2 + 16 + 4
1
=√ Substitute x with 0
2
0 + 16 + 4
1 1 1
=√ = =
16 + 4 4+4 8
3.2. Finding Limits Using Rules and One-Sided Limits 107

(g) Solution.
√ √ p
1 − 1 − x2 (1 − 1 − x2 )(1 + 1 − x2 ) Rationalize the
lim = lim p
x→0 x x→0 x(1 + 1 − x2 ) numerator
1 − (1 − x2 ) Distribute the numer-
= lim √
x→0 x(1 + 1 − x2 ator using (3.2.1)
1 − 1 + x2
= lim p
x→0 x(1 + 1 − x2 )
x2
= lim p
x→0 x(1 + 1 − x2 )
x
= lim √ Simplify
x→0 1 + 1 − x2
0
= √ Substitute x with 0
1 + 1 − 02
0 0 0
= √ = = = 0
1+ 1 1+1 2
(h) Solution.
√ √
x−2 4−2
lim = Substitute x with 4
x→4 −3x + 11 −3(4) + 11
2−2 0
= = =0
−12 + 11 −1
Note. Here we don’t need to rationalize because when we substitute x with 4,
0
we get −1 which is a defined fraction. (A fraction is undefined only when the
denominator is 0.)
√ √
(i) Solution. Substituting x with 2 into f (x) = 2x+5− x−2
x+7
, we get 00 , which is
undefined. To find lim f (x), we need to first rationalize the numerator of f (x).
x→2
√ √ √ √ √ √
2x + 5 − x + 7 ( 2x + 5 − x + 7)( 2x + 5 + x + 7)
lim = lim √ √
x→2 x−2 x→2 (x − 2)( 2x + 5 + x + 7)
(2x + 5) − (x + 7) Distribute the
= lim √ √
x→2 (x − 2)( 2x + 5 + x + 7) numerator
x−2
= lim √ √
x→2 (x − 2)( 2x + 5 + x + 7)
1
= lim √ √ Simplify
x→2 2x + 5 + x + 7
1
=p √ Substitute x with 2
2(2) + 5 + 2 + 7
1 1 1
=√ √ = =
9+ 9 3+3 6
108 Chapter 3. Limits and Derivatives

(j) Solution. Substituting x with 3, we get 00 , which is undefined. As before, we


need to rationalize the denominator first.

x−3 (x − 3)( x2 − 5x + 10 + 2)
lim √ = lim √ √
x→3 x2 − 5x + 10 − 2 x→3 ( x2 − 5x + 10 − 2)( x2 − 5x + 10 + 2)

(x − 3)( x2 − 5x + 10 + 2) Distribute the
= lim
x→3 (x2 − 5x + 10) − 4 denominator

(x − 3)( x2 − 5x + 10 + 2)
= lim
x→3 x2 − 5x + 6

(x − 3)( x2 − 5x + 10 + 2) Factor the de-
= lim
x→3 (x − 2)(x − 3) nominator

x2 − 5x + 10 + 2
= lim Simplify
x→3 x−2
p
32 − 5(3) + 10 + 2
= Substitute x with 3
3−2
√ √
9 − 15 + 10 + 2 4+2 2+2
= = = =4
1 1 1
4. Solution. [Exercise on page 14]

(a) By definition, f (x) = 5 when x = 2. So f (2) = 5.


(b) From the definition of limit, the expression “x → 2” means that x gets closer
and closer to 2 (but is never equal to 2). So in the expression “lim f (x)”, x is
x→2
different from 2. Since f (x) = 1 + x3 for x 6= 2, it follows that

lim f (x) = lim (1 + x3 ) = 1 + (2)3 = 1 + 8 = 9


x→2 x→2

(c) From the definition of g(x), we have g(−3) = −4.



(d) Since g(x) = x2 + 16 for x 6= −3, it follows that
√ p √ √
lim g(x) = lim x2 + 16 = (−3)2 + 16 = 9 + 16 = 25 = 5
x→−3 x→−3

5. Solution. [Exercise on page 14]

(a) The function f has two pieces: x2 + x + 1 for x ≤ 1, and x + 2 for x > 1. (A
function that has more that one piece is called a piecewise function.) To find
f (−2) we use the first piece because −2 ≤ 1. So

f (−2) = (−2)2 + (−2) + 1 = 4 − 2 + 1 = 3

(b) Since 1 ≤ 1, we use the first piece to get f (1).

f (1) = 12 + 1 + 1 = 1 + 2 = 3
3.2. Finding Limits Using Rules and One-Sided Limits 109

(c) To find f (2), we use the second piece (x + 2) since 2 > 1.

f (2) = 2 + 2 = 4

(d) Since the second piece of g(x), x2 − 10, is defined for x ≥ −2, and since 0 ≥ −2,
it follows that
g(0) = 02 − 10 = −10

(e) We want to find lim f (x). When we write x → 1, it means that x can get closer
x→1
to 1 from the left (x < 1) or x can approach 1 from the right (x > 1). Since
f (x) consists of two pieces and the piece to the left of 1 (x2 + x + 1) is different
from the piece to the right of 1 (x + 2), we need to find one-sided limits.
ˆ Limit from the left.

lim f (x) = lim− (x2 + x + 1) = 12 + 1 + 1 = 3


x→1− x→1

ˆ Limit from the right.

lim f (x) = lim+ (x + 2) = 1 + 2 = 3


x→1+ x→1

Since the limit from the left is equal to the limit from the right, it follows that
lim f (x) exists and is equal to 3. That is, lim f (x) = 3.
x→1 x→1

(f ) We need to find one-sided limits.


ˆ Limit from the left.

lim f (x) = lim − (−3 − x) = −3 − (−2) = −3 + 2 = −1


x→−2− x→−2

ˆ Limit from the right.

lim f (x) = lim + (x2 − 10) = (−2)2 − 10 = 4 − 10 = −6


x→−2+ x→−2

Since the limit from the left is not equal to limit from the right, lim g(x) does
x→−2
not exist.

6. Solution. [Exercise on page 14]

(a) The function f consists of three pieces: First piece: x − 5 for x < −2. Second
2
piece: 0 for −2 ≤ x < 4. Third piece: x2 − 8 for x ≥ 4. To find f (−2), we use
the second piece (because −2 ≤ −2 < 4) and we get f (−2) = 0.
(b) Since 4 ≥ 4, f (4) is obtained by substituting x with 4 into the third piece, that
is,
42 16
f (4) = −8= −8=8−8=0
2 2
(c) To find lim f (x), we need to find one-sided limits.
x→−2
110 Chapter 3. Limits and Derivatives

ˆ Limit from the left.

lim f (x) = lim − (x − 5) = −2 − 5 = −7


x→−2− x→−2

ˆ Limit from the right.

lim f (x) = lim + 0 = 0


x→−2+ x→−2

Since the limit from the left does not match the limit from the right, it follows
that lim f (x) does not exist.
x→−2

(d) Again, we need to find one-sided limits.


ˆ Limit from the left.
lim f (x) = lim− 0 = 0
x→4− x→4

ˆ Limit from the right.

x2 42
 
lim f (x) = lim+ −8 = −8=0
x→4+ x→4 2 2

Since lim− f (x) = lim+ f (x) = 0, it follows that lim f (x) = 0.


x→4 x→4 x→4

7. [Exercise on page 15]

(a) Solution.

Absolute Value
The absolute value of a number x, demoted |x|, is defined as x if x ≥ 0.
For x < 0, the absolute value of x is defined to be −x. So
(
x if x ≥ 0
|x| =
−x if x < 0

For example, |5| = 5, |0| = 0, | − 3| = −(−3) = 3.


Note. By definition, the absolute value of a number is always greater
than or equal to 0.
To take the limit of a function that contains an absolute value symbol,
one can proceed as follows.

Step 1. Remove all the absolute values by using the definition and
rewrite the function as a piecewise function.

Step 2. Find one-sided limits and conclude.

We want to find lim |x − 2|.


x→2
3.2. Finding Limits Using Rules and One-Sided Limits 111

ˆ Let f (x) = |x − 2|. First, we need to take away the absolute value symbol.
By definition,

 
x−2 if x − 2 ≥ 0 x−2 if x − 2 ≥ 0
f (x) = =
−(x − 2) if x − 2 < 0 −x + 2 if x − 2 < 0

x−2 if x ≥ 2
=
−x + 2 if x < 2

ˆ The one-sided limits are

lim f (x) = lim− (−x + 2) = −(2) + 2 = 0


x→2− x→2

lim f (x) = lim+ (x − 2) = (2) − 2 = 0


x→2+ x→2

Since these limits are equal, it follows that lim |x − 2| = 0.


x→2

(b) Let f (x) = 3x + |x + 6|. Then

 
3x + (x + 6) if x + 6 ≥ 0 3x + x + 6 if x + 6 ≥ 0
f (x) = =
3x + (−(x + 6)) if x + 6 < 0 3x − x − 6 if x + 6 < 0
 
4x + 6 if x + 6 ≥ 0 4x + 6 if x ≥ −6
= =
2x − 6 if x + 6 < 0 2x − 6 if x < −6

The one-sided limits are

lim f (x) = lim − (2x − 6) = 2(−6) − 6 = −12 − 6 = −18


x→−6− x→−6

lim f (x) = lim + (4x + 6) = 4(−6) + 6 = −24 + 6 = −18


x→−6+ x→−6

Since these limits match, lim 3x + |x + 6| = −18.


x→−6
x2 +2x−3
(c) First we need to write the function f (x) = |x−1|
without the absolute value
symbol.
( 2 (
x +2x−3 x2 +2x−3
x−1
if x−1≥0 x−1
if x − 1 ≥ 0
f (x) = x2 +2x−3 = x2 +2x−3
−(x−1)
if x−1<0 −x+1
if x − 1 < 0
( 2
x +2x−3
x−1
if x≥1
= x2 +2x−3
−x+1
if x<1

Now we find the one-sided limits.


x2 + 2x − 3 (x − 1)(x + 3)
lim− f (x) = lim− = lim− Factor
x→1 x→1 −x + 1 x→1 −(x − 1)
112 Chapter 3. Limits and Derivatives

x+3
= lim− Simplify
x→1 −1
1+3
= = −4 Substitute x with 1
−1

x2 + 2x − 3 (x − 1)(x + 3)
lim+ f (x) = lim+ = lim+ Factor
x→1 x→1 x−1 x→1 x−1
x+3
= lim+ Simplify
x→1 1
1+3
= =4 Substitute x with 1
1

Since the limit from the left is not equal to the limit from the right, it follows
2 +2x−3
that lim x |x−1| does not exist.
x→1

3.3 Infinite Limits and Vertical Asymptotes


1. [Exercise on page 15]
x+1
Solution. Substituting x with 3 into −3x+9 , we get 04 , which is undefined. To solve
x+1
such a problem, we need to find the one sided-limits of −3x+9 . In general, we have
the following.
k
Limits of the Form 0
, k 6= 0

Suppose we want to find lim f (x). If we substitute x with a and we run into
x→a
a problem of the form k0 , where k 6= 0, we have to find one-sided limits. To do
this, one can perform the so-called Sign Analysis of f (x). (This is illustrated
below step by step.)

ˆ If f (x) is negative when x is close to a and x < a, then the limit from
the left is lim− f (x) = −∞.
x→a

ˆ If f (x) is positive when x is close to a and x < a, then the limit from the
left is lim− f (x) = ∞.
x→a

ˆ If f (x) is negative when x is close to a and x > a, then the limit from
the right is lim+ f (x) = −∞.
x→a

ˆ If f (x) is positive when x is close to a and x > a, then the limit from the
right is lim+ f (x) = ∞.
x→a

x+1
Let us explain how to perform the sign analysis with the limit lim .
x→3 −3x+9
3.3. Infinite Limits and Vertical Asymptotes 113

Step 1. Find the numbers that make the numerator 0. Set x + 1 = 0.


Solving this for x, we get x = −1.
Step 2. Find the numbers that make the denominator 0. Set −3x + 9 = 0.
Solving this for x, we get x = 3.
Step 3. Mark the numbers found in steps 1 and 2 on the real line. We
get Figure 3.1.

−∞ −1 3 ∞

Figure 3.1: Marking the points −1 and 3 on the real line

Step 4. Find the sign of f (x). The numbers −1 and 3 define three intervals:
(−∞, −1), (−1, 3), and (3, ∞). The goal of this step is to find the sign
of f (x) on each interval. To get the sign on a given interval, choose an
arbitrary number in that interval. Then substitute that number into the
function. If the result is positive then the function is positive on the
interval. If the result is negative, then the function is negative on the
x+1
interval. Let us illustrate this with our function f (x) = −3x+9 .
ˆ Finding the sign of f (x) on the interval (−∞, −1). Choose for exam-
−2+1
ple −2. Then f (−2) = −3(−2)+9 = −1
15
. Since the result is negative,
the function f (x) is negative on the interval (−∞, −1) as shown in
Figure 3.2.


−∞ −1 3 ∞
choose −2
−1
f (−2) = 15
<0
x+1
Figure 3.2: Sign of f (x) = −3x+9
on the interval (−∞, −1)

ˆ Finding the sign of f (x) on the interval (−1, 3). Choose for example
0+1
0. Then f (0) = −3(0)+9 = 19 > 0. Because the result is positive, the
function is positive on the interval (−1, 3) as shown in Figure 3.3.

+
−∞ −1 3 ∞
choose 0
1
f (0) = 9
>0
x+1
Figure 3.3: Sign of f (x) = −3x+9
on the interval (−1, 3)
114 Chapter 3. Limits and Derivatives


−∞ −1 3 ∞
choose 4
f (4) = − 35 < 0
x+1
Figure 3.4: Sign of f (x) = −3x+9
on the interval (3, ∞)

ˆ Finding the sign of f (x) on the interval (3, ∞). Choose for example
4+1 5
4. Then f (4) = −3(4)+9 = −3 < 0. Since the result is negative, the
function is negative on the interval (3, ∞) as shown in Figure 3.4.
Putting all these together, we get the sign of f (x) as shown in Figure 3.5.

− + −
−∞ −1 3 ∞

x+1
Figure 3.5: Sign Analysis for f (x) = −3x+9

Step 5. Find the one-sided limits and conclude.


ˆ Limit from the left. When x gets closer and closer to 3 from the left
(that is, x approaches 3 and x < 3), the denominator −3x + 9 gets
1
closer and closer to 0, and therefore −3x+9 gets bigger and bigger. On
the other hand, when x gets closer and closer to 3, the numerator,
1 x+1
x+1, gets closer and closer to 4. So the product (x+1)× x−3 = −3x+9
gets bigger and bigger as x approaches 3 from the left. Because the
x+1
function f (x) = −3x+9 is positive when x → 3− (thanks to the sign
analysis), we have that

x+1
lim− = ∞.
x→3 −3x + 9

ˆ Limit from the right. The sign analysis shows that f (x) is negative
when x is close to 3 and x > 3. So a similar reasoning as before
enables us to conclude that
x+1
lim+ = −∞.
x→3 −3x + 9

Conclusion: Because the limit from the left is not equal to the limit from the right,
x+1
we can conclude that lim −3x+9 does not exist.
x→3

2. [Exercise on page 15]


4−x
Solution. Substituting x with 2 into f (x) = x2 −4x+4 , we get 20 , which is a problem
of the form k0 . So we need to perform the sign analysis of f (x).
3.3. Infinite Limits and Vertical Asymptotes 115

ˆ Finding the numbers that make the numerator 0. Set 4 − x = 0. Solving this
equation for x, we get x = 4.
ˆ Finding the numbers that make the denominator 0. Set x2 −4x+4 = 0. Factoring
the left hand side, we get (x − 2)(x − 2) = 0. This implies that x = 2.
ˆ Marking the numbers 2 and 4 on the real line, we get Figure 3.6.

−∞ 2 4 ∞

Figure 3.6

ˆ The sign analysis of f (x) is shown in Figure 3.7.

+ + −
−∞ 2 4 ∞
choose 0 choose 3 choose 5
4 1
f (0) = 4
>0 f (3) = 1
>0 f (5) = − 19 < 0
4−x
Figure 3.7: Sign Analysis for f (x) = x2 −4x+4

ˆ The sign analysis tells us that the one-sided limits are


4−x 4−x
lim− =∞ and lim+ = ∞.
x→2 x2 − 4x + 4 x→2 x2 − 4x + 4
Since the one-sided limits are both equal to ∞, it follows that
4−x
lim = ∞.
x→2 x2 − 4x + 4
3. [Exercise on page 15]
−5 −5
Solution. Substituting x with −2 into (x+2) 2 , we get 0
, which is a problem of the
k
form 0 . So we need to perform the sign analysis.
ˆ Finding the numbers that make the numerator 0. The numerator is −5, which
is different from 0 for any x. So there is no value of x for which the numerator
is 0.
ˆ Finding the numbers that make the denominator 0. Set (x + 2)2 = 0. Solving
this for x, we get x = −2.
ˆ The sign analysis of f (x) = −5
(x+2)2
is shown in Figure 3.8.
ˆ From the sign analysis, we deduce the one-sided limits:
−5 −5
lim − = −∞ and lim + = −∞.
x→−2 (x + 2)2 x→−2 (x + 2)2
Since the one-sided limits are both equal to −∞, it follows that
−5
lim = −∞.
x→−2 (x + 2)2
116 Chapter 3. Limits and Derivatives

− −
−∞ −2 ∞
choose −3 choose 0
f (−3) = − 51 < 0 f (0) = − 54 < 0
−5
Figure 3.8: Sign Analysis for f (x) = (x+2)2

Alternate Solution
−5
For the function f (x) = (x+2)2 , the numerator, −5, is always negative. And the
2
denominator, (x+2) , is a perfect square; so it always positive for every x 6= −2.
−5
So the fraction (x+2)2 is always negative for every x 6= −2. And therefore the
−5
limit from either side is −∞. This implies that lim (x+2) 2 = −∞.
x→−2

4. [Exercise on page 15]


Solution. Substituting x with 0, we get 10 , which is a problem of the form k0 . Since
the numerator, 1, is always positive. And the denominator, x2 , is always positive for
every x 6= 0, it follows that the function x12 is positive for every x 6= 0. Therefore the
limit from either side is ∞. This implies that lim x12 = ∞.
x→0

5. [Exercise on page 15]


Solution. Substituting x with 1, we get 00 , which is a problem of form 00 . Since
the limit is of the form 00 , we have to use algebra to simplify the fraction. Since the
numerator and denominator are both polynomials, we can factor and simplify. (If
there was a radical symbol, we would have rationalized first and then simplified.)

x2 + 3x − 4 (x − 1)(x + 4)
lim 2
= lim Factor
x→1 x − 2x + 1 x→1 (x − 1)2
x+4
= lim Simplify
x→1 x − 1

1+4
= Substitute x with 1
1−1
5 k
= Problem of the form
0 0

Because we have a problem of the form k0 , we need to find the one-sided limits and see
if they are equal or not. First, we need to perform the sign analysis of the function
f (x) = x+4
x−1
.

ˆ The numerator is 0 when x = −4.


ˆ The denominator is 0 when x = 1.
ˆ The sign analysis of f (x) = x+4
x−1
is shown in Figure 3.9.
3.3. Infinite Limits and Vertical Asymptotes 117

+ − +
−∞ −4 1 ∞
choose −5 choose 0 choose 2
−1 4 6
f (−5) = −6
>0 f (0) = −1
<0 f (2) = 1
>0
x+4
Figure 3.9: Sign Analysis for f (x) = x−1

From the sign analysis, we deduce the one-sided limits:

x+4 x+4
lim− = −∞ and lim+ = ∞.
x→1 x−1 x→1 x−1
x+4
Since the one-sided limits do not match, lim does not exist.
x→1 x−1

6. [Exercise on page 15]

7x 7(5) 35
(a) Substituting x with 5 into f (x) = (x−5) 3 , we get (5−5)3 = 0 , which is a problem

of the form k0 . So we need to find the sign analysis of f (x).


ˆ The numerator is 0 when 7x = 0 or x = 0.
ˆ The denominator is zero when (x − 5)3 = 0 or x = 5.
ˆ The sign analysis of f (x) is shown in Figure 3.10.

+ − +
−∞ 0 5 ∞
choose −1 choose 3 choose 6
−7 21 42
f (−1) = −216
>0 f (3) = −8
<0 f (6) = 1
>0
7x
Figure 3.10: Sign Analysis for f (x) = (x−5)3

From the sign analysis, we have

7x 7x
lim− = −∞ and lim+ =∞
x→5 (x − 5)3 x→5 (x − 5)3

Since these limits do not match, it follows that lim f (x) does not exist.
x→5

(b) Since lim− f (x) = −∞ (or since lim+ f (x) = ∞), it follows (by the definition
x→5 x→5
of vertical asymptote) that the line x = 5 is a vertical asymptote of the curve
y = f (x).

7. Solution. [Exercise on page 15]


118 Chapter 3. Limits and Derivatives

How to Find the Vertical Asymptotes

Let y = f (x) be a function of the form f (x) = p(x)


q(x)
, where p(x) and q(x) are
both functions (not necessarily polynomials). To find the vertical asymptotes
of f , one can proceed as follows.

Step 1. Set q(x) = 0 and solve for x.

Step 2. For every solution found in Step 1, find the limit of f (x) as x
approaches that solution. Let a be a solution to the equation q(x) =
0.

ˆ If one of the limits lim f (x), lim− f (x), or lim+ f (x) is ±∞,
x→a x→a x→a
then the line x = a is a vertical asymptote of f .
ˆ Otherwise, the line x = a is not a vertical asymptote.

Note. Polynomials have no vertical asymptotes. Indeed, it is not possible to


find a number that makes the denominator of a polynomial zero.

x2 +x−6
We want to find the vertical asymptotes of f (x) = x2 −x−2
.

ˆ Let q(x) be the denominator of f (x). Then q(x) = x2 − x − 2 = (x − 2)(x + 1).


Solving the equation q(x) = 0, we find x = 2 or x = −1.
ˆ Finding the limit as x approaches 2. Substituting x with 2 into f (x), we run
into a problem of the form 00 . To get get rid of this problem, we need to simplify
the function.
x2 + x − 6 (x − 2)(x + 3)
lim f (x) = lim = lim Factor
x→2 x→2 x2 − x − 2 x→2 (x − 2)(x + 1)

x+3
= lim Simplify
x→2 x + 1

2+3 5
= = Substitute x with 2
2+1 3
Since lim f (x) is a number, it follows that the line x = 2 is not a vertical
x→2
asymptote.
ˆ Finding the limit as x approaches −1. Substituting x with −1 into f (x), we
run into a problem of the form k0 with k 6= 0 (actually k = −6). To solve this
problem, we need to perform the sign analysis of f (x).
The numerator of f (x) is 0 when x = 2 or x = −3. And the denomina-
tor is 0 when x = 2 or x = −1. These numbers determine four intervals:
(−∞, −3), (−3, −1), (−1, 2), and (2, ∞). The sign of f (x) in each of these in-
tervals is shown in Figure 3.11.
From the sign analysis, we have

lim f (x) = −∞ and lim f (x) = ∞


x→−1− x→−1+
3.4. Limits at Infinity and Horizontal Asymptotes 119

+ − + +
−∞ −3 −1 2 ∞
choose −4 choose −2 choose 0 choose 3
1 3
f (−4) = 3
>0 f (−2) = −1 < 0 f (0) = 3 > 0 f (3) = 2
>0

x2 +x−6
Figure 3.11: Sign Analysis of f (x) = x2 −x−2

Since lim f (x) = −∞ (or since lim + f (x) = ∞), it follows that the line
x→−1− x→−1
x = −1 is a vertical asymptote of the curve y = f (x).

3.4 Limits at Infinity and Horizontal Asymptotes


1. Solution. [Exercise on page 15]

(a) In section 3.2, we used nine rules to find the limits of various functions as x
approaches a number. In this section we are interested in limits at infinity. We
will need the following rules.

Rules for Limits (continued)

10. Let n be a positive real number. Then

lim xn = ∞.
x→∞

11. Let n be a natural number, n > 0. Then



n −∞ if n is odd
lim x =
x→−∞ ∞ if n is even

12. Let n be a positive real number. Then


1 1
lim =0 and lim = 0.
x→∞ xn x→−∞ xn

Using Rule 10, we get lim x4 = ∞.


x→∞
5
(b) lim x = ∞ by Rule 10.
x→∞

(c) Using Rule 11, we get lim x8 = ∞.


x→−∞

(d) Using Rule 11, we get lim x7 = −∞.


x→−∞

(e)

3
1
lim x = lim x 3
x→∞ x→∞
120 Chapter 3. Limits and Derivatives

=∞ Rule 10

Some Properties for Limits at Infinity


Let k be a real number, k 6= 0. Let a be a real number or a = −∞ or
a = ∞, and let f (x) be a function.

(i) If k > 0 and lim f (x) = ∞, then lim kf (x) = ∞.


x→a x→a

(ii) If k > 0 and lim f (x) = −∞, then lim kf (x) = −∞.
x→a x→a

(iii) If k < 0 and lim f (x) = ∞, then lim kf (x) = −∞.


x→a x→a

(iv) If k < 0 and lim f (x) = −∞, then lim kf (x) = ∞.


x→a x→a

(f ) For the limit lim 3x2 , we have k = 3 and lim x2 = ∞. Using Property (i), we
x→∞ x→∞
have that lim 3x2 = ∞,
x→∞

(g) Using Property (iii), we get lim − 2x10 = −∞.


x→∞

(h) Using Property (iii) again, we get lim − 5x10 = −∞.


x→−∞

(i) Using Property (ii), we get lim 5x9 = −∞.


x→−∞

(j) Using Property (iv), we get lim − 6x31 = ∞.


x→−∞

2. Solution. [Exercise on page 15]

Limits of Polynomials at Infinity

Let P (x) be a polynomial. Then the limit of P (x) is the limit of the term with
respect to the highest power. That is, if P (x) = an xn +an−1 xn−1 +· · ·+a1 x+a0
is a polynomial of degree n, then

lim P (x) = lim an xn and lim P (x) = lim an xn .


x→∞ x→∞ x→−∞ x→−∞

(a) Using this, we get lim x5 + x3 + 1 = lim x5 = ∞.


x→∞ x→∞
3 5 5
(b) lim 2x − x + 7x − 1 = lim 7x = ∞.
x→∞ x→∞

(c) lim − 4x + x − 1 = lim − 4x3 = −∞.


3
x→∞ x→∞
2 2020
(d) lim 1 − x + x = lim x2020 = ∞.
x→−∞ x→−∞

(e) lim 1 − 8x + 2x = lim − 8x5 = ∞.


5 2
x→−∞ x→−∞

3. [Exercise on page 16]


3.4. Limits at Infinity and Horizontal Asymptotes 121

1
(a) Solution. Using Rule 12, we get lim 4 = 0.
x→−∞ x

(b) Solution.
5 5 1
lim 9
= lim 9
x→−∞ 3x 3 x→−∞ x
5
= (0) Rule 12
3
=0

(c) Solution. Using Rule 12, we have lim √1 = lim 1


1 = 0.
x→∞ x x→∞ x 2

(d) Solution.

Limits of Rational Functions at Infinity


To find the limit of a rational function, one can proceed as follows.

Step 1. Divide each term on top and bottom by the highest power of
x in the denominator.

Step 2. Simplify as much as possible.

Step 3. Use the rules for limits, including the rules for limits at infin-
ity.

x2 +1
For the limit lim3x 2 +x+1 , the highest power of x in the denominator is x2 . So
x→∞
we need to divide each term by x2 .

x 2
x2 + 1 x2
+ x12
lim = lim 2 Divide each term by x2
x→∞ 3x2 + x + 1 x→∞ 3x + x + 1
x2 x2 x2

1 + x12
= lim Simplify
x→∞ 3 + 1 + 12
x x

lim 1 + x12

x→∞
= 1 1
 Rule 4
lim 3+ x
+ x2
x→∞

lim (1) + lim x12



x→∞ x→∞
= Rule 2
lim (3) + lim x1 + lim 1
 
x→∞ x→∞ x→∞ x2

1+0
= Rule 1 and Rule 12
3+0+0
1
=
3

7x3
(e) Solution. For the limit lim 3 , the highest power of x in the denominator
x→−∞ −4
x
122 Chapter 3. Limits and Derivatives

is x3 . So we need to divide each term by x3 .

7x 3
7x3 3
lim 3 = lim x3 x Divide each term by x3
x→−∞ x − 4 x→−∞ − 4
x3 x3
7
= lim Simplify
x→−∞ 1 − x43
lim (7)
x→−∞
= 4
 Rule 4
lim 1− x3
x→−∞

lim (7)
x→−∞
= 4
 Rule 2
lim (1) − lim x3
x→−∞ x→−∞

lim (7)
x→−∞
= 1
 Rule 1
lim (1) − 4 lim x3
x→−∞ x→−∞

7
= Rule 1 and Rule 12
1 − 4(0)
7
= =7
1−0

(f ) Solution.

−3x 5
−3x + 5 x2
+ x2
lim = lim x2
Divide each term by x2
x→−∞ x2 − x + 1 x→−∞ x2
− x
x2
+ 1
x2
−3
+ x52 x
= lim Simplify
x→−∞ 1 − 1 + 12
x x
−3 5

lim x
+ x2
x→−∞
= 1 1
 Rule 4
lim 1− x
+ x2
x→−∞

0+0 Rule 2, Rule 1, and


=
1−0+0 Rule 12
0
= =0
1
3.4. Limits at Infinity and Horizontal Asymptotes 123

(g) Solution.

3x 2
3x2 − 2x x3
− 2x
x3
lim = lim 3 2 Divide each term by x3
x→∞ −5x3 + x2 − x x→∞ − 5x + x − x
x3 x3 x3
3
− x22 x
= lim Simplify
x→∞ −5 + 1 − 1
x x2
3 2

lim x
− x2
x→∞
= 1 1
 Rule 4
lim −5 + x
− x2
x→∞

0−0 Rule 2, Rule 1, and


=
−5 + 0 − 0 Rule 12
0
= =0
−5

(h) Solution.

2x3 2
2x3 − x2 + 3 x2
− xx2 + x32 Divide each
lim = lim
x→∞ −4x2 + 3x − 1 x→∞ −4x2
+ 3x − x12 term by x2
x2 x2

2x − 1 + x32
= lim Simplify
x→∞ −4 + x3 − x12
3

lim 2x − 1 + x2
x→∞
= 3 1
 Rule 4
lim −4 + x
− x2
x→∞

lim (2x − 1) + lim x32



x→∞ x→∞
= 3
 3
 Rule 2
lim (−4) + lim x − lim x
x→∞ x→∞ x→∞

lim (2x − 1) + 0
x→∞
= Rule 12
−4 + 0 − 0
lim (2x − 1) 1
x→∞
= = − lim (2x − 1)
−4 4 x→∞
1 1
= − lim (2x) = − (∞) = −∞
4 x→∞ 4
124 Chapter 3. Limits and Derivatives

Solution.
x7 x3
x 7 − x3 + 1 x4
− x4
+ x14 Divide each
lim = lim
x→−∞ −x4 + 3x2 + 8 x→−∞ −x4
+ 3x2
+ x84 term by x4
x4 x4

x3 − x1 + x14
= lim Simplify
x→−∞ −1 + x32 + x84
1 1
 
lim (x3 ) − lim + lim
x x4
x→−∞ x→−∞ x→−∞ Rule 4 and
= 3
 8

lim (−1) + lim x2
+ lim x4
Rule 2
x→−∞ x→−∞ x→−∞

lim (x3 ) − 0 + 0
x→−∞
= Rule 12
−1 + 0 + 0
lim (x3 )
x→−∞
=
−1
−∞
= =∞
−1

(i) Solution.
√ 1
x2 2
x + x2 x2
+ xx2
lim = lim 2x x2 Divide each term by x2
x→∞ 2x − x2 x→∞ − x2
x2
1
3 +1
= lim x22 Simplify
x→∞
x
− 1
0+1
= Rule 4, Rule 2, and Rule 12
0−1
1
= = −1
−1

4. [Exercise on page 16]

(a) Solution. We will use the following fact.

Square Root of a Perfect Square

ˆ The absolute value of a real number x, denoted |x|, is defined as



x if x ≥ 0
|x| =
−x if x < 0

For example, |3| = 3, |0| = 0, | − 2| = −(−2) = 2, | − 15| = 15.

ˆ If x is a real number, then the square root of x2 is the absolute value


of x. That is, √
x2 = |x|.
3.4. Limits at Infinity and Horizontal Asymptotes 125

ˆ The square root of x4 is x2 . Indeed,


√ p
x4 = (x2 )2 = |x2 |
= x2 Because x2 ≥ 0 for all x


For the limit lim x2 + 3 − x, we first need to rationalize.
x→∞

√ √
√ ( x2 + 3 − x)( x2 + 3 + x)
lim x2 + 3 − x = lim √
x→∞ x→∞ x2 + 3 + x
(x2 + 3) − x2
= lim √
x→∞ x2 + 3 + x
3
= lim √ .
x→∞ x2 + 3 + x

From there, we can use the same method as above (that is, divide each term by
the largest power of x in the denominator). It turns out that this does not work
very well when dealing with radicals and limits as x goes to ±∞. Luckily, there
is another technique, that consists of pulling out the largest power
√ of x in the
2
numerator and denominator, and then simplify. For example, x + 3 becomes

s   √ r r r
3 3 3 3
x2 1 + 2 = x2 1 + 2 = |x| 1 + 2 = x 1 + 2
x x x x

The latter equality follows from the fact that when x goes to positive infinity, x
is positive, and therefore |x| = x. (If it was x → −∞, then |x| = −x.) So the
limit above becomes

√ 3
lim x2 + 3 − x = lim q
x→∞ x→∞
x 1 + x32 + x
3
= lim q  Factor the denominator
x→∞ 3
x 1+ x2
+1
3
= lim q x
x→∞
1 + x32 + 1
0 0
=√ = =0 Use the rules
1+0+1 2
126 Chapter 3. Limits and Derivatives

(b) Solution. First, we need to rationalize.


√ √
√ ( x 2 + x + 1 − x)( x2 + x + 1 + x)
lim x2 + x + 1 − x = lim √
x→∞ x→∞ x2 + x + 1 + x
x2 + x + 1 − x2
= lim √ Distribute the numerator
x→∞ x2 + x + 1 + x

x+1
= lim √
x→∞ x2 + x + 1 + x

x 1 + x1
 Pull out x on the nu-
= lim q merator and x2 in the
x→∞
x2 1 + xx2 + x12 + x

radical symbol

x 1 + x1

= lim √ q
x→∞
x2 1 + xx2 + x12 + x



x 1 + x1

x2 = |x| = x (x → ∞ im-
= lim q
x→∞
x 1 + xx2 + x12 + x plies that x > 0)

1
1+ x
= lim q Simplify
x→∞ 1 1
1+ x
+ x2
+1

1+0
=√ Use the rules
1+0+0+1
1 1
=√ =
1+1 2
5. [Exercise on page 16]
3
(a) f (x) = 4x +1
x3 −x
Solution.

Horizontal Asymptotes
Let f be a function, and let b be a real number.

ˆ The line y = b is a horizontal asymptote of f (x) if lim f (x) = b or


x→∞
lim f (x) = b.
x→−∞

ˆ To find the horizontal asymptotes of f (x) , we have to calculate two


limits: lim f (x) and lim f (x). If one of these is a real number, then
x→∞ x→−∞
there is a horizontal asymptote. Otherwise, there is no horizontal
asymptote.

4x3 +1
Consider the function f (x) = x3 −x
.
3.5. Continuity 127

ˆ Finding the limit lim f (x).


x→∞
4x 3
4x3 + 1 x3
+ x13 4+ 1
x3 4+0
lim f (x) = lim 3 = lim x3 = lim 1 = = 4.
x→∞ x→∞ x − x x→∞ − xx3 x→∞ 1 − 1−0
x3 x2

ˆ Similarly, one has lim f (x) = 4.


x→−∞
So the line y = 4 is a horizontal asymptote of f (x).
(b) Solution. We need to find lim f (x) and lim f (x).
x→∞ x→−∞
x 1
x x2 x 0
lim f (x) = lim = lim x2
= lim 1 = = 0.
x→∞ x→∞ x2 + 1 x→∞ x2
+ 1
x2
x→∞ 1+ x2
1+0
Similarly, one has lim f (x) = 0. So the line y = 0 is a horizontal asymptote
x→−∞
of f (x).
(c) Solution. We need to find lim f (x) and lim f (x).
x→∞ x→−∞

x3
x3 x2 x ∞
lim f (x) = lim 2 = lim x2 1
= lim 1 = = ∞.
x→∞ x→∞ x + 1 x→∞ + x→∞ 1 + x2 1+0
x2 x2

Likewise, lim f (x) = −∞. Since these are not real numbers, there is
x→−∞
no horizontal asymptote.

6. [Exercise on page 16]


Solution.
ˆ Finding the limit.
C(x) C(x)
lim C(x) = lim C(x) =
x→∞ x→∞ x x
13x + 8000
= lim C(x) = 13x + 8000
x→∞ x
13x 8000
x
+ x
= lim x Divide each term by x
x→∞
x

13 + 8000
x
= lim Simplify
x→∞ 1
13 + 0 k
= = 13 lim =0
1 x→∞ x
ˆ Interpreting the answer. The average cost approaches $13 as the number of
items gets larger and larger.

3.5 Continuity
1. [Exercise on page 16]
Solution.
128 Chapter 3. Limits and Derivatives

Definition of Continuity at a Point


A function f is said to be continuous at a point x = c if the following three
conditions are satisfied:

(1) f (c) is defined

(2) lim f (x) exists


x→c

(3) lim f (x) = f (c)


x→c

A function f is discontinuous (or not continuous) at x = a if one of the condi-


tions above is not satisfied. Intuitively, a function f is discontinuous if there is
a hole or gap on the graph of f .

In Figure 3.5, there are four points where the graph is not continuous:

ˆ At x = −5. The function is discontinuous there because condition (1) is not


satisfied as f (−5) is undefined.
ˆ At x = −2.
– Condition (1) is satisfied because f (−2) is defined as f (−2) = 6.
– Condition (2) is satisfied because lim f (x) = 5.
x→−2
– But condition (3) is not satisfied because the limit lim f (x) is not equal
x→−2
to f (−2).
So f is discontinuous at x = −2.
ˆ At x = 2.
– Condition (1) holds since f (2) is defined as f (2) = 4.
– Condition (2) fails because the limit lim f (x) does not exist. Indeed, the
x→2
limit from the left, lim− f (x) = 5, is not equal to the limit from the right,
x→2
lim+ f (x) = 3.
x→2
So f is discontinuous at x = 2.
ˆ At x = 4.
– Condition (1) is satisfied since f (4) is defined as f (4) = −2.
– Condition (2) is not satisfied because the limit lim f (x) does not exist.
x→4
Indeed, the limit from the left (which is lim− f (x) = 1) is not equal to the
x→4
limit from the right (which is lim+ f (x) = −2).
x→4
So f is discontinuous at x = 4.

2. [Exercise on page 16]

ˆ The function f is not continuous at 2 because condition (1) is not satisfied.


2 −4
Indeed, f (2) = 22−2 4−4
= 2−2 = 00 , and 00 is undefined.
3.5. Continuity 129

ˆ Regarding g(x),
– condition (1) is satisfied because g(2) is defined as g(2) = 3.
– For condition (2), we need to find the limit of g(x) as x approaches 2.

x2 − 4
lim g(x) = lim
x→2 x→2 x−2
(x − 2)(x + 2)
= lim Factor the numerator
x→2 x−2
= lim (x + 2) Simplify
x→2

=2+2=4 Substitute x with 2

Because the limit exists, condition (2) is satisfied.


– Condition (3) is not satisfied since lim g(x) 6= g(2).
x→2

So g is not continuous at 2.
ˆ For the function h(x),
– condition (1) is satisfied because h(2) is defined as h(2) = 4.
– For condition (2), we have

x2 − 4
lim h(x) = lim = 4.
x→2 x→2 x−2

Because the limit exists, condition (2) is satisfied.


– Condition (3) holds since lim h(x) = h(2).
x→2

Since the three conditions are satisfied, it follows that h is continuous at 2.

3. [Exercise on page 17]

(a) Solution. The function f consists of two pieces: x + 2 and x2 . The first piece
is defined for every x less than 0, and the second piece is defined for every x
greater than 0. None of these is defined precisely at x = 0. So the function f
is discontinuous at 0 because it is not defined there. (Condition (1) is not
satisfied.)
(b) Solution.
2
ˆ The function f has two pieces: 2x − 1 and x+1 x
. The first piece is defined
for every x less than or equal to 1. In particular, the first piece is defined
at x = 1. The second piece is not defined at 1 because we have a strict
inequality. So f (1) is defined and f (1) = 2(1) − 1 = 2 − 1 = 1. This implies
that condition (1) is satisfied.
ˆ Checking condition (2). We want to find lim f (x) (if it exists). Since the
x→1
pieceto the left of 1 (2x − 1) is not the same as the piece to the right of 1

x2
x+1
, we need to find the one-sided limits and see if they are equal.
130 Chapter 3. Limits and Derivatives

– Limit from the left. As x approaches 1 from the left, x is less than 1.
So, for the limit from the left, we use the piece 2x − 1:

lim f (x) = lim− (2x − 1) = 2(1) − 1 = 1.


x→1− x→1

– Limit from the right. As x approaches 1 from the right, x is greater


x2
than 1. So, for the limit from the right, we use x+1 :

x2 12 1
lim+ f (x) = lim+ = = .
x→1 x→1 x+1 1+1 2
Since the limit from the left is not equal to the limit from the right, it follows
that lim f (x) does not exist. So f is not continuous at 1.
x→1

(c) Solution.

ˆ Condition
√ (1) is satisfied because f (−2) is defined as f (−2) = −2 + 3 =
1 = 1.
ˆ Checking condition (2). Because the piece to the left of −2 is not the same
as the piece to the right of −2, we need to find one-sided limits.

lim f (x) = lim− x2 + 2x = (−2)2 + 2(−2) = 4 − 4 = 0,


x→−2− x→1

√ √
lim + f (x) = lim + x+3= −2 + 3 = 1.
x→−2 x→−2

Since the limit from the left is not equal to the limit from the right, lim f (x)
x→1
does not exist. Therefore f is not continuous at −2.

(d) Solution.
ˆ Condition (1) is satisfied because f (0) is defined as f (0) = 2.
ˆ Checking condition (2). Because the piece to the left of 0 is not the same
as the piece to the right of 0, we need to find one-sided limits.

lim f (x) = lim− e2x = e2(0) = e0 = 1,


x→0− x→0

lim f (x) = lim+ x3 + 1 = 03 + 1 = 1.


x→0+ x→0

Since the limit from the left equals the limit from the right, the limit of f (x)
as x approaches 0 exists and lim f (x) = 1.
x→0
ˆ Condition (3) is not satisfied because lim f (x) 6= f (0).
x→0

So f is not continuous at 0.
(e) Solution.
ˆ Condition (1) holds because f (3) is defined as

f (3) = −2 + ln(3 − 2) = −2 + ln(1) = −2 + 0 = −2.


3.5. Continuity 131

ˆ Checking condition (2). The limit of f (x) as x approaches 3 from the left is

x2 − 2x − 3
lim− f (x) = lim−
x→3 x→3 −2x + 6
(x − 3)(x + 1)
= lim− Factor the numerator
x→3 −2(x − 3)
x+1
= lim− Simplify
x→3 −2
3+1
= = −2. Substitute x with 3
−2
And the limit from the right is

lim f (x) = lim+ (−2 + ln(x − 2)) = −2 + ln(3 − 2) = −2.


x→3+ x→3

Since the limit from the left equals the limit from the right, the limit of f (x)
as x approaches 3 exists and lim f (x) = −2.
x→3
ˆ Condition (3) is satisfied because lim f (x) = f (3).
x→3
Since all the conditions are satisfied, the function f is continuous at x = 3.
(f ) Solution.
1−1 0
ˆ Condition (1) holds because f (1) is defined as f (1) = e 2 = e2 = 12 .
ˆ Checking condition (2). The limit of f (x) as x approaches 1 from the left is

ex−1 e1−1 1
lim− f (x) = lim− = = .
x→1 x→1 2 2 2
And the limit from the right is

x−1
lim+ f (x) = lim+
x→1 x→1 x−1
√ √
( x − 1)( x + 1)
= lim+ √ Rationalize the numerator
x→1 (x − 1)( x + 1)
x−1
= lim+ √ Distribute the numerator
x→1 (x − 1)( x + 1)
1
= lim+ √ Simplify
x→1 x+1
1 1
=√ = Substitute x with 1
1+1 2

Because the limit from the left equals the limit from the right, the limit of
f (x) as x approaches 1 exists and lim f (x) = 21 .
x→1
ˆ Condition (3) holds because lim f (x) = f (1).
x→1
So the function f is continuous at x = 1.
132 Chapter 3. Limits and Derivatives

4. [Exercise on page 18]


(a) Solution.
ˆ Condition (1) is satisfied because f (4) is defined as f (4) = 0.
ˆ Condition (2):
lim f (x) = lim− log3 (7 − x) = log3 (7 − 4) = log3 (3) = 1.
x→4− x→4

lim f (x) = lim+ (x2 − 4x + 1) = 42 − 4(4) + 1 = 1.


x→4+ x→4

So lim f (x) exists and is equal to 1.


x→4
ˆ Condition (3) is not satisfied because lim f (x) 6= f (4).
x→4
Thus, f is not continuous at 4.
(b) Solution. To make f continuous at 4, the limit has to be equal to f (4). Since
the limit is 1, to make f continuous at 4, f (4) should be equal to 1.
5. [Exercise on page 18]

(a) Solution.
ˆ Condition (1) holds because f (2) is defined (for every k) as f (2) = k(2)2 −
6 = 4k − 6.
ˆ Condition (2):
lim− f (x) = lim− (kx2 − 6) = k(2)2 − 6 = 4k − 6.
x→2 x→2

lim f (x) = lim+ 3(2) + k = 6 + k.


x→2+ x→2
The limit exists if and only the limit from the left equals the limit from the
right. That is,
4k − 6 = 6 + k
3k = 12 Add −k + 6 to both sides
12
k= Divide both sides by 3
3
=4
So condition (2) holds if and only if k = 4.
ˆ Condition (3): When k = 4, we have f (2) = 4k −6 = 4(4)−6 = 16−6 = 10.
On the other hand, we have
lim f (x) = 4k − 6 = 4(4) − 6 = 10
x→2−

and
lim f (x) = 6 + k = 6 + 4 = 10.
x→2+

So lim f (x) = 10 = f (2). And therefore, condition (3) holds for the value
x→2
k = 4.
3.5. Continuity 133

Conclusion: The function f is continuous at 2 if and only if k = 4.


(b) Solution.
ˆ Condition (1) is satisfied for every k because f (−1) is defined as f (−1) =
k(−1) + 5 = −k + 5.
ˆ Condition (2). Since the piece to the left of −1 is the same as the piece to
the right of −1, we don’t need to find one-sided limits.

2x2 − 3x − 5
lim f (x) = lim
x→−1 x→−1 x+1
(x + 1)(2x − 5)
= lim Factor the numerator
x→−1 x+1
2x − 5
= lim Simplify
x→−1 1
= 2(−1) − 5 = −7 Substitute x with −1

So condition (2) holds for every k.


ˆ Condition (3) holds if and only if lim f (x) = f (−1):
x→−1

−7 = −k + 5
−12 = −k Subtract 5 from both sides
12 = k Divide both sides by −1

So condition (3) holds if and only if k = 12.


Conclusion: The function f is continuous at −1 if and only if k = 12.

(c) Solution.
ˆ Condition (1) is satisfied (for every k) because f (1) is defined as f (1) =
k 2 − 5k.
ˆ Condition (2):

lim f (x) = lim− (−2x3 − 4x) = −2(1)3 − 4(1) = −6.


x→1− x→1

And
lim f (x) = lim+ (−6x) = −6(1) = −6.
x→1+ x→1

Since the limit from the left is equal to the limit from the right, it follows
that lim− f (x) exists and is equal to −6. So condition (2) holds (for every
x→1
k).
ˆ Condition (3) holds if and only if lim f (x) = f (1), that is,
x→1

−6 = k 2 − 5k
0 = k 2 − 5k + 6 Add 6 to both sides
k 2 − 5k + 6 = 0
134 Chapter 3. Limits and Derivatives

(k − 2)(k − 3) = 0 Factor
k = 2 or k = 3

So condition (3) holds if and only if k = 2 or k = 3.


Conclusion: The function f is continuous at 1 if and only if k = 2 or k = 3

6. [Exercise on page 18]


Solution.

Continuity on an Open Interval

A function f is continuous on an open interval (a, b) if it is continuous at every


point in the interval.

Continuity From the Right and From the Left

ˆ A function f is said to be continuous from the right at x = c if lim+ f (x) =


x→c
f (c).

ˆ A function f is said to be continuous from the left at x = c if lim− f (x) =


x→c
f (c).

Continuity on a Closed Interval

A function f is said to be continuous on a closed interval [a, b] if it satisfies the


following three conditions:

(1) f is continuous on the open interval (a, b)

(2) f is continuous from the right at x = a

(3) f is continuous from the left at x = b

Consider the function f whose graph is shown in Figure 3.5.

ˆ On the interval (−7, −5). The function is continuous there because, from the
graph, it is continuous at every point between −7 and −5.
ˆ On the interval (−6, −4). The function is not continuous on (−6, −4) because,
from the graph, it is discontinuous at −5 (as f (−5) is undefined).
ˆ On the interval [0, 2].
– Clearly f is continuous on the open interval (0, 2) because it is continuous
at every point in (0, 2).
– The function is continuous from the right at x = 0 because lim+ f (x) = 5 =
x→0
f (0).
3.5. Continuity 135

– The function is not continuous from the left at x = 2 because the limit
lim− f (x) is not equal to f (2). Indeed, lim− f (x) = 5 and f (2) = 4.
x→2 x→2
Thus, f is not continuous on the closed interval [0, 2].
ˆ On the interval [5, 6] the function is continuous because it clearly satisfies the
three conditions above.
ˆ On the interval [4, 6].
– The function is continuous on the open interval (4, 6) because it is continuous
at every point there.
– The function is continuous from the right at x = 4 because lim+ f (x) =
x→4
−2 = f (4).
– The function is continuous from the left at x = 6 because lim− f (x) = 2 =
x→6
f (6).
Thus, f is continuous on the closed interval [4, 6].
ˆ On the interval [3, 4].
– The function is continuous on the open interval (3, 4) because it is continuous
at every point there.
– The function is continuous from the right at x = 3 because lim+ f (x) = 2 =
x→3
f (3).
– The function is not continuous from the left at x = 4 because the limit
lim− f (x) is not equal to f (4). Indeed, lim− f (x) = 1 and f (4) = −2.
x→4 x→4
Hence, f is not continuous on the closed interval [3, 4].

7. [Exercise on page 18]


Solution.

ˆ Checking the continuity on the open interval (−3, 3). Let c be a number in
(−3, 3).

– f (c) = 9 − c2 is defined because c is between −3 and 3.
√ √
– lim f (x) = lim 9 − x2 = 9 − c2 .
x→c x→c
– lim f (x) = f (c).
x→c
Thus f is continuous at c.
ˆ Checking the continuity from the right at x = −3. We have
√ p √
lim + f (x) = lim + 9 − x2 = 9 − (−3)2 = 0 = 0 = f (−3).
x→−3 x→−3

Thus, f is continuous from the right at x = −3.


ˆ Checking the continuity from the left at x = 3. We have
√ p √
lim− f (x) = lim− 9 − x2 = 9 − (−3)2 = 0 = 0 = f (3).
x→3 x→3

Thus, f is continuous from the left at x = 3.


136 Chapter 3. Limits and Derivatives

Hence, f is continuous on the closed interval [−3, 3].

8. [Exercise on page 18]


Solution.

ˆ On the open interval (1, 4), f is continuous because it is a polynomial and


polynomials are continuous everywhere.
ˆ Checking the continuity from the right at x = 1.

lim f (x) = lim+ (x2 − 3x + 8)


x→1+ x→1
2
= 1 − 3(1) + 8 = 1 − 3 + 8 = 6.

On the other hand f (1) = 7(1) − 2 = 5. Since lim+ f (x) 6= f (1), it follows that
x→1
f is not continuous from the right at x = 1.
ˆ Checking the continuity from the left at x = 4.

lim f (x) = lim− (x2 − 3x + 8)


x→4− x→4
2
= 4 − 3(4) + 8 = 16 − 12 + 8 = 12 = f (4).

Thus f is continuous from the left at x = 4.

So f is not continuous on the closed interval [1, 4] because it is not continuous from
the right at x = 1.

9. [Exercise on page 18]


Solution. The absolute value of x, denoted |x|, is defined as

x if x ≥ 0
|x| =
−x if x < 0

ˆ The first piece, x, is continuous for all x > 0 because it is a polynomial.


ˆ The second piece, −x, is continuous for all x < 0 because it is a polynomial.
ˆ We now need to check the continuity at 0. Clearly f is defined at 0 and f (0) = 0.
To check if the limit exists, we need to find one-sided limits.

lim f (x) = lim− (−x) = −(0) = 0,


x→0− x→0

and
lim f (x) = lim+ (x) = 0.
x→0+ x→0

Thus lim f (x) exists and is equal to 0. Moreover, lim f (x) = f (0). So f is
x→0 x→0
continuous at 0.

Conclusion. The function f (x) = |x| is continuous for all x.

10. [Exercise on page 19]


Solution.
3.5. Continuity 137

ˆ The pieces x + 2 and x2 are continuous everywhere (for every k) because they
are polynomials.
ˆ Continuity at x = k.
– The function is defined at k and f (k) = k + 2.
– Finding one-sided limits:
lim f (x) = lim− (x + 2) = k + 2,
x→k− x→k

lim f (x) = lim+ x2 = k 2 .


x→k+ x→k

The limit exists if and only if k 2 = k + 2, that is, k 2 − k − 2 = 0. Factoring


this, we get (k + 1)(k − 2) = 0, which implies that k = −1 or k = 2.
– Clearly, when k = −1 or k = 2, lim f (x) = f (k).
x→k
Hence, f is continuous on R if and only if k = −1 or k = 2.
11. [Exercise on page 19]
Solution.

Continuity of Some Key Functions

ˆ Polynomials. Every polynomial function f (x) is continuous for all x.


In other words, polynomial functions are continuous on the interval R =
(−∞, ∞).
P (x)
ˆ Rational Function. Any rational function, f (x) = Q(x)
, is continuous
for all x except where Q(x) = 0.

ˆ Root
pFunction. If f (x) is continuous on an interval I, then the function
y = f (x) is continuous for all x in I where f (x) ≥ 0.

ˆ Exponential Function. Any exponential function, y = ax , a > 0, is


continuous for all x.

ˆ Logarithmic Function. Any logarithmic function, y = loga x, a >


0, a 6= 1, is continuous for all x > 0.

(a) The function f (x) = x5 − 3x4 + 7x − 1 is continuous for all x because it is a


polynomial.
2
(b) The function g(x) = x2x−16 is continuous for all x except where x2 − 16 = 0.
Since x2 − 16 = (x − 4)(x + 4), the function g is continuous for all x except
at −4 and 4.

(c) The function h(x) = 2x − 6 is continuous for all x where 2x − 6 ≥ 0. Solving
this latter inequality, we get 2x ≥ 6. Dividing both sides by 2, we get x ≥ 3.
Thus, h is continuous for all x ≥ 3.
(d) The function k(x) = e2x is continuous for all x.
(e) First, we recall the following properties of inequalities.
138 Chapter 3. Limits and Derivatives

Multiplying an Inequality by a Number


Let k be a nonzero real number, and let a and b be real numbers.

ˆ If a < b and k is positive, then ka < kb.

ˆ If a < b and k is negative, then ka > kb.

ˆ Considering the other type of inequalities, we have the following


table.

a<b a≤b a>b a≥b


k > 0 ka < kb ka ≤ kb ka > kb ka ≥ kb
k < 0 ka > kb ka ≥ kb ka < kb ka ≤ kb

The function l(x) = ln(−3x + 12) is continuous for all x where −3x + 12 > 0.
We need to solve this inequality.

−3x + 12 > 0
−3x > −12 Subtract 12 from both sides
 
1 1
x < −12 − Multiply both sides by k = −
3 3
x<4

So the function l is continuous for all x < 4.

12. [Exercise on page 19]


Solution.
2x+1
(a) The function f (x) = (2x−3)(x+7) is discontinuous where (2x − 3)(x + 7) = 0.
3
Solving this for x, we get x = 2 or x = −7. Thus, f is discontinuous at −7 and
3
2
.
x3
(b) The function f (x) = x2 −3x+2 is discontinuous where x2 − 3x + 2 = 0. Factoring
this, we get (x − 1)(x − 2) = 0. This latter equation implies that x = 1 or x = 2.
So f is discontinuous at 1 and 2.
|x−6|
(c) The function f (x) = x−6
is discontinuous at x = 6 because the denominator is
zero when x = 6.
(d) Since f (x) = x4 − x + 1 is a polynomial, it is continuous for all x. Thus, there
is no x for which f is discontinuous.

13. [Exercise on page 19]


Solution. Since each piece of the function f is a polynomial, the only x-values where
f might be discontinuous here are 0 and 3.
3.5. Continuity 139

ˆ Checking the continuity at 0.

lim f (x) = lim− (x + 5) = 0 + 5 = 5,


x→0− x→0

lim− f (x) = lim− (x2 − 2x + 4) = 4.


x→0 x→0

Since the limit from the left is not equal to the limit from the right, the function
is not continuous at 0.
ˆ Checking the continuity at 3.

lim f (x) = lim− (x2 − 2x + 4) = 32 − 2(3) + 4 = 7,


x→3− x→4

lim f (x) = lim+ (10 − x) = 10 − 3 = 7.


x→3+ x→3

This shows that the limit exists and lim f (x) = 7. Furthermore, f (3) = 10−3 =
x→4
7 = lim f (x). Thus, f is continuous at 3.
x→3

Conclusion. The function f is discontinuous at x = 0.

14. [Exercise on page 19]


Solution.

The Intermediate Value Theorem


Theorem 3.5.1. Let f be a function. If

(i) f is continuous on a closed interval [a, b], and

(ii) N is a number between f (a) and f (b) (where f (a) 6= f (b)),

then there exists a number c in (a, b) such that f (c) = N .

Conditions (i) and (ii) are called hypotheses. And the sentence “then there
exists a number c in (a, b) such that f (c) = N ” is called the conclusion or
thesis. To apply the theorem, we have to check the hypotheses first.

ˆ Let f (x) = x3 +x−1. Clearly f is continuous on the closed interval [0, 1] because
it is a polynomial.
ˆ Finding the values of f at the endpoints: f (0) = −1 and f (1) = 1 + 1 − 1 = 1.
ˆ Finding N . The original equation is f (x) = 0. So N = 0. (If the equation is
f (x) = B, then N is B.)
ˆ Clearly N is between f (0) and f (1).

So by the Intermediate Value Theorem, there exists c in (0, 1) such that f (c) = 0.

15. [Exercise on page 19]


Solution. Let f (x) = x5 − 2x2
140 Chapter 3. Limits and Derivatives

ˆ The function f is continuous on the closed interval [−1, 2] because it is a poly-


nomial.
ˆ The values of f at the endpoints are f (−1) = (−1)5 − 2(−1)2 = −1 − 2 = −3
and f (2) = 25 − 2(2)2 = 32 − 8 = 24.
ˆ The number N = 7 is between f (−1) and f (2).

So by the Intermediate Value Theorem, there exists c in (−1, 2) such that f (c) = 7.

16. [Exercise on page 19]


Solution.

ˆ If one unit costs $15, x units will cost 15x, and this holds for all x ≤ 280.
ˆ If one unit costs $12, x units will cost 12x, and this holds for all x > 280.
ˆ So the cost function is

15x if x ≤ 280
C(x) =
12x if x > 280

(a) The cost of producing 250 units is C(250) = 15(250) = $3750.


(b) The cost of producing 280 units is C(280) = 15(280) = $4200.
(c) The cost of producing 300 units is C(300) = 12(300) = $3600.
(d) The cost of producing 500 units is C(500) = 12(500) = $6000.
(e) The function C is a piecewise function and each piece is continuous because it
is a polynomial. It remains to check the continuity at 280.

lim C(x) = lim − 15x = 15(280) = 4200


x→280− x→280

lim C(x) = lim + 12x = 12(280) = 3360


x→280+ x→280

Since the limit from the left is not equal to the limit from the right, the function
C is not continuous at x = 280. Thus, C is discontinuous at x = 280.

3.6 Rates of Change


1. [Exercise on page 19]

(a) Solution.

Average Rate of Change


Let f be a function. The average rate of change of f with respect to x
over the interval [a, b] (or as x changes from a to b) is

f (b) − f (a)
b−a
3.6. Rates of Change 141

Here f (x) = 10, a = 1, b = 2. The average rate of change of f (x) over the
interval [1, 2] is
f (2) − f (1) 10 − 10 0
= = =0
2−1 1 1
(b) Solution. On the interval from x = −3 to x = 5, the average rate of f (x) = −π 2
is
f (5) − f (−3) −π 2 − (π 2 ) −π 2 + π 2 0
= = = =0
5 − (−3) 5+3 8 8
(c) Solution. The average rate of change of f (x) = 4x + 5 over the interval [0, 7] is
f (7) − f (0) [4(7) + 5] − [4(0) + 5] (28 + 5) − (0 + 5)
= =
7−0 7 7
33 − 5 28
= = =4
7 7
(d) Solution. First we need to find f (−2) and f (8).
f (−2) = −(−2)2 + 3(−2) + 8 = −(4) − 6 + 8 = −4 − 6 + 8 = −2
f (8) = −(8)2 + 3(8) + 8 = −64 + 24 + 8 = −32
The average rate of change over the interval [−2, 8] is
f (8) − f (−2) −32 − (−2) −32 + 2 −30
= = = = −3
8 − (−2) 8+2 10 10
(e) Solution. First we need to find f (−11) and f (−1).
f (−11) = 6 + (−11) − (−11)3 = 6 − 11 − (−1331) = −5 + 1331 = 1326
f (−1) = 6 + (−1) − (−1)3 = 6 − 1 − (−1) = 5 + 1 = 6
The average rate of change over the interval [−11, −1] is
f (−1) − f (−11) 6 − 1326 6 − 1326 −1320
= = = = −132
−1 − (−11) −1 − (−11) −1 + 11 10

√ On the interval from x = −3 to x = 0, the average rate of change of


(f ) Solution.
f (x) = 2x + 9 is
p p
f (0) − f (−3) 2(0) + 9 − 2(−3) + 9
=
0 − (−3) 0+3
√ √ √
9− 3 3− 3
= = ≈ 0.423
3 3
(g) Solution. The average rate of change of f (x) = e2x over the interval 14 , 12 is
 

1 1 1
f ( 12 ) − f ( 14 ) e2( 2 ) − e2( 4 ) e1 − e 2
1 = =
2
− 41 2
4
− 41 1
4
√  a ac
= 4 e − e ≈ 4.278 b
=
c
b
142 Chapter 3. Limits and Derivatives

(h) Solution. The average rate of change of f (x) = ln(1 + x2 ) over the interval
from x = − 13 to x = 51 is
1
f ( 15 ) − f (− 13 ) − ln 1 + 91
 
ln 1 + 25
1 =
5
− (− 13 ) 1
5
+ 13
26
− ln 10
 
ln 25 9 a c ad + bc
= 3+5 + =
15
b d bd
26
− ln 10
      
ln 25 9 15 26 10
= 8 = ln − ln ≈ −0.124
15
8 25 9

2. [Exercise on page 20]

(a) Solution.

Instantaneous Rate of Change


Let f be a function and a be a number. The instantaneous rate of change
of f at x = a is given by

f (a + h) − f (a)
lim (3.6.1)
h→0 h

Here f (x) = 3x and a = 1. Using (3.6.1), the instantaneous rate of change of


f (x) = 3x at x = 1 is
f (1 + h) − f (1) 3(1 + h) − 3(1)
lim = lim
h→0 h h→0 h
3 + 3h − 3 3h
= lim = lim
h→0 h h→0 h

= lim 3 Simplify
h→0

=3

(b) Solution. The instantaneous rate of change of f (x) = 4x2 at x = 3 is

f (3 + h) − f (3) 4(3 + h)2 − 4(3)2


lim = lim
h→0 h h→0 h
4(9 + 6h + h2 ) − 4(9)
= lim (a + b)2 = a2 + 2ab + b2
h→0 h
36 + 24h + 4h2 − 36 24h + 4h2
= lim = lim
h→0 h h→0 h
h(24 + 4h)
= lim Factor the numerator
h→0 h
= lim (24 + 4h) Simplify
h→0

= 24 + 4(0) = 24 + 0 = 24 Substitute h with 0


3.6. Rates of Change 143

(c) Solution. The instantaneous rate of change of f (x) = −5x2 at x = −1 is

f (−1 + h) − f (−1) −5(−1 + h)2 − [−5(−1)2 ]


lim = lim
h→0 h h→0 h
−5(1 − 2h + h2 ) − (−5) (a+b)2 = a2 +
= lim
h→0 h 2ab + b2
−5 + 10h − 5h2 + 5 10h − 5h2
= lim = lim
h→0 h h→0 h
h(10 − 5h)
= lim Factor the numerator
h→0 h
= lim (10 − 5h) Simplify
h→0

= 10 − 5(0) = 10 + 0 = 10 Substitute h with 0

(d) Solution. The instantaneous rate of change of f (x) = −x2 + 7 at x = −6 is

f (−6 + h) − f (−6) [−(−6 + h)2 + 7] − [−(−6)2 + 7]


lim = lim
h→0 h h→0 h
[−(36 − 12h + h2 ) + 7] − [−(36) + 7]
= lim
h→0 h
2
(−36 + 12h − h + 7) − (−36 + 7)
= lim
h→0 h
2
−29 + 12h − h − (−29)
= lim
h→0 h
−29 + 12h − h2 + 29
= lim
h→0 h
2
12h − h
= lim
h→0 h
h(12 − h)
= lim Factor the numerator
h→0 h
= lim (12 − h) Simplify
h→0

= 12 − 0 = 12 Substitute h with 0

3. [Exercise on page 20]

(a) Solution.

Average Velocity and Instantaneous Velocity

Let s(t) be the position of an object moving along a straight line at time
t.

ˆ The average velocity of the object as t changes from t = t1 to t = t2


is nothing but the average rate of change of the position over the
144 Chapter 3. Limits and Derivatives

interval [t1 , t2 ]. In other words,

s(t2 ) − s(t1 )
Average velocity between t1 and t2 =
t2 − t1

ˆ The instantaneous velocity of the object at time t = t1 is the in-


stantaneous rate of change of the position s when t = t1 . That
is,

s(t1 + h) − s(t1 )
Instantaneous velocity at time t1 = lim
h→0 h

If the position is in kilometers (km) and time is in hours (h), then the unit
of the average (or instantaneous) velocity is km/h.

Here the position is s(t) = 25t2 . The average velocity of the car between t = 0
and t = 2 is
s(2) − s(0) 25(2)2 − 25(0)2 25(4) − 0 100
= = = = 50 km/h
2−0 2 2 2

(b) Solution. The instantaneous velocity of the car at t = 2 is

s(2 + h) − s(2) 25(2 + h)2 − 25(2)2


lim = lim
h→0 h h→0 h
25(4 + 4h + h2 ) − 25(4)
= lim (a + b)2 = a2 + 2ab + b2
h→0 h
100 + 100h + 25h2 − 100 100h + 25h2
= lim = lim
h→0 h h→0 h
h(100 + 25h)
= lim Factor the numerator
h→0 h
= lim (100 + 25h) Simplify
h→0

= 100 + 25(0) = 100 km/h Substitute h with 0

4. [Exercise on page 20]

(a) Solution. The instantaneous velocity when t = 9 is

s(9 + h) − s(9)
lim
h→0 h
Finding s(9 + h). This is obtained by substituting t with 9 + h into s(t). Since
s(t) = 31 t2 − 5t + 19,

1
s(9 + h) = (9 + h)2 − 5(9 + h) + 19
3
3.6. Rates of Change 145

1
= (81 + 18h + h2 ) − 5(9 + h) + 19 (a + b)2 = a2 + 2ab + b2
3
1
= 27 + 6h + h2 − 45 − 5h + 19 Distribute
3
1
= 1 + h + h2
3
Finding s(9). Substituting t with 9 into s(t), we get
1
s(9) = (9)2 − 5(9) + 19 = 27 − 45 + 19 = 1
3
The required instantaneous velocity is

1 + h + 31 h2 − (1)

s(9 + h) − s(9)
lim = lim
h→0 h h→0 h
1 2
h + 3h
= lim
h→0 h
h 1 + 13 h

= lim Factor the numerator
h→0 h
 
1
= lim 1 + h Simplify
h→0 3
1
= 1 + (0) = 1 Substitute h with 0
3
Here the position is in feet (ft) and time is in seconds (s). So the instantaneous
velocity when t = 9 is 1 ft/s.
(b) Solution. First we need to find s(13 + h)
1
s(13 + h) = (13 + h)2 − 5(13 + h) + 19
3
1
= (169 + 26h + h2 ) − 5(13 + h) + 19 (a + b)2 = a2 + 2ab + b2
3
169 26 1
= + h + h2 − 65 − 5h + 19 Distribute
3 3 3
31 11 1 2 a a ± bc
= + h+ h ±c=
3 3 3 b b
Substituting t with 13 into s(t), we get
1 169 169 169 − 138 31
s(13) = (13)2 − 5(13) + 19 = − 65 + 19 = − 46 = =
3 3 3 3 3
The instantaneous velocity when t = 13 is
31 11
+ 13 h2 − 31

s(13 + h) − s(13) 3
+ 3
h 3
lim = lim
h→0 h h→0 h
11
3
h + 31 h2
= lim
h→0 h
146 Chapter 3. Limits and Derivatives

11
+ 13 h

h 3
= lim Factor the numerator
h→0 h
 
11 1
= lim + h Simplify
h→0 3 3
11 1
= + (0) ≈ 3.67 ft/s Substitute h with 0
3 3
5. [Exercise on page 20]
(a) Solution. The average rate of change in the cost as the number of items
produced changes from 2 to 3 is
C(3) − C(2) [200 + 21(3) − (3)2 ] − [200 + 21(2) − (2)2 ]
=
3−2 1
(200 + 63 − 9) − (200 + 42 − 4) 254 − 238
= = = 16
1 1
(b) Solution. If the production is at the level of x = 2 items, the cost of producing
one additional item is
C(3) − C(2) = [200 + 21(3) − (3)2 ] − [200 + 21(2) − (2)2 ] = 254 − 238 = 16
We notice that this coincides with the answer found in part (a). This is true in
general.

Cost of Producing one Additional Item and Average Cost

Let C(x) be the cost of producing x items. Let n be a nonnegative number.

ˆ If the production is at the level of x = n items, the cost of producing


one additional item is

C(n + 1) − C(n)

ˆ The average cost as x changes from n to n + 1 is

C(n + 1) − C(n) C(n + 1) − C(n)


=
(n + 1) − n n+1−n
C(n + 1) − C(n)
= = C(n + 1) − C(n)
1

So if the production is at the level of x = n items, the cost of


producing one additional item is equal to the average cost as x
changes from n to n + 1.

(c) Solution. The instantaneous rate of change of cost C(x) = 200 + 21x − x2 when
x = 2 is
C(2 + h) − C(2) [200 + 21(2 + h) − (2 + h)2 ] − [200 + 21(2) − (2)2 ]
lim = lim
h→0 h h→0 h
3.6. Rates of Change 147

[200 + 42 + 21h − (4 + 4h + h2 )] − (200 + 42 − 4)


= lim
h→0 h
(242 + 21h − 4 − 4h − h2 ) − (238)
= lim
h→0 h
2
238 + 17h − h − 238
= lim
h→0 h
h(17 − h)
= lim Factor the numerator
h→0 h
Simplify and sub-
= lim (17 − h) = 17 − 0 = 17
h→0 stitute h with 0
This is close to the answer found in part (b). In general, we have the following
important fact.

Cost of producing one additional item ≈ instantaneous rate of


change of cost

Let C(x) be the cost of producing x items. And let n be a nonnegative


number. Then if the production is at the level of x = n items, the cost
of producing one additional item can be approximated by the
instantaneous rate of change at x = n. That is,

C(n + h) − C(n)
C(n + 1) − C(n) ≈ lim
h→0 h
As we will see in the next section, the instantaneous rate of change is
nothing but the derivative.

6. [Exercise on page 20]


(a) Solution. The average rate of change of profit when the number of items sold
changes from 1 to 2 is
P (2) − P (1) [3(2)2 − 9(2) + 8] − [3(1)2 − 9(1) + 8]
=
2−1 1
(12 − 18 + 8) − (3 − 9 + 8)
= = (2) − (2) = 0
1
(b) Solution. The average rate of change of profit when the number of items sold
changes from 2 to 5 is
P (5) − P (2) [3(5)2 − 9(5) + 8] − [3(2)2 − 9(2) + 8]
=
5−2 3
(75 − 45 + 8) − (12 − 18 + 8) 38 − 2
= = = 12
3 3
(c) Solution. The instantaneous rate of change of profit when x = 1 is
P (1 + h) − P (1)
lim
h→0 h
148 Chapter 3. Limits and Derivatives

Finding P (1 + h). Substituting x with 1 + h into P (x) = 3x2 − 9x + 8, we get

P (1 + h) = 3(1 + h)2 − 9(1 + h) + 8 = 3(1 + 2h + h2 ) − 9(1 + h) + 8


= 3 + 6h + 3h2 − 9 − 9h + 8 = 2 − 3h + 3h2

Substituting x with 1 into P (x), we get

P (1) = 3(1)2 − 9(1) + 8 = 3 − 9 + 8 = 2

We now find the instantaneous rate of change of profit when x = 1:

P (1 + h) − P (1) 2 − 3h + 3h2 − 2 h(−3 + 3h)


lim = lim = lim
h→0 h h→0 h h→0 h
= lim (−3 + 3h) = −3 + 3(0) = −3
h→0

This means that at the level of production of x = 1 item, the profit is decreasing
at the rate of $3 per item.
(d) Solution. The instantaneous rate of change of profit when x = 5 is

P (5 + h) − P (5)
lim
h→0 h

Finding P (5 + h). Substituting x with 5 + h into P (x) = 3x2 − 9x + 8, we get

P (5 + h) = 3(5 + h)2 − 9(5 + h) + 8 = 3(25 + 10h + h2 ) − 9(5 + h) + 8


= 75 + 30h + 3h2 − 45 − 9h + 8 = 38 + 21h + 3h2

Substituting x with 5 into P (x), we get

P (5) = 3(5)2 − 9(5) + 8 = 75 − 45 + 8 = 38

We now find the instantaneous rate of change of profit when x = 5:

P (5 + h) − P (5) 38 + 21h + 3h2 − 38 h(21 + 3h)


lim = lim = lim
h→0 h h→0 h h→0 h
= lim (21 + 3h) = 21 + 3(0) = 21
h→0

This means that when 5 items are manufactured, the profit is increasing at the
rate of $21 per item.

3.7 Definition of the Derivative


1. [Exercise on page 21]

(a) Solution.
3.7. Definition of the Derivative 149

Definition of the Derivative


The derivative of a function f at a point a, denoted f 0 (a), is defined by

f (a + h) − f (a)
f 0 (a) = lim (3.7.1)
h→0 h
if this limit exists.

Geometric Interpretation of the Derivative

ˆ Let f be a function, and let a be a number such that f 0 (a) exists.


Geometrically, f 0 (a) represents the slope of the tangent line
T to the curve y = f (x) at the point P (a, f (a)). This is
shown in the following figure.

y
y = f (x)
T

P
f (a)
f 0 (a) = slope of T

a x

ˆ If the limit (3.7.1) does not exist, there is no tangent at the point
(a, f (a)).

ˆ Recall that the slope of the line through two points P1 (x1 , y1 ) and
−y1
P2 (x2 , y2 ) is m = xy22 −x 1
.

Consider the function whose graph is shown in Figure 3.12.


ˆ The derivative f 0 (−1) is the slope of the tangent line T1 to the curve at the
point P1 (−1, −4).
ˆ Since the line T1 passes through the points (−1, −4) and (0, 5), the slope is
5 − (−4) 5+4 9
m= = = = 9.
0 − (−1) 0+1 1
ˆ Thus, f 0 (−1) = 9.
(b) Solution.
ˆ The derivative of f at 1, f 0 (1), is the slope of the tangent line at (1, f (1)).
From Figure 3.12, the tangent line at (1, f (1)) is the line T2 that passes
150 Chapter 3. Limits and Derivatives

y
T1
6

−2 −1 1 2 3 4 x

−2

P1 −4

T2
−6

Figure 3.12

through the points (1, −2) and (0, 1). Since the slope of T2 is

1 − (−2) 1+2 3
m= = = = −3,
0−1 −1 −1

it follows that f 0 (1) = −3.


ˆ Equation of T2 . Using the slope-intercept formula and the point (1, −2),
the equation of the line T2 is y − (−2) = −3(x − 1). This is equivalent to
y + 2 = −3x + 3. Subtracting 2 from both sides, we get y = −3x + 1.
(c) Solution. The derivative at a point a is zero if and only if the slope of the
tangent line to the curve at (a, f (a)) is zero. And we know that the slope of a
line is 0 if and only if it is horizontal. So finding the points where the derivative is
zero amounts to finding the points (a, f (a)) where the tangent line is horizontal.
From Figure 3.12, there are two points on the graph with that property: (0, 0)
and (2, −4). Thus, the derivative of f is zero at 0 and 2.
(d) Solution. No. Indeed, the tangent line to the graph of f at (3, f (3)) is an
increasing line. Since an increasing line has a positive slope, the derivative f 0 (3)
is positive.

2. [Exercise on page 21]


Solution.
3.7. Definition of the Derivative 151

ˆ The tangent line to the curve at (1, f (1)) passes through the points (0, −1) and
(1, 0) as shown in Figure 3.7.
0−(−1)
ˆ So the slope of the tangent line is m = 1−0
= 0+1
1−0
= 1.
ˆ Therefore, the derivative of f at 1 is f 0 (1) = 1.

3. [Exercise on page 22]


Solution.

Finding the Derivative f 0 (a) Using the Definition

To find
f (a + h) − f (a)
f 0 (a) = lim ,
h→0 h
one can proceed as follows.

Step 1. Find f (a + h). To get f (a + h), we substitute x with a + h. This


means that anytime you see x, replace it with a + h.

ˆ For example, if f (x) = x2 − 4x + 1, then

f (a + h) = (a + h)2 − 4(a + h) + 1.
x 2(a+h)−ea+h
ˆ If f (x) = 2x−e

5x3 − x+7
, then f (a + h) = √
5(a+h)3 − a+h+7
.

Step 2. Find an simplify f (a + h) − f (a).

f (a+h)−f (a)
Step 3. Divide f (a + h) − f (a) by h. The fraction h
is called the
difference quotient.

Step 4. Take the limit of the difference quotient as h approaches 0 to get


f 0 (a) (if this limit exists): f 0 (a) = lim f (a+h)−f
h
(a)
.
h→0

We want to find the equation of the tangent line to the curve y = −x2 at (1, −1).
f (1+h)−f (1)
ˆ First we need to find the slope, which is given by m = f 0 (1) = lim h
.
h→0
– Finding f (1 + h). We have
Replace x with 1 + h
f (1 + h) = −(1 + h)2
into f (x) = −x2
= −(1 + h)(1 + h) = −(1 + h + h + h2 ) Distribute
= −1 − 2h − h2

– Finding f (1 + h) − f (1). We have

f (1 + h) − f (1) = −1 − 2h − h2 − (−(−1)2 )
152 Chapter 3. Limits and Derivatives

= −1 − 2h − h2 − (−1)
= −1 − 2h − h2 + 1 = −2h − h2

f (1+h)−f (1) −2h−h2


– Dividing by h, we get h
= h
.
– Taking the limit as h approaches 0, we get

f (1 + h) − f (1)
f 0 (1) = lim
h→0 h
−2h − h2
= lim
h→0 h
h(−2 − h)
= lim Factor the numerator
h→0 h
= lim (−2 − h) Simplify
h→0

= −2 − 0 = −2. Substitute h with 0

Thus, the slope of the tangent line is m = f 0 (1) = −2.

ˆ An equation of the tangent line to the curve f (x) = −x2 at (1, −1) is

y − (−1) = m(x − 1)
y − (−1) = −2(x − 1) Substitute m with −2
y + 1 = −2x + 2
y = −2x + 1 Subtract 1 from both sides

4. [Exercise on page 22]

Solution.

f (4+h)−f (4)
ˆ We first need to find the slope, which is given by m = f 0 (4) = lim h
.
h→0


– Finding f (4 + h). We have f (4 + h) = 4 + h.
– Finding f (4 + h) − f (4). We have

√ √ √
f (4 + h) − f (4) = 4+h− 4= 4 + h − 2.


f (4+h)−f (4) 4+h−2
– Dividing by h, we get h
= h
.
3.7. Definition of the Derivative 153

– Taking the limit as h approaches 0, we get



0 f (4 + h) − f (4) 4+h−2
f (4) = lim = lim
h→0 h h→0 h
√ √
( 4 + h − 2)( 4 + h + 2) Rationalize the
= lim √
h→0 h( 4 + h + 2) numerator
(4 + h) − 4 Distribute the
= lim √
h→0 h( 4 + h + 2) numerator
4+h−4 h
= lim √ = lim √
h→0 h( 4 + h + 2) h→0 h( 4 + h + 2)
1
= lim √ Simplify
h→0 4+h+2
1 1
=√ =√ Substitute h with 0
4+0+2 4+2
1 1
= =
2+2 4

Thus, the slope of the tangent line is m = f 0 (4) = 41 .



ˆ An equation of the tangent line to the curve f (x) = x at (4, 2) is

1
y − 2 = (x − 4)
4
1
y−2= x−1 Distribute the right hand side
4
1
y = x+1 Add 2 to both sides
4

5. [Exercise on page 22]


f (2+h)−f (2)
Solution. By definition, the derivative of f at a = 2 is f 0 (2) = lim h
.
h→0

ˆ Finding f (2 + h).

f (2 + h) = −3(2 + h)2 + 4(2 + h)


= −3(22 + 2(2)h + h2 ) + 4(2 + h) (a + b)2 = a2 + 2ab + b2
= −3(4 + 4h + h2 ) + 4(2 + h)
= −12 − 12h − 3h2 + 8 + 4h Distribute
2
= −4 − 8h − 3h

ˆ Finding f (2 + h) − f (2).

f (2 + h) − f (2) = −4 − 8h − 3h2 − (−3(2)2 + 4(2))


154 Chapter 3. Limits and Derivatives

= −4 − 8h − 3h2 − (−12 + 8)
= −4 − 8h − 3h2 − (−4)
= −4 − 8h − 3h2 + 4 = −8h − 3h2
f (2+h)−f (2) −8h−3h2
ˆ Dividing by h, we get h
= h
.
ˆ Taking the limit as h approaches 0, we get
f (2 + h) − f (2) −8h − 3h2
f 0 (2) = lim = lim
h→0 h h→0 h
h(−8 − 3h)
= lim Factor
h→0 h
= lim (−8 − 3h) Simplify
h→0

= −8 − 3(0) = −8. Substitute h with 0

6. [Exercise on page 22]


f (−1+h)−f (−1)
Solution. By definition, f 0 (−1) = lim h
.
h→0

ˆ Finding f (−1 + h).


f (−1 + h) = (−1 + h)3 − (−1 + h) = (−1 + h)2 (−1 + h) − (−1 + h)
= (1 − 2h + h2 )(−1 + h) + 1 − h
= (−1 + h + 2h − 2h2 − h2 + h3 ) + 1 − h
= −1 + 3h − 3h2 + h3 + 1 − h = 2h − 3h2 + h3 .

ˆ Finding f (−1 + h) − f (−1).


f (−1 + h) − f (−1) = 2h − 3h2 + h3 − ((−1)3 − (−1))
= 2h − 3h2 + h3 − (−1 + 1)
= 2h − 3h2 + h3 − 0 = 2h − 3h2 + h3
f (−1+h)−f (−1) 2h−3h2 +h3
ˆ Dividing by h, we get h
= h
.
ˆ Taking the limit as h approaches 0, we get
f (−1 + h) − f (−1) 2h − 3h2 + h3
f 0 (−1) = lim = lim
h→0 h h→0 h
h(2 − 3h + h2 )
= lim Factor
h→0 h
= lim (2 − 3h + h2 ) Simplify
h→0

= 2 − 3(0) + (0)2 = 2 + 0 = 2 Substitute h with 0

7. [Exercise on page 22]


f (3+h)−f (3)
Solution. By definition, f 0 (3) = lim h
.
h→0
3.7. Definition of the Derivative 155

ˆ Finding f (3 + h). We have f (3 + h) = 3+h


(3+h)+1
= 3+h
4+h
.
ˆ Finding f (3 + h) − f (3). We have
 
3+h 3 3+h 3
f (3 + h) − f (3) = − = −
4+h 3+1 4+h 4
4(3 + h) − 3(4 + h) a c ad − bc
= − =
4(4 + h) b d bd
12 + 4h − 12 − 3h h
= =
16 + 4h 16 + 4h

ˆ Dividing by h, we get
h
f (3 + h) − f (3) 16+4h
=
h h
a
h b a
= =
(16 + 4h)h c bc
ˆ Taking the limit as h → 0, we get
f (3 + h) − f (3) h
f 0 (3) = lim = lim
h→0 h h→0 (16 + 4h)h

1
= lim Simplify
h→0 16 + 4h

1 1
= = Substitute h with 0
16 + 4(0) 16
8. [Exercise on page 22]

(a) Solution.

The Derivative as a Function


Let f be a function. The derivative of f , denoted f 0 , is the function
defined by
f (x + h) − f (x)
f 0 (x) = lim , (3.7.2)
h→0 h
provided this limit exists. Geometrically, f 0 (x) represents the slope of the
tangent line to the graph of y = f (x) at (x, f (x)).

We want to find the derivative of f (x) = 10 by considering the slope of the


tangent line.
ˆ Since f (x) = 10 is a constant function, its graph is a horizontal line.
ˆ So the tangent line to the graph of f (x) = 10 is the line of equation y = 10.
(The tangent line to a line y = mx + b at any point is the line of equation
y = mx + b, that is, the same line.)
ˆ Since the slope of a horizontal line is 0, it follows that f 0 (x) = 0.
156 Chapter 3. Limits and Derivatives

Short Solution
Since f (x) = 10 = 0x + 10 is a line, and since the slope of that line is
m = 0, it follows that f 0 (x) = 0.

(b) Solution. The function f (x) = −2x + 3 is a line with slope m = −2. So
f 0 (x) = −2.

(c) Solution. Since f (x) = − 23 = 0x − 2


3
is a line with slope m = 0, it follows that
f 0 (x) = 0.

(d) Solution. Since

5 − 4x 5 4 a±b a b
f (x) = = − x = ±
6 6 6 c c c
5 2
= − x,
6 3

the slope is m = − 23 . Therefore, f 0 (x) = − 32 .

9. [Exercise on page 22]

(a) Solution.

f (x + h) − f (x)
f 0 (x) = lim Use Definition (3.7.2)
h→0 h
k−k
= lim f (x + h) = k
h→0 h
0
= lim = lim (0) = 0.
h→0 h h→0

(b) Solution.

f (x + h) − f (x)
f 0 (x) = lim Use Definition (3.7.2)
h→0 h
(x + h) − x Since f (x) = x, it follows
= lim
h→0 h that f (x + h) = x + h
x+h−x h
= lim = lim
h→0 h h→0 h

= lim (1) Simplify


h→0

=1
3.7. Definition of the Derivative 157

(c) Solution.

f (x + h) − f (x)
f 0 (x) = lim Use Definition (3.7.2)
h→0 h
(x + h)2 − x2
= lim f (x + h) = (x + h)2
h→0 h
(x2 + 2xh + h2 ) − x2
= lim (a + b)2 = a2 + 2ab + b2
h→0 h
2xh + h2 h(2x + h)
= lim = lim Factor
h→0 h h→0 h
= lim (2x + h) Simplify
h→0

= 2x + 0 = 2x Substitute h with 0

(d) Solution.

f (x + h) − f (x)
f 0 (x) = lim Use Definition (3.7.2)
h→0 h
(x + h)3 − x3
= lim f (x + h) = (x + h)3
h→0 h
x3 + 3x2 h + 3xh2 + h3 − x3
= lim (a + b)3 = a3 + 3a2 b + 3ab2 + b3
h→0 h
3x2 h + 3xh2 + h3
= lim
h→0 h
h(3x2 + 3xh + h2 )
= lim Factor
h→0 h
= lim (3x2 + 3xh + h2 ) Simplify
h→0

= 3x2 + 3x(0) + 02 = 3x2 Substitute h with 0


158 Chapter 3. Limits and Derivatives

(e) Solution.

f (x + h) − f (x)
f 0 (x) = lim Use Definition (3.7.2)
h→0 h
√ √
x+h− x √
= lim f (x + h) = x+h
h→0 h
√ √  √ √
x + h − x ( x + h + x)
= lim √ √ Rationalize
h→0 h( x + h + x)
(x + h) − x
= lim √ √ Distribute the numerator
h→0 h( x + h + x)
h
= lim √ √
h→0 h( x + h + x)
1
= lim √ √ Simplify
h→0 x+h+ x
1
=√ √ Substitute h with 0
x+0+ x
1 1
=√ √ = √
x+ x 2 x

(f ) Solution.

f (x + h) − f (x)
f 0 (x) = lim Use Definition (3.7.2)
h→0 h
1 1
x+h
− x 1
= lim f (x + h) =
h→0 h x+h
x−x−h
(x+h)x a c ad − bc
= lim − =
h→0 h b d bd
−h a
(x+h)x −h b a
= lim = lim =
h→0 h h→0 h(x + h)x c bc
−1
= lim Simplify
h→0 (x + h)x
1 1
=− = − 2.
(x + 0)x x

10. [Exercise on page 22]


3.7. Definition of the Derivative 159

(a) Solution.
f (x + h) − f (x)
f 0 (x) = lim Use Definition (3.7.2)
h→0 h
−4(x + h) + 1 − (−4x + 1)
= lim f (x + h) = −4(x + h) + 1
h→0 h
−4x − 4h + 1 + 4x − 1 −4h
= lim = lim
h→0 h h→0 h
= lim (−4) = −4 Simplify
h→0

Since f 0 (x) = −4, it follows that f 0 (−3) = −4, f 0 (0) = −4, and f 0 (2) = −4.
(b) Solution.
ˆ Finding f (x + h). Substituting x with x + h into f (x) = 3x2 − 5x, we get

f (x + h) = 3(x + h)2 − 5(x + h)


= 3(x2 + 2xh + h2 ) − 5(x + h) (a + b)2 = a2 + 2ab + b2
= 3x2 + 6xh + 3h2 − 5x − 5h Distribute

ˆ Finding f (x + h) − f (x).

f (x + h) − f (x) = (3x2 + 6xh + 3h2 − 5x − 5h) − (3x2 − 5x)


= 3x2 + 6xh + 3h2 − 5x − 5h − 3x2 + 5x
= 6xh + 3h2 − 5h
2 −5h
ˆ Dividing this by h, we get f (x+h)−f
h
(x)
= 6xh+3h
h
.
ˆ Taking the limit as h approaches 0, we get
f (x + h) − f (x)
f 0 (x) = lim Use Definition (3.7.2)
h→0 h
6xh + 3h2 − 5h
= lim
h→0 h
h(6x + 3h − 5)
= lim Factor
h→0 h
= lim (6x + 3h − 5) Simplify
h→0

= 6x + 3(0) − 5 = 6x − 5 Substitute h with 0

Since f 0 (x) = 6x − 5, it follows that

f 0 (−3) = 6(−3) − 5 = −18 − 5 = −23


f 0 (0) = 6(0) − 5 = 0 − 5 = −5
f 0 (2) = 6(2) − 5 = 12 − 5 = 7.

(c) Solution.
160 Chapter 3. Limits and Derivatives

ˆ Finding f (x + h). Substituting x with x + h into f (x) = 2x−1


3x+7
, we get
2(x + h) − 1 2x + 2h − 1
f (x + h) = = .
3(x + h) + 7 3x + 3h + 7
ˆ Finding f (x + h) − f (x).
f (x + h) − f (x)
2x + 2h − 1 2x − 1
= −
3x + 3h + 7 3x + 7
(2x + 2h − 1)(3x + 7) − (2x − 1)(3x + 3h + 7)
=
(3x + 3h + 7)(3x + 7)
(6x2 + 14x + 6xh + 14h − 3x − 7) − (6x2 + 6xh + 14x − 3x − 3h − 7)
=
(3x + 3h + 7)(3x + 7)
6x2 + 14x + 6xh + 14h − 3x − 7 − 6x2 − 6xh − 14x + 3x + 3h + 7
=
(3x + 3h + 7)(3x + 7)
17h
= .
(3x + 3h + 7)(3x + 7)
ˆ Dividing this by h, we get
17h
f (x + h) − f (x) (3x+3h+7)(3x+7)
=
h h
a
17h b a
= =
h(3x + 3h + 7)(3x + 7) c bc
ˆ Taking the limit as h approaches 0, we get
17h
f 0 (x) = lim
h→0 h(3x + 3h + 7)(3x + 7)
17
= lim Simplify
h→0 (3x + 3h + 7)(3x + 7)
17
= Substitute h with 0
(3x + 3(0) + 7)(3x + 7)
17 17
= =
(3x + 7)(3x + 7) (3x + 7)2
Using this, we have
17 17 17
f 0 (−3) = 2
= 2
=
(3(−3) + 7) (−2) 4
17 17 17
f 0 (0) = 2
= 2 =
(3(0) + 7) 7 49
17 17 17
f 0 (2) = = =
(3(2) + 7)2 132 169
3.7. Definition of the Derivative 161

(d) Solution.

f (x + h) − f (x)
f 0 (x) = lim Use Definition (3.7.2)
h→0 h
p √
3(x + h) − 3x p
= lim f (x + h) = 3(x + h)
h→0 h
√ √ √ √
( 3x + 3h − 3x)( 3x + 3h + 3x)
= lim √ √ Rationalize
h→0 h( 3x + 3h + 3x)
(3x + 3h) − 3x
= lim √ √ Distribute the numerator
h→0 h( 3x + 3h + 3x)
3h
= lim √ √
h→0 h( 3x + 3h + 3x)
3
= lim √ √ Simplify
h→0 3x + 3h + 3x
3
=p √ Substitute h with 0
3x + 3(0) + 3x
3 3
=√ √ = √ .
3x + 3x 2 3x

Using this, we have


ˆ f 0 (−3) = √ 3 = √3 ,
2 −9
which does not exist because the square root
2 3(−3)
of a negative number is undefined.
ˆ f 0 (0) = √3 = 2(0)
3
= 30 , which does not exist because the division by 0
2 3(0)
is undefined.
ˆ f 0 (2) = √3 = 2
3

6
.
2 3(2)

11. [Exercise on page 23]

Solution.

Differentiability
A function f is said to be differentiable at a point a if the limit

f (a + h) − f (a)
lim
h→0 h
exists. If this limit does not exist, we say that f is not differentiable at x = a.

To show that f (x) = |x − 1| is not differentiable at x = 1, we need to show that the


162 Chapter 3. Limits and Derivatives

f (1+h)−f (1)
limit f 0 (1) = lim h
does not exist.
h→0

f (1 + h) − f (1) |(1 + h) − 1| − |1 − 1|
lim = lim f (1 + h) = |(1 + h) − 1|
h→0 h h→0 h
|1 + h − 1| − |0| |h| − 0
= lim = lim
h→0 h h→0 h
|h|
= lim .
h→0 h

By definition of the absolute value, we have



h if h ≥ 0
|h| =
−h if h < 0.
|h|
So to find lim , we need to find one-sided limits.
h→0 h

ˆ Limit from the left. As h approaches 0 from the left, h < 0, and therefore
|h| = −h. So
|h| −h
lim− = lim− = lim− (−1) = −1.
h→0 h h→0 h h→0

ˆ Limit from the right. As h approaches 0 from the right, h > 0, and therefore
|h| = h. So
|h| h
lim+ = lim+ = lim+ (1) = 1.
h→0 h h→0 h h→0

Because the limit from the left is not equal to the limit from the right, the limit
lim |h|
h
does not exist. Hence, f is not differentiable at x = 1.
h→0

12. [Exercise on page 23]

(a) Solution.

Continuity vs Differentiability
Theorem 3.7.1. Let f be a function.
ˆ If f is differentiable at a, then f is continuous at a. So we have the
implication
f is differentiable at a =⇒ f is continuous at a.
The symbol “ =⇒ ” means “implies”.
ˆ If f is not continuous at a, then f is not differentiable at a. So we
have the implication
f is not continuous at a =⇒ f is not differentiable at a.

Note. The theorem does not say that if f is not differentiable, then f is
not continuous. For example, the absolute value function f (x) = |x − 1|
3.7. Definition of the Derivative 163

is not differentiable at 1 (as we saw in the preceding exercise) but it is


continuous at 1.

We want to determine if the function defined by the graph in Figure 3.8 is


continuous and differentiable at the points −6, −4, −3, −1, 0, 1, 3, 4, 5.
ˆ At x = −6.
– Continuity. The function f clearly satisfies the three conditions of the
definition of continuity. Thus, f is continuous at −6.
– Differentiability. The graph of f in the interval [−7, −4] is the straight
line through (−6, 0) and (−4, 3). So the tangent line at x = −6 is the
3−0
same straight line. Since the slope of that line exists (m = −4+6 = 23 ),
it follows that f is differentiable at −6 and f 0 (−6) = 32 .
ˆ At x = −4.
– Continuity. The function f clearly satisfies the three conditions of the
definition of continuity. Thus, f is continuous at −4.
– Differentiability. Since the tangent line from the left is not the same as
the tangent line from the right, f is not differentiable at −4. The
point (−4, 3) is called a sharp point or corner.
ˆ At x = −3.
– Continuity. The function f is not continuous at −3 because f is
undefined there.
– Differentiability. Since f is not continuous at −3, it follows (by Theo-
rem 3.7.1) that f is not differentiable at −3.
ˆ At x = −1.
– Continuity. The function f clearly satisfies the three conditions of the
definition of continuity. Thus, f is continuous at −1.
– Differentiability. If we draw the tangent line from the right, we will see
that it is not the same as the tangent line from the left. So f is not
differentiable at −1. Actually, the point (−1, 0) is a corner.
ˆ At x = 0.
– Continuity. The function f clearly satisfies the three conditions of the
definition of continuity. Thus, f is continuous at 0
– Differentiability. The tangent line at x = 0 is the y-axis, which is a
vertical line. Since the slope of a vertical line is undefined, it follows
that f is not differentiable at x = 0.
ˆ At x = 1.
– Continuity. The one-sided limits are
lim f (x) = 2 and lim f (x) = 3.
x→1− x→1+

Because the limit from the left is not equal to the limit from the right,
the limit of f (x) as x approaches 1 does not exist. Therefore the second
condition of the definition of continuity is not satisfied. This implies
that f is not continuous at 1.
164 Chapter 3. Limits and Derivatives

– Differentiability. Since f is not continuous at x = 1, it follows (by


Theorem 3.7.1) that f is not differentiable at 1.
ˆ At x = 3, f is continuous and not differentiable (the justification is the
same as the one we provided at x = −1).
ˆ At x = 4.
– Continuity. The function f is not continuous at 4 because it is not
defined there.
– Differentiability. Since f is not continuous at x = 4, it follows that f is
not differentiable at 4.
ˆ At x = 5, f is clearly continuous and differentiable.
(b) Find f 0 (−5), f 0 (−2), and f 0 (2).
Solution.
ˆ Finding f 0 (−5). The tangent line at the point where x = −5 passes through
3−0
the points (−6, 0) and (−4, 3). Since the slope of that line is m = −4−(−6) =
3 0 3
2
, it follows that f (−5) = 2 .
ˆ Finding f 0 (−2). The tangent line at the point where x = −2 passes through
the points (−2, 1) and (−1, 0). Since the slope of that line is
0−1 −1 −1
m= = = = −1,
−1 − (−2) −1 + 2 1

it follows that f 0 (−2) = −1.


ˆ Finding f 0 (2). In the interval (1, 3) the graph of f is a horizontal line. Since
the slope of a horizontal line is 0, it follows that f 0 (2) = 0. Actually, the
derivative of f at every point in the open interval (1, 3) is zero.

3.8 Graphical Differentiation


1. [Exercise on page 23]
Solution. We will use the graphical interpretation of the derivative.

Geometric Interpretation of the Derivative

Let f be a function. The derivative of f at x, f 0 (x), can be interpreted as the


slope of the tangent line to the graph of f at the point (x, f (x)).

ˆ Here the graph of f is a line. So the tangent line at every point coincides with
the graph of f .
ˆ Since the graph of f is a horizontal line, and since the slope of a horizontal line
is 0, it follows that the derivative at every point is 0. That is, f 0 (x) = 0 for
every x.
ˆ So the graph of f 0 is the line of equation y = 0, that is, the x-axis as shown
below.
3.8. Graphical Differentiation 165

y
f
2
f0
−3 3 x

2. [Exercise on page 24]


Solution. Since the graph of f is a straight line, the tangent line at every point
coincides with the graph of f . So the derivative at every point is the slope of the
graph of f . To find the slope, we need to choose two points on the line. Let’s choose
for example the points (1, 2) and (3, 4). Remembering the formula for the slope
(m = xy22 −y
−x1
1
), we have
4−2 2
m= = =1
3−1 2
So f 0 (x) = 1 for every x. And the graph of f 0 is the line of equation y = 1 as shown
below.

y f
4
3
2
f0
1

−3 −2 −1 1 2 3 x
−1

3. [Exercise on page 24]


Solution. Using the same approach as in the preceding exercise, all we have to do
is to find the slope of the graph of f (since it is a straight line). Choosing the points
(−2, 3) and (−1, 1) on the graph of f , the slope is

1−3 1−3 −2
m= = = = −2
−1 − (−2) −1 + 2 1

So f 0 (x) = −2 for every x. And the graph of f 0 is the line of equation y = −2 as


shown below.
166 Chapter 3. Limits and Derivatives

y
3
2
1

−3 −2 −1 1 2 3 x
−1
f0
−2
−3
f

4. [Exercise on page 24]

Solution. Here the graph of f is not a straight line. So the derivative is not a
constant function as before. To sketch the graph of f 0 , the key thing is to be able to
estimate the derivative f 0 (a) for a point x = a.

How to Estimate the Derivative Graphically?


Let f be a function defined by a graph. To estimate the derivative of f at
x = a, one can proceed as follows.

Step 1. Draw the tangent line to the graph of f at the point (a, f (a)).

Step 2. Choose two points on the tangent line and find the slope between
the two points.

Step 3. If the slope found in Step 2 is m, then f 0 (a) ≈ m. And therefore


the graph of f 0 passes through the point (a, m).

Note. If the tangent line is horizontal, the slope is 0, and therefore the graph
of f 0 passes through the point (a, 0). Or, equivalently, the graph of f 0 intersects
the x-axis at a. Situations where f 0 (a) = 0 include the following.
f (a, f (a))
Tangent line Tangent line
(a, f (a)) f
f 0 (a) = 0 f 0 (a) = 0

f
Tangent line
(a, f (a)) Tangent line
(a, f (a))
0
f (a) = 0 f 0 (a) = 0 f
3.8. Graphical Differentiation 167

How to Sketch the Graph of the Derivative?


Let f be a function defined by a graph. If the graph of f is a straight line,
and if the slope of the line is m, then the graph of f 0 is the horizontal line of
equation y = m. Suppose the graph of f is not a straight line. To sketch the
graph of f 0 , one can go through the following steps.

Step 1. Find the points (if any) where the tangent line is horizontal or
where the function is not differentiable. These points subdivide
the x-axis into a certain number of intervals (n points will produce
n + 1 intervals).

Step 2. Mark the x-coordinate of each point where the tangent line is hor-
izontal. Specifically, if (a, f (a)) is a point on the graph such that
f 0 (a) = 0, plot the point (a, 0).

Step 3. For each interval found in Step 1, choose one or two numbers in
the interval and estimate the derivative at those numbers. (Choose
more numbers if you want to improve the accuracy of the graph of
the derivative.) If f 0 (a) ≈ m, then plot the point (a, m).

Step 4. The graph of f 0 is obtained by connecting the points (smoothly)


plotted in Steps 2 and 3.

Here we want to sketch the graph of the derivative of the following function.

y
f

1
1 3 5 x
−1

ˆ From the graph of f , we see that the tangent line at the point where x = 2 is
horizontal. This determines two open intervals: (−∞, 2) and (2, ∞).
ˆ Since the tangent line is horizontal at (2, −1), it follows that f 0 (2) = 0. Plotting
the point (2, 0), we have the following

y
f

1
1 3 5 x
−1
168 Chapter 3. Limits and Derivatives

ˆ On the interval (−∞, 2), we need to choose a number of points (one or two) and
estimate the derivative at those points.

– Choose for example x = 0. To estimate the derivative f 0 (0),

* first we draw the tangent line to the graph of f at (0, 3):


y

5 f

1 3 5 x
−1

* Then we choose two points on the tangent line and compute the slope
m between the two points. Choosing for example the points (0, 3) and
(1, −1), we get

−1 − 3 −4
m= = = −4.
1−0 1

So f 0 (0) ≈ −4.
Note. We are just estimating the slope of the tangent line. So your
answer may be slightly different.
Plotting the point (0, −4), we get

5 f

1
1 3 5 x
−1

−3

– Choose another point in the interval (−∞, 2). Choose for example x = 1.
To estimate the derivative f 0 (1), we first draw the tangent line to the graph
of f at (1, 0).
3.8. Graphical Differentiation 169

5 f

1
1 3 5 x
−1

−3

Then we select two points on the tangent line. Choose for example the
points (0, 2) and (1, 0). The slope between these points is
0−2 −2
m= = = −2
1−0 1
So f 0 (1) ≈ −2. Plotting the point (1, −2), we have the following
y

5 f

1
1 3 5 x
−1

−3

ˆ Estimating the derivative at some points on the interval (2, ∞). We are going
to choose two points.
– Select the point (3, 0). The tangent line to the graph of f at the point (3, 0)
is shown below.
y

5 f

1
1 3 5 x
−1

−3

Using the points (2, −2) and (3, 0), the slope of the tangent line is
0 − (−2) 0+2 2
m= = = =2
3−2 1 1
170 Chapter 3. Limits and Derivatives

So f 0 (3) ≈ 2. Plotting the point (3, 2), we get


y

5 f

1
1 3 5 x
−1

−3

– Select another point in the interval (2, ∞). Choose for example the point
x = 4 (the corresponding point on the graph of f is (4, 3)). Using the same
approach as before, an estimate of the derivative at 4 is m = 4. So f 0 (4) ≈ 4.
Plotting the point (4, 4), we get
y

5 f

1
1 3 5 x
−1

−3

ˆ Connecting the points (0, −4), (1, −2), (2, 0), (3, 2) and (4, 4), we get the graph
of the derivative of f :
y
f f0
5

1
−1 1 3 5 7 x

−3

−5

5. [Exercise on page 25]


Solution. We use the same method/approach as in Exercise 4.
ˆ First, observe that the tangent line to the graph of f is horizontal when x = −1.
So f 0 (−1) = 0. Plotting the point (−1, 0), we have the following.
3.8. Graphical Differentiation 171

3
2
1

−3 −2 −1 1 2 x
−1
−2

ˆ The point x = −1 determines two intervals: (−∞, −1) and (−1, ∞).
ˆ Choose two points (or more if we want to improve the accuracy of the graph)
in the interval (−∞, −1). Also choose two points in the interval (−1, ∞). Then
estimate the derivative at those points. Select for example the points −3, −2, 0,
and 1. The estimates at these points are given in the following table. (See
Exercise 4 where we cover some estimates of the derivative in detail.)

x −3 −2 −1 0 1
f 0 (x) ≈ 4 2 0 −2 −4

ˆ Plotting the points (−3, 4), (−2, 2), (−1, 0), (0, −2), (1, −4) and connecting them,
we get the graph of the derivative as shown in the following figure.

y
0
f 4

−4 −2 2 x

−2

−4 f

6. [Exercise on page 25]

Solution. We use the same method/approach as in Exercise 4.

ˆ First, observe that the tangent line at (x, f (x)) is horizontal when x = 0 or
x = 2. This means that f 0 (0) = 0 and f 0 (2) = 0. Plotting the points (0, 0) and
(2, 0), we get the following.
172 Chapter 3. Limits and Derivatives

y
4
3
2
1

−1 1 2 3 4 x
−1
−2

ˆ The points 0 and 2 determine three intervals: (−∞, 0), (0, 2), and (2∞).
– On the interval (−∞, 0), tangent lines have a negative slope. So f 0 (x) < 0
for x < 0. This means that the graph of the derivative is below the x-axis
on (∞, 0).
– On the interval (0, 2), tangent lines have a positive slope. So f 0 (x) > 0 for
0 < x < 2. This means that the graph of the derivative is above the x-axis
on (0, 2).
– Similarly, the graph of f 0 is below the x-axis on the interval (2, ∞).
ˆ Notice from the graph of f that if we draw tangent lines at (x, f (x)), the largest
slope occurs at x = 1. This means that the largest value of f 0 is f 0 (1).
ˆ Choose the points −0.5, 1, 2.5. The estimates of the derivative at these points
are given in the following table. (See Exercise 4 where we cover some estimates
of the derivative in detail.)
x −0.5 0 1 2 2.5
f 0 (x) ≈ −3.75 0 3 0 −3.75
ˆ Connecting the points (−0.5, −3.75), (0, 0), (1, 3), (2, 0), (2.5, −3.75), we get the
graph of f 0 as shown in the following figure.
y
4

−1 1 3 x

−2

−4 f0 f

7. [Exercise on page 25]


Solution. This is similar to Exercise 6. From the graph of f , we have the following
estimates of the derivative.
3.8. Graphical Differentiation 173

x −2 −1 0.5 2 3
0
f (x) ≈ 4 0 −2.25 0 4

Connecting the points (−2, 4), (−1, 0), (0.5, −2.25), (2, 0), (3, 4), we get the graph of
f 0 as shown in the following figure.

y
f0 f
4

−4 −2 2 4 x
−2

−4

8. [Exercise on page 26]


Solution.

y
4

−2 2 4x

−2

ˆ First, observe that the point (1, 3) is a sharp point or corner. (Remember that
a sharp point is a point where the tangent line from the left does not match the
tangent line from the right.) This means that f is not differentiable at 1. So
f 0 (1) does not exist.
ˆ The point x = 1 defines two intervals: (−∞, 1) and (1, ∞).
ˆ On the interval (∞, 1), the graph of f is a straight line. This means that the
tangent line at every (x, f (x)), x < 1, coincides with the graph of f . Using the
points (−2, 0) and (0, 2) on the graph of f , the slope is
2−0 2 2
m= = = =1
0 − (−2) 0+2 2

So f 0 (x) = 1 for every x < 1.


174 Chapter 3. Limits and Derivatives

ˆ Similarly, on the interval (1, ∞), the derivative is constant. Using the points
(2, 1) and (3, −1) on the graph of f , we find

−1 − 1 −2
m= = = −2
3−2 1
So f 0 (x) = −2 for every x > 1.

We thus obtain an explicit formula for the derivative:



 1 if x < 1
0
f (x) = does not exist if x = 1
−2 if x > 1

The graph of f 0 is shown in the following figure.

y
4

2
f0
f

−2 2 4 x
f0
−2

9. [Exercise on page 26]


Solution.

y
4

−6 −4 −2 2 x

ˆ First, we observe from the graph of f that the tangent line is horizontal when
x = −1. So f 0 (−1) = 0. We also observe that f is not differentiable at −2 and
0 as the points (−2, 1) and (0, 1) are sharp points. So f 0 (−2) and f 0 (0) do not
exist.
3.8. Graphical Differentiation 175

ˆ The points −2, −1, and 0 determine four intervals: (−∞, −2), (−2, −1), (−1, 0),
and (0, ∞).

ˆ On the interval (−∞, −2), the graph of f is a straight line. So f 0 is constant for
x < −2. Using the points (−5, 0) and (−2, 1), we find

1−0 1 1
m= = =
−2 − (−5) −2 + 5 3

This means that f 0 (x) = 1


3
for every x < −2.

ˆ On the interval (−2, −1), the graph is not a straight line. So we need to estimate
the derivative at some points. Choose for example x = −1.5. Drawing the
tangent line at the point where x = −1.5, choosing two points on the tangent
line, we get the estimate f 0 (−1.5) ≈ 2.

ˆ On the interval (−1, 0), we choose x = −0.5. An estimate of the derivative at


the selected point is f 0 (−0.5) ≈ −2.

ˆ On the interval (0, ∞), the graph of f is a straight line. This means that f 0 is
constant for x > 0. Using the points (0, 1) and (2, 0), we get

0−1 −1
m= =
2−0 2

So f 0 (x) = − 12 for every x > 0.

ˆ Putting all the above information together, we get the graph of f 0 as shown in
the following figure.

y
4

2
0
f

−6 −4 −2 2 4 x
f
−2

−4

10. [Exercise on page 26]

Solution.
176 Chapter 3. Limits and Derivatives

y
3

−5 5 x

ˆ First, observe that the tangent line at 0 is a vertical line. Since the slope of a
vertical line is undefined, it follows that f is not differentiable at 0. So f 0 (0)
does not exist (DNE).

ˆ The point x = 0 determines two intervals: (∞, 0) and (0, ∞).

ˆ On the intervals (∞, 0) and (0, ∞), we need to estimate the derivative at some
points. Choose for example the points −5, −1, 1, 5. Estimating the derivative at
these points, we get the following table.

x −5 −1 0 1 5
0
f (x) ≈ −0.114 −0.33 DNE 0.33 0.114

Plotting these points, and noticing from the graph of f that the slope of the
tangent line at (x, f (x)) approaches ∞ as x approaches 0, we get the graph of
the derivative.

f
2

f0
−5 5 x

−2

11. [Exercise on page 27]


3.8. Graphical Differentiation 177

y
4

−2 2 4 6 x

(a) Solution. From the graph, the tangent line at x = 0 has a negative slope. This
means that f 0 (0) < 0. On the other hand, f 0 (2) = 0 since the tangent line to
the graph of f at 2 is horizontal. So f 0 (2) is bigger than f 0 (0).
(b) Solution. From the graph the slope of the tangent line at x = 1.5 is positive,
while the slope at x = 4 is 0. So f 0 (1.5) is bigger than f 0 (4).
(c) Solution. By definition, the instantaneous rate of change of f when x = 0.5 is
f 0 (0.5). But f 0 (0.5) is the slope the line through the points (−1, 3) and (0, 2),
that is,
2−3 −1 −1
f 0 (0.5) = = = = −1
0 − (−1) 0+1 1
So the instantaneous rate of change of f when x = 0.5 is −1.
(d) Solution.
ˆ For x < 1, the graph of f is a straight line. And according to the answer
from part (c), f 0 (x) = −1 for all x < 1.
ˆ For x > 3, the graph of f is a horizontal line. This means that f 0 (x) = 0
for every x > 3
ˆ On the interval (1, 3), we choose the points 1.5, 2, 2.5, and the estimates of
the derivative at these points are given in the following table.
x 1.5 2 2.5
0
f (x) ≈ 0.75 0 0.75
ˆ Using the facts that f (x) = −1 for x < 1, f 0 (x) = 0 for x > 3, and
0

the points (1.5, 0.75), (2, 0), (2.5, 0.75), we get the graph of the derivative as
shown in the following figure.
y
4
f

f0
−2 2 4 6 x
178 Chapter 3. Limits and Derivatives

12. [Exercise on page 27]

−4 −2 2 4x
−2

−4

(a) Solution. From the graph of f , the slope of the tangent line at x = −0.5 is
bigger than the slope of the tangent line at x = 2. So f 0 (−0.5) > f 0 (2).

(b) Solution. Estimating the derivative at the points −3, −1, −0.5, 0.5, 1, 3, we get
the following table.

x −3 −1 −0.5 0.5 1 3
0
f (x) ≈ 0.11 1 4 4 1 0.11

Connecting these points, we get the graph of f 0 :

2
f0
−4 −2 2 f 4x
−2

−4

13. [Exercise on page 28]

Solution.
3.8. Graphical Differentiation 179

(b) 10 (a)

−2 2 4 6x

−5

We will use the fact if the tangent line to a graph at x = a is horizontal, then the
graph of the derivative crosses the x-axis at x = a. So we will look at the points
where the tangent line is horizontal.
Consider the graph (a). The tangent line is horizontal at the points where x = −1
or x = 1 or x = 4. Since the graph (b) crosses the x-axis exactly at the same points
(−1, 1, and 4), it follows that (b) is the derivative of (a). So (a) is the graph of f
and (b) is the graph of f 0 .

14. [Exercise on page 28]


Solution.

y
(a) 5 (b)

−4 −2 2 x

−5

−10

−15

From the graph of (a), we see that the tangent line is horizontal at three points
including the point where x = −1. Since (b) does not cross the x-axis at x = −1, it
follows that (b) cannot be the derivative of (a). So (a) is the derivative of (b). This
means that (b) is the graph of f and (a) is the graph of f 0 .

15. [Exercise on page 28]


Solution. As in Exercise 13, we use the fact that if the graph of f has a horizontal
tangent line at the point where x = a, then the graph of f 0 must cross the x-axis at
the same point. This implies that if there are n points where the tangent line to the
graph of f is horizontal, then the graph of f 0 will cross the x-axis at n points.
180 Chapter 3. Limits and Derivatives

ˆ From the graph (a), we observe that the tangent line is horizontal exactly at one
point (x = 0). So the graph of the derivative of (a) crosses the x-axis exactly at
one point as well. Only (iv) has that property. Thus, (iv) is the derivative
of (a).
ˆ The graph (b) has exactly two points where the tangent line is horizontal. So
the graph of its derivative intersects the x-axis exactly at two points. Only (i)
has that property. This means that (i) is the derivative of (b).
ˆ The graph (c) is not differentiable at 0 (because it is a sharp point). This means
that the graph of its derivative is not defined at 0. Only (ii) has that property.
So (ii) is the derivative of (c).
ˆ Lastly, (iii) is the derivative of (d).

16. [Exercise on page 29]


Solution.

ˆ The graphs in (a) and (c) look similar. They have a horizontal tangent line at
0. This means that the graphs of their derivatives cross the x-axis at the origin.
Only (ii) and (iv) have that property. But (c) is not differentiable at two points,
while (a) is differentiable everywhere. So the derivative of (c) is undefined at
two points and the derivative of (a) is defined everywhere. This implies that
(iv) is the derivative of (c) and (ii) is the derivative of (a).
ˆ Now, the derivative of (b) is either (i) or (iii). From the graph of (b), observe
that the tangent lines have a negative slope for x < 0 and a positive slope for
x > 0. This means that the graph of the derivative of (b) is below the x-axis for
x < 0, and above the x-axis for x > 0. This corresponds to (iii). So (iii) is the
derivative of (b). Therefore, (i) is the derivative of (d).

17. [Exercise on page 30]

(a) Solution. First, remember that the rate of change is the derivative and the
derivative is the slope of the tangent line. From the graph of P , the tangent
lines have a positive slope for 0 ≤ x < 3. So the rate of change of profit is
positive on the interval [0, 3). (Note that 3 is not included because P 0 (3) = 0.)
(b) Solution. From the graph of P , we see that the tangent line is horizontal at
3. This means that P 0 (3) = 0. So the rate of change of profit at the level of
production of 300 units is $0/unit.
(c) Solution. Estimating the derivative of P (or the marginal profit) at the points
1, 2, 3, 4, we get the following table.
x 1 2 3 4
0
P (x) ≈ 1.33 0.67 0 −0.67

Connecting the points (1, 1.33), (2, 0.67), (3, 0), (4, −0.67), we get the graph of
the marginal profit as shown in the following figure.
3.8. Graphical Differentiation 181

2
Profit
1

1 2 3 4 x

−1 Marginal profit

The graph of the marginal profit tells us two things.


ˆ The marginal profit is decreasing as the number of units increases.
ˆ The marginal profit (or rate of change or the derivative of P ) is negative
if more than 3 × 100 = 300 units are sold. This means that increasing
production beyond that level will reduce profit.
182 Chapter 3. Limits and Derivatives
Chapter4

Differentiation Rules

4.1 Basic Rules of Differentiation


1. [Exercise on page 31]

(a) Solution.

Constant Rule
The derivative of a constant is 0. That is, if k is a constant, then
d
[k] = 0.
dx

Since f (x) = 20 is a constant function, it follows that f 0 (x) = d


dx
[20] = 0.
(b) Solution. We have f 0 (x) = 0 since 34 is a constant (34 is a constant because it
does not depend on x).
(c) Solution. Since π = 3.14159 · · · is a constant, it follows that π 2 = π × π is also
a constant. Therefore, f 0 (x) = dx
d
[π 2 ] = 0 (NOT 2π !).
(d) Solution.

Power Rule
For any real number n,
d n
[x ] = nxn−1 .
dx

For the function f (x) = x6 , n = 6. Using the power rule, we get


d 6
f 0 (x) = [x ] = 6x6−1 = 6x5 .
dx

(e) f (t) = t100 .


Solution. Using the power rule, we get
d  100 
f 0 (t) = t = 100t100−1 = 100t99 .
dt
183
184 Chapter 4. Differentiation Rules

(f ) Solution.

The Constant Multiple Rule

Let u(x) be a differentiable function, and let k be a constant. Then the


function ku(x) is differentiable, and

d d
[ku(x)] = k [u(x)].
dx dx
In words, this says that the derivative of a constant times a function is
the constant times the derivative of the function.

Let’s find the derivative of f (x) = 3x−2 .

d
f 0 (x) = [3x−2 ]
dx
d
= 3 [x−2 ] Constant Multiple Rule
dx
= 3 −2x−2−1

Power Rule
6 1
= 3 −2x−3 = −6x−3 = − 3 x−m =

x xm

(g) Solution.

d x2
   
0 d 1 2 a 1
f (x) = = x = a
dx 3 dx 3 b b
1 d  2
= x Constant Multiple Rule
3 dx
1
= (2x2−1 ) Power Rule
3
1 2
= (2x1 ) = x
3 3

(h) Solution.
 
0 d 1 4 1 d h 4i
f (t) = t =3 t3 Constant Multiple Rule
dt 2 2 dt
 
1 4 4 −1
= t 3 Power Rule
2 3
4 1 a a−b
= t3 −1=
6 b b
2 1
= t3
3
4.1. Basic Rules of Differentiation 185

(i) Solution.

d h√ 3 i d h 3i √ m
f 0 (x) = x = x2 xm = x 2
dx dx
3 3
= x 2 −1 Power Rule
2
3 1 3√ a a−b 1 √
= x2 = x −1= and x 2 = x
2 2 b b

(j)
 
0 d 2 d  −3  1
f (x) = 3
= 2x = x−n
dx x dx xn
d −3
=2 [x ] = 2(−3)x−3−1 Constant Multiple Rule and Power Rule
dx
−6
= −6x−4 = 4
x

(k) Solution.


   
0 d 1 d 1 1
f (x) = √ = n
x = xn
dx 3 x dx x 13
d h −1 i 1
= x 3 = x−n
dx xn
1 1
= − x− 3 −1 Power Rule
3
1 4 −1 a a−b
= − x− 3 = 4 −1=
3 3x 3 b b

(l) Solution.

d hp i d h 5i √ m
f 0 (p) = 3
p5 = p3 n
pm = p n
dp dp
5 5 5 2 a a−b
= p 3 −1 = p 3 Power Rule and −1=
3 3 b b

(m) Solution.
√

  
0 d 8
x d 1 1 n
1
f (x) = = x8 x = xn
dx 2 dx 2
 
1 d h 1i 1 1 1 −1
= x8 = x8 Constant Multiple Rule and Power Rule
2 dx 2 8
1 −7 1 1 1 a a−b 1
= x 8 = 7 = 7 −1= and x−n = n
16 16 x 8 16x 8 b b x
186 Chapter 4. Differentiation Rules

(n) Solution.

d e2
   
0 2 d 1
f (q) = =e Constant Multiple Rule
dq q dq q
d  −1  1
= e2 q = q −1
dq q
= e2 (−1)q −1−1 = e2 (−q −2 )

Power Rule
e2
 
1 1
2
=e − 2 =− 2 q −n =
q q qn

2. [Exercise on page 31]

(a) Solution.

The Sum Rule


Let f and g be two differentiable functions. Then the function f + g is
differentiable and
d d d
[f (x) + g(x)] = [f (x)] + [g(x)] .
dx dx dx
In words, this says that the derivative of a sum is the sum of the deriva-
tives.

d
f 0 (x) = [x + 1]
dx
d d
= [x] + [1] Sum Rule
dx dx
=1+0=1

(b) Solution.

d
f 0 (x) = [3x2 + 11x]
dx
d d
= [3x2 ] + [11x] Sum Rule
dx dx
d d
= 3 [x2 ] + 11 [x] Constant Multiple Rule
dx dx
d n
= 3(2x) + 11(1) = 6x + 11 [x ] = nxn−1
dx

(c) Solution.
4.1. Basic Rules of Differentiation 187

The Difference Rule


Let u and v be two differentiable functions. Then the function u − v is
differentiable and
d d d
[u(x) − v(x)] = [u(x)] − [v(x)] .
dx dx dx
In words, this says that the derivative of a difference is the difference of
the derivatives.

d d d
f 0 (x) = [x − 5] = [x] − [5] Difference Rule
dx dx dx
=1−0=1

(d) Solution.
 
0 d 2 2z
f (z) = z −
dz 5
 
d  2 d 2z
= z − Difference Rule
dz dz 5
2 d
= 2z − [z] Power Rule and Constant Multiple Rule
5 dz
2 2
= 2z − (1) = 2z −
5 5

(e) Solution.
√ 
√
  
0 d t− t d 1
f (t) = = t− t
dt 2 dt 2
√i 1 d d h√ i
 
1d h Constant Multiple Rule and
= t− t = [t] − t
2 dt 2 dt dt Difference Rule

 
1 d h 1i 1
= 1− t2 t = t2
2 dt
   
1 1 1 −1 1 1 −1
= 1− t 2 = 1− t 2 Power Rule and ab − 1 = a−b
b
2 2 2 2
 
1 1 1 1
= 1− 1 t−n =
2 2 t2 tn
  √
1 1 2 t−1 b a b ac − b
= 1− √ = √ a− = − =
2 2 t 4 t c 1 c c
188 Chapter 4. Differentiation Rules

(f ) Solution.

d 
f 0 (x) = −3x4 − 2x3 + x2 − 1

dx
d  d  3 d  2 d Sum and Difference
−3x4 −

= 2x + x − [1]
dx dx dx dx Rules
d d d d
= −3 [x4 ] − 2 [x3 ] + [x2 ] − [1] Constant Multiple Rule
dx dx dx dx
Power Rule and Con-
= −3(4x3 ) − 2(3x2 ) + 2x − 0
stant Rule
= −12x3 − 6x2 + 2x

(g) Solution.

d 2x3 − 3x2
   
0 d 1 3 2

f (x) = = 2x − 3x
dx 4 dx 4
1 d  3
2x − 3x2

= Constant Multiple Rule
4 dx
 
1 d 3 d 2
= [2x ] − [3x ] Difference Rule
4 dx dx
1 Constant Multiple Rule and
2(3x2 ) − 3(2x)

=
4 Power Rule
1 3 3
= (6x2 − 6x) = x2 − x
4 2 2

(h) Solution.

d h 5 2
i d h 5i d h 2i
f 0 (x) = x3 − x3 = x3 − x3 Difference Rule
dx dx dx
5 5 2 2
= x 3 −1 − x 3 −1 Power Rule
3 3
5 2 2 1 5 2 2 a a−b
= x 3 − x− 3 = x 3 − 1 −1=
3 3 3 3x 3 b b

(i) Solution.
   
0 d 2 1 d 2 d 1
f (x) = x − = [x ] − Difference Rule
dx x dx dx x
d
  1
1 Power Rule and dx =
= 2x − − 2 x
x d
dx
[x−1 ] = −x−2 = − x12
1
= 2x +
x2
4.1. Basic Rules of Differentiation 189

(j) Solution.
d
f 0 (x) = [1.4x5 − 2.5x2 + 3.8]
dx
d d d
= [1.4x5 ] − [2.5x2 ] + [3.8] Sum and Difference Rules
dx dx dx
d d d
= 1.4 [x5 ] − 2.5 [x2 ] + [3.8] Constant Multiple Rule
dx dx dx
= 1.4(5x4 ) − 2.5(2x) + 0 = 7x4 − 5x Power Rule and Constant Rule

(k) Solution.

   
0 d x d x 1
f (x) = √ = x = x2
dx x dx x 12
d h 1− 1 i xm
= x 2 n
= xm−n
dx x
d h 1 i 1 −1 1
= x2 = x 2 = √ Power Rule
dx 2 2 x

(l) Solution.
√  √ 
0 d x+x d x x a+b a b
f (x) = = + 2 = +
dx x2 dx x2 x c c c
" 1 #
d x2 x d h 1 −2 1−2
i √ 1 xm
= + = x 2 + x x = x 2 and n = xm−n
dx x2 x2 dx x
d h −3 −1
i d h −3 i d  −1 
= x +x
2 = x 2 + x Sum Rule
dx dx dx
3 3 3 5
= − x− 2 −1 + (−1)x−1−1 = − x− 2 − x−2 Power Rule
2 2
3 1 1
=− 5 − 2 x−n =
2x 2 x xn

3. [Exercise on page 31]


Solution. Let f (x) = 2x3 − x2 + 2.

ˆ First, we need to find the derivative:


d d d d
f 0 (x) = [2x3 − x2 + 2] = [2x3 ] − [x2 ] + [2] Sum Rule
dx dx dx dx
Constant Multiple Rule, Power
= 2(3x2 ) − 2x − 0 = 6x2 − 2x.
Rule, and Constant Rule

ˆ The slope of the tangent line at (1, 3) is

m = f 0 (1) = 6(1)2 − 2(1) = 6 − 2 = 4.


190 Chapter 4. Differentiation Rules

ˆ So the equation of the tangent line at P (1, 3) is y − 3 = m(x − 1), that is,
y − 3 = 4(x − 1). This equation is equivalent to y − 3 = 4x − 4. Adding 3 to
both sides, we get y = 4x − 1.

4. [Exercise on page 31]


Solution.

ˆ The tangent line is horizontal when the derivative is 0. And the derivative is
dy d 3 d 3 d d
= [x − 3x + 1] = [x ] − 3 [x] + [1]
dx dx dx dx dx
2
= 3x − 3.

ˆ Set dy
dx
= 0. Then

3x2 − 3 = 0
3(x2 − 1) = 0 Factor
3(x − 1)(x + 1) = 0 a2 − b2 = (a − b)(a + b)
x = 1 or x = −1

ˆ The y-coordinate for x = −1 is

f (−1) = (−1)3 − 3(−1) + 1 = −1 + 3 + 1 = 3.

This gives the point (−1, 3).


ˆ The y-coordinate for x = 1 is

f (1) = 13 − 3(1) + 1 = 1 − 3 + 1 = −1.

This gives the point (1, −1).

Thus, the points where the tangent line to the curve y = x3 − 3x + 1 is horizontal are
(−1, 3) and (1, −1).

5. [Exercise on page 31]

(a) Solution.

Marginal Analysis – Marginal Cost, Marginal Revenue, and


Marginal Profit

Let C(x) be the cost of producing x units of a certain product, and let
R(x) be the revenue function. Then the profit function, P (x), is given by
P (x) = R(x) − C(x).

ˆ The marginal cost function is the derivative of C(x). That is,

d
Marginal Cost = C 0 (x) = [C(x)]
dx
4.1. Basic Rules of Differentiation 191

ˆ The The marginal revenue function is the derivative of R(x). That


is,
d
Marginal Revenue = R0 (x) = [R(x)]
dx
ˆ The marginal profit function is the derivative of the profit function.
That is,
d
Marginal Profit = P 0 (x) = [P (x)]
dx

The marginal cost function is


d d d d
C 0 (x) = [3x2 + 75x + 400] = [3x2 ] + [75x] + [400]
dx dx dx dx
d d
= 3 [x2 ] + 75 [x] + 0 = 3(2x) + 75(1) = 6x + 75
dx dx
(b) Solution.

How to Interpret the Marginal Cost, Marginal Revenue, and


Marginal Profit

Let C(x) be the cost of producing x units of some item.

ˆ Then the cost of producing x + 1 units is C(x + 1).

ˆ If x units have been produced, the cost of producing one more unit
is C(x + 1) − C(x). That is,

Cost of producing the (x + 1)st unit = C(x + 1) − C(x)

The figure below shows that the marginal cost C 0 (x) is close to C(x + 1) −
C(x). When x is very large, C 0 (x) is very close to C(x + 1) − C(x).
y
y = C(x)

C(x + 1)

C(x + 1) − C(x)
C 0 (x)
P
C(x)
1

x x+1 x
192 Chapter 4. Differentiation Rules

The conclusion here is that the marginal cost, C 0 (x), can be


interpreted as the approximate cost of producing the (x + 1)st
unit. In other words, the actual cost for producing the (x + 1)st unit is
approximately C 0 (x).

Similarly, if R(x) and P (x) represent the revenue and profit functions
respectively,

ˆ the marginal revenue, R0 (x), can be interpreted as the ap-


proximate revenue of producing the (x + 1)st unit.

ˆ The marginal profit, P 0 (x), can be interpreted as the ap-


proximate profit of producing the (x + 1)st unit.

From part (a) the marginal cost function is C 0 (x) = 6x + 75. When x = 4,

C 0 (4) = 6(4) + 75 = 24 + 75 = $99 per unit.

This means that the approximate cost of producing the 5th unit is $99.
(c) Solution. The marginal cost for the production level x = 35 units is

C 0 (35) = 6(35) + 75 = 210 + 75 = $285 per unit.

This means that the approximate cost of producing the 36th unit is
$285.

6. [Exercise on page 32]

(a) Solution.
ˆ We first need to find the revenue function R(q). Since the demand function
is p = −0.01q + 60 and the price per unit is p, we have that

R(q) = pq = (−0.01q + 60)q = −0.01q 2 + 60q.

ˆ The marginal revenue function is

d d d
R0 (q) = [−0.01q 2 + 60q] = −0.01 [q 2 ] + 60 [q]
dq dq dq
= −0.01(2q) + 60(1) = −0.02q + 60

(b) Solution.
ˆ First we need to find the profit function P (q).

P (q) = R(q) − C(q)


= (−0.01q 2 + 60q) − (3500 − 15q + 0.02q 2 )
= −0.01q 2 + 60q − 3500 + 15q − 0.02q 2 = −0.03q 2 + 75q − 3500.
4.2. The Product and Quotient Rules 193

ˆ The marginal profit function is


d
P 0 (q) = [−0.03q 2 + 75q − 3500]
dq
d 2 d d
= −0.03 [q ] + 75 [q] − [3500]
dq dq dq
= −0.03(2q) + 75 = −0.06q + 75

(i) The marginal profit for the production level q = 400 is

P 0 (400) = −0.06(400) + 75 = −24 + 75 = $51 per unit.

This means that the approximate profit to produce the 401st unit
is $51.
(ii) The marginal profit for the production level q = 935 is

P 0 (935) = −0.06(935) + 75 = $18.9 per unit.

One can interpret this as follows. After 935 units have been produced,
the profit to produce one more unit will be approximately $18.9.
(c) Solution. The marginal profit is 0 when −0.06q + 75 = 0. Subtracting 75
from both sides, we get −0.06q = −75. Dividing both sides by −0.06, we get
q = 1250.
(d) Solution. When the marginal profit is 0, q = 1250, and the profit is

P (1250) = −0.03(1250)2 + 75(1250) − 3500


= −46875 + 93750 − 3500 = $43375.

4.2 The Product and Quotient Rules


1. [Exercise on page 32]
(a) Solution.

The Product Rule


Let f (x) = u(x)v(x). Suppose u(x) and v(x) are both differentiable. Then
the product u(x)v(x) is differentiable and
   
d d d
[u(x)v(x)] = [u(x)] v(x) + u(x) [v(x)] .
dx dx dx

If one uses the notation u0 (x) and v 0 (x), then

f 0 (x) = u0 (x)v(x) + u(x)v 0 (x).

In words, this says that the derivative of the product of two functions is
the derivative of the first function times the second plus the first function
194 Chapter 4. Differentiation Rules

times the derivative of the second.

Warning. In general, the derivative of a product is not equal to the


product of the derivatives. That is,
d d d
[u(x)v(x)] 6= [u(x)] [v(x)].
dx dx dx

The derivative of f (x) = (5x + 7)(6x2 ) is

d
f 0 (x) = [(5x + 7)(6x2 )]
dx
d d
= [5x + 7](6x2 ) + (5x + 7) [6x2 ] Product Rule
dx dx
= (5 + 0)(6x2 ) + (5x + 7)12x
= 30x2 + 60x2 + 84x = 90x2 + 84x

Alternate Solution
One can find f 0 (x) without using the product rule:

d
f 0 (x) = [(5x + 7)(6x2 )]
dx
d
= [30x3 + 42x2 ] Distribute
dx
d d
= [30x3 ] + [42x2 ] = 90x2 + 84x
dx dx
Note. In Questions (b)–(e) below, where the given function is a product
of two functions, one can find the derivative in two different ways: (1) by
distributing first; (2) by using the product rule. We will use the product
rule. In next sections, we will see products of functions where the product
rule is essential. Examples of such functions include

f (x) = x2 (x3 + 1)8 , g(x) = xex , h(x) = x ln x.

(b) Solution.

d
f (x) = [(3x + 4)(x − 5)]
dx
d d
= [3x + 4](x − 5) + (3x + 4) [x − 5] Product Rule
dx dx
= (3 + 0)(x − 5) + (3x + 4)(1 − 0)
= 3x − 15 + 3x + 4 = 6x − 11 Distribute
4.2. The Product and Quotient Rules 195

(c) Solution.

d
f 0 (x) = [(5x2 − 2)(x3 + 3x)]
dx
d  2 d  3
5x − 2 (x3 + 3x) + (5x2 − 2)
 
= x + 3x Product Rule
dx dx
= 10x(x3 + 3x) + (5x2 − 2)(3x2 + 3)
= 10x4 + 30x2 + 15x4 + 15x2 − 6x2 − 6 Distribute
= 25x4 + 39x2 − 6

(d) Solution.

d
f 0 (x) = [(x3 + 1)(2x2 − 4x − 1)]
dx
= 3x2 (2x2 − 4x − 1) + (x3 + 1)(4x − 4) Product Rule
= (6x4 − 12x3 − 3x2 ) + (4x4 − 4x3 + 4x − 4) Distribute
= 10x4 − 16x3 − 3x2 + 4x − 4

(e) Solution.

d √
f 0 (x) = [(2 + x)(x2 − 3x)]
dx
d √ √ d
= [2 + x](x2 − 3x) + (2 + x) [x2 − 3x] Product Rule
dx dx
1 √ d √  1
= √ (x2 − 3x) + (2 + x)(2x − 3) x = √
2 x dx 2 x
√ √
x2 − 3x + 2 x(2 + x)(2x − 3) a a + bc
= √ +c=
2 x b b
√ √ √
x2 − 3x + 2 x(4x − 6 + 2x x − 3 x)
= √ Distribute
2 x
√ √
x2 − 3x + 8x x − 12 x + 4x2 − 6x
= √ Distribute
2 x
√ √
5x2 − 9x + 8x x − 12 x
= √
2 x

2. [Exercise on page 32]

(a) Solution.
196 Chapter 4. Differentiation Rules

The Quotient Rule

Let f (x) = u(x)


v(x)
with v(x) 6= 0. Suppose u(x) and v(x) are both differen-
tiable. Then f (x) is differentiable and
d d
− u(x) dx
 
d u(x) dx
[u(x)]v(x) [v(x)]
= 2
.
dx v(x) (v(x))

If one uses the notation u0 (x) and v 0 (x), then

u0 (x)v(x) − u(x)v 0 (x)


f 0 (x) = .
(v(x))2

In words, this says that the derivative of a quotient is the derivative of the
numerator times the denominator minus the numerator times the deriva-
tive of the denominator, all divided by the square of the denominator:
d d
[Top] × Bottom − Top × dx
 
d Top dx
[Bottom]
=
dx Bottom (Bottom)2

Warning. In general, the derivative of a quotient is not equal to the


quotient of the derivatives. That is,
  d
d u(x) [u(x)]
6= dx
d
.
dx v(x) dx
[v(x)]

 
0 d 5x + 1
f (x) =
dx 5x − 1
d d
dx
[5x + 1](5x − 1) − (5x + 1) dx [5x − 1]
= 2
Quotient Rule
(5x − 1)
5(5x − 1) + (5x + 1)(5)
=
(5x − 1)2
25x − 5 − (25x + 5)
= Distribute
(5x − 1)2
25x − 5 − 25x − 5 −10
= 2
=
(5x − 1) (5x − 1)2
4.2. The Product and Quotient Rules 197

(b) Solution.
 
0 d 1 + 2x
f (x) =
dx 3 − 4x
d d
dx
[1 + 2x](3 − 4x) − (1 + 2x) dx [3 − 4x]
= 2
Quotient Rule
(3 − 4x)
2(3 − 4x) − (1 + 2x)(−4) 6 − 8x − (−4 − 8x)
= =
(3 − 4x)2 (3 − 4x)2
6 − 8x + 4 + 8x 10
= 2
=
(3 − 4x) (3 − 4x)2

(c) Solution.

d x2 + 1
f 0 (x) = [ ]
dx x3 − 1
d d
dx
[x2 + 1](x3 − 1) − (x2 + 1) dx [x3 − 1]
= Quotient Rule
(x3 − 1)2
2x(x3 − 1) − (x2 + 1)(3x2 )
=
(x3 − 1)2
2x4 − 2x − (3x4 + 3x2 )
=
(x3 − 1)2
2x4 − 2x − 3x4 − 3x2 −x4 − 2x − 3x2
= =
(x3 − 1)2 (x3 − 1)2

(d) Solution.

x3 + 3x
 
0 d
f (x) =
dx x2 − 4x + 3
d d
dx
[x3 + 3x](x2 − 4x + 3) − (x3 + 3x) dx [x2 − 4x + 3]
= Quotient Rule
(x2 − 4x + 3)2
(3x2 + 3)(x2 − 4x + 3) − (x3 + 3x)(2x − 4)
=
(x2 − 4x + 3)2
(3x4 − 12x3 + 9x2 + 3x2 − 12x + 9) − (2x4 − 4x3 + 6x2 − 12x)
=
(x2 − 4x + 3)2
3x4 − 12x3 + 9x2 + 3x2 − 12x + 9 − 2x4 + 4x3 − 6x2 + 12x
=
(x2 − 4x + 3)2
x4 − 8x3 + 6x2 + 9
=
(x2 − 4x + 3)2
198 Chapter 4. Differentiation Rules

(e) Solution.
 
0 d 1
f (x) =
dx x3 + 2x2 − 1
d d
dx
[1](x3 + 2x2 − 1) − (1) dx [x3 + 2x2 − 1]
= Quotient Rule
(x3 + 2x2 − 1)2
0(x3 + 2x2 − 1) − (3x2 + 4x)
=
(x3 + 2x2 − 1)2
0 − 3x2 − 4x −3x2 − 4x
= 3 = 3
(x + 2x2 − 1)2 (x + 2x2 − 1)2

(f ) Solution.
 √ 
0 d x
f (x)
dx 3 + x
d √ √ d
dx
[ x](3 + x) − x dx [3 + x]
= 2
Quotient Rule
(3 + x)
1 √ 3+x √

2 x
(3 + x) − x(1) √ −
2 x
x
= =
(3 + x)2 (3 + x)2
(3+x)−2x

2 x 3 + x − 2x 3−x
= = √ = √
(3 + x)2 2 x(3 + x) 2 2 x(3 + x)2

(g) Solution.

d 2x5 + x4 − 6x
 
0
f (x) =
dx x
d d
dx
[2x5
+ x4 − 6x]x − (2x5 + x4 − 6x) dx [x]
= 2
Quotient Rule
x
(10x + 4x − 6)x − (2x5 + x4 − 6x)(1)
4 3
=
x2
10x5 + 4x4 − 6x − 2x5 − x4 + 6x
=
x2
8x5 + 3x4 x2 (8x3 + 3x2 )
= =
x2 x2
Simplify (this holds
= 8x3 + 3x2
for all x 6= 0)

Alternate Solution
If we first simplify the function, then we can find f 0 (x) without using the
4.2. The Product and Quotient Rules 199

Quotient Rule.

d 2x5 + x4 − 6x d 2x5 x4 6x
   
0
f (x) = = + −
dx x dx x x x
d  4
2x + x3 − 6

= Simplify (this holds for all x 6= 0)
dx
d d d
= [2x4 ] + [x3 ] − [6] = 8x3 + 3x2
dx dx dx
Note. This example shows that sometimes it is much easier to simplify
the original function first before taking its derivative.

(h) Solution. There are many ways of calculating f 0 (x): we can use the quotient
rule or simplify the function first and use the power rule or use the product rule.
We will simplify the function first because it is easier.

x2 + 4x + 3 x2 4x 3
f (x) = √ =√ +√ +√
x x x x
x2 x 3 3 1 1 √ 1 xm
= 1 +4 1 + 1 = x 2 + 4x 2 + 3x− 2 . x = x 2 and = xm−n
x2 x2 x2 xn
Now, by using the sum rule, the constant multiple rule, and the power rule, we
get
   
0 3 1 1 −1 1 −3
f (x) = x + 4
2 x 2 +3 − x 2
2 2 2
3 1 1 3 3 3 1 2 3
= x 2 + 2x− 2 − x− 2 = x 2 + 1 − 3
2 2 2 x2 2x 2

(i) Solution.
d
x + x1 − x dx d
x + x1
  
0 dx
[x]
f (x) = Quotient Rule
(x + x1 )2
x + x1 − x 1 − x12
  
d 1 1
= =− 2
(x + x1 )2 dx x x
x + x1 − x + x
x2
1
x
+ xx2
= =
(x + x1 )2 (x + x1 )2
2x
x2 2x 2
= 1 2 = 1 2 = 2 Simplify
(x + x
) x2 (x + x) x x + x1

(j) Solution. Using the quotient rule first, and then the product rule we get

d (6x + 1)(3x2 − 4)
 
0
f (x) =
dx 7x − 2
200 Chapter 4. Differentiation Rules

d d
dx
+ 1)(3x2 − 4)](7x − 2) − (6x + 1)(3x2 − 4) dx
[(6x [7x − 2]
=
(7x − 2)2
d d d

dx
[6x + 1](3x2 − 4) + (6x + 1) dx [3x2 − 4] − (6x + 1)(3x2 − 4) dx [7x − 2]
=
(7x − 2)2
(6(3x2 − 4) + (6x + 1)(6x)) (7x − 2) − (6x + 1)(3x2 − 4)(7)
=
(7x − 2)2
(18x2 − 24 + 36x2 + 6x) (7x − 2) − (18x3 − 24x + 3x2 − 4)(7)
=
(7x − 2)2
(54x2 + 6x − 24)(7x − 2) − (126x3 − 168x + 21x2 − 28)
=
(7x − 2)2
378x3 − 108x2 + 42x2 − 12x − 168x + 48 − 126x3 + 168x − 21x2 + 28
=
(7x − 2)2
252x3 − 87x2 − 12x + 76
=
(7x − 2)2

Alternate Solution
(6x+1)(3x2 −4)
Distributing the numerator of f (x) = 7x−2
, we find

18x3 + 3x2 − 24x − 4


f (x) =
7x − 2
Using now the quotient rule, we get
d d
[18x3 + 3x2 − 24x − 4](7x − 2) − (18x3 + 3x2 − 24x − 4) dx [7x − 2]
f 0 (x) = dx
2
(7x − 2)
(54x2 + 6x − 24)(7x − 2) − (18x3 + 3x2 − 24x − 4)(7)
=
(7x − 2)2
378x3 − 108x2 + 42x2 − 12x − 168x + 48 − 126x3 − 21x2 + 168x + 28
=
(7x − 2)2
252x3 − 87x2 − 12x + 76
=
(7x − 2)2

3. [Exercise on page 32]


Solution.
ˆ We first need to find the derivative:
d d
dy dx
[x3 − x](x2 + 1) − (x3 − x) dx [x2 + 1]
= Quotient Rule
dx (x2 + 1)2
(3x2 − 1)(x2 + 1) − (x3 − x)(2x)
=
(x2 + 1)2
4.2. The Product and Quotient Rules 201

3x4 + 3x2 − x2 − 1 − (2x4 − 2x2 )


=
(x2 + 1)2
3x4 + 2x2 − 1 − 2x4 + 2x2 x4 + 4x2 − 1
= = .
(x2 + 1)2 (x2 + 1)2
ˆ The slope is obtained by substituting x with 1 into the derivative:
(1)4 + 4(1)2 − 1 1+4−1 4 4
m= 2 2
= 2
= 2 = = 1.
((1) + 1) (1 + 1) 2 4
ˆ The equation of the tangent line at P (1, 0) is y−0 = 1(x−1). That is, y = x−1.
4. [Exercise on page 32]
(a) Solution.

Average Cost and Marginal Average Cost

Let C(x) be the cost to manufacture x items.

ˆ The average cost per item, denoted C(x), is

C(x)
C(x) = .
x

ˆ The marginal average cost is the derivative of the average cost func-
tion. That is,

0 d  
Marginal average cost = C (x) = C(x)
dx

ˆ For C(x) = 4000 + 3x, the average cost function is C(x) = 4000+3x
x
.
4000+3(8)
(i) When x = 8, C(8) = 8
= 503.
4000+3(20)
(ii) When x = 20, the average cost is C(20) = 20
= 203.
(b) Solution. The marginal average cost function is
 
0 d   d 4000 + 3x
C (x) = C(x) =
dx dx x
d d
dx
[4000+ 3x]x − (4000 + 3x) dx [x]
= 2
Quotient Rule
x
(0 + 3)x − (4000 + 3x)(1)
=
x2
3x − 4000 − 3x −4000
= 2
=
x x2
(c) Solution. Remembering that the rate of change of a function is the derivative
of that function, the rate of change of the average cost when x = 15 is
0 4000 4000
C (15) = − 2 = − ≈ −17.78
15 225
202 Chapter 4. Differentiation Rules

This means that at the level of production of 15 units, the average cost is de-
creasing at the rate of $17.78 per unit.
5. [Exercise on page 33]
(a) Solution. First, the average cost is
4x2 +100 a
C(x) 3x+2 4x2 + 100 b a
C(x) = = = =
x x (3x + 2)x c bc
4x2 + 100
= .
3x2 + 2x
The marginal average cost is
d 4x2 + 100
 
0 d
C (x) = [C(x)] =
dx dx 3x2 + 2x
d d
dx
[4x2 + 100](3x2 + 2x) − (4x2 + 100) dx [3x2 + 2x]
= Quotient Rule
(3x2 + 2x)2
(8x)(3x2 + 2x) − (4x2 + 100)(6x + 2)
=
(3x2 + 2x)2
24x3 + 16x2 − 24x3 − 8x2 − 600x − 200
=
(3x2 + 2x)2
8x2 − 600x − 200
=
(3x2 + 2x)2
(b) Solution.

Marginal Average Revenue and Marginal Average Profit

Let R(x) and P (x) be the revenue and profit functions respectively.
R(x)
ˆ The average revenue per item, denoted R(x), is R(x) = x
.

ˆ The marginal average revenue is the derivative of the average revenue


function.
P (x)
ˆ The average profit per item, denoted P (x), is P (x) = x
.

ˆ The marginal average profit is the derivative of the average profit


function.

R(x) 7x+200
First, the average revenue is R(x) = x
= x
. The marginal average
revenue is
 
0 d   d 7x + 200
R (x) = R(x) =
dx dx x
d d
dx
[7x + 200]x − (7x + 200) dx [x] (7)x − (7x + 200)(1) Quotient
= 2
=
x x2 Rule
4.3. The Chain Rule 203

7x − 7x − 200 −200
= =
x2 x2
(c) Solution.
ˆ First, the profit function is
4x2 + 100
P (x) = R(x) − C(x) = 7x + 200 −
3x + 2
2
(7x + 200)(3x + 2) − (4x + 100)
=
3x + 2
21x2 + 14x + 600x + 400 − 4x2 − 100 17x2 + 614x + 300
= = .
3x + 2 3x + 2
ˆ The average profit is
P (x) 17x2 + 614x + 300 17x2 + 614x + 300
P (x) = = = .
x x(3x + 2) 3x2 + 2x
0
Now the marginal average profit, P (x), is the derivative of the average profit function.
d
Using the quotient rule and the fact that dx [17x2 + 614x + 300] = 34x + 614 and
d
dx
[3x2 + 2x] = 6x + 2, we get

0 (34x + 614)(3x2 + 2x) − (17x2 + 614x + 300)(6x + 2)


P (x) =
(3x2 + 2x)2
102x3 + 68x2 + 1842x2 + 1228x − (102x3 + 34x2 + 3684x2 + 1228x + 1800x + 600)
=
(3x2 + 2x)2
102x3 + 1910x2 + 1228x − 102x3 − 3718x2 − 3028x − 600
=
(3x2 + 2x)2
−1808x2 − 1800x − 600
=
(3x2 + 2x)2

4.3 The Chain Rule


1. [Exercise on page 33]
(a) Solution.

Composition of Functions
Suppose f and g are functions. Assume that for all x in the domain of f ,
f (x) is in the domain of g. The composition of g and f , denoted g ◦ f , is
the function defined as

(g ◦ f )(x) = g(f (x)).

Similarly, one can define f ◦ g as

(f ◦ g)(x) = f (g(x)).
204 Chapter 4. Differentiation Rules

ˆ To find g(f (x)), we first need to find f (x).

ˆ To find f (g(x)), we first need to find g(x).

(i) Finding g(f (0)).


ˆ First, f (0) = 02 − 3(0) = 0.
ˆ Then g(f (0)) = g(0) = 5(0) + 4 = 4.
(ii) First, f (1) = 12 − 3(1) = 1 − 3 = −2. Then
g(f (1)) = g(−2) = 5(−2) + 4 = −10 + 4 = −6.
(iii) To find f (g(−2)), we first need to find g(−2). Substituting x with −2 into
g(x), we get g(−2) = 5(−2) + 4 = −10 + 4 = −6. Substituting now x with
−6 into f (x), we get
f (g(−2)) = f (−6) = (−6)2 − 3(−6) = 36 + 18 = 54.
(iv) First, g(3) = 5(3) + 4 = 15 + 4 = 19. Then
f (g(3)) = f (19) = 192 − 3(19) = 361 − 57 = 304.
(b) Solution. Substituting x with f (x) into g(x), we get
g(f (x)) = g(x2 − 3x) = 5(x2 − 3x) + 4 = 5x2 − 15x + 4.

(c) Solution.
f (g(x)) = f (5x + 4) = (5x + 4)2 − 3(5x + 4)
= (5x)2 + 2(5x)(4) + 42 − 3(5x + 4) (a + b)2 = a2 + 2ab + b2
= 25x2 + 40x + 16 − 15x − 12 = 25x2 + 25x + 4

2. [Exercise on page 33]

(a) Solution.

Inner and Outer Functions


ˆ Let f and g be two functions. If y = f (g(x)), the function g(x) is
called the inside or inner function, and the function f (x) is called
the outer function.

ˆ If y has the form y = [g(x)]n , where n is a real number, the inner


function if g(x) and the outer function is f (x) = xn .

ˆ For example, if y = (3x + 4)7 , the inner function is g(x) = 3x + 4


and the outer function is f (x) = x7 .

To write h(x) as the composition of two functions, the idea is to find the inner
and outer functions. For h(x) = (x2 + 3)5 , the inner function is g(x) = x2 + 3
and the outer function is f (x) = x5 . So h(x) = f (g(x)).
4.3. The Chain Rule 205

1
(b) Solution. Here h(x) = (5x + 4) 2 , and the inner function is g(x) = 5x + 4 and
1 √
the outer function is f (x) = x 2 = x. So h(x) = f (g(x)).

(c) Solution. Here h(x) = f (g(x)) where g(x) = 10 + 7x and f (x) = − 3 x.
(d) Solution. The function
1 3
h(x) = (x2 + x) 4 − 3(x2 + x) 4 + 11
1 3
can be written as h(x) = f (g(x)), where g(x) = x2 +x and f (x) = x 4 −3x 4 +11.

Alternate Solution
Rewriting h(x) as
1 1
h(x) = (x2 + x) 4 − 3[(x2 + x) 4 ]3 + 11,
1
we have h(x) = f (g(x)) where g(x) = (x2 + x) 4 and f (x) = x − 3x3 + 11.

3. [Exercise on page 33]

(a) Solution.

The Chain Rule


Let y = f (u) be a function of u and u = g(x) be a function of x. Then
y = f (g(x)) is a function of x. The inner function is u = g(x) and the
outer function is y = f (u).
ˆ Suppose that f (u) and g(x) are both differentiable. The chain rule
says that the derivative of y with respect to x is the derivative of the
outer function (with respect to u) times the derivative of the inner
function (with respect to x). That is,
dy dy du
= · (4.3.1)
dx du dx
ˆ Using the fact that
dy d dy d
= [f (g(x))], = [f (u)] = f 0 (u) = f 0 (g(x)),
du dx du du
du d
and dx
= dx
[g(x)], Equation (4.3.1) can be rewritten as
d
[f (g(x))] = f 0 (g(x)) · g 0 (x) (4.3.2)
dx
This latter equation is an alternate form of the chain rule.
An important consequence of the chain rule is the following. Suppose f
dy
is of the form f (u) = un , where n is a real number, then du = nun−1 by
the power rule. Therefore, Equation (4.3.1) becomes
d d
[(g(x))n ] = n[g(x)]n−1 [g(x)] (4.3.3)
dx dx
206 Chapter 4. Differentiation Rules

Equation (4.3.3) is known in the literature as the generalized power rule.


But we will still refer to it as the chain rule.
Warning. While using the chain rule, do not forget to multiply by the
derivative of the inner function.

For y = (2x)5 , the inner function is u = 2x and the outer function is y = f (u) =
u5 . Using the chain rule, we get

dy dy du
= ·
dx du dx
d 5 d
= [u ] [2x]
du dx
4
= (5u )(2)
= 10u4 = 10(2x)4 Replace u with 2x

Alternate Solution
Since y is of the form y = (g(x))n with g(x) = 2x, we can use (4.3.3):

dy d  d
(2x)5 = 5(2x)5−1 [2x]

= Chain Rule
dx dx dx
d
= 5(2x)4 (2) = 10(2x)4 [2x] = 2
dx

(b) Solution. Since y is of the form y = (g(x))n with g(x) = −3x + 4, we can use
Equation 4.3.3:

dy d  d
(−3x + 4)8 = 8(−3x + 4)7 [−3x + 4]

= Chain Rule
dx dx dx
d
= 8(−3x + 4)7 (−3) [−3x + 4] = −3
dx
= 8(−3)(−3x + 4)7 = −24(−3x + 4)7

(c) Solution.

dy d h 2 3
i 3 3 d
= (x − 1) = (x2 − 1) 2 −1 [x2 − 1]
2 Chain Rule
dx dx 2 dx
3 1 3 1
= (x2 − 1) 2 (2x) = (2x)(x2 − 1) 2
2 2
1
= 3x(x2 − 1) 2

(d) Solution.

dy d 
(−x3 + 2x + 1)15

=
dx dx
4.3. The Chain Rule 207

d
= 15(−x3 + 2x + 1)14 [−x3 + 2x + 1] Chain Rule
dx
= 15(−x + 2x + 1) (−3x2 + 2)
3 14

= 15(−3x2 + 2)(−x3 + 2x + 1)14

(e) Solution.

dy d 
3(5x − 1)7

=
dx dx
d 
(5x − 1)7

=3 Constant Multiple Rule
dx
d
= 3(7)(5x − 1)6 [5x − 1] Chain Rule
dx
= 21(5x − 1) (5) = 105(5x − 1)6
6

(f ) Solution.

dy d 
−2(5x6 − 2x)4

=
dx dx
d  6
(5x − 2x)4

= −2 Constant Multiple Rule
dx
d
= −2(4)(5x6 − 2x)3 [5x6 − 2x] Chain Rule
dx
= −8(5x6 − 2x)3 (30x5 − 2) = −8(30x5 − 2)(5x6 − 2x)3

(g) Solution.

dy d h√ 2 i d h 2 1
i √ 1
= x −x = (x − x) 2 X = X2
dx dx dx
1 1 d
= (x2 − x) 2 −1 [x2 − x] Chain Rule
2 dx
1 1 1 1 1
= (x2 − x)− 2 (2x − 1) = (2x − 1) 1 X −n =
2 2 2
(x − x) 2 Xn
1
(2x − 1) 2x − 1
= 2√ = √
x2 − x 2 x2 − x

Alternate Solution
p
Let y be a function of the form y = g(x). Using the chain rule, one can
easily show that
d
d hp i [g(x)]
g(x) = dxp . (4.3.4)
dx 2 g(x)
208 Chapter 4. Differentiation Rules

Using this, we have

dy d h√ 2 i d
[x2 − x] 2x − 1
= x − x = dx√ = √
dx dx 2 x2 − x 2 x2 − x

(h) Solution.
" #

 
dy d 1 d 1 n 1
= √ = X = Xn
dx dx 3 x2 − 1 dx (x2 − 1) 13
d h 2 1
i 1
= (x − 1)− 3 = X −n
dx Xn
1 1 d
= − (x2 − 1)− 3 −1 [x2 − 1] Chain Rule
3 dx
1 4 d 2
= − (x2 − 1)− 3 (2x) [x − 1] = 2x
3 dx
2 4 −2x
= − x(x2 − 1)− 3 = 4
3 3(x2 − 1) 3

4. [Exercise on page 33]

(a) Solution.
ˆ We first need to find the marginal revenue function.
d p
R0 (x) = [16 3 (x2 + x)2 ]
dx
d h 2 2
i
= 16 (x + x) 3 Constant Multiple Rule
dx
 
2 2 − 31
= 16 (x + x) (2x + 1) Chain Rule
3
32 1 32(2x + 1)
= (2x + 1) 1 = 1
3 (x2 + x) 3 3(x2 + x) 3
(i) When x = 100,
32(2(100) + 1) 32(201) 2144
R0 (100) = 1 = = √
1
3
≈ $99.19
3(1002 + 100) 3 3(10100) 3 10100

(ii) When x = 250,


32(2(250) + 1) 32(501) 5344
R0 (250) = 1 = = √
1
3
≈ $134.48
3(2502 + 250) 3 3(62750) 3 62750

(b) Solution. The average revenue from the sale of x items is


p 2
R(x) 16 3 (x2 + x)2 16(x2 + x) 3
R(x) = = = .
x x x
4.3. The Chain Rule 209

(c) Solution. The marginal average revenue is the derivative of the average revenue
R(x). That is,
d d
[R(x)]x − R(x) dx
 
0 d d R(x) dx
[x]
R (x) = [R(x)] = = 2
Quotient Rule
dx dx x x
xR0 (x) − R(x) d
= [x] = 1
x2 dx
 
32(2x+1) 2
x 1 − 16(x2 + x) 3
2
3(x +x) 3 32(2x + 1)
= R0 (x) = 1 by part (a)
x2 3(x2 + x) 3
32x(2x+1) 2
1 − 16(x2 + x) 3
3(x2 +x) 3
=
x2
32x(2x+1)−48(x2 +x)
3(x2 +x) 3
1
a a − bc
= −c= and am · an = am+n
x2 b b
a
32x(2x + 1) − 48(x2 + x) b a
= 1 =
3x2 (x2 + x) 3 c bc
64x2 + 32x − 48x2 − 48x 16x2 − 16x
= 1 = 1
3x2 (x2 + x) 3 3x2 (x2 + x) 3
16x(x − 1) 16(x − 1)
= 1 = 1
3x2 (x2 + x) 3 3x(x2 + x) 3
5. [Exercise on page 34]
(a) Solution.
ˆ First, we need to find the revenue function. The revenue is R = pq. To
express this only in terms of q, we need to express p in terms of q.

– Squaring the equation q = 16000 − 2.5p, we get q 2 = 16000 − 2.5p.
Subtracting 16000 from both sides, we get q 2 −16000 = −2.5p. Dividing
both sides by −2.5, we get
q 2 − 16000 q 2 − 16000
 
2 2
p= = 5 = (q − 16000)
−2.5 −2 −5
2q 2 − 32000 2
= = − q 2 + 6400.
−5 5
– The revenue function is
 
2 2 2
R = pq = − q + 6400 q = − q 3 + 6400q
5 5
ˆ Now, we find the profit function P in terms of q.
2
P (q) = R(q) − C(q) = − q 3 + 6400q − (1500q + 4000)
5
2 3 2
= − q + 6400q − 1500q − 4000 = − q 3 + 4900q − 4000
5 5
210 Chapter 4. Differentiation Rules

ˆ The marginal profit is


 
0 d d 2 3
P (q) = [P ] = − q + 4900q − 4000
dq dq 5
2 6
= − (3q 2 ) + 4900(1) − 0 = − q 2 + 4900
5 5
(b) Solution.
ˆ When p = 6000,
p √ √
q = 16000 − 2.5(6000) = 16000 − 15000 = 1000.

ˆ The corresponding marginal profit is


√ 6 √
P 0 ( 1000) = − ( 1000)2 + 4900
5
6
= − (1000) + 4900 = −1200 + 4900 = $3700
5
(c) Solution. The derivative of q with respect to p is
dq d hp i
= 16000 − 2.5p
dp dp
d
dp
− 2.5p]
[16000 d p d
[g(x)]
√ [ g(x)] = dxp
2 16000 − 2.5p dx 2 g(x)
−2.5 d
= √ [16000 − 2.5p] = 0 − 2.5 = −2.5
2 16000 − 2.5p dp

6. [Exercise on page 34]

(a) Solution. For the function y = (−5x + 4)(x3 + 1)6 , we must use the product
rule before using the chain rule. (We first use the product rule because it applies
to the whole function.)

dy d 
(−5x + 4)(x3 + 1)6

=
dx dx
d d  3
[−5x + 4] (x3 + 1)6 + (−5x + 4) (x + 1)6

= Product Rule
dx dx
d
= −5(x3 + 1)6 + (−5x + 4)6(x3 + 1)5 (3x2 ) dx
[−5x+4] = −5 and Chain
Rule
3 6 2 3 5
= −5(x + 1) + 18x (−5x + 4)(x + 1)
Factor out (x3 +1)5 , which is
= (x3 + 1)5 −5(x3 + 1) + 18x2 (−5x + 4)
 
the greatest common factor
= (x3 + 1)5 (−5x3 − 5 − 90x3 + 72x2 ) Distribute inside brackets
= (x3 + 1)5 (−95x3 + 72x2 − 5)
4.3. The Chain Rule 211

(b) Solution.

dy d h √ i
= x 2 − x2
dx dx
d √ 2
d h√ 2
i
= [x] 2 − x + x 2−x Product Rule
dx dx
! d
√ d 2
hp i
[2 − x ] [g(x)]
d
g(x) = dx √ by
= (1) 2 − x2 + x dx√ dx 2 g(x)
2 2 − x2 the Chain Rule

 
−2x
= 2 − x2 + x √
2 2 − x2
√ x2
= 2 − x2 − √
2 − x2
(2 − x2 ) − x2 2 − 2x2 a − cb = ac−b and
= √ =√ √ √ c
2 − x2 2 − x2 ( X)( X) = X

(c) Solution.

dy d  2
(x + 1)3 (x2 + 2)6

=
dx dx
d  2 d  2
(x + 1)3 (x2 + 2)6 + (x2 + 1)3 (x + 2)6
 
= Product Rule
dx dx
= 3(x2 + 1)(2x)(x2 + 2)6 + (x2 + 1)3 6(x2 + 2)5 (2x) Chain Rule
= 6x(x2 + 1)2 (x2 + 2)6 + 12x(x2 + 1)3 (x2 + 2)5
Factor out 6x(x2 +
= 6x(x + 1) (x + 2) x2 + 2 + 2(x2 + 1)
2 2 2 5
 
1)2 (x2 +2)5 , the great-
est common factor
= 6x(x2 + 1)2 (x2 + 2)5 (x2 + 2 + 2x2 + 2)
= 6x(x2 + 1)2 (x2 + 2)5 (3x2 + 4)

(d) Solution.
d d
dy dx
[(x + 1)5 ] (x5 + 1) − (x + 1)5 dx [x5 + 1]
= Quotient Rule
dx (x5 + 1)2
5(x + 1)4 (1)(x5 + 1) − (x + 1)5 (5x4 ) Chain Rule and
= d
(x5 + 1)2 dx
[x5 + 1] = 5x4
5(x + 1)4 [(x5 + 1) − (x + 1)(x4 )]
= Factor out 5(x + 1)4
(x5 + 1)2
5(x + 1)4 (x5 + 1 − x5 − x4 )
=
(x5 + 1)2
5(x + 1)4 (1 − x4 )
=
(x5 + 1)2
212 Chapter 4. Differentiation Rules

(e) Solution.

x2
 
dy d
= √
dx dx x3 + 1
d

3 + 1 − x2 d
2
√ 
[x ] x x 3+1
= dx √
dx
2 Quotient Rule
3
x +1
√ d 3 
[x +1]
2x x3 + 1 − x2 dx √ d
hp i
[g(x)]
2 x3 +1 d
g(x) = √
dx
by
= √ 2 dx 2 g(x)
x3 + 1 the Chain Rule
√  2

2x x3 + 1 − x2 2√3x x3 +1 d 3
= √ 2 [x + 1] = 3x2
x3 + 1 dx
√ 4
2x x3 + 1 − 2√3x x3 +1
√ 2
= X =X
x3 + 1
4x(x3 +1)−3x4

2 x3 +1 b ac − b
= a− =
x3 + 1 c c
a
4x4 + 4x − 3x4 b a
= √ =
2(x3 + 1) x3 + 1 c bc
x4 + 4x
= √
2(x3 + 1) x3 + 1

(f ) Solution.
d
 x  hp i d
[g(x)]
r 
dy d x 2 d
g(x) = √
dx
by
= = dxpx +3 dx 2 g(x)
dx dx 2
x +3 2 x2x+3 the Chain Rule
d d
dx
[x](x2 +3)−x dx [x2 +3]
2
(x +3)2
= Quotient Rule
2 x2x+3
p

(1)(x2 +3)−x(2x) x2 +3−2x2


(x2 +3)2 (x2 +3)2
= =
2 x2x+3 2 x2x+3
p p

a
−x2 + 3 b a
= =
2(x2 + 3)2 x2x+3
p
c bc
s
−x2 + 3 x2 + 3
=
2(x2 + 3)2 x

(g) Solution.
" 5 # 4
x4 + 1
 4
d x4 + 1
 
dy d x +1
= =5 Chain Rule
dx dx x2 + 1 x2 + 1 dx x2 + 1
4.3. The Chain Rule 213

4 !
d d
x4 + 1 [x4
+ 1](x2 + 1) − (x4 + 1) dx [x2 + 1]

dx
=5 Quotient Rule
x2 + 1 (x2 + 1)2
 4 4  3 2
4x (x + 1) − (x4 + 1)2x

x +1
=5
x2 + 1 (x2 + 1)2
 4 4  5
4x + 4x3 − 2x5 − 2x

x +1
=5
x2 + 1 (x2 + 1)2
 4 4  5
2x + 4x3 − 2x

x +1
=5
x2 + 1 (x2 + 1)2
4
2x(x4 + 2x2 − 1) 10x(x4 + 2x2 − 1)(x4 + 1)4
  4
x +1
=5 =
(x2 + 1)2 x2 + 1 (x2 + 1)6
7. [Exercise on page 34]
Solution.
ˆ We first need to find the derivative.
dy d 2√ 3
= [x x − 2]
dx dx
d 2√ 3 d √
= [x ] x − 2 + x2 [ x3 − 2] Product Rule
dx dx
! d
√ d 3
hp i
[x − 2] [g(x)]
d
g(x) = dx √ by
= 2x x3 − 2 + x2 dx√ dx 2 g(x)
2 x3 − 2 the Chain Rule
√ 3x2
 
2 d 3
3
= 2x x − 2 + x √ [x − 2] = 3x2
2 x3 − 2 dx
√ 3x4
= 2x x3 − 2 + √
2 x3 − 2
4x(x3 − 2) + 3x4 b ac + b √  √ 
= √ a+ = and X X =X
2 x3 − 2 c c
4x4 − 8x + 3x4 7x4 − 8x
= √ = √
2 x3 − 2 2 x3 − 2
ˆ The slope of the tangent line at (3, 45) is obtained by replacing x with 3 into dy
dx
:
7(3)4 − 8(3) 567 − 24 543
m= √ = √ = .
3
2 3 −2 2 25 10
ˆ The equation of the tangent line is y − 45 = 543 10
(x − 3). Multiplying out the
right hand side, we get y − 45 = 543
10
x − 1629
10
. Adding 45 to both sides, we get
543 1179
y= x− .
10 10
8. [Exercise on page 34]
Solution.
214 Chapter 4. Differentiation Rules

ˆ First we need to find the derivative.


 
dy d x
=
dx dx (x2 + 28)4
d d
dx
[x](x2 + 28)4 − x dx [(x2 + 28)4 ]
= Quotient Rule
[(x2 + 28)4 ]2
(1)(x2 + 28)4 − x(4)(x2 + 28)3 (2x) d
= [x] = 1 and Chain Rule
(x2 + 28)8 dx
(x2 + 28)3 (x2 + 28 − 8x2 )
= Factor out (x2 + 28)3
(x2 + 28)8
(x2 + 28)3 (−7x2 + 28)
=
(x2 + 28)8
ˆ The tangent line is horizontal when the slope is 0. Since the slope is the deriva-
dy
tive, the tangent line is horizontal when dx = 0, that is,
(x2 + 28)3 (−7x2 + 28)
= 0.
(x2 + 28)8
Multiplying both sides of this latter equation by (x2 + 28)8 , we get (x2 +
28)3 (−7x2 + 28) = 0. This implies that (x2 + 28)3 = 0 or −7x2 + 28 = 0.
2 2
√ equation is equivalent to x +28 = 0. This implies that x = −28
– The former
or x = ± −28. Since the square root of a negative number is undefined,
the equation (x2 + 28)3 = 0 has no solution.
– The equation −7x2 + 28 = 0 is equivalent to −7x2 = √ −28. Dividing both
2
sides by −7, we get x = 4. This implies that x = ± 4 = ±2.
* The y-coordinate for x = −2 is obtained by substituting x with −2 into
x
y = (x2 +28)4:

−2 2 2 1
y= 2 4
=− 4
=− =− .
((−2) + 28) (4 + 28) 1048576 524288
1

This gives the point −2, − 524288 .
* The y-coordinate for x = 2 is
2 2 2 1
y= = = = .
((2)2
+ 28) 4 (4 + 28) 4 1048576 524288
1

This gives the point 2, 524288 .
x
Hence, the tangent line to the curve y = (x2 +28)4
is horizontal at the points
1 1
 
−2, − 524288 and 2, 524288 .

4.4 Derivatives of Exponential Functions


1. [Exercise on page 34]

(a) f (x) = 2ex


4.4. Derivatives of Exponential Functions 215

The Derivative of ex
The derivative of the natural exponential function is ex . That is,
d x
[e ] = ex .
dx

d
f 0 (x) = [2ex ]
dx
d
= 2 [ex ] Constant Multiple Rule
dx
= 2ex

(b) Solution.

d √
f 0 (x) = [1 + 2x − ex ]
dx
d d h√ i d
= [1] + 2x − [ex ] Sum and Difference Rules
dx dx dx
d
d
hp i
[2x] [g(x)]
= 0 + √ − ex
dx
d
dx
g(x) = dx√ by
2 g(x)
2 2x the Chain Rule
2 1
= √ − ex = √ − ex
2 2x 2x

(c) Solution.

d  3 x d 3 x d
f 0 (x) = xe = [x ]e + x3 [ex ] Product Rule
dx dx dx
= 3x e + x e = (3x + x )ex
2 x 3 x 2 3
Factor out ex

(d) Solution.

d d
[x2 ](ex + 3) − x2 dx [ex + 3]
 2 
0 d x dx
f (x) = = Quotient Rule
dx ex + 3 (ex + 3)2
2x(ex + 3) − x2 (ex + 0) 2xex + 6x − x2 ex
= =
(ex + 3)2 (ex + 3)2

2. [Exercise on page 34]

(a) Solution.
216 Chapter 4. Differentiation Rules

The Derivative of eg(x)


Let y be a function of the form y = eg(x) , and let u = g(x). Then y = eu ,
and by the chain rule,
dy dy du d u d d
= · = [e ] · [g(x)] = eu · [g(x)].
dx du dx du dx dx
Replacing u with g(x), we get

d  g(x)  d
e = eg(x) [g(x)].
dx dx

dy d 3x d
= [e ] = e3x [3x] Chain Rule
dx dx dx
d
= e3x (3) = 3e3x [3x] = 3
dx
(b) Solution.

dy d −5x d
= [e ] = e−5x [−5x] Chain Rule
dx dx dx
= e (−5) = −5e−5x
−5x

(c) Solution.
dy d h x2 +5x+1 i 2 d
= e = ex +5x+1 [x2 + 5x + 1] Chain Rule
dx dx dx
2 +5x+1 2 +5x+1
= ex (2x + 5) = (2x + 5)ex

(d) Solution.
dy d d
= [−e−x ] = − [e−x ] Constant Multiple Rule
dx dx dx
 
−x d
=− e [−x] Chain Rule
dx
= − e−x (−1) = −(−1)e−x = e−x


(e) Solution.
dy d  d  −1.5x 
−4e−1.5x = −4

= e Constant Multiple Rule
dx dx dx
d
= −4e−1.5x [−1.5x] Chain Rule
dx
= −4e−1.5x (−1.5) = −4(−1.5)e−1.5x = 6e−1.5x
4.4. Derivatives of Exponential Functions 217

(f ) Solution.
dy d h −x2 +1 i d h −x2 +1 i
= 7e =7 e Constant Multiple Rule
dx dx dx
2 d
= 7e−x +1 [−x2 + 1] Chain Rule
dx
2 +1 2 +1
= 7e−x (−2x + 0) = 7(−2x)e−x
2 +1
= −14xe−x

(g) Solution.
dy d  2x d 2x d
e + e−2x = [e ] + [e−2x ]

= Sum Rule
dx dx dx dx
d d
= e2x [2x] + e−2x [−2x] Chain Rule
dx dx
= e (2) + e (−2) = 2e2x − 2e−2x
2x −2x

3
(h) Solution. To take the derivative of y = x2 ex , we must use the product rule
before using the chain rule. The reason we first use the product rule is because
it applies to the whole function, while the chain rule applies only to the term
3
ex .
dy d h x3 i
= xe
dx dx
d 3 d h x3 i
= [x]ex + x e Product Rule
dx dx
 
x3 x3 d 3
= (1)e + x e [x ] Chain Rule
dx
3 3 3 3
= ex + xex (3x2 ) = (1 + 3x3 )ex Factor out ex

(i) Solution.
dy d  2 −4x  d 2 −4x d  −4x 
= xe = [x ]e + x2 e Product Rule
dx dx dx dx
 
−4x 2 −4x d
= 2xe +x e [−4x] Chain Rule
dx
= 2xe−4x + x2 e−4x (−4) = 2xe−4x − 4x2 e−4x
= (2x − 4x2 )e−4x Factor out e−4x

(j) Solution.
d
[e−2x ](x2 + 5) − e−2x dx
d
[x2 + 5]
 −2x 
dy d e dx
= = Quotient Rule
dx dx x2 + 5 (x2 + 5)2
e−2x dx
d
[−2x](x2 + 5) − e−2x dx
d
[x2 + 5]
= Chain Rule
(x2 + 5)2
218 Chapter 4. Differentiation Rules

−2e−2x (x2 + 5) − e−2x (2x)


=
(x2 + 5)2
e−2x [−2(x2 + 5) − 2x] e−2x (−2x2 − 10 − 2x)
= = Factor out e−2x
(x2 + 5)2 (x2 + 5)2
(−2x2 − 2x − 10)e−2x
=
(x2 + 5)2

(k) Solution.

dy d h x√2x+1 i
= e
dx dx
√ d h √ i
= ex 2x+1 x 2x + 1 Chain Rule
dx
d √

 i
x 2x+1 d h 1
=e [x] 2x + 1 + x (2x + 1) 2 Product Rule
dx dx


  
x 2x+1 1 − 21
=e (1) 2x + 1 + x (2x + 1) (2) Chain Rule
2
!
√ √ x 1 1
= ex 2x+1 2x + 1 + (2) 1 X −n = n
2 (2x + 1) 2 X


 
x 2x+1 x
=e 2x + 1 + √
2x + 1
a + cb = ac+b

 
(2x + 1) + x and
= ex 2x+1 √ √ √ c
2x + 1 ( X)( X) = X
 
3x + 1 √
= √ ex 2x+1
2x + 1

(l) Solution.
" 2
#
dy d xex
=
dx dx x + e−x
h i
x2 2 d
d
dx
xe (x + e−x ) − xex dx [x + e−x ]
= Quotient Rule
(x + e−x )2
 h i
x2 2 2
d d
(x + e−x ) − xex d d
[e−x ]

[x]e + x ex [x] +
dx dx dx dx Product and
=
(x + e−x )2 Sum Rules

 
x2 x2 2
e + x(2x)e (x + e−x ) − xex (1 + (−1)e−x )
= Chain Rule
(x + e−x )2
2 2 2 2 2 2
xex + ex e−x + 2x3 ex + 2x2 ex e−x − xex + xex e−x
=
(x + e−x )2
4.4. Derivatives of Exponential Functions 219

2
(e−x + 2x3 + 2x2 e−x + xe−x ) ex
=
(x + e−x )2

(m)

dy d 
(1 + xe4x )5

=
dx dx
d 
= 5(1 + xe4x )4 1 + xe4x

Chain Rule
dx
 
4x 4 d d  4x 
= 5(1 + xe ) [1] + xe Sum Rule
dx dx
 
4x 4 d 4x d 4x
= 5(1 + xe ) 0 + [x]e + x [e ] Product Rule
dx dx
4x 4 4x 4x

= 5(1 + xe ) e + x 4e Chain Rule
= 5(1 + xe4x )4 (1 + 4x)e4x

(n) Solution.

dy d h x2 +1 √ i
= e 2x + 3
dx dx
d h x2 +1 i √ 2 d h√ i
= e 2x + 3 + ex +1 2x + 3 Product Rule
dx dx
x2 +1 d 2
√ x2 +1 d
h 1
i
=e [x + 1] 2x + 3 + e (2x + 3) 2 Chain Rule
dx dx

 
x2 +1 x2 +1 1 − 21 d
=e (2x) 2x + 3 + e (2x + 3) [2x + 3] Chain Rule
2 dx
!
x2 +1
√ x2 +1 1 1
= 2xe 2x + 3 + e (2)
2 (2x + 3) 21
2
x2 +1
√ ex +1
= 2xe 2x + 3 + √
2x + 3
2 2 2
2x(2x + 3)ex +1 + ex +1 (4x2 + 6x + 1)ex +1
= √ = √
2x + 3 2x + 3

3. [Exercise on page 35]

(a) Solution.

The Derivatives of ax and ag(x)


Let a 6= 1 and a > 0, and let y = ax be the exponential function of base
a.

ˆ To find the derivative of y = ax , we use the change-of-base theorem


for exponentials to rewrite ax as an exponential function with base
220 Chapter 4. Differentiation Rules

e:
ax = ex ln a .
Using now the chain rule, we get
dy d  x ln a  d
= e = ex ln a [x ln a] = (ln a)ex ln a = (ln a)ax .
dx dx dx
So
d x
[a ] = (ln a)ax . (4.4.1)
dx
ˆ Likewise, if y is of the form ag(x) , then by using the change-of-base
theorem for exponentials and the chain rule, we get
d  g(x)  d
a = (ln a)ag(x) [g(x)]. (4.4.2)
dx dx

For y = 5x , we have
dy d x d  x ln 5 
= [5 ] = e Change the base
dx dx dx
d
= ex ln 5 [x ln 5] Chain Rule
dx
x ln 5
=e (ln 5) = (ln 5)ex ln 5 = (ln 5)5x

Alternate Solution
Using (4.4.1), we get
d x
[5 ] = (ln 5)5x .
dx

(b) Solution.
dy h x2 i d h x2 ln 3 i
= 3 = e Change the base
dx dx
2 d
= ex ln 3 [x2 ln 3] Chain Rule
dx
2 d 2 d
= ex ln 3
(2x ln 3) [x ln 3] = (ln 3) [x2 ] = (ln 3)(2x)
dx dx
2 2
= (2x ln 3)ex ln 3
= (2x ln 3)3x

Alternate Solution
Using (4.4.2), we get

d x2 2 d 2 2
[3 ] = (ln 3)3x [x2 ] = (ln 3)3x (2x) = (2x ln 3)3x .
dx dx
4.4. Derivatives of Exponential Functions 221

(c) Solution.
dy d h √ i
= 2·5 x
dx dx
d h √x i
=2 5 Constant Multiple Rule
dx
√ d √
 
x
= 2 (ln 5)5 [ x] Formula (4.4.2)
dx
  
√ 1 d √  1
x
= 2 (ln 5)5 √ x = √
2 x dx 2 x

x
(ln 5)5
= √
x

4. [Exercise on page 35]


Solution.

ˆ We first need to find the derivative.


dy d h x2 −6x i
= e
dx dx
2 d
= ex −6x [x2 − 6x] Chain Rule
dx
2 −6x 2 −6x
= ex (2x − 6) = (2x − 6)ex

ˆ The tangent line is horizontal where the derivative is 0. So we need to solve the
dy 2 2
equation dx = 0. Set (2x − 6)ex −6x = 0. Then 2x − 6 = 0 or ex −6x = 0.
– The equation 2x − 6 = 0 is equivalent to 2x = 6. Dividing both sides by 2,
we get x = 3.
2
– The equation ex −6x = 0 has no solution because the exponential of a number
is always positive (eX > 0 for every number X).
So the derivative is zero when x = 3.
2 −6x 2 −6(3)
ˆ Substituting this into ex , we get y = e3 = e9−18 = e−9 .

Thus, the tangent line is horizontal at the point (3, e−9 ).

5. [Exercise on page 35]


Solution. The marginal cost is

0 d h√ −x
i
C (x) = 600 − 500 · 1.2
dx
d
[600 − 500 · 1.2−x ] hp i d
[g(x)]
= √ dx d
dx
g(x) = dx√
2 600 − 500 · 1.2−x 2 g(x)

d
dx
[600] d
− 500 dx [1.2−x ] Difference Rule and Constant
= √
2 600 − 500 · 1.2−x Multiple Rule
222 Chapter 4. Differentiation Rules

0 − 500(ln 1.2)1.2−x dx
d
[−x] d  g(x)  d
= √ a = (ln a)ag(x) [g(x)]
2 600 − 500 · 1.2−x dx dx
−500(ln 1.2)1.2−x (−1) d
= √ [−x] = −1
2 600 − 500 · 1.2−x dx
250(ln 1.2) · 1.2−x
=√ Simplify
600 − 500 · 1.2−x
(i) When x = 0,

0 250(ln 1.2) · 1.20 250(ln 1.2) · (1)


C (0) = √ =p
600 − 500 · 1.2 0 600 − 500 · (1)
250(ln 1.2) 250(ln 1.2)
= √ = = 25 ln 1.2 ≈ $4.56
100 10
(ii) When x = 15,
250(ln 1.2) · 1.2−15 250(0.182321)(0.064905)
C 0 (15) = √ = p ≈ $0.12
600 − 500 · 1.2 −15 600 − 500(0.064905)

4.5 Derivatives of Logarithmic Functions


1. [Exercise on page 35]

(a) Solution.

The Derivative of ln x
Let x be a positive number. By definition,

ln x = y if and only if ey = x.

Replacing y with ln x, we get eln x = x. Taking the derivative of both sides


with respect to x, we get
d  ln x  d
e = [x]
dx dx
d d g(x) d d
eln x [ln x] = 1 [e ] = eg(x) [g(x)] and [x] = 1
dx dx dx dx
d
x [ln x] = 1 eln x = x
dx
d 1
[ln x] = Divide both sides by x
dx x
Thus, the derivative of ln x is x1 , that is,

d 1
[ln x] = . (4.5.1)
dx x
4.5. Derivatives of Logarithmic Functions 223

d
f 0 (x) = [5 ln x] Constant Multiple Rule
dx
 
1 d 1
=5 [ln x] = by (4.5.1)
x dx x
5
=
x
(b) Solution.
d  3 d 3 d
f 0 (x) =

x − ln x = [x ] − [ln x] Difference Rule
dx dx dx
1 d 1
= 3x2 − [ln x] = by (4.5.1)
x dx x
3
3x − 1 b ac − b
= a− =
x c c
(c) Solution.
d  2
f 0 (x) =

x ln x
dx
d 2 d
= [x ] ln x + x2 [ln x] Product Rule
dx dx
1 d 1
= 2x ln x + x2 [ln x] = by (4.5.1)
x dx x
= 2x ln x + x

(d) Solution.
 
0 d x
f (x) =
dx 1 + ln x
d d
dx
[x](1 + ln x) − x dx [1 + ln x]
= 2
Quotient Rule
(1 + ln x)
d d d

dx
+ ln x) − x dx
[x](1 [1] + dx
[ln x]
= 2
Sum Rule
(1 + ln x)
(1)(1 + ln x) − x 0 + x1

d 1
= [ln x] = by (4.5.1)
(1 + ln x)2 dx x
1 + ln x − 1 ln x
= 2
=
(1 + ln x) (1 + ln x)2

2. [Exercise on page 35]

(a) Solution.
224 Chapter 4. Differentiation Rules

The Derivative of ln g(x)

Let y be a function of the form y = ln g(x). Let u = g(x). Then y = ln u


and by the chain rule,
dy dy du d d 1 d
= · = [ln u] · [g(x)] = [g(x)].
dx du dx du dx u dx
Replacing u with g(x), we get
d
d dx
[g(x)]
[ln g(x)] = . (4.5.2)
dx g(x)

dy d
= [ln(3x)]
dx dx
d
[3x]
= dx By (4.5.2)
3x
3 1
= =
3x x
(b) Solution. Using (4.5.2), we get
d
dy d dx
[−5x + 4] −5
= [ln(−5x + 4)] = = .
dx dx −5x + 4 −5x + 4

(c) Solution. Using (4.5.2), we have


d
dy d [x2 + 9] 2x
= [ln(x2 + 9)] = dx
= .
dx dx x2 + 9 x2+9

(d) Solution.
d d
dy d [x− ex ] d [g(x)]
= [ln(x − ex )] = dx
[ln g(x)] = dx
dx dx x − ex dx g(x)
d d
[x]− dx [ex ] 1 − ex d x
= dx
= [e ] = ex
x − ex x − ex dx

(e) Solution.

dy d d
= [4 ln(−2x)] = 4 [ln(−2x)] Constant Multiple Rule
dx dx dx
d d
[−2x] d [g(x)]
= 4 dx [ln g(x)] = dx
−2x dx g(x)
−2 −8 4
=4 = =
−2x −2x x
4.5. Derivatives of Logarithmic Functions 225

(f ) Solution.

dy d d
= [ln(4x)] + [ln(4x)] Sum Rule
dx dx dx
d d d
[4x] [−4x] d [g(x)]
= dx + dx [ln g(x)] = dx
4x −4x dx g(x)
4 4 1 1 2
= − = + =
4x −4x x x x
(g) Solution.

dy d d d
= [x ln(x2 )] = [x] ln(x2 ) + x [ln(x2 )] Product Rule
dx dx dx dx
d d
2 dx
[x2 ] d dx
[g(x)]
= (1) ln(x ) + x [ln g(x)] =
x2 dx g(x)
2x 2x2
= ln(x2 ) + x 2
= ln(x 2
) + 2
= ln (x2 ) + 2
x x
Note. Do not use the property ln(xn ) = n ln x to simplify further! Indeed, the
function ln(x2 ) + 2 is valid for all numbers except 0 while the function 2 ln x + 2
is valid only for x > 0.
(h) Solution.

dy d 3 d
= [x ] ln(4x2 − 6x + 3) + x3 [ln(4x2 − 6x + 3)] Product Rule
dx dx dx
d d
[4x2 − 6x + 3] d [g(x)]
= 3x2 ln(4x2 − 6x + 3) + x3 dx 2 [ln g(x)] = dx
4x − 6x + 3 dx g(x)
8x − 6
= 3x2 ln(4x2 − 6x + 3) + x3 2
4x − 6x + 3
2 2
8x4 − 6x3
= 3x ln (4x − 6x + 3) +
4x2 − 6x + 3
(i) Solution.
dy d h x2 i
= e ln(3x)
dx dx
d x2 2 d
= [e ] ln(3x) + ex [ln(3x)] Product Rule
dx dx
d d
 g(x) 
d e = eg(x) dx [g(x)],
 
x2 d 2 [3x] dx
= e [x ] ln(3x) + ex dx
2
d
d
[g(x)]
dx 3x dx
[ln g(x)] = dx
g(x)

2 3
 2 
= ex (2x) ln(3x) + ex
3x
x2
 
x2 e 1 2
= 2xe ln(3x) + = 2x ln (3x) + ex
x x
226 Chapter 4. Differentiation Rules

(j) Solution.
dy d h √ i
= ln 6x + 7
dx dx
d
√ d
dx
[ 6x + 7] d dx
[g(x)]
= √ [ln g(x)] =
6x + 7 dx g(x)
√6
2 6x+7 d hp i g 0 (x)
=√ g(x) = p
6x + 7 dx 2 g(x)
3 3
=√ √ =
6x + 7 6x + 7 6x + 7
(k) Solution.

d x d
dy d h √ i [xe + 8] d [g(x)]
= ln(xe x + 8) = dx

x
[ln g(x)] = dx
dx dx xe + 8 dx g(x)

d d
dx
[xe x ] + dx [8]
= √ Sum Rule
xe x +8
√ √
d d
dx
[x]e x + x dx [e x ]
= √ Product Rule
xe x + 8

d √
√ √
x
e + xe x dx [ x] d  g(x)  d
= √ e = eg(x) [g(x)]
xe x + 8 dx dx
√ √
x 1
x √
e + xe 2 x d √  1
= √
x
x = √
xe +8 dx 2 x
  √  √  √
2 x+x
1 + 2√x x e x √
2 x
e x b ac + b
= √
x
= √ a+ =
+8 xe xe x + 8 c c
√ √
a
(x + 2 x)e x b a
= √  √ =
xe x + 8 2 x c bc

(l) Solution.

dy d h√ −x i d
[e−x + ln 2x] d hp i d
[g(x)]
= e + ln 2x = dx√ −x g(x) = dxp
dx dx 2 e + ln 2x dx 2 g(x)
d d
 
d
dx
[e−x ] + dx
d
[ln 2x] −e−x + 2x
2
dx
eg(x) = eg(x) dx [g(x)],
= √ = √ −x d
d
[g(x)]
2 e−x + ln 2x 2 e + ln 2x dx
[ln g(x)] = dx
g(x)
−x 1 −x
−e + 1 − xe x b ac + b ab a
= √ −x = √ a+ = , =
2 e + ln 2x 2x e−x + ln 2x c c c bc

(m) Solution.
xe−x
 
dy d
=
dx dx 1 + ln(5x)
4.5. Derivatives of Logarithmic Functions 227

d
dx
[xe−x ](1
+ ln(5x)) − xe−x dx d
[1 + ln(5x)]
= Quotient Rule
(1 + ln(5x))2
d
[x]e−x + x dx
d
[e−x ] (1 + ln(5x)) − xe−x dx d

dx
[1 + ln(5x)]
= Product Rule
(1 + ln(5x))2
(e−x + x(−e−x )) (1 + ln(5x)) − xe−x dx d d

[1] + dx [ln(5x)]
=
(1 + ln(5x))2

(e−x − xe−x ) (1 + ln(5x)) − xe−x 0 + 5



5x
=
(1 + ln(5x))2
e−x + e−x ln(5x) − xe−x − xe−x ln(5x) − e−x
=
(1 + ln(5x))2
e−x ln(5x) − xe−x − xe−x ln(5x)
=
(1 + ln(5x))2

3. [Exercise on page 35]

(a) Solution.

The Derivative of loga x


Let a be a real number such that a > 0 and a 6= 1. Then the derivative of
y = loga x is
 
d d ln x Change-of-Base Theo-
[loga x] =
dx dx ln a rem for Logarithms
1 d
= [ln x] Constant Multiple Rule
ln a dx
 
1 1 d 1
= [ln x] = by (4.5.1)
ln a x dx x
1
=
(ln a)x

So
d 1
[loga x] = (4.5.3)
dx (ln a)x

Using (4.5.3), we get


d 1
[log3 x] = .
dx (ln3)x

(b) Solution.

dy d d
= [7 log8 x] = 7 [log8 x] Constant Multiple Rule
dx dx dx
228 Chapter 4. Differentiation Rules

1 d 1
=7 [loga x] =
(ln 8)x dx (ln a)x
7
=
(ln8)x
(c) Solution.
d ln(−2x3 + x2 )
 
dy d  3 2
 ln X
= log2 (−2x + x ) = loga X =
dx dx dx ln 2 ln a
1 d
= [ln(−2x3 + x2 )] Constant Multiple Rule
ln 2 dx
d d
1 dx [−2x3 + x2 ] d dx
[g(x)]
= [ln g(x)] =
ln 2 −2x3 + x2 dx g(x)
1 −6x2 + 2x −6x2 + 2x
= =
ln 2 −2x3 + x2 (ln2)(−2x3 + x2 )

Alternate Solution

The Derivative of loga g(x)

Let a > 0 and a 6= 0. Let y be a function of the form y = loga g(x).


The derivative of y with respect to x is
 
d d ln g(x) ln X
[loga g(x)] = loga X =
dx dx ln a ln a
1 d
= [ln g(x)] Constant Multiple Rule
ln a dx !
d
1 dx
[g(x)]
= By (4.5.2)
ln a g(x)
d
dx
[g(x)]
=
(ln a)g(x)

Using this, we get


d
d [−2x3 + x2 ]
[log2 (−2x3 + x2 )] = dx
dx (ln 2)(−2x3 + x2 )
−6x2 + 2x
=
(ln2)(−2x3 + x2 )

5
(d) Solution. For the function y = log(x2 − x) 2 , the base is a = 10. So
h 5
i
d 2
dy d h 5
i dx
(x − x) 2
d d
[g(x)]
= log(x2 − x) 2 = 5 log a g(x) = dx
dx dx (ln 10)(x2 − x) 2 dx (ln a)g(x)
4.5. Derivatives of Logarithmic Functions 229

5 3
d
2
(x2 − x) 2 dx [x2 − x]
= 5 Chain Rule
(ln 10)(x2 − x) 2
5 3
2
(x2 − x) 2 (2x − 1) 5(2x − 1)
= 5 = Simplify
(ln 10)(x2 − x) 2 2(ln10)(x2 − x)

4. [Exercise on page 35]

(a) Solution.

The Derivative of the Logarithm of an Absolute Value


Recall the definition of the absolute value function:

x if x ≥ 0
|x| =
−x if x < 0
d
Using (4.5.1), dx
[ln x] = x1 . Using the chain rule for logarithms,
d
d dx
[−x] −1 1
[ln(−x)] = = = .
dx −x −x x
d d
So dx
[ln x] = dx
[ln(−x)]. This implies that

d 1
[ln |x|] = .
dx x
Similarly, one has
d
d dx
[g(x)]
[ln |g(x)|] =
dx g(x)
d 1
[loga |x|] =
dx (ln a)x
d
d dx
[g(x)]
[loga |g(x)|] = .
dx (ln a)g(x)

Note. An absolute value inside a logarithm has no effect on the derivative.

dy
For y = ln |x|, dx
= x1 .
(b) Solution.

dy d
= [ln | ln x|]
dx dx
d d
dx
[ln x] d dx
[g(x)]
= [ln |g(x)|] =
ln x dx g(x)
1 a
x
1 d 1 b a
= = [ln x] = and =
ln x x ln x dx x c bc
230 Chapter 4. Differentiation Rules

(c) Solution.

d d
dy d hp i
dx
[ln |6 − x|] d hp i
dx
[g(x)]
= ln |6 − x| = p g(x) = p
dx dx 2 ln |6 − x| dx 2 g(x)
d
dx
[6−x] d
d [g(x)]
= p 6−x [ln |g(x)|] = dx
2 ln |6 − x| dx g(x)
−1 a
6−x −1 b a
= p = p =
2 ln |6 − x| 2(6 − x) ln|6 − x| c bc

(d) Solution.

dy d  2
x log5 |x3 + 1|

=
dx dx
d 2 d 
[x ] log5 |x3 + 1| + x2 log5 |x3 + 1|

= Product Rule
dx dx
d d
[x3 + 1] d [g(x)]
= 2x log5 |x3 + 1| + x2 dx [loga |g(x)|] = dx
(ln 5)(x3 + 1) dx (ln a)g(x)
3x2
= 2x log5 |x3 + 1| + x2
(ln 5)(x3 + 1)
3x4
= 2xlog5 |x3 + 1| +
(ln5)(x3 + 1)

5. [page 35]

(a) Solution.

dy d  2x 
= x
dx dx
d  2x ln x 
= e AB = eB ln A
dx
d d  g(x)  d
= e2x ln x [2x ln x] e = eg(x) [g(x)]
dx dx dx
 
2x ln x d d
=e [2x] ln x + 2x [ln x] Product Rule
dx dx
 
1
= e2x ln x 2 ln x + 2x
x
= e2x ln x (2 ln x + 2)
= 2(1 + lnx)e2xlnx = 2(1 + lnx)x2x
4.5. Derivatives of Logarithmic Functions 231

Alternate Solution – Logarithmic Differentiation

An alternate solution to find the derivative of f (x) = x2x is to proceed as


follows.

Step 1. Take the natural logarithm of both sides.

f (x) = x2x
ln(f (x)) = ln(x2x ) = 2x ln x ln(xn ) = n ln x

Step 2. Take the derivative of both sides.

ln(f (x)) = 2x ln x
d d
[ln(f (x))] = [2x ln x]
dx dx
d
f 0 (x) d d dx
[g(x)]
= [2x ln x] [ln g(x)] =
f (x) dx dx g(x)
f 0 (x) d d
= [2x] ln x + 2x [ln x] Product Rule
f (x) dx dx
f 0 (x)
 
1
= 2 ln x + 2x
f (x) x
f 0 (x)
= 2 ln x + 2
f (x)

f 0 (x)
Step 3. Solve for f 0 (x). From the equation f (x)
= 2 ln x + 2, it
follows that

f 0 (x) = (2 ln x + 2)f (x) = 2(ln x + 1)x2x

This process is called logarithmic differentiation.

(b) Solution.

dy d  2 d h 3x ln(x2 +1) i
(x + 1)3x = AB = eB ln A

= e
dx dx dx
2 d d  g(x)  d
= e3x ln(x +1) [3x ln(x2 + 1)] e = eg(x) [g(x)]
dx dx dx
 
2 d d
= e3x ln(x +1) [3x] ln(x2 + 1) + 3x [ln(x2 + 1)] Product Rule
dx dx
  d
3x ln(x2 +1) 2 2x d dx
[g(x)]
=e 3 ln(x + 1) + 3x 2 [ln g(x)] =
x +1 dx g(x)
6x2
 
2
= e3xln(x +1) 3ln(x2 + 1) + 2
x +1
232 Chapter 4. Differentiation Rules

(c) Solution. Using the same rules as before, we have

dy d h 2
i d h (x2 +7) ln |1+e2x | i
= |1 + e2x |x +7 = e
dx dx dx
2 2x d 
= e(x +7) ln |1+e | (x2 + 7) ln |1 + e2x |

dx
 
(x2 +7) ln |1+e2x | d 2 2x 2 d  2x

=e [x + 7] ln |1 + e | + (x + 7) ln |1 + e |
dx dx
!
d 2x
2 2x [1 + e ]
= e(x +7) ln |1+e | (2x) ln |1 + e2x | + (x2 + 7) dx
1 + e2x
2e2x
 
(x2 +7) ln |1+e2x | 2x 2
=e (2x) ln |1 + e | + (x + 7)
1 + e2x
2(x2 + 7)e2x
 
(x2 +7)ln|1+e2x | 2x
=e 2xln|1 + e | +
1 + e2x

6. [Exercise on page 36]


Solution.

ˆ We first need to find the derivative:


dy d x d x ln x
= [x ] = [e ] AB = eB ln A
dx dx dx
d d  g(x)  d
= ex ln x [x ln x] e = eg(x) [g(x)]
dx dx dx
 
x ln x d d
=e [x] ln x + x [ln x] Product Rule
dx dx
 
1 d d 1
= ex ln x ln x + x [x] = 1 and [ln x] =
x dx dx x
= ex ln x (ln x + 1) = (1 + ln x) ex ln x Simplify

ˆ The derivative is zero if (1 + ln x) ex ln x = 0. This implies that 1 + ln x = 0 or


ex ln x = 0.
– The first equation is equivalent to ln x = −1. Taking the natural exponential
on both sides, we get x = e−1  . The corresponding
 y-coordinate is y =
−1 e−1 −1 −1 e−1
(e ) . This gives the point e , (e ) .
– The second equation, ex ln x = 0, has no solution because the exponential of
every number is always positive.
 
−1
Thus, the tangent line is horizontal at the point e−1 , (e−1 )e .

7. [Exercise on page 36]


Solution.
4.6. Derivatives of Trigonometric Functions 233

The average cost is C(x) = C(x)


x
= 3 log2xx+10 . And the marginal average cost is
 
0 d   d 3 log2 x + 10
C (x) = C(x) =
dx dx x
d d
dx
[3 log2 x + 10]x − (3 log2 x + 10) dx [x]
= 2
Quotient Rule
x
d d

3 dx [log2 x] + dx [10] x − (3 log2 x + 10)(1)
=
x2
 
3 (ln12)x + 0 x − 3 log2 x − 10 d 1
= [loga x] =
x2 dx (ln a)x
 
3
(ln 2)x
x − 3 log2 x − 10
=
x2
3
− 3 ln x
− 10 3
− 3lnln2x − 10lnln2 2 ln x
= ln 2 ln 2
= ln 2
loga x =
x2 x2 ln a
3 − 3 ln x − 10ln2
=
(ln2)x2

4.6 Derivatives of Trigonometric Functions


1. [Exercise on page 36]
(a) Solution. First, we recall the derivatives of basic trigonometric functions.

Derivatives of Basic Trigonometric Functions

There are six basic trigonometric functions: sine (sin), cosine (cos), tan-
gent (tan), cotangent (cot), secant (sec), and cosecant (csc) functions.

ˆ The derivative of the sine function is the cosine function, that is,
d
[sin x] = cos x
dx
ˆ The derivative of the cosine function is the opposite of the sine func-
tion, that is,
d
[cos x] = − sin x
dx
ˆ The tangent function, denoted tan, is defined as tan x = sin x
cos x
. And
its derivative is given by
d 1
[tan x] = sec2 x =
dx cos2 x
ˆ The cotangent function, denoted cot, is defined as cot x = cos x
sin x
. This
1
is precisely the inverse of tangent, that is, cot x = tan x . And its
234 Chapter 4. Differentiation Rules

derivative is
d 1
[cot x] = − csc2 x = − 2
dx sin x
ˆ The secant function, denoted sec, is defined as the inverse of the
cosine function, that is, sec x = cos1 x . Its derivative is given by

d
[sec x] = sec x tan x
dx

ˆ The cosecant function, denoted csc, is defined as the inverse of the


sine function, that is, csc x = sin1 x . Its derivative is

d
[csc x] = − csc x cot x
dx

We want to find the derivative of f (x) = −5 cos x.


d
f 0 (x) = [−5 cos x]
dx
d
= −5 [cos x] Constant Multiple Rule
dx
d
= −5(− sin x) [cos x] = − sin x
dx
= 5 sin x

(b) Solution.
   
0 d sin x 3 d sin x d
f (x) = −x = − [x3 ] Difference Rule
dx 4 dx 4 dx
1 d Constant Multiple Rule,
= [sin x] − 3x2
4 dx Power Rule
1 d
= cos x − 3x2 [sin x] = cos x
4 dx

(c) Solution.
d  4  d 4 d
π + 3 tan x = [π ] + [3 tan x] Sum Rule
dx dx dx
d Constant Rule, Con-
= 0 + 3 [tan x]
dx stant Multiple Rule
d
= 3sec2 x [tan x] = sec2 x
dx
(d) Solution.
d
f 0 (x) = [3 sin x − 8 cos x + 2 tan x]
dx
4.6. Derivatives of Trigonometric Functions 235

d d d
= [3 sin x] − [8 cos x] + [2 tan x]
dx dx dx
d d d
= 3 [sin x] − 8 [cos x] + 2 [tan x]
dx dx dx
d
dx
[sin x] = cos x
2 d
= 3 cos x − 8(− sin x) + 2 sec x dx
[cos x] = − sin x
d
dx
[tan x] = sec2 x
= 3 cos x + 8 sin x + 2 sec2 x

(e) Solution.

d
f 0 (x) = [7 sec x − csc x + cot x]
dx
d d d
= 7 [sec x] − [csc x] + [cot x]
dx dx dx
d
dx
[sec x] = sec x tan x
2 d
= 7 sec x tan x − (− csc x cot x) + (− csc x) dx
[csc x] = − csc x cot x
d
dx
[cot x] = − csc2 x
= 7 secx tan x + cscx cot x − csc2 x

(f ) Solution.
d d d
f 0 (x) = [x cot x] = [x] cot x + x [cot x] Product Rule
dx dx dx
= (1) cot x + x(− csc x) = cot x − x csc2 x
2

(g) Solution.
d d d
f 0 (x) = [sin x cos x] = [sin x] cos x + sin x [cos x] Product Rule
dx dx dx
= (cos x) cos x + sin x(− sin x) = cos x − sin2 x
2

(h) Solution.
d d
− sin x dx
 
0 d sin x dx
[sin x]x [x]
f (x) = = 2
Quotient Rule
dx x x
cos x(x) − sin x(1) d
= [sin x] = cos x
x2 dx
x cos x − sin x
=
x2
(i) Solution.
d d
d h sec x i [sec x] csc x − sec x dx [csc x]
f 0 (x) = = dx
Quotient Rule
dx csc x (csc x)2
236 Chapter 4. Differentiation Rules

sec x tan x csc x − sec x(− csc x cot x) d


dx
[sec x] = sec x tan x
= d
(csc x)2 dx
[csc x] = − csc x cot x
sec x tan x csc x + sec x csc x cot x
=
csc2 x
sec x csc x(tan x + cot x)
=
csc2 x
(j) Solution.
d d h cos x i
f 0 (x) = [x sin x] +
dx dx x
d d
d d dx
[cos x]x − cos x dx [x] Product Rule,
= [x] sin x + x [sin x] +
dx dx x 2 Quotient Rule
− sin x(x) − cos x(1)
= (1) sin x + x cos x +
x2
−x sin x − cos x
= sin x + x cos x +
x2
(k) Solution.
√ 
0 d 9 d 9x d
f (x) = [x ] + + [tan x]
dx dx 2 dx
1 d √ d
= 9x8 + x + sec2 x [tan x] = sec2 x
9

2 dx dx
8 1 d h 1i
= 9x + x 9 + sec2 x
2 dx
 
8 1 1 −8
= 9x + x 9 + sec2 x
2 9
1
= 9x8 + 8 + sec x
2
18x 9
(l) Solution.
d  2 d 2 d d
f 0 (x) =

x cos x − 2 tan x + 3 = [x cos x] − 2 [tan x] + [3]
dx dx dx dx
d 2 d
= [x ] cos x + x2 [cos x] − 2 sec2 x + 0
dx dx
= 2x cos x + x2 (− sin x) − 2 sec2 x
= 2x cos x − x2 sin x − 2 sec2 x

(m) Solution.
d d
f 0 (x) = [x cos x] + [x2 sin x]
dx dx
d d d d
= [x] cos x + x [cos x] + [x2 ] sin x + x2 [sin x] Product Rule
dx dx dx dx
4.6. Derivatives of Trigonometric Functions 237

= (1) cos x + x(− sin x) + 2x sin x + x2 cos x


= cos x − x sin x + 2x sin x + x2 cos x
= cos x + x sin x + x2 cos x

(n) Solution.
 
0 d sin x
f (x) =
dx 1 + tan x
d d
dx
[sin x](1 + tan x) − sin x dx [1 + tan x]
= Quotient Rule
(1 + tan x)2
cos x(1 + tan x) − sin x(0 + sec2 x)
=
(1 + tan x)2
cos x + cos x tan x − sin x sec2 x
=
(1 + tan x)2

(o) Solution.

csc x √
 
0 d 1
f (x) = + − x 7

dx cos x 3
d √
 
d 1 d h csc x i 
= + − 7
x
dx cos x dx 3 dx
d d
dx
− 1 dx
[1] cos x [cos x] 1 d d h 1i
= + [csc x] − x7
cos2 x 3 dx dx
(0) cos x − (− sin x) 1 1 1
= 2
+ (− csc x cot x) − x 7 −1
cos x 3 7
sin x 1 1
= 2
− csc x cot x − 6
cos x 3 7x 7
(p) Solution. First we have f (x) = (x cos x) sin x. Now we have
d d
f 0 (x) = [x cos x] sin x + x cos x [sin x] Product Rule
dx dx
 
d d
= [x] cos x + x [cos x] sin x + x cos x(cos x) Product Rule
dx dx
= (cos x − x sin x) sin x + x cos2 x
= cos x sin x − x sin2 x + x cos2 x

(q) Solution.
d  d 
f 0 (x) = 5 sin4 x = 5 (sin x)4
 
dx dx
 
4−1 d Chain Rule:
= 5 4(sin x) [sin x] d d
dx dx
[(g(x))n ] = n(g(x))n−1 dx [g(x)]
238 Chapter 4. Differentiation Rules

d
= 5 4(sin x)3 cos x
 
[sin x] = cos x
dx
= 20 cos x sin3 x

Warning. Sine of x raised to the power of n is written sinn x. That is, sinn x =
(sin x)n . But sine of xn is written sin xn . That is, sin xn = sin(xn ). Note that
sinn x is not equal to sin xn in general. The same remark applies to the other
trigonometric functions.
(r) Solution.
d 
f 0 (x) = x(1 + tan x)8

dx
d d 
[x](1 + tan x)8 + x (1 + tan x)8

= Product Rule
dx dx
 
8 7 d
= (1)(1 + tan x) + x 8(1 + tan x) [tan x] Chain Rule
dx
d
= (1 + tan x)8 + x 8(1 + tan x)7 sec2 x [tan x] = sec2 x
 
dx
= (1 + tan x)8 + 8x(1 + tan x)7 sec2 x
= (1 + tan x)7 1 + tan x + 8x sec2 x Factor out (1 + tan x)7


2. [Exercise on page 36]

(a) f (x) = sin(2x)


Solution.

Chain Rule (for Trigonometric Functions)

Let y be a function of the form y = f (g(x)). The function f is the


outer function and g is the inner function. In Section 4.3, we saw that an
alternate form of the chain rule is
d d
[f (g(x))] = f 0 (g(x)) [g(x)]
dx dx
In words, this says that the derivative of f (g(x)) with respect to x is the
derivative of the outer function f evaluated at g(x) times the derivative
of the inner function g.

ˆ Suppose y is of the form y = sin(g(x)). Then the outer function is


f (u) = sin u and the inner function is u = g(x). Since

– the derivative of the outer function is f 0 (u) = cos u;


– the derivative of the outer function evaluated at g(x) is
f 0 (g(x)) = cos(g(x));

it follows that the derivative of sin(g(x)) with respect to x is


4.6. Derivatives of Trigonometric Functions 239

d
cos(g(x)) dx [g(x)]. That is,

d d
[sin(g(x))] = cos(g(x)) [g(x)]
dx dx

ˆ Now, suppose that y is of the form y = cos(g(x)). Then the outer


function is f (u) = cos u and the inner function is u = g(x). Since
f 0 (u) = − sin(u) = − sin(g(x)), it follows that

d d
[cos(g(x))] = − sin(g(x)) [g(x)]
dx dx

ˆ Similarly, we have the following:

d d
[tan(g(x))] = sec2 (g(x)) [g(x)]
dx dx
d d
[cot(g(x))] = − csc2 (g(x)) [g(x)]
dx dx
d d
[sec(g(x))] = sec(g(x)) tan(g(x)) [g(x)]
dx dx
d d
[csc(g(x))] = − csc(g(x)) cot(g(x)) [g(x)]
dx dx

For y = sin(2x), the inner function is g(x) = 2x, and the outer function is
f (x) = sin x. So
d d
[sin(2x)] = cos(2x) [2x] = cos(2x)(2) = 2 cos (2x)
dx dx
(b) Solution.
dy d
= [cos(−3x + 1)]
dx dx
d d d
= − sin(−3x + 1) [−3x + 1] [cos(g(x))] = − sin(g(x)) [g(x)]
dx dx dx
= − sin(−3x + 1)(−3) = 3 sin ( − 3x + 1)

(c) Solution.
dy d 
tan(x3 − x2 )

=
dx dx
d d d
= sec2 (x3 − x2 ) [x3 − x2 ] [tan(g(x))] = sec2 (g(x)) [g(x)]
dx dx dx
= sec (x − x )(3x2 − 2x)
2 3 2

= (3x2 − 2x) sec2 (x3 − x2 )


240 Chapter 4. Differentiation Rules

(d) Solution.

dy d
= [x sin(x2 + 1)]
dx dx
d d 
[x] sin(x2 + 1) + x sin(x2 + 1)

= Product Rule
dx dx
d d
[sin(g(x))] =
= (1) sin(x2 + 1) + x cos(x2 + 1) [x2 + 1] dx
d
dx cos(g(x)) dx [g(x)]
= sin(x2 + 1) + x cos(x2 + 1)(2x)
= sin (x2 + 1) + 2x2 cos (x2 + 1)

(e) Solution.

dy d  2 
= 3x cos(πx − 1)
dx dx
d d
= [3x2 ] cos(πx − 1) + 3x2 [cos(πx − 1)] Product Rule
dx dx
d
 
2 d [cos(g(x))] =
= 6x cos(πx − 1) + 3x − sin(πx − 1) [πx − 1] dx
d
dx − sin(g(x)) dx [g(x)]
= 6x cos(πx − 1) − 3x2 sin(πx − 1)(π)
= 6x cos (πx − 1) − 3πx2 sin (πx − 1)

(f ) Solution.

dy d
= [sin(cos x)]
dx dx
d d d
= cos(cos x) [cos x] [sin(g(x))] = cos(g(x)) [g(x)]
dx dx dx
d
= cos(cos x)(− sin x) [cos x] = − sin x
dx
= − sin x cos ( cos x)

(g) Solution.

dy d
= [cos(sec(4x))]
dx dx
d d d
= − sin(sec(4x)) [sec(4x)] [cos(g(x))] = − sin(g(x)) [g(x)]
dx dx dx
d d
[sec(g(x))] =
= − sin(sec(4x)) sec(4x) tan(4x) [4x] dx
d
dx sec(g(x)) tan(g(x)) dx [g(x)]
= − sin(sec(4x)) sec(4x) tan(4x)(4)
= −4 sin ( sec (4x)) sec (4x) tan (4x)
4.6. Derivatives of Trigonometric Functions 241

(h) Solution.

dy d h √ 2
i
= sin 1 + x
dx dx
√ d h√ i d d
= cos 1 + x2 1 + x2 [sin(g(x))] = cos(g(x)) [g(x)]
dx dx dx
!
√ d
[1 + x 2
] d p
h i d
[g(x)]
= cos 1 + x2 dx√ g(x) = dxp
2 1 + x2 dx 2 g(x)

 
2x
= cos 1 + x 2 √
2 1 + x2
√ 
x cos 1 + x2
= √
1 + x2

(i) Solution.
 
dy d 1
= x sin
dx dx x
    
d 1 d 1
= [x] sin +x sin Product Rule
dx x dx x
d
      
1 1 d 1 dx
[sin(g(x))] =
= (1) sin + x cos d
x x dx x cos(g(x)) dx [g(x)]
       
1 1 1 d 1 1
= sin + x cos − 2 =− 2
x x x dx x x
   
1 x 1
= sin − 2 cos
x x x
1 1 1
= sin ( ) − cos ( )
x x x

(j) Solution.

dy d h 3
4 i
= sec(x + 1)
dx dx
3 d  d d
= 4 sec(x3 + 1) sec(x3 + 1) [(g(x))n ] = n(g(x))n−1 [g(x)]

dx dx dx
 
d
= 4 sec3 (x3 + 1) sec(x3 + 1) tan(x3 + 1) [x3 + 1] Chain Rule
dx
= 4 sec3 (x3 + 1) sec(x3 + 1) tan(x3 + 1)(3x2 )
= 12x2 sec4 (x3 + 1) tan (x3 + 1)

(k) Solution.

dy d 
sin 3e5x

=
dx dx
242 Chapter 4. Differentiation Rules

 d d d
= cos 5e5x [3e5x ] [sin(g(x))] = cos(g(x)) [g(x)]
dx dx dx
d
= cos 5e5x 3 [e5x ]

Constant Multiple Rule
dx
d d  g(x)  d
= cos 5e5x 3e5x [5x] = eg(x) [g(x)]

e
dx dx dx
= cos 5e5x 3e5x (5)


= 15 cos (5e5x )e5x


(l) Solution.
dy d 
ln | cos2 x|

=
dx dx
d d
[(cos x)2 ] d [g(x)]
= dx [ln |g(x)|] = dx
cos2 x dx g(x)
d
2(cos x) dx [cos x] d d
= [(g(x))n ] = n(g(x))n−1 [g(x)]
cos2 x dx dx
2(cos x)(− sin x) d
= [cos x] = − sin x
cos2 x dx
−2 sin x sin x
= = −2 tan x tan x =
cos x cos x
(m) Solution.
dy d  cot(−x) 
= 3
dx dx
d d  g(x)  d
= (ln 3)3cot(−x) [cot(−x)] a = (ln a)ag(x) [g(x)]
dx dx dx
d d
[cot(g(x))] =
= (ln 3)3cot(−x) (− csc2 (−x)) [−x] dx
d
dx − csc2 (g(x)) dx [g(x)]
= (ln 3)3cot(−x) (− csc2 (−x))(−1)
= (ln 3)3cot(−x) csc2 (−x)
(n) Solution.
dy d  csc(log x) 
= e
dx dx
d d  g(x)  d
= ecsc(log x) [csc(log x)] e = eg(x) [g(x)]
dx dx dx
d
 
csc(log x) d [csc(g(x))] =
=e − csc(log x) cot(log x) [log x] dx
d
dx − csc(g(x)) cot(g(x)) dx [g(x)]
log x = log10 x,
 
csc(log x) 1
=e − csc(log x) cot(log x) d
[loga x] = (ln1a)x
(ln 10)x dx

ecsc(log x) csc(log x) cot(log x)


=−
(ln 10)x
4.6. Derivatives of Trigonometric Functions 243

(o) Solution. Using the rules (quotient rule, product rule, chain rule, sum rule,
constant rule, constant multiple rule, power rule), we get

d x5 csc(3x)
 
dy
=
dx dx 1 + 4 cot x2
d d
dx
[x5
csc(3x)](1 + 4 cot x2 ) − x5 csc(3x) dx [1 + 4 cot x2 ]
=
(1 + 4 cot x2 )2
d d d d
 
dx
[x5 ] csc(3x) + x5 dx [csc(3x)] (1 + 4 cot x2 ) − x5 csc(3x) dx
[1] + 4 dx [cot x2 ]
=
(1 + 4 cot x2 )2
d

5x4 csc(3x) + x5 (− csc(3x) cot(3x)) dx [3x] (1 + 4 cot x2 )
=
(1 + 4 cot x2 )2
d

x5 csc(3x) 0 + 4(− csc2 (x2 ) dx [x2 ])

(1 + 4 cot x2 )2
(5x4 csc(3x) − x5 csc(3x) cot(3x)(3)) (1 + 4 cot x2 ) + 4x5 csc(3x) csc2 (x2 )(2x)
=
(1 + 4 cot x2 )2
(5x4 csc(3x) − 3x5 csc(3x) cot(3x)) (1 + 4 cot x2 ) + 8x6 csc(3x) csc2 (x2 )
=
(1 + 4 cot x2 )2

3. [Exercise on page 36]


Solution.

ˆ First, we need to find the derivative of y = cos(3x).

dy d
= [cos(3x)]
dx dx
d d d
= − sin(3x) [3x] [cos(g(x))] = − sin(g(x)) [g(x)]
dx dx dx
− sin(3x)(3) = −3 sin(3x)

ˆ The slope of the tangent line at π 1 π



,
9 2
is obtained by substituting x with 9
into
the derivative:
  π  π  √ ! √
3 3 3
m = −3 sin 3 = −3 sin = −3 =−
9 3 2 2

ˆ The equation of the tangent line to the curve y = cos(3x) at the point π 1

,
9 2
is

1 3 3 π
y− =− x−
2 2 9
√ √
1 3 3 3π 3
y− =− x+
2 2 18
√ √
3 3 3π 3 1
y=− x+ +
2 18 2
244 Chapter 4. Differentiation Rules

4. [Exercise on page 36]


Solution.

ˆ First we need to find the derivative. Using the Chain Rule, we have
dy d
= [sin(sin x)]
dx dx
d d d
= cos(sin x) [sin x] [sin(g(x))] = cos(g(x)) [g(x)]
dx dx dx
= cos(sin x)(cos x) = cos x cos(sin x)

ˆ The slope at (π, 0) is then

m = cos π cos(sin π) = (−1) cos(0) = (−1)(1) = −1

ˆ Equation of the tangent line: y − π = −(x − 0) or y = −x + π.

5. [Exercise on page 37]

(a) Solution. The first derivative of f (x) = x cos(4x) is


d d
f 0 (x) = [x] cos(4x) + x [cos(4x)] Product Rule
dx dx
d
 
d [cos(g(x))] =
= (1) cos(4x) + x − sin(4x) [4x] dx
d
dx − sin(g(x)) dx [g(x)]
= cos(4x) + x (− sin(4x)(4))
= cos(4x) − 4x sin(4x)

The second derivative is


d 0 d
f 00 (x) = [f (x)] = [cos(4x) − 4x sin(4x)]
dx dx
d d
= [cos(4x)] − [4x sin(4x)]
dx dx
 
d d d
= − sin(4x) [4x] − [4x] sin(4x) + 4x [sin(4x)]
dx dx dx
 
d
= − sin(4x)(4) − 4 sin(4x) + 4x cos(4x) [4x]
dx
= −4 sin(4x) − (4 sin(4x) + 4x cos(4x)(4))
= −4 sin (4x) − 4 sin (4x) − 16x cos (4x)

(b) Solution. The first derivative of f (x) = sin6 (x) is


d 
f 0 (x) = (sin x)6

dx
d d d
= 6(sin x)5 [sin x] [(g(x))n ] = n(g(x))n−1 [g(x)]
dx dx dx
4.6. Derivatives of Trigonometric Functions 245

d
= 6(sin x)5 cos x [sin x] = cos x
dx
The second derivative is
d 0 d 
f 00 (x) = 6(sin x)5 cos x

[f (x)] =
dx dx
d 
(sin x)5 cos x

=6
dx
 
d  5
 5 d
=6 (sin x) cos x + (sin x) [cos x] Product Rule
dx dx
 
4 d 5
= 6 5(sin x) [sin x](cos x) + (sin x) (− sin x)
dx
= 6 5(sin x)4 cos x(cos x) − (sin x)5 sin x


= 30 sin4 x cos2 x − 6 sin6 x

6. [Exercise on page 37]


(a) Solution. The derivative of R with respect to t is

0 d h π  i
R (t) = 250 sin t + 0.1 + 700
dt 8
d h π i d
= 250 sin t + 0.1 + [700]
dt 8 dt
d h π i
= 250 sin t + 0.1 + 0
dt 8
π  d hπ i
= 250 cos t + 0.1 t + 0.1
8 dt 8
π  π 
= 250 cos t + 0.1
8 8
125π π
= cos ( t + 0.1)
4 8
(b) Solution. Remember that the instantaneous rate of change of nothing but the
derivative. So the instantaneous rate of change of the revenue when t = 3 is
125π π 
R0 (3) = cos (3) + 0.1 ≈ $28.33/month
4 8
Note. Remember that to calculate T (x), where T is a basic trigonometric
function (sin, cos, tan, cot, sec or csc) we need to put the calculator in radian
mode.
(c) Solution. Substituting t with 4 into R0 (t), we get
125π π 
R0 (4) = cos (4) + 0.1 ≈ −$9.8/month
4 8
This means that after 4 months, that is, in January, the revenue is decreasing
at the rate of $9.8 per month.
246 Chapter 4. Differentiation Rules

4.7 Exponential Growth and Decay


1. [Exercise on page 37]
dy
(a) Solution. Suppose y is of the form y = y0 ekt . We want to show that dt
= ky.

dy d  kt  d  kt 
= y0 e = y0 e Constant Multiple Rule
dt dt dt
 
kt d d  g(t)  d
= y0 e [kt] e = eg(t) [g(t)]
dt dt dt
= y0 ekt (k)


= ky0 ekt = k y0 ekt = ky y = y0 ekt




(b) Solution.
i. For y = 10e7t , the initial quantity is y0 = 10 and the growth constant is
k = 7.
ii. For y = 53e0.3t , the quantity present at time t = 0 is y0 = 53 and the
growth constant is k = 0.3.
iii. For y = 110e−0.04t , the initial quantity is y0 = 110 and the decay constant
is k = −0.04.
iv. First we need to express y = 284e2−5t in the form y0 ekt .

y = 284e2−5t = 284e2 e−5t ea−b = ea e−b

This tells us that y0 = 284e2 ≈ 2098.49 and k = −5.

2. [Exercise on page 37]


Solution.

(a) If the initial quantity is 17 and the rate of change of y is 4y, then y0 = 17 and
k = 4. Therefore, y = 17e4t . Since k is positive, y grows exponentially.
(b) Here the initial quantity is 300 and dydt
= −0.05y. So y0 = 300, k = −0.05, and
−0.05t
y = 300e . Since k is negative, y decays exponentially.

3. [Exercise on page 37]

(a) Solution. Since y grows or decays exponentially, it is of the form y(t) = y0 ekt ,
where y0 and k are to be found.
ˆ The condition y(0) = 75 means that the initial quantity is y0 = 75.
ˆ Now, we find k. We will use the information y(2) = 105.

y(2) = 105
75e2k = 105 y(t) = 75ekt , y(2) = 75e2k
105
e2k = Divide both sides by 75
75
4.7. Exponential Growth and Decay 247

 
105
2k = ln Take the natural logarithm of both sides
75
ln 105

75
k= ≈ 0.17 Divide both sides by 2
2
So y = 75e0.17t .
(b) Solution. Because y grows or decays exponentially, y has the form y(t) = y0 ekt .
Since y(0) = 10000, it follows that y0 = 10000. So y = 10000ekt . To find k, we
solve the equation y(5) = 7046.88 or 10000e5k = 7046.88.
10000e5k = 7046.88
7046.88
e5k = Divide both sides by 10000
10000
 
7046.88
5k = ln Take the natural logarithm of both sides
10000
ln 7046.88

10000
k= ≈ −0.07 Divide both sides by 5
5
Thus, y = 10000e−0.07t .
(c) Solution. Again, y is of the form y(t) = y0 ekt , where y0 and k are to be found.
ˆ Substituting t with 2 into y(t) = y0 ekt , we get y(2) = y0 e2k . From the
problem, y(2) = 265.46. So
y0 e2k = 265.46 (4.7.1)
ˆ Similarly, the condition y(7) = 308.42 leads us to
y0 e7k = 308.42 (4.7.2)
To find y0 and k, we need to solve the system formed by the equations (4.7.1)
and (4.7.2). From (4.7.1), we have y0 = 265.46e2k
. Substituting this into (4.7.2), we
have
 
265.46 7k
e = 308.42
e2k
ea
265.46e5k = 308.42 b
= ea−b
e
308.42
e5k = Divide both sides by 265.46
265.46
 
308.42
5k = ln Take the natural logarithm of both sides
265.46
ln 308.42

265.46
k= ≈ 0.03
5
Using the equation y0 = 265.46
e2k
, we find
265.46 265.46
y0 = = ≈ 250
e2(0.03) e0.06
So y = 250e0.03t .
248 Chapter 4. Differentiation Rules

(d) Solution. We use the same approach as in part (c). First, the equation y(3) =
1182.80 is equivalent to y0 e3k = 1182.8. The second condition, y(8) = 558.72, is
equivalent to y0 e8k = 558.72. Dividing both sides of this latter equation by e8k ,
we get
558.72
y0 =
e8k
Substituting this into y0 e3k = 1182.8, we have

558.72 3k
e = 1182.8 or 558.72e−5k = 1182.8
e8k

Dividing both sides of the latter equation by 558.72, we get e−5k= 1182.8
558.72
. Taking
1182.8
the natural logarithm on both sides, we have −5k = ln 558.72 . Dividing both
sides by −5, we get
1182.8

ln 558.72
k= ≈ −0.15
−5
558.72
Using the equation y0 = e8k
, we find that

558.72 558.72
y0 = 8(−0.15)
= −0.12 ≈ 1855
e e
Hence, y = 1855e−0.15t .

4. [Exercise on page 38]

(a) Solution.
ˆ The sentence “grows exponentially” means that y is of the form y(t) = y0 ekt .
ˆ The sentence “A bacteria culture initially contains 2000 bacteria” means
that y0 = 2000. So y(t) = 2000ekt .
ˆ The sentence “After 4 hours the culture contains 3502 bacteria” means that
y(4) = 3502.
ˆ Since y(4) = 2000e4k , the equation y(4) = 3502 becomes

2000e4k = 3502
3502
e4k = Divide both sides by 2000
2000
 
3502
4k = ln Take the natural logarithm of both sides
2000
3502

ln 2000
k= ≈ 0.14
4
So y = 2000e0.14t .
(b) Solution. The number of bacteria after 7 hours is

y(7) = 2000e0.14(7) = 2000e0.98 ≈ 5329


4.7. Exponential Growth and Decay 249

(c) Solution. The rate of growth after 7 hours is the derivative of y at 3, that is,
y 0 (3). To get y 0 (3), first we need to find the derivative y 0 (t).
d  d  0.14t 
y 0 (t) = 2000e0.14t = 2000

e Constant Multiple Rule
dt dt
d d  g(t)  d
= 2000e0.14t [0.14t] e = eg(t) [g(t)]
dt dt dt
0.14t
= 2000e (0.14) = 280e0.14t
We now have
y 0 (7) = 280e0.14(7) = 280e0.98 ≈ 746.05
Thus, after 7 hours, the population of bacteria is increasing at the rate of ap-
proximately 746 bacteria per hour.
(d) Solution. We need to solve the equation y(t) = 10000 for t.
2000e0.14t = 10000
e0.14t = 5 Divide both sides by 2000
0.14t = ln 5 Take the natural logarithm of both sides
ln 5
t= ≈ 11.5 Divide both sides by 0.14
0.14
So, after approximately 11.5 hours, there will be 10000 bacteria.

5. [Exercise on page 38]

(a) Solution. Let y(t) be the number of bacteria after t hours. We want to find an
expression for y(t).
ˆ The sentence “grows at a rate proportional to its size” means that y grows
exponentially, that is, y is of the form y(t) = y0 ekt .
ˆ The sentence “A culture initially contains 33000 bacteria” means that y0 =
33000. So y(t) = 33000ekt .
ˆ The sentence “After 7 hours the culture contains 54000 bacteria” means
that y(7) = 54000. This is equivalent to
33000e7k = 54000
54000
e7k = Divide both sides by 33000
33000
 
54000
7k = ln Take the natural logarithm of both sides
33000
ln 54000

33000
k= ≈ 0.07 Divide both sides by 7
7
So y(t) = 33000e0.07t .
(b) Solution. The number of bacteria after 8 hours is
y(8) = y(t) = 33000e0.07(8) = 33000e0.56 ≈ 57772
250 Chapter 4. Differentiation Rules

(c) Solution. The rate of growth after 8 hours is y 0 (8). But

d  d  0.07t 
y 0 (t) = 33000e0.07t = 33000

e
dt dt
d
= 33000e0.07t [0.07t] = 33000e0.07t (0.07) = 2310e0.07t
dt
Substituting t with 8 into y 0 (t), we get

y 0 (8) = 2310e0.07(8) = 2310e0.56 ≈ 4044 bacteria per hour

(d) Solution. We want to find t such that y(t) = 2y0 = 2(33000) = 66000.

y(t) = 66000
33000e0.07t = 66000
e0.07t = 2 Divide both sides by 33000
0.07t = ln 2 Take the natural logarithm of both sides
ln 2
t= ≈ 9.9 Divide both sides by 0.07
0.07
So the population of bacteria will double after approximately 9.9 hours.

6. [Exercise on page 38]

(a) Solution. Let m(t) be the mass after t years. We want to find a formula for
m(t). Since the sample decays exponentially, m(t) is of the form m(t) = m0 ekt ,
where m0 is the initial mass. From the sentence “Initially, the sample weighs 10
g”, it follows that m0 = 10. So

m(t) = 10ekt

Now, the sentence “After 6 years, the sample weighs 7 g” means that m(6) = 7.
But m(6) = 10ek(6) = 10e6k . So the equation m(6) = 7 becomes

10e6k = 7
e6k = 0.7 Divide both sides by 10
6k = ln(0.7) Take the natural logarithm of both sides
ln(0.7)
k= ≈ −0.06 Divide both sides by 6
6

So the mass remaining after t years is m(t) = 10e−0.06t .


(b) Solution. The mass of the sample after 13 years is

m(13) = 10e−0.06(13) ≈ 4.58 g


4.7. Exponential Growth and Decay 251

(c) Solution. The derivative of m(t) is


d  −0.06  d  −0.06 
m0 (t) = 10e = 10 e Constant Multiple Rule
dt dt
 
−0.06 d d  g(t)  d
= 10 e [−0.06t] e = eg(t) [g(t)]
dt dt dt
= 10 e−0.06t (−0.06) = −0.6e−0.06t


So the rate of change of the mass of the sample after 13 years is

m0 (13) = −0.6e−0.06(13) ≈ −0.275 g per year

This means that after 13 years, the mass of the sample is decreasing at the rate
of 0.275 g per year.
m0
(d) Solution. We need to solve the equation m(t) = 2
for t.
m0
m0 e−0.06t =
2
1
e−0.06t = Divide both sides by m0
2
−0.06t = ln(0.5) Take the natural logarithm of both sides
ln(0.5)
t= ≈ 11.55 Divide both sides by −0.06
−0.06
Thus, half of the weight of the sample will remain after approximately 11.55
years.

7. [Exercise on page 38]


Solution.

Half-life
Let y = y(t) be a quantity that decays over time (for example a radioactive
substance), and let y0 be the initial quantity. The half-life of y is the time
required for y0 to be reduced by one-half. So, if T is the half-life of y, then
y0
y(T ) =
2
y0
To find the half-life of y, all we have to do is to solve the equation y(t) = 2
for t.

Here we have m(t) = m0 e−0.02476t . To find the half-life, we need to solve the equation
m(t) = m20 for t.
m0
m0 e−0.02476t =
2
1
e−0.02476t = Divide both sides by m0
2
252 Chapter 4. Differentiation Rules

−0.02476t = ln(0.5) Take the natural logarithm of both sides


ln(0.5)
t= ≈ 28 Divide both sides by −0.02476
−0.02476

So the half-life is approximately 28 days.

8. [Exercise on page 38]

(a) Solution. Let m(t) be the mass remaining after t days. First, we need to find
an expression for m(t) of the form m0 ekt .
ˆ From the sentence “A sample has a mass of 35 mg initially”, it follows that
m0 = 35. So m(t) = 35ekt .
ˆ Since the half-life is 140 days, it follows that m(140) = 35
2
. But m(140) =
35e140k . So the equation m(140) = 35 2
becomes

35
35e140k =
2
1
e140k = Divide both sides by 35
2
140k = ln(0.5) Take the natural logarithm of both sides
ln(0.5)
k= ≈ −0.005 Divide both sides by 140
140

So m(t) = 35e−0.005t .
Now, the mass remaining after 200 days is

m(200) = 35e−0.005(200) = 35e−1 ≈ 12.88 mg

(b) Solution. We need to solve the equation m(t) = 6 for t.

35e−0.005t = 6
6
e−0.005t = Divide both sides by 35
35
 
6 Take the natural loga-
−0.005t = ln
35 rithm of both sides
6

ln 35
t= ≈ 352.72 days Divide both sides by −0.005
−0.005

So it takes about 352.72 days for a sample weighting 35 mg to decay to a mass


of 6 mg.

9. [Exercise on page 38]

(a) Solution. We will use the following theorem which was stated in Exercise 2.
4.7. Exponential Growth and Decay 253

Exponential Growth or Decay – Theorem


Let y be a quantity that depends on time t, and let y0 be the initial
quantity.
Theorem 4.7.1. Suppose the rate of change of y, dy dt
, is proportional to
dy
y, say dt = ky, where k is a constant. Then y is of the form

y(t) = y0 ekt .

Let y be the world population (in billions) after t years.


ˆ From the problem, the population present at time t = 0 is 6.143 billion. So
y0 = 6.143.
ˆ From the sentence “the population grows at the rate of approximately 1.31%
per year”, it follows that the rate of change of y is 1.31%y, that is,

dy
= 0.0131y
dt
This means that k = 0.0131.
So, by Theorem 4.7.1, the population after t years is

y(t) = 6.143e0.0131t billion

(b) Solution. Since t = 0 corresponds to 2000, in 2021, t = 21. So the estimated


population in 2021 is

y(21) = 6.143e0.0131(21) ≈ 8.09 billion

In 2040, t = 40 and the estimated population is

y(40) = 6.143e0.0131(40) ≈ 10.37 billion

(c) Solution. In 2025, t = 25 and the required rate of change is y 0 (25). But

d  d  0.0131t 
y 0 (t) = 6.143e0.0131t = 6.143

e Constant Multiple Rule
dt dt
d d  g(t)  d
= 6.143e0.0131t [0.0131t] e = eg(t) [g(t)]
dt dt dt
0.0131t
= 6.143e (0.0131) = 0.08e0.0131t

Substituting t with 25 into y 0 (t), we get

y 0 (25) = 0.08e0.0131(25) ≈ 0.11

This means that in 2025, the world population will be increasing at a rate of
approximately 0.11 billion per year.
254 Chapter 4. Differentiation Rules

(d) Solution. We need to solve the equation y(t) = 2y0 for t.


6.143e0.0131t = 2(6.143)
e0.0131t = 2 Divide both sides by 6.143
0.0131t = ln 2 Take the natural logarithm of both sides
ln 2
t= ≈ 53 Divide both sides by 0.0131
0.0131
Thus, if the population continues to grow at the rate of approximately 1.31% per
year, after approximately 53 years, that is, around 2053, the world population
will double (the population in 2000).
10. [Exercise on page 39]
(a) Solution. Remember that if interest is compounded continuously, the com-
pound amount, A, is given by the formula
A = P ert
where P is the initial amount and r is the interest rate. So, if P = 4000, t = 7,
and A = 9000,
4000e7r = 9000
e7r = 2.25 Divide both sides by 4000
Take the natural loga-
7r = ln(2.25)
rithm of both sides
ln(2.25)
r= ≈ 0.1158 = 11.58% Divide both sides by 7
7
(b) Here P = 11000, t = 5, and A = 15700. Substituting these into the equation
A = P ert , we have 11000e5r = 15700. Solving this latter equation for r, we get
r ≈ 7.12%.
11. [Exercise on page 39]
(a) Solution. We know that the compound amount is given by the formula
A(t) = P ert
where P is the initial amount and r is the rate of interest.
ˆ From the sentence “Suppose you invest $25500 in an account”, it follows
that P = 25500. So A(t) = 25500ert .
ˆ From the sentence “suppose that after 1 year your amount on deposit is
$27076.83”, it follows that A(1) = 27076.83. But A(1) = 25500er(1) =
25500er . So the equation A(1) = 27076.83 becomes
25500er = 27076.83
27076.83
er = Divide both sides by 25500
25500
 
27076.83 Take the natural logarithm
r = ln ≈ 0.06 = 6%
25500 of both sides
4.7. Exponential Growth and Decay 255

Thus, A(t) = 25500e0.06t .


(b) Solution. The compound amount after 5 years is

A(5) = 25500e0.06(5) = 25500e0.3 ≈ $34421.40

(c) Solution. The rate of growth of A after 5 years is the derivative of A evaluated
at 5, that is, A0 (5). So we first need to find A0 (t).
d  d  0.06t 
A0 (t) = 25500e0.06t = 25500

e Constant Multiple Rule
dt dt
d d  g(t)  d
= 25500e0.06t [0.06t] e = eg(t) [g(t)]
dt dt dt
0.06t
= 25500e (0.06) = 1530e0.06t

Now, substituting t with 5 into A0 (t), we have

A0 (5) = 1530e0.06(5) = $2065.28 per year

This means that after 5 years the amount on deposit is increasing at the rate of
approximately $2065.28 per year.

12. [Exercise on page 39]


Solution. First, we need to find a formula for the amount of money on deposit,
A(t) = P ert , after t years.

ˆ Since the initial amount is $7500, it follows that P = 7500.


ˆ Since after 2 years the amount on deposit is $8247.44, it follows that A(2) =
8247.44. Substituting t with 2 int A(t), we have A(2) = P er(2) = 7500e2r . So
the equation A(2) = 8247.44 becomes

7500e2r = 8247.44
8247.44
e2r = Divide both sides by 7500
7500
 
8247.44 Take the natural logarithm
2r = ln
7500 of both sides
8247.44

ln 7500
r= ≈ 0.0475 = 4.75% Divide both sides by 2
2
So
A(t) = 7500e0.0475t
Substituting t with 8 into A(t), we get the amount on deposit after 8 years:

A(8) = 7500e0.0475(8) = 7500e0.38 ≈ $10967.13

13. [Exercise on page 39]

(a) Solution.
256 Chapter 4. Differentiation Rules

Present Value
ˆ Suppose P dollars is invested at an interest rate r compounded m
times per year. Then the compound amount (denoted A) after t
years is given by the formula
 r tm
A=P 1+ (4.7.3)
m
If A, r, m, and t are given, then P can be found using (4.7.3). The
amount P is called the present value of A dollars. In other words,
the present value of A dollars is the amount of money that should
be invested today in order to end up with A dollars after t years.

ˆ If the rate of interest is compounded continuously, then A is given


by the formula A = P ert . Dividing both sides by ert , we have P =
A
ert
= Ae−rt . So, if interest is compounded continuously, the present
value is given by
P = Ae−rt

Here A = $14000, r = 5% = 0.05, t = 9, and m = 2 (“semiannually” indicates


tm
that m = 2). Substituting these into the formula A = P 1 + mr , we have
 9(2)
0.05
P 1+ = 14000
2
 9(2)
14000 0.05
P = 9(2) Divide both sides by 1 +
1 + 0.05 2
2
14000
= ≈ $8976.32
(1.025)18

(b) Solution. Here A = 14000, r = 3% = 0.03, t = 6, and m = 4 (“quarterly”


indicates that m = 4). So
 6(4)
0.03
P 1+ = 14000
4
 6(4)
14000 0.03
P = 6(4) Divide both sides by 1 +
1 + 0.03 2
4
14000
= ≈ $11701.64
(1.0075)24

14. [Exercise on page 39]


Solution. Here A = $78500, r = 2.78% = 0.0278, t = 4, and interest is compounded
continuously. So the present value is

P = Ae−rt = 78500e−0.0278(4) ≈ $70238.64


4.7. Exponential Growth and Decay 257

15. [Exercise on page 39]


Solution. Since A = $40000, r = 3.9% = 0.039, t = 5, and since interest is com-
pounded continuously, the present value is

P = Ae−rt = 40000e−0.039(5) ≈ $32913.39


258 Chapter 4. Differentiation Rules
Chapter5

Applications of the Derivative I

5.1 Increasing and Decreasing Functions


1. [Exercise on page 41]

(a) Solution.

Critical Numbers and Critical Points


Let f be a function.

ˆ A number c in the domain of f is called a critical number of f if


f 0 (c) = 0 or f 0 (c) does not exist.

ˆ If c is a critical number of f , the point (c, f (c)) is called critical point


for f .

To find the critical numbers of f one can proceed as follows.

(i) Find the derivative of f (x).

(ii) Find all numbers for which the derivative is zero. To do this, set the
derivative f 0 (x) equal to zero and solve for x.

(iii) Find all numbers c for which f 0 (c) is undefined and f (c) is defined.

(iv) The critical numbers for f are the numbers found in steps (ii) and
(iii).

We want to find the critical numbers for f (x) = −2x2 + 8x − 3.


ˆ The derivative is f 0 (x) = −4x + 8.
ˆ Set f 0 (x) = 0. Then −4x + 8 = 0. Subtracting 8 from both sides, we get
−4x = −8. Dividing both sides by −4, we get x = 2.
ˆ There is no number c for which f 0 (c) does not exist because f 0 (x) is defined
for all x as it is a polynomial.
ˆ So the only critical number for f is 2.
(b) Solution.

259
260 Chapter 5. Applications of the Derivative I

ˆ The derivative of f (x) = x3 + x2 + 5 is f 0 (x) = 3x2 + 2x.


ˆ Set f 0 (x) = 0. Then 3x2 + 2x = 0. Factoring out x, we get x(3x + 2) = 0.
This implies x = 0 or x = − 32 .
ˆ The derivative is defined everywhere as it is a polynomial. So there is no c
for which f 0 (c) does not exist.
ˆ Thus, f has two critical numbers: − 23 and 0.
(c) Solution.
ˆ The derivative of f (x) = 3x4 + 8x3 − 18x2 is f 0 (x) = 12x3 + 24x2 − 36x.
ˆ Set f 0 (x) = 0. Then 12x3 + 24x2 − 36x = 0. Factoring out 12x, we get
12x(x2 + 2x − 3) = 0. Factoring this further, we get 12x(x − 1)(x + 3) = 0.
This latter equation implies that x = 0 or x = 1 or x = −3.
ˆ The derivative is defined everywhere as it is a polynomial. So there is no c
for which f 0 (c) does not exist.
ˆ Hence, the critical numbers for f are −3, 0, and 1.
(d) Solution.

ˆ The derivative of f (x) = 3
x is
d h 1 i 1 −2 1
f 0 (x) = x3 = x 3 = 2 .
dx 3 3x 3
2
ˆ Set f 0 (x) = 0. Then 12 = 0. Multiplying both sides by 3x 3 , we get 1 = 0,
3x 3
which is not true. So the equation f 0 (x) = 0 has no solution.
ˆ Here the derivative is a fraction; so it is undefined where the denominator
2
is 0, that is, 3x 3 = 0. Dividing both sides of this latter equation by 3, we
2
get x 3 = 0. This implies that x = 0. So the derivative does not exist when
x = 0. We now have to check if 0 is in the domain of f . In other √ words, we
have to check if f (0) is defined. This is the case because f (0) = 3 0 = 0.
ˆ So c = 0 is a critical number for f .
(e) Solution.

ˆ The derivative of f (x) = 1 − x2 is
d h√ i
f 0 (x) = 1 − x2
dx
d d
[1 − x2 ] d hp i [g(x)]
= dx√ g(x) = dxp
2 1 − x2 dx 2 g(x)
−2x −x
= √ =√
2 1−x 2 1 − x2
ˆ Setting the derivative equal to 0, we get √1−x
−x
2 = 0. Multiplying both sides

2
by 1 − x , we get −x = 0. This implies that x = 0.

ˆ The derivative does not exist where the denominator is 0. Set 1 − x2 = 0.
Squaring both sides, we get 1 − x2 = 0. Factoring the left hand side, we get
(1 − x)(1 + x) = 0. This implies that x = 1 or x = −1. So the derivative
5.1. Increasing and Decreasing Functions 261

does not exist at −1 and 1. Moreover, the function f is defined at these


numbers because
p √ √
f (−1) = 1 − (−1)2 = 1 − 1 = 0 and f (1) = 1 − 12 = 0

ˆ Thus, the critical numbers for f are −1, 0, and 1.


(f ) Solution.
ˆ The derivative of f (x) = 1
x
is

d −1 1
f 0 (x) = [x ] = (−1)x−2 = − 2
dx x
ˆ Set f 0 (x) = 0. Then − x1 = 0. Multiplying both sides by x, we get −1 = 0,
which is not true. So the equation f 0 (x) = 0 has no solution.
ˆ The derivative does not exist where the denominator is 0, that is, where
x2 = 0 or x = 0. So f 0 (0) does not exist. But the function is undefined at
0 because f (0) = 01 and the division by 0 is not defined. (In other words,
x = 0 is not in the domain of f .) So 0 is not a critical number.
ˆ Thus, the function f (x) = x1 has no critical number.
(g) Solution.
ˆ The derivative of f (x) = 2x+1
x−3
is
d d
0 dx
[2x + 1](x − 3) − (2x + 1) dx [x − 3]
f (x) = 2
Quotient Rule
(x − 3)
2(x − 3) − (2x + 1)(1) 2x − 6 − 2x − 1 −7
= 2
= 2
=
(x − 3) (x − 3) (x − 3)2

ˆ Set (x−3)
−7 2
2 = 0. Then, by multiplying both sides by (x − 3) , we get −7 = 0,

which is not true. So there is no number c such that f 0 (c) = 0.


ˆ The derivative is undefined where (x − 3)2 = 0. Taking the square root of
both sides, we get x − 3 = 0. Adding 3 to both sides, we get x = 3. So
the derivative does not exist when x = 3. We now have to check if f (x)
is defined at 3. Substituting x with 3 into f (x), we get f (3) = 2(3)+1
3−3
= 70 .
Since the division by 0 is not possible, f (3) is undefined. In other words, 3
is not in the domain of f .
ˆ Hence, f has no critical number.
(h) Solution.
ˆ The derivative of f (x) = x ln(3x) is
d d
f 0 (x) = [x] ln(3x) + x [ln(3x)] Product Rule
dx dx
d d
[3x] d [g(x)]
= (1) ln(3x) + x dx [ln g(x)] = dx
3x dx g(x)
3
= ln(3x) + x = ln(3x) + 1 Simplify
3x
262 Chapter 5. Applications of the Derivative I

ˆ Set ln(3x) + 1 = 0. Subtracting 1 from both sides, we get ln(3x) = −1.


Taking the natural exponential on both sides, we get 3x = e−1 . Dividing
−1
both sides by 3, we get x = e 3 .
ˆ The derivative is undefined for all x ≤ 0 (logarithm functions are defined
only for positive numbers). And the original function is also undefined for
the same values of x. So there is no c such that f 0 (c) does not exist and
f (c) is defined.
e−1
ˆ Hence, the only critical number of f is 3
.
(i) Solution.
ˆ The derivative of f (x) = x2 · 3−x is
d 2 d  −x 
f 0 (x) = [x ] · 3−x + x2 3 Product Rule
dx dx
d d  g(x)  d
= 2x · 3−x + x2 (ln 3)3−x [−x] a = (ln a)ag(x) [g(x)]
dx dx dx
d
= 2x · 3−x − x2 (ln 3)3−x [−x] = −1
dx
= x · 3−x (2 − x ln 3) Factor out x · 3−x
ˆ Set x · 3−x (2 − x ln 3) = 0. Then x = 0 or 3−x = 0 or 2 − x ln 3 = 0.
– The equation 3−x = 0 has no solution because the exponential of any
number is always positive.
– Solving 2 − x ln 3 = 0. Subtracting 2 from both sides, we get −x ln 3 =
−2. Dividing both sides by − ln 3, we get x = ln23 .
So the derivative is 0 when x = 0 or x = ln23 .
ˆ There is no c such that f 0 (c) does not exist because f 0 (x) is defined for all
x. (Recall that ax is defined for all x.)
ˆ Thus, the critical numbers for f are 0 and ln23 .

2. [Exercise on page 41]

(a) Solution.

Increasing and Decreasing Functions – Definition


Let f be a function, and let I be an interval.

ˆ The function f is said to be increasing on I if for every x, y in I,

f (x) < f (y) whenever x < y

ˆ The function f is said to be decreasing on I if for every x, y in I,

f (x) > f (y) whenever x < y

ˆ The function f constant on I if for every x, y in I, f (x) = f (y).


5.1. Increasing and Decreasing Functions 263

Increasing/Decreasing Test

Let f be a function, and let I be an interval. Suppose that f is differen-


tiable on I.

ˆ If f 0 (x) > 0 for all x in I, then f is increasing on I.

ˆ If f 0 (x) < 0 for all x in I, then f is decreasing on I.

ˆ If f 0 (x) = 0 for all x in I, then f is constant on I.

How to find the intervals where f is increasing or decreasing?

(i) Find the critical numbers for f .

(ii) Mark the critical numbers on the real line, as well as any points
where f is undefined. These points determine a certain number of
open intervals.

(iii) To find the sign of f 0 (x) on an open interval I found in step (ii),
choose any number, say t, in I. Then substitute x with t into
the derivative f 0 (x) . If f 0 (t) is positive, the function is increasing
on I. If f 0 (t) is negative, the function is decreasing on I.

We want to find the intervals where f (x) = −2x2 + 8x − 3 is increasing and


where it is decreasing.
ˆ From Problem 1(a), the critical number for f (x) = −2x2 + 8x − 3 is 2.
ˆ Plotting the critical number 2 on the real line, we get Figure 5.1. This point
determines two open intervals: (−∞, 2) and (2, ∞).

−∞ ∞
2

Figure 5.1: Marking the point 2 on the real line

ˆ Finding the sign of f 0 (x) on each interval.


– On the interval (−∞, 2), choose for example x = 0. Substituting x with
0 into the derivative, we get f 0 (0) = −4(0) + 8 = 8. Since f 0 (0) > 0, the
function is increasing on the interval (−∞, 2).
– On the interval (2, ∞), choose x = 3. Since f 0 (3) = −4(3) + 8 =
−12+8 = −4 is negative, the function f is decreasing on the interval
(2, ∞).
– All this is summarized in Figure 5.2
(b) Solution.
ˆ From Problem 1(b), the critical numbers for f (x) = x3 + x2 + 5 are − 32 and
0.
264 Chapter 5. Applications of the Derivative I

+ −
−∞ 2 ∞
choose 0 choose 3
f 0 (0) = 8 > 0 f 0 (3) = −4 < 0

Figure 5.2: Sign of f 0 (x) = −4x + 8

ˆ The sign of f 0 (x) = 3x2 + 2x is shown in Figure 5.3.

+ − +
−∞ ∞
− 23 0
choose −1 choose −0.5 choose 1
f 0 (−1) = 1 > 0 f 0 (−0.5) = −0.25 < 0 f 0 (1) = 5 > 0

Figure 5.3: Sign of f 0 (x) = 3x2 + 2x

ˆ From Figure 5.3,


– the function f is increasing on the intervals (−∞, − 23 ) and (0, ∞),
and
– decreasing on the interval − 23 , 0 .


(c) Solution.
ˆ From Problem 1(c), the critical numbers for f are −3, 0, and 1.
ˆ These numbers determine four intervals, and the sign of f 0 (x) in each of
these is shown in Figure 5.4. We choose 0.5 as the test value in the interval
(0, 1): f 0 (0.5) = 10.5 > 0.

− + − +
−∞ −3 0 1 ∞
choose −4 choose −2 choose 2
f 0 (−4) = −240 < 0 f 0 (−2) = 72 > 0 f 0 (2) = 120 > 0

Figure 5.4: Sign of f 0 (x) = 12x3 + 24x2 − 36x

ˆ From Figure 5.4, f is increasing on the intervals (−3, 0) and (1, ∞),
and decreasing on the intervals (−∞, −3) and (0, 1).
5.1. Increasing and Decreasing Functions 265

(d) Solution.
2
ˆ We first need to find the critical numbers of f (x) = (x − 1) 3 .
– The derivative is
2 1 d 2 1 2
f 0 (x) = (x − 1)− 3 [x − 1] = 1 (1) = 1 .
3 dx 3 (x − 1) 3 3(x − 1) 3
2 1
– Set 1 = 0. Multiplying both sides by 3(x − 1) 3 , we get 2 = 0,
3(x−1) 3
which is not true. So the equation f 0 (x) = 0 has no solution.
– The derivative does not exist when x = 1. Moreover, f (1) is defined
2
since f (1) = (1 − 1) 3 = 0.
– Thus, the critical number for f is 1.
ˆ The critical number 1 determines two open intervals: (−∞, 1) and (1, ∞).
And the sign of f 0 (x) in each of these is shown in Figure 5.5.

− +
−∞ 1 ∞
choose 0 choose 2
f 0 (0) = − 23 < 0 f 0 (2) = 2
3
>0

Figure 5.5: Sign of f 0 (x) = 2


1
3(x−1) 3

ˆ From Figure 5.5, the function f is decreasing on the interval (−∞, 1)


and increasing on the interval (1, ∞).
(e) Solution.
ˆ From Problem 1(g), the function f (x) = 2x+1x−3
has no critical number.
ˆ Marking the points on the real line. There is no critical number to mark.
However, the function is undefined at x = 3. So we need to mark 3 on the
real line. This gives two open intervals: (−∞, 3) and (3, ∞).
ˆ The sign of f 0 (x) is shown in Figure 5.6.

− −
−∞ 3 ∞
choose 0 choose 4
f 0 (0) = − 79 < 0 f 0 (4) = −7 < 0
−7
Figure 5.6: Sign of f 0 (x) = (x−3)2
266 Chapter 5. Applications of the Derivative I

ˆ From Figure 5.6, the function f is decreasing on the intervals (−∞, 3)


and (3, ∞).
Note. The graph of f does not cross the vertical line x = 3 because f is
undefined at 3. Actually, the line x = 3 is a vertical asymptote.

Alternate Solution
−7
The derivative of f (x) is f 0 (x) = (x−3) 2 . Since the denominator is always

greater than or equal to 0, and since the numerator is negative, it follows


that f 0 (x) < 0 for all x 6= 3. Therefore, f is decreasing on the intervals
(−∞, 3) and (3, ∞).

(f ) Solution.
ˆ We first need to find the critical numbers of f (x) = x2x+1 .
– The derivative is
d d
[x](x2 + 1) − x dx [x2 + 1]
 
0 d x dx
f (x) = = Quotient Rule
dx x2 + 1 (x2 + 1)2
1(x2 + 1) − x(2x) x2 + 1 − 2x2 1 − x2
= = =
(x2 + 1)2 (x2 + 1)2 (x2 + 1)2
2
– Set (x1−x 2 2 2
2 +1)2 = 0. Multiplying both sides by (x + 1) , we get 1 − x = 0.

Factoring this, we get (1 − x)(1 + x) = 0. The latter equation implies


that x = 1 or x = −1.
– Since x2 + 1 is always positive, there is no number c for which f 0 (c) does
not exist.
– So the critical numbers for f are −1 and 1.
ˆ These numbers determine three open intervals: (−∞, −1), (−1, 1), and (1, ∞).
The sign of f 0 (x) in each of these is shown in Figure 5.7

− + −
−∞ −1 1 ∞
choose −2 choose 0 choose 2
f 0 (−2) = − 25
3
<0 f 0 (0) = 1 > 0 f 0 (2) = − 25
3
<0

1−x2
Figure 5.7: Sign of f 0 (x) = (x2 +1)2

ˆ From Figure 5.7, f is increasing on the interval (−1, 1) and decreasing


on the intervals (−∞, −1) and (1, ∞).
(g) Solution.
3
ˆ We first need to find the critical numbers for f (x) = xe−x .
5.1. Increasing and Decreasing Functions 267

– The derivative is
d 3 d 3
f 0 (x) = [x]e−x + x [e−x ] Product Rule
dx dx
3 3 d d  g(x)  d
= (1)e−x + xe−x [−x3 ] e = eg(x) [g(x)]
dx dx dx
3 3 3 3
= e−x + xe−x (−3x2 ) = e−x − 3x3 e−x
3 3
= (1 − 3x3 )e−x Factor out e−x
3 3
– Set (1 − 3x3 )e−x = 0. Then 1 − 3x3 = 0. (Note that e−x 6= 0 for all
x.) Subtracting 1 from both sides, we get −3x3 = −1. Dividing both
sides by −3, we get x3 = 13 . Taking the cube root of both sides, we have
q
x = 3 13 .
– Since the derivative is defined everywhere, there is no c for which f 0 (c)
does not exist. q
– So the critical number for f is 3 13 .
ˆ This determines two open intervals, and the sign of f 0 (x) in each of these is
shown in Figure 5.8

+ −
−∞ q ∞
3 1
3

choose 0 choose 1
f 0 (0) = 1 > 0 f 0 (1) = −2e−1 < 0
3
Figure 5.8: Sign of f 0 (x) = (1 − 3x3 )e−x

 q 
ˆ Thus, f is increasing on the interval −∞, 3 13 and decreasing on
q 
the interval 3 13 , ∞ .
(h) Solution.
 
7x2 +6
ˆ We first need to find the critical numbers for f (x) = ln x2 +3
.
– The derivative is
h 2 i
d 7x +6 d
0 dx x2 +3 d dx
[g(x)]
f (x) = 7x2 +6
[ln g(x)] =
x2 +3
dx g(x)
d d
dx
[7x2 +6](x2 +3)−(7x2 +6) dx [x2 +3]
2
(x +3)2
= 7x2 +6
Quotient Rule
x2 +3
268 Chapter 5. Applications of the Derivative I

14x(x2 +3)−(7x2 +6)(2x)


d
(x2 +3)2 dx
[7x2 + 6] = 14x and
= d
7x2 +6
dx
[x2 + 3] = 2x
x2 +3
14x3 +42x−14x3 −12x
(x2 +3)2
= 7x2 +6
Distribute the numerator
x2 +3
a
(x2 + 3)
 
30x b a d
= 2 c =
(x + 3)2 (7x2 + 6) d
b c
30x
= Simplify
(x2 + 3)(7x2 + 6)
30x 2 2
– Set (x2 +3)(7x 2 +6) = 0. Multiplying both sides by (x + 3)(7x + 6), we

get 30x = 0. Dividing both sides by 30, we get x = 0.


– Since x2 + 3 and 7x2 + 6 are both always positive, there is no c for which
f 0 (c) is undefined.
– So the critical number for f is 0.
ˆ This number determines two intervals, and the sign of f 0 (x) in each of these
is shown in Figure 5.9.

− +
−∞ 0 ∞
choose −1 choose 1
−15
f 0 (−1) = 26
<0 f 0 (1) = 15
26
>0

Figure 5.9: Sign of f 0 (x) = 30x


(x2 +3)(7x2 +6)

ˆ Thus, f is decreasing on the interval (−∞, 0) and increasing on the


interval (0, ∞).

3. [Exercise on page 41]

(a) Solution.
ˆ The profit function is

P (x) = R(x) − C(x) = 0.936x2 − 0.0004x3 − (0.28x2 − 0.00006x3 )


= 0.936x2 − 0.0004x3 − 0.28x2 + 0.00006x3
= −0.00034x3 + 0.656x2

ˆ The derivative is
d d
P 0 (x) = [−0.00034x3 ] + [0.656x2 ] = −000034(3x2 ) + 0.656(2x)
dx dx
= −0.00102x2 + 1.312x = x(−0.00102x + 1.312)
5.2. Relative (or Local) Extrema 269

ˆ Set x(−0.00102x + 1.312) = 0. Then x = 0 or −0.00102x + 1.312 = 0. The


latter equation implies that −0.00102x = −1.312. Dividing both sides by
−0.00102, we get x ≈ 1286.27.
ˆ Thus, the critical numbers for P are 0 and 1286.27.
ˆ The sign of P 0 (x) is shown in Figure 5.10

+ −
0 1286.27 4000
choose 1 choose 1300
P 0 (1) = 1.31 > 0 P 0 (1300) = −18.2 < 0

Figure 5.10: Sign of P 0 (x) = −0.00102x2 + 1.312x

ˆ From Figure 5.10, we deduce that the profit is increasing on the interval
(0, 1286.27).
(b) Solution. Again from Figure 5.10, the profit is decreasing when the produc-
tion level is between 1286.27 and 4000 items.

5.2 Relative (or Local) Extrema


1. [Exercise on page 42]
Solution.

Relative Maximum and Minimum


Let f be a function and c be a number in the domain of f . Then

ˆ f (c) is a relative maximum (or local maximum) value of f if f (c) ≥ f (x)


for all x near c.

ˆ f (c) is a relative minimum (or local minimum) value of f if f (c) ≤ f (x)


for all x near c.

ˆ f (c) is a relative extremum if it is either a relative maximum or a relative


minumum. (The plural form of extremum is extrema.)

Graphically, a relative maximum is a peak while a relative minimum is a valley.

The First Derivative Test


Let f be a continuous function and c be a critical number of f .
270 Chapter 5. Applications of the Derivative I

ˆ If

f 0 (x) > 0 for all x near c, x < c and f 0 (x) < 0 for all x near c, x > c

then f (c) is relative maximum of f .

ˆ If

f 0 (x) < 0 for all x near c, x < c and f 0 (x) > 0 for all x near c, x > c

then f (c) is relative minimum of f .

ˆ If the derivative f 0 (x) is positive for all x near c, or negative for all x near
c, then f has no relative minimum or maximum at c.

In this problem, the function is defined by a graph.

ˆ If we look at the graph, we see that f is increasing on the interval (−∞, 0)


and decreasing on the interval (0, 2). So the sign of the derivative changes from
positive to negative at x = 0. By the first derivative test, f (0) = 0 is a relative
maximum of f .
ˆ Similarly, the sign of the derivative changes from negative to positive at x = 2.
So f (2) = −4 is a relative minimum of f .

2. [Exercise on page 42]

(a) Solution.

How to Find the Relative Extrema?


(i) Find the intervals where the function is increasing or decreasing.

(ii) Use the first derivative test to determine the relative maximum
and/or minimum (if any).

For the function f (x) = −2x2 + 8x − 3, the intervals where f is increasing


or decreasing were found in Section 5.1–Problem 2(a). This is summarized in
Figure 5.11 below.
ˆ From Figure 5.11, the derivative changes from positive to negative at x = 2.
So
f (2) = −2(2)2 + 8(2) − 3 = −8 + 16 − 3 = 5
is a relative maximum of f (by the first derivative test).
ˆ There is no relative minimum.
(b) Solution. In Section 5.1–Problem 2(b) we found the intervals where f is in-
creasing or decreasing. These are shown in Figure 5.12.
5.2. Relative (or Local) Extrema 271

+ −
−∞ 2 ∞
choose 0 choose 3
f 0 (0) = 8 > 0 f 0 (3) = −4 < 0

Figure 5.11: Sign of f 0 (x) = −4x + 8

+ − +
−∞ ∞
− 23 0
choose −1 choose −0.5 choose 1
f 0 (−1) = 1 > 0 f 0 (−0.5) = −0.25 < 0 f 0 (1) = 5 > 0

Figure 5.12: Sign of f 0 (x) = 3x2 + 2x

ˆ From Figure 5.12, the relative maximum of f is


   3  2
2 2 2 8 4
f − = − + − +5=− + +5=
3 3 3 27 9
8 12 135 139
+ − + = .
27 27 27 27
ˆ Again from Figure 5.12, the relative minimum of f is

f (0) = 03 + 02 + 5 = 5.

(c) Solution. In Section 5.1–Problem 2(c), we found the intervals where f is in-
creasing or decreasing. These are summarized in Figure 5.13 below.

− + − +
−∞ −3 0 1 ∞
choose −4 choose −2 choose 2
f 0 (−4) = −240 < 0 f 0 (−2) = 72 > 0 f 0 (2) = 120 > 0

Figure 5.13: Sign of f 0 (x) = 12x3 + 24x2 − 36x

ˆ From Figure 5.13, the relative maximum of f is

f (0) = 3(0)4 + 8(0)3 − 18(0)2 = 0.


272 Chapter 5. Applications of the Derivative I

ˆ The relative minimums are

f (−3) = 3(−3)4 + 8(−3)3 − 18(−3)2 = 243 − 216 − 162 = −135,

and
f (1) = 3(1)4 + 8(1)3 − 18(1)2 = 3 + 8 − 18 = −7.
(d) Solution. In Section 5.1–Problem 2(d), we found the intervals where f is
increasing or decreasing. These are shown in Figure 5.14.

− +
−∞ 1 ∞
choose 0 choose 2
f 0 (0) = − 23 < 0 f 0 (2) = 2
3
>0

Figure 5.14: Sign of f 0 (x) = 2


1
3(x−1) 3

ˆ From Figure 5.14, the relative minimum of f is


2 2
f (1) = (1 − 1) 3 = 0 3 = 0.

ˆ There is no relative maximum.


(e) Solution. In Section 5.1–Problem 2(e), we found the intervals where f is in-
creasing or decreasing. These are shown in Figure 5.15.

− −
−∞ 3 ∞
choose 0 choose 4
f 0 (0) = − 79 < 0 f 0 (4) = −7 < 0
−7
Figure 5.15: Sign of f 0 (x) = (x−3)2

From Figure 5.15, there is no relative extremum because the sign of the
derivative does not change.
(f ) Solution. In Section 5.1–Problem 2(f), we found the intervals where f is in-
creasing or decreasing. These are shown in Figure 5.16.
ˆ From Figure 5.16, the relative minimum of f is f (−1) = −1
(−1)2 +1
= − 12 .
ˆ The relative maximum is f (1) = 1
12 +1
= 12 .
(g) Solution.
5.2. Relative (or Local) Extrema 273

− + −
−∞ −1 1 ∞
choose −2 choose 0 choose 2
f 0 (−2) = − 25
3
<0 f 0 (0) = 1 > 0 f 0 (2) = − 25
3
<0

1−x2
Figure 5.16: Sign of f 0 (x) = (x2 +1)2

ˆ The derivative of f (x) = 4x ln(5x) is


d d
f 0 (x) = [4x] ln(5x) + 4x [ln(5x)] Product Rule
dx dx
d d
[5x] d [g(x)]
= 4 ln(5x) + 4x dx [ln g(x)] = dx
5x dx g(x)
5
= 4 ln(5x) + 4x = 4 ln(5x) + 4 = 4(1 + ln(5x))
5x
ˆ Set f 0 (x) = 0. Then 4(1 + ln(5x)) = 0. Dividing both sides by 4, we get
1 + ln(5x) = 0. Subtracting 1 from both sides, we get ln(5x) = −1. Taking
the natural exponential on both sides, we get 5x = e−1 . Dividing both sides
−1
by 5, x = e 5 .
−1
ˆ So the critical number for f is c = e 5 .
ˆ The sign of the derivative is shown in Figure 5.17. Note that f (x) is
defined only for positive values of x because the logarithm function loga x
(a > 0, a 6= 1) is defined for all x > 0.

− +
0 e−1 ∞
5
≈ 0.074
choose 0.05 choose 1
f 0 (0.05) ≈ −1.55 < 0 f 0 (1) ≈ 10.44 > 0

Figure 5.17: Sign of f 0 (x) = 4(1 + ln(5x))

ˆ From Figure 5.17, the relative minimum of f is


 −1   −1   −1 
e e e
f =4 ln 5
5 5 5
4e−1
 −1 
e  4e−1
=4 ln e−1 = (−1) = −
5 5 5
ˆ There is no relative maximum.
274 Chapter 5. Applications of the Derivative I

(h) Solution. In Section 5.1–Problem 2(g), we found the intervals where f is


increasing or decreasing. These are shown in Figure 5.18.

+ −
−∞ q ∞
3 1
3

choose 0 choose 1
f 0 (0) = 1 > 0 f 0 (1) = −2e−1 < 0
3
Figure 5.18: Sign of f 0 (x) = (1 − 3x3 )e−x

ˆ From Figure 5.18, the relative maximum is


r !   13 !   13  1 13 3   13
3 1 1 1 − (3) 1 1
f =f = e = e− 3 (am )n = amn
3 3 3 3
1
1 − 31 e− 3 1 1 1 1 √
= 1 e = √3
= 1√3
= √ √3
= √
3
an = n
a
(3) 3 3 e3 3 3
e 3 3e

ˆ There is no relative minimum.


(i) Solution.
ˆ The derivative of f (x) = (e2x − 3)5 is

d 2x d d
f 0 (x) = 5(e2x − 3)4 [e − 3] [(g(x))n ] = n(g(x))n−1 [g(x)]
dx dx dx
 
d 2x d
= 5(e2x − 3)4 [e ] − [3] Difference Rule
dx dx
 
2x 4 2x d d  g(x)  d
= 5(e − 3) e [2x] − 0 e = eg(x) [g(x)]
dx dx dx
= 5(e2x − 3)4 2e2x = 10e2x (e2x − 3)4


ˆ Set f 0 (x) = 0. Then 10e2x (e2x − 3)4 = 0. This implies that (e2x − 3)4 = 0.
(Note that 10e2x 6= 0 for all x.) Taking the 4th root of both sides, we get
e2x − 3 = 0. Adding 3 to both sides, we get e2x = 3. Taking the natural
logarithm on both sides, we get 2x = ln 3. Dividing both sides by 2, we get
x = ln23 .
ˆ So the critical number for f is c = ln23 .
ˆ The sign of the derivative is shown in Figure 5.19.
ˆ From Figure 5.19, there is no relative extremum.
(j) Solution. In Section 5.1–Problem 2(h), we found the intervals where f is
increasing or decreasing. These are shown in Figure 5.20.
5.2. Relative (or Local) Extrema 275

+ +
−∞ ∞
ln 3
2
≈ 0.55
choose 0 choose 1
f 0 (0) = 160 > 0 f 0 (1) = 10e2 (e2 − 3)4 > 0

Figure 5.19: Sign of f 0 (x) = 10e2x (e2x − 3)4

− +
−∞ 0 ∞
choose −1 choose 1
−15
f 0 (−1) = 26
<0 f 0 (1) = 15
26
>0

Figure 5.20: Sign of f 0 (x) = 30x


(x2 +3)(7x2 +6)

ˆ From Figure 5.20, the relative minimum of f is

7(0)2 + 6
   
6
f (0) = ln 2
= ln = ln (2)
0 +3 3

ˆ There is no relative maximum.

3. [Exercise on page 42]

(a) Solution.
ˆ We first need to find the profit function P (q). Using the formula P (q) =
R(q) − C(q), where R(q) = pq, is the revenue, we get

P (q) = pq − C(q) = (−3q + 80)q − (8q + 100)


= −3q 2 + 80q − 8q − 100 = −3q 2 + 72q − 100

ˆ The derivative of P (q) is

d d
P 0 (q) = [P (q)] = [−3q 2 + 72q − 100] = −6q + 72
dq dq

ˆ Set P 0 (q) = 0. Then −6q + 72 = 0. Subtracting 72 from both sides, we get


−6q = −72. Dividing both sides by −6, we get q = 12.
ˆ So the critical number for P is 12.
ˆ The sign of P 0 (q) is shown in Figure 5.21.
From Figure 5.21, the number of units that produces maximum profit is q = 12.
276 Chapter 5. Applications of the Derivative I

+ −
0 12 ∞
choose 1 choose 13
P 0 (1) = 66 > 0 P 0 (13) = −6 < 0

Figure 5.21: Sign of P 0 (q) = −6q + 72

(b) Solution. The price that produces maximum profit is

p = −3(12) + 80 = −36 + 80 = 44

(c) Solution. The maximum profit is

P (12) = −3(12)2 + 72(12) − 100 = −432 + 864 − 100 = 332

4. [Exercise on page 42]

(a) Solution.
ˆ The profit function is

P (q) = R(q) − C(q) = pq − (120 + 50qe−0.02q )


= 100qe−0.02q − 120 − 50qe−0.02q = 50qe−0.02q − 120

ˆ The derivative is
d  d
P 0 (q) = 50qe−0.02q − [120]

Difference Rule
dq dq
d d  −0.02q  Product Rule and
= [50q]e−0.02q + 50q e −0
dq dq Constant Rule
d d  g(q)  d
= 50e−0.02q + 50qe−0.02q [−0.02q] e = eg(q) [g(q)]
dq dq dq
d
= 50e−0.02q + 50qe−0.02q (−0.02) [−0.02q] = −0.02
dq
= 50e−0.02q − qe−0.02q = (50 − q)e−0.02q Factor out e−0.02q

ˆ Set P 0 (q) = 0. Then (50 − q)e−0.02q = 0. This implies that 50 − q = 0. (Note


that e−0.02q 6= 0 for all q.) Solving this latter equation, we get q = 50, which
is the critical number for P .
ˆ The sign of P 0 (q) is shown in Figure 5.22.
From Figure 5.22, the number of units that produces maximum profit is q = 50.
(b) Solution. The price that produces maximum profit is

p = 100e−0.02(50) = 100e−1 ≈ 36.79


5.2. Relative (or Local) Extrema 277

+ −
0 50 ∞
choose 1 choose 51
P 0 (1) = 49e−0.02 > 0 P 0 (51) = −e−1.02 < 0

Figure 5.22: Sign of P 0 (q) = (50 − q)e−0.02q

(c) Solution. The maximum profit is

P (50) = 50(50)e−0.02(50) − 120 = 2500e−1 − 120 ≈ 799.7

5. [Exercise on page 42]

(a) Solution.
ˆ The profit function is

P (x) = R(x) − C(x)


= 0.936x2 − 0.0004x3 − (0.28x2 − 0.00006x3 )
= 0.936x2 − 0.0004x3 − 0.28x2 + 0.00006x3
= −0.00034x3 + 0.656x2

ˆ In Section 5.1–Problem 3, we found the intervals where P is increasing or


decreasing. These are shown in Figure 5.23.

+ −
0 1286.27 4000
choose 1 choose 1300
P 0 (1) = 1.31 > 0 P 0 (1300) = −18.2 < 0

Figure 5.23: Sign of P 0 (x) = −0.00102x2 + 1.312x

ˆ From Figure 5.23, the number of units that should be sold in order to
maximize the profit is approximately 1286.27.
(b) Solution. The maximum profit is

P (1286.27) = −0.00034(1286.27)3 + 0.656(1286.27)2 ≈ $361784.46


278 Chapter 5. Applications of the Derivative I

5.3 Concavity
1. [Exercise on page 43]
Solution.

Concavity and Inflection Point


Let f be a function and I be an interval.

ˆ The function f is concave upward on I if the tangent line to the graph of


f at each point x in I lies below the graph.

ˆ The function f is concave downward on I if the tangent line to the graph


of f at each point x in I lies above the graph.

ˆ An inflection point of f is a point on the graph of f where the concavity


changes.

ˆ The function f whose graph is shown in Figure 5.2 is concave downward on the
interval (−∞, 1) and concave upward on the interval (1, ∞).
ˆ Since the concavity changes at x = 1, the point (1, −2) is an inflection point.

2. [Exercise on page 43]

(a) Solution.

Second Derivative
Let f be a function.

ˆ The derivative of f , f 0 (x), is sometimes called the first derivative of


f.

ˆ The second derivative of f , denoted f 00 (x), is the derivative of the


first derivative. That is,
d 0
f 00 (x) = [f 0 (x)]0 = [f (x)] .
dx

ˆ For f (x) = 3x4 − 2x3 + 7x − 11, the first derivative is

f 0 (x) = 12x3 − 6x2 + 7

ˆ The second derivative is


d 0 d
f 00 (x) = [f (x)] = [12x3 − 6x2 + 7] = 36x2 − 12x
dx dx
5.3. Concavity 279

(b) Solution.
d d 0 d
f 0 (x) = [−5x + 6] = −5 and f 00 (x) = [f (x)] = [−5] = 0
dx dx dx
(c) Solution. The first derivative is
d h −x3 i 3 d d  g(x)  d
f 0 (x) = e = e−x [−x3 ] e = eg(x) [g(x)]
dx dx dx dx
3 3
= e−x (−3x2 ) = −3x2 e−x

The second derivative is


d h 3
i d  3 d h −x3 i
f 00 (x) = −3x2 e−x = −3x2 e−x − 3x2

e Product Rule
dx dx dx
3
 3
 3 3
= −6xe−x − 3x2 −3x2 e−x = −6xe−x + 9x4 e−x
3 3
= (−6x + 9x4 )e−x Factor out e−x

(d) Solution. The first derivative is


d d
d  [x2 + 1] d [g(x)]
f 0 (x) = ln(x2 + 1) = dx dx

[ln g(x)] =
dx x2 + 1 dx g(x)
2x
= 2
x +1
The second derivative is
 
00 d 0 d 2x
f (x) = [f (x)] =
dx dx x2 + 1
d d
dx
[2x](x2 + 1) − 2x dx [x2 + 1]
= Quotient Rule
(x2 + 1)2
2(x2 + 1) − 2x(2x) 2x2 + 2 − 4x2 2 − 2x2
= = = 2
(x2 + 1)2 (x2 + 1)2 (x + 1)2

(e) Solution. The first derivative of f (x) = x2 e−3x is


d 2 −3x d
f 0 (x) = [x ]e + x2 [e−3x ] Product Rule
dx dx
 
−3x 2 −3x d d  g(x)  d
= 2xe +x e [−3x] e = eg(x) [g(x)]
dx dx dx
= 2xe−3x + x2 e−3x (−3)


= 2xe−3x − 3x2 e−3x = (2x − 3x2 )e−3x Factot out e−3x

The second derivative is


d 
f 00 (x) = (2x − 3x2 )e−3x

dx
280 Chapter 5. Applications of the Derivative I

d d
= [2x − 3x2 ]e−3x + (2x − 3x2 ) [e−3x ] Product Rule
dx dx
−3x 2 −3x
= (2 − 6x)e + (2x − 3x )e (−3)
= (2 − 6x)e−3x + (−6x + 9x2 )e−3x
= (2 − 6x − 6x + 9x2 )e−3x = (2 − 12x + 9x2 )e−3x

(f ) Solution. Recalling that log10 X is abbreviated log X, the first derivative of


f (x) = log(x + e−x ) is

0 d  −x
 d
dx
[x + e−x ] d d
dx
[g(x)]
f (x) = log(x + e ) = [loga g(x)] =
dx (ln 10)(x + e−x ) dx (ln a)g(x)
d
[x]
+ dxd
[e−x ] 1 − e−x d −x
= dx
= [e ] = −e−x
(ln 10)(x + e−x ) (ln 10)(x + e−x ) dx

The second derivative is


1 − e−x
 
00 d
f (x) =
dx (ln 10)(x + e−x )
d 1 − e−x
 
1 Constant
=
(ln 10) dx x + e−x Multiple Rule
1 dx d
[1 − e−x ](x + e−x ) − (1 − e−x ) dx
d
[x + e−x ]
= Quotient Rule
(ln 10) (x + e−x )2
1 (0 − (−e−x ))(x + e−x ) − (1 − e−x )(1 − e−x )
=
(ln 10) (x + e−x )2
e−x (x + e−x ) − (1 − e−x )(1 − e−x )
=
(ln10)(x + e−x )2

3. [Exercise on page 43]

(a) Solution.

Higher Derivatives
Let f be a function.

ˆ The third derivative of f , denoted f 000 (x), is the derivative of the


second derivative. That is,
d 00
f 000 (x) = [f 00 (x)]0 = [f (x)] .
dx

ˆ The fourth derivative of f , denoted f (4) (x), is the derivative of the


third derivative. That is,
d 000
f (4) (x) = [f 000 (x)]0 = [f (x)] .
dx
5.3. Concavity 281

ˆ For n ≥ 4, the nth derivative of f , denoted f (n) (x), is the derivative


of the (n − 1)th derivative. That is,

d  (n−1) 
f (n) (x) = f (x) .
dx

For f (x) = −x4 + 2x3 − 5x + 6,


ˆ The first derivative is
d
f 0 (x) = [−x4 + 2x3 − 5x + 6] = −4x3 + 6x2 − 5
dx
ˆ The second derivative is
d 0 d
f 00 (x) = [f (x)] = [−4x3 + 6x2 − 5] = −12x2 + 12x
dx dx
ˆ The third derivative is
d 00 d
f 000 (x) = [f (x)] = [−12x2 + 12x] = −24x + 12
dx dx
ˆ The fourth derivative is
d 000 d
f (4) (x) = [f (x)] = [−24x + 12] = −24
dx dx
(b) Solution.
d 2x d
f 0 (x) = [e ] = 2e2x , f 00 (x) = [2e2x ] = 2(2e2x ) = 4e2x ,
dx dx
d
f 000 (x) = [4e2x ] = 8e2x , f (4) (x) = 16e2x
dx
4. [Exercise on page 43]

(a) Solution.

Concavity Test
Let f be a function.

ˆ If f 00 (x) > 0 for all x in an interval I, then f is concave upward on


I.

ˆ If f 00 (x) < 0 for all x in an interval I, then f is concave downward


on I.

We will write C.D for concave downward and C.U for concave upward.

For f (x) = x2 − 6x + 3, the first derivative is f 0 (x) = 2x − 6. And the second


derivative is f 00 (x) = 2. Since f 00 (x) > 0 for all x, the function f is concave
upward on the interval (−∞, ∞). Since the concavity does not change,
there is no inflection point.
282 Chapter 5. Applications of the Derivative I

(b) Solution.
ˆ The first derivative of f (x) = x3 − 3x2 + 5 is f 0 (x) = 3x2 − 6x, and the
second derivative is
f 00 (x) = 6x − 6.
ˆ Set f 00 (x) = 0. Then 6x − 6 = 0. Adding 6 to both sides, we get 6x = 6.
Dividing both sides by 6, we get x = 1.
ˆ The sign of f 00 (x) is shown in Figure 5.24.

concave down concave up


− +
−∞ 1 ∞
choose 0 choose 2
f 00 (0) = −6 < 0 f 00 (2) = 6 > 0

Figure 5.24: Sign of f 00 (x) = 6x − 6

ˆ From Figure 5.24, f is concave downward on the interval (−∞, 1),


and concave upward on the interval (1, ∞).
ˆ Since the concavity changes at x = 1, there is an inflection point there.
Since f (1) = 13 − 3(1)2 + 5 = 1 − 3 + 5 = 3, the inflection point is (1, 3).
(c) Solution.
ˆ The first derivative is f 0 (x) = 4x3 − 12x2 , and the second derivative is

f 00 (x) = 12x2 − 24x = 12x(x − 2)


ˆ Set f 00 (x) = 0. Then 12x(x − 2) = 0. This implies that x = 0 or x = 2.
ˆ The sign of f 00 (x) is shown in Figure 5.25.

concave up concave down concave up


+ − +
−∞ 0 2 ∞
choose −1 choose 1 choose 3
f 00 (−1) = 36 > 0 f 00 (1) = −12 < 0 f 00 (3) = 36 > 0

Figure 5.25: Sign of f 00 (x) = 12x2 − 24x

ˆ From Figure 5.25, f is concave upward on the intervals (−∞, 0) and


(2, ∞), and concave downward on (0, 2).
ˆ Since the concavity changes at x = 0 and x = 2, there are two inflection
points. Since
f (0) = 04 − 4(0)3 = 0 and f (2) = 24 − 4(2)3 = 16 − 32 = −16,
the inflection points are (0, 0) and (2, −16).
5.3. Concavity 283

(d) Solution.
ˆ The first derivative of f (x) = −3
x+1
is
d d
[−3](x + 1) − (−3) dx [x + 1]
f 0 (x) = dx
Quotient Rule
(x + 1)2
0(x + 1) + 3(1) 3
= 2
=
(x + 1) (x + 1)2
The second derivative is
d d
d 0 [3](x + 1)2 − 3 dx [(x + 1)2 ]
f 00 (x) = [f (x)] = dx
Quotient Rule
dx (x + 1)4
0(x + 1)2 − 3(2(x + 1)(1))
=
(x + 1)4
−6(x + 1) −6
= 4
=
(x + 1) (x + 1)3
ˆ Set f 00 (x) = 0.Then (x+1)
−6 3
3 = 0. Multiplying both sides by (x + 1) , we get

−6 = 0, which is not true. So the equation f 00 (x) = 0 has no solution.


ˆ For the sign of f 00 (x) (see Figure 5.26), we need to plot the point −1 on the
real line because the original function is not defined when x = −1.
concave up concave down
+ −
−∞ −1 ∞
choose −2 choose 0
f 00 (−2) = 6 > 0 f 00 (0) = −6 < 0
−6
Figure 5.26: Sign of f 0 (x) = (x+1)3

ˆ From Figure 5.26, f is concave upward on the interval (−∞, −1) and
concave downward on the interval (−1, ∞).
ˆ Although the concavity changes at −1, there is no inflection point because
f (x) is not defined at −1.
(e) Solution.
ˆ Using the quotient rule, the first derivative of f (x) = x
x2 +1
is
d d
0 dx
[x](x2 + 1) − x dx [x2 + 1] (1)(x2 + 1) − x(2x)
f (x) = =
(x2 + 1)2 (x2 + 1)2
x2 + 1 − 2x2 −x2 + 1
= =
(x2 + 1)2 (x2 + 1)2
The second derivative is
d −x2 + 1
 
00 d 0
f (x) = [f (x)] =
dx dx (x2 + 1)2
284 Chapter 5. Applications of the Derivative I

−2x(x2 + 1)2 − (−x2 + 1)2(x2 + 1)(2x)


= Quotient Rule
(x2 + 1)4
(x2 + 1)[−2x(x2 + 1) − 4x(−x2 + 1)]
=
(x2 + 1)4
(x2 + 1)(−2x3 − 2x + 4x3 − 4x)
=
(x2 + 1)4
(x2 + 1)(2x3 − 6x) 2x(x2 − 3)
= = Simplify and Factor
(x2 + 1)4 (x2 + 1)3
2
−3)
ˆ Set f 00 (x) = 0. Then 2x(x
(x2 +1)3
= 0. Multiplying both sides by (x2 + 1)3 , we
2 2
get 2x(x√ − 3) = 0. This implies that x = 0 or x = 3. That is, x = 0 or
x = ± 3.
ˆ The sign of f 00 (x) is shown in Figure 5.27.

concave down concave up concave down concave up


− + − +
−∞ √ √ ∞
− 3 0 3
choose −2 choose −1 choose 1 choose 2
f 00 (−2) = − 125
4
< 0 f 00 (−1) = 1
2
> 0 f 00 (1) = − 12 < 0 f 00 (2) = 4
125
>0

2x(x2 −3)
Figure 5.27: Sign of f 00 (x) = (x2 +1)3


ˆ From Figure
√ 5.27, f is concave downward on the intervals (−∞,
√ − 3)
√ (0, 3), and concave upward on the intervals (− 3, 0) and
and
( 3, ∞).

ˆ Again
√ from Figure 5.27, there are three inflection points: at − 3, 0, and
3. Since,
√ √ √ √
√ 3 3 0 √ 3 3
f (− 3) = =− , f (0) = 2 = 0, f ( 3) = = ,
3+1 4 0 +1 3+1 4
 √ √  √ √ 
the inflection points are − 3, − 43 , (0, 0), and 3, 43 .
(f ) Solution.
ˆ The first derivative of f (x) = xe−4x is
d d d
f 0 (x) = [xe−4x ] = [x]e−4x + x [e−4x ] Product Rule
dx dx dx
d  g(x)  d
= e−4x + x e−4x (−4) = eg(x) [g(x)]

e
dx dx
= (1 − 4x)e−4x Factor out e −4x

ˆ The second derivative is


d 0 d 
f 00 (x) = (1 − 4x)e−4x

[f (x)] =
dx dx
5.3. Concavity 285

d d
= [1 − 4x]e−4x + (1 − 4x) [e−4x ] Product Rule
dx dx
= (−4)e−4x + (1 − 4x)(−4e−4x ) = −4e−4x + (−4 + 16x)e−4x
= (−4 − 4 + 16x)e−4x = (−8 + 16x)e−4x Factor out e−4x

ˆ Set (−8 + 16x)e−4x = 0. Then −8 + 16x = 0. (Remember that ex 6= 0 for all


x.) Adding 8 to both sides of the latter equation, we get 16x = 8. Dividing
both sides by 16, we have x = 12 .
ˆ The sign of f 00 (x) is shown in Figure 5.28.

concave down concave up


− +
−∞ 1 ∞
2
choose 0 choose 1
f 00 (0) = −8 < 0 f 00 (1) = 8e−4 > 0

Figure 5.28: Sign of f 00 (x) = (−8 + 16x)e−4x

ˆ From Figure 5.28, f is concave downward on the interval (−∞, 12 ) and


concave upward on the interval ( 12 , ∞).
ˆ Since the concavity changes at x = 21 , there is one inflection point there.
Since  
1 1 1 1
f = e−4( 2 ) = e−2 ,
2 2 2
the inflection point is 21 , 12 e−2 .


(g) Solution.
ˆ The first derivative of f (x) = ln(x2 + 4) is
d d
0 dx
[x2 + 4] 2x d dx
[g(x)]
f (x) = = 2 [ln g(x)] =
x2 + 4 x +4 dx g(x)
The second derivative is
 
00 d 0 d 2x
f (x) = [f (x)] =
dx dx x2 + 4
d d
dx
[2x](x2 + 4) − 2x dx [x2 + 4]
= Quotient Rule
(x2 + 4)2
2(x2 + 4) − 2x(2x) 2x2 + 8 − 4x2
= =
(x2 + 4)2 (x2 + 4)2
−2x2 + 8 −2(x2 − 4) −2(x − 2)(x + 2)
= = =
(x2 + 4)2 (x2 + 4)2 (x2 + 4)2

ˆ Set f 00 (x) = 0. Then −2(x−2)(x+2)


(x2 +4)2
= 0. Multiplying both sides by (x2 + 4)2 ,
we get −2(x − 2)(x + 2) = 0. This implies that x = 2 or x = −2.
286 Chapter 5. Applications of the Derivative I

concave down concave up concave down


− + −
−∞ ∞
−2 2
choose −3 choose 0 choose 3
f 00 (−3) = − 169
10
<0 f 00 (0) = 1
2
>0 f 00 (3) = − 169
10
<0

−2(x−2)(x+2)
Figure 5.29: Sign of f 00 (x) = (x2 +4)2

ˆ The sign of f 00 (x) is shown in Figure 5.29.


ˆ From Figure 5.29, the function f is concave downward on the intervals
(−∞, −2) and (2, ∞), and concave upward on the interval (−2, 2).
ˆ There are two inflection points: at −2 and 2. Since

f (−2) = ln((−2)2 + 4) = ln(8) and f (2) = ln(22 + 4) = ln 8,

the inflection points are (−2, ln 8) and (2, ln 8).


(h) Solution.
ˆ The first derivative of f (x) = (x − 1)4 is

d d d
f 0 (x) = 4(x − 1)3 [x − 1] [(g(x))n ] = n(g(x))n−1 [g(x)]
dx dx dx
= 4(x − 1)3 (1) = 4(x − 1)3

ˆ The second derivative is


d  d
f 00 (x) = 4(x − 1)3 = 4 [(x − 1)3 ]

Constant Multiple Rule
dx dx
= 4 3(x − 1) = 12(x − 1)2
2


ˆ Set f 00 (x) = 0. Then 12(x − 1)2 = 0. Dividing both sides by 12, we get
(x − 1)2 = 0. This implies that x = 1.
ˆ The sign of f 00 (x) is shown in Figure 5.30.

concave up concave up
+ +
−∞ 1 ∞
choose 0 choose 2
f 00 (0) = 12 > 0 f 00 (2) = 12 > 0

Figure 5.30: Sign of f 00 (x) = 12(x − 1)2

ˆ From Figure 5.30, f is concave upward on the intervals (−∞, 1) and


(1, ∞).
5.3. Concavity 287

ˆ Since the concavity does not change, there is no inflection point.

5. [Exercise on page 44]

(a) Solution.

The Second Derivative Test


Let f be a function, and let c be a critical number for f .

ˆ If f 00 (c) > 0, then f (c) is a local (or relative) minimum.

ˆ If f 00 (c) < 0, then f (c) ia a local (or relative) relative maximum.

The test is inconclusive if f 00 (c) = 0 or f 00 (c) does not exist. In that case,
we use the first derivative test.

ˆ Finding the critical numbers. The derivative of f (x) = 1 − 8x − x2 is

f 0 (x) = −8 − 2x

Setting f 0 (x) = 0, we get −2x − 8 = 0. Adding 8 to both sides, we get


−2x = 8. Dividing both sides by −2, we get x = −4. So the only critical
number for f is −4.
ˆ Sign of the second derivative. The second derivative is f 00 (x) = −2. Since
f 00 (−4) = −2 < 0, it follows (by the Second Derivative Test) that f (−4) is
a relative maximum. But

f (−4) = 1 − 8(−4) − (−4)2 = 1 + 32 − 16 = 17

So 17 is a relative maximum of f .
(b) Solution.
ˆ The first derivative is

f 0 (x) = −3x2 + 12x = −3x(x − 4)

Setting f 0 (x) = 0, we get −3x = 0 or x − 4 = 0. This implies that x = 0 or


x = 4. So the critical numbers are 0 and 4.
ˆ The second derivative is
d
f 00 (x) = [−3x2 + 12x] = −6x + 12
dx
Evaluating this at the critical numbers, we get

f 00 (0) = −6(0) + 12 = 12 and f 00 (4) = −6(4) + 12 = −24 + 12 = −12.

– Since f 00 (0) > 0, f (0) = −03 + 6(02 ) + 7 = 7 is a relative minimum.


– Since f 00 (4) < 0, f (4) = −43 + 6(4)2 + 7 = −64 + 96 + 7 = 39 is a
relative maximum.
288 Chapter 5. Applications of the Derivative I

(c) Solution.
ˆ The first derivative is
f 0 (x) = 4(x + 1)3
Setting 4(x + 1)3 = 0, we get x = −1. So the critical number of f is −1.
ˆ The second derivative is
f 00 (x) = 12(x + 1)2
Evaluating this at the critical number, we have f 00 (−1) = 12(−1 + 1)2 = 0.
Since f 00 (−1) = 0, the second derivative test gives no information about
relative extrema.
ˆ So we will use the first derivative test. The sign of the first derivative is
shown in Figure 5.31.

− +
−∞ −1 ∞
choose −2 choose 0
f 0 (−2) = −4 < 0 f 0 (0) = 4 > 0

Figure 5.31: Sign of f 0 (x) = 4(x + 1)3

ˆ From Figure 5.31, the function is decreasing on the interval (−∞, −1) and
increasing on (−1, ∞). So f (−1) = (−1 + 1)4 = 0 is a relative minimum.
6. [Exercise on page 44]
(a)
Point of Diminishing Returns

Let y = f (x) be a function (for example the revenue function). The point
of diminishing returns is nothing but the inflection point of f .

We want to find the inflection point of R(x) = −x3 + 12x2 + 100, where 0 ≤ x ≤
10.
ˆ The first derivative is R0 (x) = −3x2 + 24x.
ˆ The second derivative is R00 (x) = −6x + 24.
ˆ Set R00 (x) = 0. Then −6x + 24 = 0. Subtracting 24 from both sides, we get
−6x = −24. Dividing both sides by −6, we get x = 4.
ˆ The sign of R00 (x) is shown in Figure 5.32.
ˆ Since the concavity changes at 4, there is an inflection point there. Evalu-
ating R(x) at 4, we get
R(4) = −43 + 12(4)2 + 100 = −64 + 192 + 100 = 228.
So the point of diminishing returns is (4, 228).
5.4. Curve Sketching 289

concave up concave down


+ −
0 4 10
choose 0 choose 5
R00 (0) = 24 > 0 R00 (5) = −6 < 0

Figure 5.32: Sign of R00 (x) = −6x + 24

(b) Solution.
ˆ The first derivative of R(x) = xe−x is
d d
R0 (x) = [x]e−x + x [e−x ] Product Rule
dx dx
d −x
= e−x + x(−e−x ) [e ] = −e−x
dx
= e−x − xe−x = (1 − x)e−x Factor out e−x
ˆ Using the same rules, the second derivative is
d d
R00 (x) = [1 − x]e−x + (1 − x) [e−x ]
dx dx
−x −x
= −e + (1 − x)(−e )
= (−1 − 1 + x)e−x = (−2 + x)e−x
ˆ Set R00 (x) = 0. Then (−2 + x)e−x = 0. This implies that −2 + x = 0.
(Remember that ex 6= 0 for all x.) Solving the latter equation, we get
x = 2.
ˆ The sign of R00 (x) is given by Figure 5.33.

concave down concave up


− +
0 2 15
choose 1 choose 3
R00 (1) = −e−1 < 0 R00 (3) = e−3 > 0

Figure 5.33: Sign of R00 (x) = (−2 + x)e−x

ˆ Since the concavity changes at x = 2, there is an inflection point there.


Evaluating R(x) at 2, we get R(2) = 2e−2 .
ˆ Thus, the point of diminishing returns is (2, 2e−2 ).

5.4 Curve Sketching


1. Solution. [Exercise on page 44]
290 Chapter 5. Applications of the Derivative I

To sketch the graph of f (x) = −x3 + 3x2 − 1, we will go through the following steps.

(a) Domain. Since f (x) = −x3 + 3x2 − 1 is a polynomial, it is defined everywhere.


So the domain of f is R = (−∞, ∞).
(b) Intercepts.

Intercepts

Let y = f (x) be a function.

ˆ The x-intercepts of f are the intersection (if any) between the graph
of f and the x-axis. They are obtained by solving the equation
f (x) = 0 for x.

ˆ The y-intercept of f is the intersection (if any) between the graph of


f and the y-axis. This is obtained by evaluating f at 0.

ˆ The y-intercept is f (0) = −03 + 3(0)2 − 1 = −1.


ˆ For the x-intercepts, we need to solve the equation f (x) = 0, that is, −x3 +
3x2 − 1 = 0. This equation looks difficult to solve, so we will omit this step.
(c) Asymptotes.
ˆ Horizontal Asymptotes. Remember that the line y = L, where L is a
real number, is a horizontal asymptote if lim f (x) = L or lim f (x) = L.
x→∞ x→−∞
So to find the horizontal asymptotes, we need to find the limits at infinity.
Remembering how to find the limit of a polynomial at infinity, we have

lim f (x) = lim (−x3 + 3x2 − 1) = lim (−x3 ) = −∞


x→∞ x→∞ x→∞

and
lim f (x) = lim (−x3 + 3x2 − 1) = lim (−x3 ) = ∞
x→−∞ x→−∞ x→−∞

Since lim f (x) is not a number and lim f (x) is not a number either, there
x→∞ x→−∞
is no horizontal asymptote.
ˆ Vertical Asymptotes. Polynomial functions do not have vertical asymp-
totes. For rational functions, we can locate the vertical asymptotes by
equating the denominator to 0 after canceling any common factors. Since
f (x) = −x3 + 3x2 − 1 is a polynomial, there is no vertical asymptote.
(d) Intervals where f is Increasing or Decreasing and Relative Extrema
ˆ The derivative is f 0 (x) = −3x2 + 6x = −3x(x − 2).
ˆ Set −3x(x − 2) = 0. Then x = 0 or x = 2. So the critical numbers are 0
and 2. Evaluating f at the critical numbers, we get

f (0) = −03 +3(0)2 −1 = −1 and f (2) = −(2)3 +3(2)2 −1 = −8+12−1 = 3.

So the critical points are (0, −1) and (2, 3).


ˆ The sign of f 0 (x) is shown in Figure 5.34.
5.4. Curve Sketching 291

− + −
−∞ 0 2 ∞
choose −1 choose 1 choose 3
f 0 (−1) = −9 < 0 f 0 (1) = 3 > 0 f 0 (3) = −9 < 0

Figure 5.34: Sign of f 0 (x) = −3x(x − 2)

ˆ From Figure 5.34, f is decreasing on the intervals (−∞, 0) and (2, ∞), and
increasing on the interval (0, 2).
ˆ Relative Extrema. Again from Figure 5.34, the relative minimum is
f (0) = −1, and the relative maximum is f (2) = 3.
(e) Concavity and Inflection Points.
ˆ The second derivative is

d 0 d
f 00 (x) = [f (x)] = [−3x2 + 6x] = −6x + 6 = −6(x − 1)
dx dx

ˆ Set f 00 (x) = 0. Then −6(x − 1) = 0. Solving this, we get x = 1.


ˆ The sign of f 00 (x) is shown in Figure 5.35.

concave up concave down


+ −
−∞ 1 ∞
choose −1 choose 2
f 00 (−1) = 12 > 0 f 00 (2) = −6 < 0

Figure 5.35: Sign of f 00 (x) = −6x + 6

ˆ From Figure 5.35, f is concave upward on the interval (−∞, 1) and concave
downward on the interval (1, ∞).
ˆ Since the concavity changes at 1, there is an inflection point there. Evalu-
ating f at 1, we get

f (1) = −(1)3 + 3(1)2 − 1 = −1 + 3 − 1 = 1.

So the inflection point is (1, 1).


(f ) Summary of All Information in a Single Table. The information above
can be summarized in the following table, where C.U stands for concave upward
and C.D stands for concave downward.
292 Chapter 5. Applications of the Derivative I

Interval (−∞, 0) (0, 1) (1, 2) (2, ∞)


Sign of f 0 − + + −
Increasing/Decreasing Decreasing Increasing Increasing Decreasing
Sign of f 00 + + − −
Concavity C.U C.U C.D C.D

(g) Sketching the Curve. To sketch the curve, one can go through the following
steps.
ˆ Plot the intercepts. Here the y-intercept is (0, −1).
ˆ Plot the critical points: (0, −1) and (2, 3).
ˆ Plot the inflection points: (1, 1).
ˆ Graph the asymptotes. Here there is no asymptote.
ˆ Connect the points with a curve by taking the concavity into account.
The graph of f is shown in Figure 5.36.

y
4

−3 −2 −1 1 2 3 4 x
−1

−2

Figure 5.36: Graph of f (x) = −x3 + 3x2 − 1

2. Solution. [Exercise on page 44]

(a) Domain. The function f (x) = x4 − 4x3 + 21 is defined everywhere as it is a


polynomial. So, the domain of f is R = (−∞, ∞).
(b) Intercepts.
ˆ The y-intercept is f (0) = 04 − 4(0)3 + 21 = 21.
ˆ To find the x-intercepts, we need to solve the equation x4 − 4x3 + 21 = 0
for x. Since this is difficult to solve, we will omit this step.
(c) Asymptotes. There is no asymptote since f is a polynomial.
5.4. Curve Sketching 293

(d) Intervals where f is Increasing or Decreasing and Relative Extrema.


The derivative of f (x) = x4 − 4x3 + 21 is
f 0 (x) = 4x3 − 12x2
Set f 0 (x) = 0. Then
4x3 − 12x2 = 0
4x2 (x − 3) = 0 Factor out 4x2
4x2 = 0 or x − 3 = 0
x = 0 or x = 3
So, the critical numbers of f are 0 and 3. Evaluating f at these numbers, we
get
f (0) = 21 and f (3) = 34 − 4(3)3 + 21 = 81 − 108 + 21 = −6
So, the critical points of f are (0, 21) and (3, −6).
The sign of f 0 (x) is shown in the following figure.

− − +
−∞ 0 3 ∞
choose −1 choose 1 choose 4
f 0 (−1) = −16 < 0 f 0 (1) = −8 < 0 f 0 (4) = 64 > 0

This shows that f is decreasing on the intervals (−∞, 0) and (0, 3), and increas-
ing on the interval (3, ∞). The relative minimum of f is f (3) = −6. There is
no relative maximum.
(e) Concavity and Inflection Points.
The second derivative of f (x) is
d 0 d
f 00 (x) = [f (x)] = [4x3 − 12x2 ] = 12x2 − 24x
dx dx
Set f 00 (x) = 0. Then
12x2 − 24x = 0
12x(x − 2) = 0 Factor out 12x
x = 0 or x = 2
The sign of f 00 (x) is shown in the following figure.
concave up concave down concave up
+ − +
−∞ 0 2 ∞
choose −1 choose 1 choose 3
f 00 (−1) = 36 > 0 f 00 (1) = −12 < 0 f 00 (3) = 36 > 0
294 Chapter 5. Applications of the Derivative I

This shows that f is concave upward on the intervals (−∞, 0) and (2, ∞), and
concave downward on the interval (0, 2). Since the concavity changes at 0 and
2, there are inflection points there. Since
f (0) = 21 and f (2) = 24 − 4(2)3 + 21 = 16 − 32 + 21 = 5,
it follows that (0, 21) and (2, 5) are the inflection points of f .
(f ) Summary of All the Information Above in a Table. In the following
table, C.U stands for concave upward, and C.D for concave downward.

Interval (−∞, 0) (0, 2) (2, 3) (3, ∞)


Sign of f 0 − − − +
Increasing/Decreasing Decreasing Decreasing Decreasing Increasing
Sign of f 00 + − + +
Concavity C.U C.D C.U C.U

(g) Graph of f . The graph of f is shown in Figure 5.37.

30

20

10

−2 −1 1 2 3 4 5 x

−10

Figure 5.37: Graph of f (x) = x4 − 4x3 + 21

3. Solution. [Exercise on page 44]


(a) Domain. Setting the denominator equal to zero, we get x2 − 1 = 0. Factoring
this, we get (x − 1)(x + 1) = 0. This implies that x = 1 or x = −1. So f (x) is
defined for all x except at −1 and 1.
(b) Intercepts.
−2(0)2
ˆ y-intercept. The y-intercept is f (0) = 02 −1
0
= −1 = 0.
−2x2
ˆ x-intercepts. Set f (x) = 0. Then x2 −1
= 0. Multiplying
both sides by
x2 − 1, we get −2x2 = 0. Dividing both sides by −2, we get x2 = 0. Taking
the square root of both sides, we get x = 0. So the x-intercept is 0.
5.4. Curve Sketching 295

(c) Asymptotes.
ˆ Horizontal Asymptotes. We need to find the limits at infinity. Remem-
bering how to find the limit of a rational function at infinity, we have

−2x2
lim f (x) = lim
x→∞ x→∞ x2 − 1
−2x2
x2
= lim 2 Divide each term by x2
x→∞ x − 1
x2 x2
−2
= lim Simplify
x→∞ 1 − x12
lim (−2)
x→∞
= 1

lim (1) − lim x2
x→∞ x→∞
 
−2 1
= lim (k) = k, lim = 0 for all n > 0
1−0 x→∞ x→∞ xn
−2
= = −2
1

Similarly, lim f (x) = −2. Thus, the line y = −2 is a horizontal asymptote.


x→−∞
ˆ Vertical Asymptotes. By using the definition of vertical asymptote, one
can show that the lines x = −1 and x = 1 are vertical asymptotes of the
curve y = f (x). (A quick way to see this is to use the fact that −1 and 1
make the denominator zero and the numerator nonzero.)
(d) Intervals where f is Increasing or Decreasing and Relative Extrema
ˆ The derivative is

d −2x2
 
0
f (x) =
dx x2 − 1
d d
dx
[−2x2 ](x2 − 1) − (−2x2 ) dx [x2 − 1]
= Quotient Rule
(x2 − 1)2
−4x(x2 − 1) + 2x2 (2x)
=
(x2 − 1)2
−4x3 + 4x + 4x3 4x
= 2 2
= 2
(x − 1) (x − 1)2

ˆ Set f 0 (x) = 0. Then (x24x


−1)2
= 0. Multiplying both sides by (x2 − 1)2 , we
get 4x = 0. Dividing both sides by 4, we get x = 0. So the critical number
is 0. Evaluating f at 0, we get f (0) = 0. Thus, (0, 0) is a critical point.
ˆ The sign of f 0 (x) is shown in the following figure.
296 Chapter 5. Applications of the Derivative I

− − + +
−∞ −1 0 1 ∞
choose −2 choose −0.5 choose 0.5 choose 2
f 0 (−2) = − 89 < 0 f 0 (−0.5) < 0 f 0 (0.5) > 0 f 0 (2) = 8
9
>0

ˆ From this figure, f is decreasing on the intervals (−∞, −1) and (−1, 0), and
increasing on the intervals (0, 1) and (1, ∞).
ˆ Relative Extrema. The relative minimum is f (0) = 0. There is no relative
maximum.
(e) Concavity and Inflection Points.
ˆ The second derivative is
 
00 d 0 d 4x
f (x) = [f (x)] =
dx dx (x2 − 1)2
d d
dx
[4x](x2 − 1)2 − 4x dx [(x2 − 1)2 ]
= Quotient Rule
(x2 − 1)4
4(x2 − 1)2 − 4x(2)(x2 − 1)(2x)
=
(x2 − 1)4
4(x2 − 1)2 − 16x2 (x2 − 1)
=
(x2 − 1)4
(x2 − 1)[4(x2 − 1) − 16x2 ]
= Factor out x2 − 1
(x2 − 1)4
(x2 − 1)(4x2 − 4 − 16x2 )
=
(x2 − 1)4
−12x2 − 4 −4(3x2 + 1) Simplify and factor out
= =
(x2 − 1)3 (x2 − 1)3 −4
2
ˆ Set f 00 (x) = 0. Then −4(3x +1)
(x2 −1)3
= 0. Multiplying both sides by (x2 − 1)3 , we
get −4(3x2 + 1) = 0. Dividing both sides by −4, we get 3x2 + 1 = 0. This
latter equation has no solution because 3x2 + 1 is always a positive number.
ˆ The sign of f 00 (x) is shown in the following figure.

concave down concave up concave down


− + −
−∞ −1 1 ∞
choose −2 choose 0 choose 2
f 00 (−2) = − 52
27
<0 f 00 (0) = 4 > 0 f 00 (2) = − 52
27
<0

ˆ From this figure, f is concave downward on the intervals (−∞, −1) and
(1, ∞), and concave upward on the interval (−1, 1).
5.4. Curve Sketching 297

ˆ Although the concavity changes at −1 and 1, these are not inflection points
because f (x) in undefined at −1 and 1.
(f ) Summary. The information above can be summarized in the following table,
where C.U stands for concave upward and C.D stands for concave downward.

Interval (−∞, −1) (−1, 0) (0, 1) (1, ∞)


Sign of f 0 − − + +
Increasing/Decreasing Decreasing Decreasing Increasing Increasing
Sign of f 00 − + + −
Concavity C.D C.U C.U C.D

(g) Graph of f . The graph of f is shown in Figure 5.38.

y
x = −1 x=1

−1 1 x

−2 y = −2

−2x2
Figure 5.38: Graph of f (x) = x2 −1

4. Solution. [Exercise on page 44]

6
(a) Domain. The function f (x) = x2 −x−6 is defined everywhere except where the
denominator is zero. Solving the equation x2 −x−6 = 0, we get (x+2)(x−3) = 0;
that is, x = −2 or x = 3. So, the domain of f is the set of all real numbers
except −2 and 3.
(b) Intercepts.
ˆ The y-intercept is f (0) = 6
02 −0−6
= −1.
298 Chapter 5. Applications of the Derivative I

ˆ The x-intercepts are obtained by solving the equation f (x) = 0 for x:


6
=0
x2 −x−6
6=0 Multiply both sides by x2 − x − 6

Since this latter equality is not true, it follows that the equation f (x) = 0
has no solution. Therefore, f has no x-intercept.
(c) Asymptotes.
ˆ Finding the horizontal asymptotes. To find the horizontal asymptotes, we
need to find two limits: lim f (x) and lim f (x).
x→∞ x→−∞

6
lim f (x) = lim
x→∞ x→∞ x2 −x−6
6
x2
= lim 2 Divide each term by x2
x→∞ x − x
− 6
x2 x2 x2
6
x2
= lim 1 Simplify
x→∞ 1− x
− x62
6

lim x2
x→∞
= 1
 6

lim (1) − lim x
− lim x
x→∞ x→∞ x→∞

0 1
= lim = 0 for all n > 0
1−0−0 x→∞ xn

0
= =0
1
Similarly, we have lim f (x) = 0. So, the line y = 0 is a horizontal asymp-
x→−∞
tote of f at both −∞ and ∞.
ˆ Finding the vertical asymptotes. The denominator of f (x), x2 − x − 6, is
zero when x = −2 or x = 3. To find the vertical asymptotes, we need to
find the limits lim f (x) and lim f (x). For the first limit, if we substitute
x→−2 x→3
x with −2 into f (x), we have 06 , which is a limit of the form k0 with k 6= 0.
Using the method from Section 3.3, we need to determine the sign of f (x).
This is given by the following figure.
+ − +
−∞ −2 3 ∞
choose −3 choose 0 choose 4
f (−3) = 1 > 0 f (0) = −1 < 0 f (4) = 1 > 0
The sign analysis for f (x) tells us that the one-sided limits of f (x) as x → −2
are
lim − f (x) = ∞ and lim + f (x) = −∞
x→−2 x→−2
5.4. Curve Sketching 299

This shows that the line x = −2 is a vertical asymptote. Likewise, the line
x = 3 is a vertical asymptote of f (x).
(d) Increasing/Dcreasing and Relative Extrema.
6
The derivative of f (x) = x2 −x−6 is
d d
[6](x2 − x − 6) − 6 dx [x2 − x − 6]
f 0 (x) = dx
Quotient Rule
(x2 − x − 6)2
0(x2 − x − 6) − 6(2x − 1)
=
(x2 − x − 6)2
−6(2x − 1)
=
(x2 − x − 6)2
Set f 0 (x) = 0. Then
−6(2x − 1)
=0
(x2 − x − 6)2
−6(2x − 1) = 0 Multiply both sides by (x2 − x − 6)2
2x − 1 = 0 Divide both sides by −6
1
x= Solve for x
2
So, the critical number for f is 12 . Evaluating f at the critical number, we get
   
1 6 6 6 −4 24
f = 1 2 1 = 1 2 24 = 25 = 6 =−
2 (2) − 2 − 6 4
−4− 4 −4 25 25

So, the critical point of f is 12 , − 24



25
.
The sign of f 0 (x) is shown in the following figure.

+ + − −
−∞ −2 1 3 ∞
2
choose −3 choose 0 choose 1 choose 4
f 0 (−3) = 7
6
>0 f 0 (0) > 0 f 0 (1) < 0 f 0 (4) = − 76 < 0

This shows that f is increasing on the intervals (−∞, −2) and (−2, 21 ), and
decreasing on the intervals ( 12 , 3) and (3, ∞). The relative maximum of f (x) is
f ( 12 ) = − 25
24
. There is no relative minimum.
(e) Concavity and Inflection Points.
The second derivative is
 
00 d 0 d −12x + 6
f (x) = [f (x)] =
dx dx (x2 − x − 6)2
300 Chapter 5. Applications of the Derivative I

d
−12(x2 − x − 6)2 − (−12x + 6)2(x2 − x − 6) dx [x2 − x − 6]
= Quotient Rule
(x2 − x − 6)4
−12(x2 − x − 6)2 − (−12x + 6)2(x2 − x − 6)(2x − 1)
=
(x2 − x − 6)4
(x2 − x − 6) [−12(x2 − x − 6) − (−12x + 6)(4x − 2)]
= Factor out x2 − x − 6
(x2 − x − 6)4
(x2 − x − 6) (−12x2 + 12x + 72 + 48x2 − 48x + 12)
=
(x2 − x − 6)4
(x2 − x − 6)(36x2 − 36x + 84)
=
(x2 − x − 6)4
12(3x2 − 3x + 7)
= Simplify
(x2 − x − 6)3
Set f 00 (x) = 0. Then
12(3x2 − 3x + 7)
=0
(x2 − x − 6)3
12(3x2 − 3x + 7) = 0 Multiply both sides by (x2 − x − 6)3
3x2 − 3x + 7 = 0 Divide both sides by 12

−b± b2 −4ac
Using the quadratic formula we have
2a
,
p √
−(−3) ± (−3)2 − 4(3)(7) 3 ± −75
x= =
2(3) 6
Since the square root of −75 is undefined, the equation f 00 (x) = 0 has no solution.
This implies that f has no inflection point.
The sign of f 00 (x) is shown in the following figure.
concave up concave down concave up
+ − +
−∞ −2 3 ∞
choose −3 choose 0 choose 4
f 00 (−3) > 0 f 00 (0) < 0 f 00 (4) > 0

(f ) Summary.

Interval (−∞, −2) (−2, 21 ) ( 12 , 3) (3, ∞)


Sign of f 0 + + − −
Increasing/Decreasing Increasing Increasing Decreasing Decreasing
Sign of f 00 + − − +
Concavity C.U C.D C.D C.U
5.4. Curve Sketching 301

(g) Graph. The graph of f is shown in Figure 5.39.

−2 3 x

6
Figure 5.39: Graph of f (x) = x2 −x−6
302 Chapter 5. Applications of the Derivative I
Chapter6

Applications of the Derivative II

6.1 Absolute Extrema


1. [Exercise on page 45]
Solution.

y
Absolute
2 maximum

−1 3 x

−2
Absolute
minimum

Figure 6.1

Absolute Extrema–Definition
Let f be a function and I be an interval. Suppose f is defined on I, and let c
be a number in I.

ˆ f (c) is the absolute maximum of f on I if

f (x) ≤ f (c) for all x in I.

ˆ f (c) is the absolute minimum of f on I if

f (x) ≥ f (c) for all x in I.

303
304 Chapter 6. Applications of the Derivative II

ˆ f (c) is an absolute extremum if it is either an absolute maximum or an


absolute minimum. (The plural form of “extremum” is “extrema”.)

So the absolute maximum of f is the largest value of f . And the absolute


minimum is the smallest value of f .

For the function whose graph is shown in Figure 6.1, the absolute maximum is 2,
and the absolute minimum is −2.

2. [Exercise on page 45]

(a) Solution.

Extreme Value Theorem


The following theorem gives conditions (see the underlined terms below)
that guarantee the existence of the absolute extrema of a function.
Theorem 6.1.1. Let f be a function. Suppose that f is continuous on a
closed interval [a, b]. Then there exist c1 and c2 in [a, b] such that f (c1 ) is
the absolute maximum and f (c2 ) is the absolute minimum of f on [a, b].

Finding Absolute Extrema

Suppose f is continuous on [a, b]. To find the absolute extrema of f on


[a, b], one can proceed as follows.

Step 1. Find the critical numbers of f on the open interval (a, b).

Step 2. Evaluate f at the critical numbers found in Step 1.

Step 3. Evaluate f at the endpoints a and b.

Step 4. The absolute maximum of f on [a, b] is the largest of the values


found in Steps 2 and 3. The absolute minimum of f on [a, b]
is the smallest of the values from Steps 2 and 3.

We want to find the absolute extrema of f (x) = 12 + 4x − x2 on [0, 5].


ˆ The derivative is f 0 (x) = 4−2x. Set f 0 (x) = 0. Then 4−2x = 0. Subtracting
4 from both sides, we get −2x = −4. Dividing both sides by −2, we get
x = 2. Moreover, 2 is in the open interval (0, 5). So the critical number of
f in (0, 5) is 2.
ˆ Evaluating f at 2, we get

f (2) = 12 + 4(2) − 22 = 12 + 8 − 4 = 16.


6.1. Absolute Extrema 305

x 0 2 5
f (x) 12 16 7

Table 6.1

ˆ Evaluating f and the endpoints 0 and 5 of the interval [0, 5], we get

f (0) = 12 + 4(0) − 02 = 12 and f (5) = 12 + 4(5) − 52 = 7.

ˆ We summarize the values of f in Table 6.1. From Table 6.1, the absolute
maximum is 16 and it occurs when x = 2. And the absolute minimum
is 7, and it occurs when x = 5.
(b) Solution.
ˆ The derivative of f (x) = −x3 + 3x2 + 1 is

f 0 (x) = −3x2 + 6x = −3x(x − 2).

Solving the equation −3x(x − 2) = 0, we get x = 0 or x = 2, which are the


critical numbers.
ˆ Evaluating f at the critical numbers in (−1, 2), we get f (0) = −(0)3 +
3(0)2 + 1 = 1.
ˆ Evaluating f at the endpoints of the interval [−1, 2], we get

f (−1) = −(−1)3 + 3(−1)2 + 1 = 5 and f (2) = −(2)3 + 3(2)2 + 1 = 5.

ˆ We summarize the values in a single table.

x −1 0 2
f (x) 5 1 5

Table 6.2

From Table 6.2, the absolute maximum is 5, and it occurs when x = −1


or x = 2. The absolute minimum is 1, and it occurs when x = 0.
(c) Solution.
ˆ The derivative of f (x) = 2x3 − 3x2 − 12x + 1 is

f 0 (x) = 6x2 − 6x − 12 = 6(x2 − x − 2) = 6(x + 1)(x − 2).

Setting f 0 (x) = 0, we get x = −1 or x = 2. The critical number 2 is to be


rejected because it is not in the open interval (−2, 1).
ˆ Evaluating f at the critical numbers in (−2, 1), we get

f (−1) = 2(−1)3 − 3(−1)2 − 12(−1) + 1 = −2 − 3 + 12 + 1 = 8.


306 Chapter 6. Applications of the Derivative II

x −2 −1 1
f (x) −3 8 −12

Table 6.3

ˆ Evaluating f at the endpoints of the interval [−2, 1], we get

f (−2) = 2(−2)3 − 3(−2)2 − 12(−2) + 1 = −16 − 12 + 24 + 1 = −3


f (1) = 2(1)3 − 3(1) − 12(1) + 1 = 2 − 3 − 12 + 1 = −12

ˆ These values are summarized in Table 6.3.


So the absolute maximum is 8, and it occurs when x = −1. The
absolute minimum is −12, and it occurs when x = 1.
(d) Solution.
x2 −4
ˆ The derivative of f (x) = x2 +4
is

2x(x2 + 4) − (x2 − 4)(2x)


f 0 (x) = Quotient Rule
(x2 + 4)2
2x3 + 8x − 2x3 + 8x 16x
= 2 2
= 2
(x + 4) (x + 4)2

ˆ Set f 0 (x) = 0. Then (x216x


+4)2
= 0. Multiplying both sides by (x2 + 4)2 , we
get 16x = 0. Dividing both sides by 16, we get x = 0.
ˆ Evaluating f at the critical numbers in (−4, 4), we get f (0) = 0−4
0+4
= −1.
ˆ Evaluating f at the endpoints of the interval [−4, 4], we get

(−4)2 − 4 16 − 4 12 3
f (−4) = 2
= = = = f (4).
4 +4 16 + 4 20 5
ˆ We summarize these values in Table 6.4.

x −4 0 4
3 3
f (x) 5
−1 5

Table 6.4

From Table 6.4, the absolute maximum is 35 , and it is located at −4 and


4. The absolute minimum is −1, and it occurs when x = 0.
(e) Solution.
ˆ The derivative of f (x) = (x2 − 4)3 is
d 2 d d
f 0 (x) = 3(x2 − 4)2 [x − 4] [(g(x))n ] = n(g(x))n−1 [g(x)]
dx dx dx
2 2 2 2
= 3(x − 4) (2x) = 6x(x − 4)
6.1. Absolute Extrema 307

Set f 0 (x) = 0. Then 6x(x2 − 4)2 = 0. This implies that

6x = 0 or (x2 − 4)2 = 0
x=0 or x2 − 4 = 0
x=0 or (x + 2)(x − 2) = 0
x=0 or x = −2 or x = 2

ˆ Evaluating f at the critical numbers in the open interval (−2, 3), we get

f (0) = (02 − 4)3 = (−4)3 = −64 and f (2) = ((2)2 − 4)3 = (4 − 4)3 = 0.

ˆ Evaluating f at the endpoints of the interval [−2, 3], we get

f (−2) = ((−2)2 − 4)3 = (4 − 4)2 = 0 and f (3) = (32 − 4)3 = 53 = 125.

ˆ These values are summarized in Table 6.5.

x −2 0 2 3
f (x) 0 −64 0 125

Table 6.5

From Table 6.5, the absolute maximum is 125, and it occurs when x = 3.
The absolute minimum is −64, and it occurs when x = 0.
(f ) Solution.
ˆ The derivative of f (x) = x
x2 −x+1
is

(1)(x2 − x + 1) − x(2x − 1)
f 0 (x) = Quotient Rule
(x2 − x + 1)2
−x2 + 1 −(x2 − 1) −(x − 1)(x + 1)
= 2 2
= 2 2
=
(x − x + 1) (x − x + 1) (x2 − x + 1)2

Set f 0 (x) = 0. Then −(x−1)(x+1)


(x2 −x+1)2
= 0. Multiplying both sides by (x2 −x+1)2 ,
we get −(x − 1)(x + 1) = 0. This implies that x = 1 or x = −1. But the
critical number −1 is not in (0, 3), so we reject it.
ˆ Evaluating f at the critical numbers in (0, 3), we get
1 1 1
f (1) = = = =1
12 −1+1 1−1+1 1
ˆ Evaluating f at the endpoints of the interval [0, 3], we get
0 3 3
f (0) = = 0 and f (3) = 2 =
02 −0+1 3 −3+1 7
ˆ These values are summarized in Table 6.6.
From Table 6.6, the absolute maximum is 1, and it occurs when x = 1.
And the absolute minimum is 0, and it occurs when x = 0.
308 Chapter 6. Applications of the Derivative II

x 0 1 3
3
f (x) 0 1 7

Table 6.6

(g) Solution.
8 2
ˆ The derivative of f (x) = 3x 3 − 108x 3 is
   
0 8 5 2 −1 d n
f (x) = 3 x 3 − 108 x 3 [x ] = nxn−1
3 3 dx
5 1
= 8x 3 − 72x− 3
5 72 1
= 8x 3 − 1 x−n =
x 3 xn
2
8x − 72 b ac−b
= 1 a− c
= c
, am an = am+n
x 3

8(x2 − 9)
= 1 Factor out 8
x3
8(x − 3)(x + 3)
= 1 a2 − b2 = (a − b)(a + b)
x3

Alternate Way of Factoring the Derivative

Remembering that a − bc = c ac − b , we have




5
!
5 1 1 8x 3
f 0 (x) = 8x 3 − 72x− 3 = x− 3 1 − 72
x− 3
1 am
= x− 3 8x2 − 72 = am−n

an
8(x2 − 9)
= 1
x3
8(x − 3)(x + 3)
= 1
x3

1
Set f 0 (x) = 0. Then 8(x+3)(x−3)
1 = 0. Multiplying both sides by x 3 , we get
x3
8(x + 3)(x − 3) = 0. This implies that x = −3 or x = 3. But −3 does not lie
in (−2, 8), so we ignore it. So 3 is a critical number in (−2, 8). Also, since
the derivative is undefined at x = 0 and f (0) is defined, it follows that 0 is
another critical number.
ˆ Evaluating f at the critical numbers, we get f (0) = 0, and
8 2
f (3) = 3(3) 3 − 108(3) 3 ≈ 56.162 − 224.649 = −168.487
6.1. Absolute Extrema 309

ˆ Evaluating f at the endpoints, we get


8 2
f (−2) = 3(−2) 3 − 108(−2) 3 ≈ 19.0488 − 171.4393 = −152.39

and
8 2
f (8) = 3(8) 3 − 108(8) 3 = 768 − 432 = 336
ˆ These values are summarized in Table 6.7.

x −2 0 3 8
f (x) −152.39 0 −168.49 336

Table 6.7

From Table 6.7, the absolute maximum is 336, and it occurs when x = 8.
The absolute minimum is approximately −168.49, and it occurs when
x = 3.

3. [Exercise on page 45]

(a) Solution.
ˆ The derivative of C(x) = x3 − 3x + 100 is

C 0 (x) = 3x2 − 3 = 3(x2 − 1) = 3(x + 1)(x − 1).

Set C 0 (x) = 0. Then 3(x + 1)(x − 1) = 0. This implies that x = −1 or


x = 1. The critical number −1 is to be rejected since it is not in the interval
(0, 20).
ˆ The value of the cost at 1 is

C(1) = 13 − 3(1) + 100 = 1 − 3 + 100 = 98.

ˆ Evaluating C(x) at the endpoints of the interval [0, 20], we get

C(0) = 100 and C(20) = 203 − 3(20) + 100 = 8000 − 60 + 100 = 8040.

ˆ We summarize the values in Table 6.8.

x 0 1 20
C(x) 100 98 8040

Table 6.8

From Table 6.8, the value of x that minimizes the cost is 1.


(b) Solution. Again from Table 6.8, the minimum cost is $98.
310 Chapter 6. Applications of the Derivative II

6.2 Optimization Problems


1. [Exercise on page 46]
Solution. We want to find x that maximizes R(x). To do this, we need to find the
relative extrema of f (x).

ˆ The derivative of R(x) = −x3 + 450x2 + 52500x is

R0 (x) = −3x2 + 900x + 52500

ˆ Set R0 (x) = 0. Then −3x2 + 900x + 52500 = 0. Dividing both


 sides by3, we

2
get −x2 + 300x + 17500 = 0. Using the quadratic formula −b± 2ab −4ac , the
solutions are
p √
−300 ± 3002 − 4(−1)(17500) −300 ± 160000 −300 ± 400
x= = =
2(−1) −2 −2
So
−300 + 400 −300 − 400
x= = −50 or x = = 350
−2 −2
The critical number −50 is to be rejected since it is not possible to have a
negative number of units.
ˆ The intervals where R(x) is increasing or decreasing are shown in Figure 6.2.

+ −
0 350 ∞
choose 1 choose 400
R0 (1) = 53397 > 0 R0 (400) = −67500 < 0

Figure 6.2: Sign of R0 (x) = −3x2 + 900x + 52500

ˆ Figure 6.2 tells us that the production level x = 350 will yield a maximum
revenue.

2. [Exercise on page 46]


Solution. We solve this step by step.

Step 1. Make sure you understand the problem.


Step 2. Draw a figure when possible. (See Figure 6.3.)
Step 3. Assign a letter to each quantity/variable, and determine which variable
is to be optimized.
Here we want to maximize the area that we denote A. The other unknown
quantities are x and y as shown in Figure 6.3.
6.2. Optimization Problems 311

No fence is needed

x x

Figure 6.3

Step 4. Express the quantity you want to optimize in terms of the other variables
from Step 3.
Here we have A = xy, which is the formula for the area of a rectangle of
sides x and y.
Step 5. If the quantity you want to optimize depends on more than one variable,
use the given information to write it as a function of one variable.
ˆ Here, from the problem, the total length of the fencing is 2000 ft,
that is, x + x + y = 2000 or 2x + y = 2000. (Remember that from the
problem no fence is needed along the river. This is why the variable y
appears only once.) The equation 2x + y = 2000 is called constraint.
ˆ From the constraint we have y = 2000 − 2x. Substituting this into
the expression for A, we get

A = xy = x(2000 − 2x) = 2000x − 2x2

Step 6. Find the interval and the absolute maximum (or absolute minimum) of
the function found in Step 5. This can be done by using the First Deriva-
tive Test or the Second Derivative Test. Or, if the interval is closed, one
could use the method of Section 6.1.
Here the largest x can be is 1000 (this uses all the fence for the depth and
none for the width). Moreover, x is a nonnegative number. So the interval
is [0, 1000]. From now on, our goal is to find the absolute maximum of
the function A(x) = 2000x − 2x2 on the interval [0, 1000].
ˆ Finding the critical numbers. The derivative is A0 (x) = 2000 − 4x.
Set A0 (x) = 0. Then 2000 − 4x = 0. Subtracting 2000 from both
sides, we get −4x = −2000. Dividing both sides by −4, we get
x = 500.
ˆ Evaluating A(x) at 500, we have

A(500) = 500(2000 − 1000) = 500000

ˆ Evaluating A(x) at the endpoints of the closed interval [0, 1000], we


get
A(0) = 0 and A(1000) = 1000(2000 − 2000) = 0
ˆ The absolute maximum is then 500000 located at x = 500. From the
constraint y = 2000 − 2x, we find y = 2000 − 2(500) = 1000.
312 Chapter 6. Applications of the Derivative II

Conclusion. The dimensions of the field that maximize the area are x = 500 ft
and y = 1000 ft.
3. [Exercise on page 46]
Solution. Consider the same picture as before (see Figure 6.3). The quantity we
want to minimize is the cost, which we denote by C.
ˆ Function. Since fencing costs $27 per meter, and since the landlord will pay
half of the cost of one of the sides perpendicular to the river, it follows that
27x 54x 27x
C = 27x + 27y + = + 27y +
2 2 2
54x + 27x 81
= + 27y = x + 27y
2 2
To express C as a function of one variable, we use the fact that the area is 1536,
that is, xy = 1536. Dividing both sides of the latter equation by x, we get
y = 1536
x
. Substituting this into the formula for the cost, we get
 
81 1536 81 41472
C = x + 27 = x+
2 x 2 x
81 41472
So the function we want to minimize is C(x) = 2
x + x
ˆ Interval. Since x can’t be 0 here (otherwise the area would be 0 and this would
contradict the fact that the area is 1536), and since there is no maximum value
for x, the interval for x is the open interval (0, ∞).
ˆ Critical numbers. The first derivative is
     
0 d 81 41472 d 81 d 41472
C (x) = x+ = x + Sum Rule
dx 2 x dx 2 dx x
 
81 41472 d 1 1
= − =− 2
2 x2 dx x x

Set C 0 (x) = 0. Then

81 41472
− =0
2 x2
81 41472
=
2 x2
81x2 = 41472(2) Multiply both sides by 2x2
41472(2)
x2 = Divide both sides by 81
81
2
x = 1024

x = ± 1024 = ±32
Since x can’t be negative, the critical number −32 is to be rejected. So, the only
critical number is 32.
6.2. Optimization Problems 313

ˆ Absolute minimum. We will use the Second Derivative Test. So we need to


find the second derivative.
 
00 d 0 d 81 41472
C (x) = [C (x)] = −
dx dx 2 x2
   
d 81 d 1
= − 41472
dx 2 dx x2
d −2 d 1
= 0 − 41472 [x ] [k] = 0, m = x−m
dx dx x
d n
= −41472 −2x−3 [x ] = nxn−1

dx
82944
=
x3

Since C 0 (32) = 0 and C 00 (32) > 0, the function C has a relative minimum at
32. That local minimum is actually the absolute minimum since C 00 (x) > 0 on
the interval (0, ∞) (which means that C is concave upward on (0, ∞)). So the
absolute minimum cost occurs when x = 32.

Alternate Way of Justifying that C(32) is the Absolute Minimum

An alternate way to show that C(32) is the absolute minimum of C(x) is


to use the First Derivative Test:

− +
0 32 ∞
choose 1 choose 33
C 0 (1) = −41431.5 < 0 C 0 (33) ≈ 2.417 > 0

From this figure, the cost function is decreasing on the interval (0, 32) and
increasing on (32, ∞). So the absolute minimum is obtained when x = 20.

ˆ Using the equation y = 1536


x
, we find y = 1536
32
= 48.

Hence, the dimensions of the field that will minimize the cost to the farmer are x = 32
m and y = 48 m.

4. [Exercise on page 46]


Solution. Let A be the total surface area of the cylinder. The quantity we want to
minimize is A, which is given by the formula

A = 2πr2 + 2πrh
314 Chapter 6. Applications of the Derivative II

ˆ The constraint states that the volume must be equal to 2500, that is, πr2 h =
2500, or h = 2500
πr2
. Substituting this into the expression for A, we get
 
2 2500 2 5000
A = 2πr + 2πr = 2πr +
πr2 r

ˆ Since the radius r can’t be negative, and since r 6= 0 (otherwise, the volume
would be 0, and this would contradict the fact that the volume is 2500), we have
r > 0. So the interval for r is (0, ∞).
ˆ We now find the critical numbers. The first derivative is Setting this equal to 0,
we get 4πr3 − 5000 = 0, that is, r3 = 5000

or r3 = 1250
π
. Taking the cube root of
q
both sides, we find r = 3 1250
π
, which is the only critical number.
ˆ Using the same rules as before, the second derivative is
   
00 d 0 d 5000 d d 5000 10000r
A (r) = [A (r)] = 4πr − 2 = [4πr] − = 4π +
dr dr r dr dr r2 r4
Since this is positive for every r in (0, ∞), theqfunction A is concave upward
and therefore the (only) critical number r = 3 1250 π
gives rise to an absolute
minimum.
q
ˆ The value of h corresponding to r = 3 1250π
is

2500 2500 2500 2500 2500


h= = 2 = 2 =  = 2 1
πr2 2
q
π 3 1250 π 1250 3
π(1250) 3
1
2
(1250) 3 π 3
π π π3

1   13
2(1250) 2(1250) 3 1250
= 2 1 = 1 =2 = 2r
(1250) 3 π 3 π3 π

Thus, to minimize the total surface area, the radius and height of the cylinder should
be
 1  1
1250 3 1250 3
r= ≈ 7.355 cm and h=2 ≈ 14.71 cm
π π

5. [Exercise on page 46]


Solution. The quantity we want to maximize is the volume of the box, which we
denote by V .

ˆ Let x represent the length of a side of the base (which is a square), and let y
denote the height of the box as shown in Figure 6.4.
Then V = x×x×y = x2 y. (Recall that the volume of a box of width a, length b,
and height c is given by the formula V = abc.) From the problem, the constraint
is that the total surface is 1875, that is, the sum of areas of all faces (except the
top) is equal to 1875, or

x2 + xy + xy + xy + xy = 1875,
6.2. Optimization Problems 315

Figure 6.4

2
that is, x2 + 4xy = 1875. Solving this for y, we get y = 1875−x
4x
. Substituting
this into V , we get
1875 − x2 x(1875 − x2 ) 1875x − x3
 
2 1
V =x = = = (1875x − x3 )
4x 4 4 4

ˆ The derivative is V 0 = 14 (1875 − 3x2 ). Setting V 0 = 0, we get 1875 − 3x2 = 0.


Solving this, we get x2 = 625, which implies that x = ±25. Rejecting a negative
length leaves x = 25.
ˆ Since the second derivative, V 00 (x) = 14 (−6x) = − 32 x, is negative for all x > 0,
the function V is concave downward, and therefore the (only) critical number
x = 25 must give rise to an absolute maximum.

1875−x2
ˆ Using the equation y = 4x
, we find
1875 − (25)2 1875 − 625 1250
y= = = = 12.5
4(25) 100 100
So, the largest possible volume of the box is
V = x2 y = (25)2 (12.5) = (625)(12.5) = 7812.5 cm3

6. [Exercise on page 46]


Solution.
(a) We want to find a formula for the cost C.
ˆ The area of the bottom is x2 . Since the material for the bottom costs 6
cents/cm2 , it follows that the cost for the bottom is 6x2 .
ˆ The area of one side is xh. So the area of the four sides is 4xh. Since the
material for the sides costs 6 cents/cm2 , it follows that the cost for the sides
is 4xh(6) = 24xh.
ˆ The area of the top is x2 . Since the material for the top costs 12 cents/cm2 ,
it follows that the cost for the top is 12x2 .
Adding up these, we get the cost of the box:
C = 6x2 + 24xh + 12x2 = 18x2 + 24xh
316 Chapter 6. Applications of the Derivative II

(b) The volume of the box is V = x2 h.


768
(c) If V = 768, then x2 h = 768. Dividing both sides by x2 , we have h = x2
.
(d) Substituting h into the formula for C, we get
 
2 768 18432
C(x) = 18x + 24x = 18x2 +
x2 x

ˆ Since x > 0, the interval for x is (0, ∞).


ˆ The derivative is
 
0 d 2 18432
C (x) = 18x +
dx x
 
d 2 d 18432
= [18x ] + Sum Rule
dx dx x
d d
d 2 dx
[18432]x − 18432 dx [x] Constant Multiple Rule,
= 18 [x ] +
dx x 2 Quotient Rule
0(x) − 18432(1)
= 18(2x) +
x2
18432
= 36x −
x2
36x3 − 18432 b ac − b
= a− =
x2 c c

Setting this equal to 0, wet get 36x3 − 18432 = 0. Adding 18432 to both
sides, we get 36x3 = 18432. Dividing both sides by√36, we get x3 = 512.
Taking now the cube root of both sides, we get x = 3 512 = 8, which is the
only critical number.
ˆ The second derivative is
   
00 d 18432 d d 18432 36864
C (x) = 36x − 2
= [36x] − 2
= 36 +
dx x dx dx x x3

Since C 0 (8) > 0, it follows by the Second Derivative Test that the relative
minimum of C(x) occurs when x = 8. Actually, C(8) is the absolute mini-
mum because the function C is concave upward on the interval (0, ∞) (since
C 00 (x) > 0 for all x > 0).
Thus, the value x = 8 is the one that should be used for the cheapest cost.

6.3 Implicit Differentiation and Related Rates


1. [Exercise on page 46]

(a) Solution.
6.3. Implicit Differentiation and Related Rates 317

Finding the Derivative of an Implicit Function

ˆ An explicit function is a function of the form y = f (x), where the


output y is expressed explicitly as a function of x. Examples include
y = x2 , y = x3 , y = ex , y = ln x....

ˆ An implicit function is a function defined by an equation where


the output is not written explicitly as a function of the input. For
example, in the equations

y − x3 = 0, xy − 2x = 1, x3 y 3 − x2 y + 1 = 0,

y is given implicitly in terms of x.

ˆ To find the derivative of an implicit function, one can proceed as


follows.

– Take the derivative of both sides of the equation with respect


to x.
dy
– Solve the resulting equation for dx
.

ˆ Note. If y is a function of x, then for every real number n,

d n d dy
[y ] = [y n ] · Chain Rule
dx dy dx
dy
= ny n−1 Power Rule
dx
So, for every real number n,
d n dy
[y ] = ny n−1
dx dx
Warning. In general, the derivative of y n with respect to x is not
d
equal to ny n−1 , that is, dx [y n ] 6= ny n−1 .

dy
We want to find dx
for the implicit function x2 + y 2 = 25.

x2 + y 2 = 25
d  2 d
x + y2 =

[25] Take the derivative of both sides
dx dx
d 2 d
[x ] + [y 2 ] = 0 Sum Rule, Constant Rule
dx dx
dy d n d n dy
2x + 2y 2−1 =0 [x ] = nxn−1 , [y ] = ny n−1
dx dx dx dx
dy
2x + 2y =0
dx
318 Chapter 6. Applications of the Derivative II

dy
2y = −2x Subtract 2x from both sides
dx
dy −2x
= Divide both sides by 2y
dx 2y
dy x
=− Simplify
dx y

(b) Solution.

x3 + y 3 = 1
d 3 d
[x + y 3 ] = [1] Take the derivative of both sides
dx dx
d 3 d
[x ] + [y 3 ] = 0 Sum Rule, Constant Rule
dx dx
dy d n d n dy
3x2 + 3y 2 =0 [x ] = nxn−1 , [y ] = ny n−1
dx dx dx dx
dy
3y 2 = −3x2 Subtract 3x2 from both sides
dx
dy −3x2 −x2
= = 2 Divide both sides by 3y 2
dx 3y 2 y

(c) Solution.

2x2 − y 2 = x
d d
[2x2 − y 2 ] = [x] Take the derivative of both sides
dx dx
d d
[2x2 ] − [y 2 ] = 1
dx dx
dy d n dy
4x − 2y =1 [y ] = ny n−1
dx dx dx
dy
−2y = 1 − 4x
dx
dy 1 − 4x 4x − 1
= =
dx −2y 2y

(d) Solution.

x4 + 3y 3 = 5y
d 4 d
[x + 3y 3 ] = [5y] Take the derivative of both sides
dx dx
d 4 d dy
[x ] + 3 [y 3 ] = 5
dx dx dx
dy dy d n dy
4x3 + 9y 2 =5 [y ] = ny n−1
dx dx dx dx
6.3. Implicit Differentiation and Related Rates 319

dy dy
9y 2 −5 = −4x3
dx dx
dy
(9y 2 − 5) = −4x3 Factor the left hand side
dx
dy −4x3
=
dx 9y 2 − 5

(e) Solution.
d √ d 2
[ x − y] = [y + 3] Take the derivative of both sides
dx dx
d √  d d 2 d
x − [y] = [y ] + [3]
dx dx dx dx
1 dy dy d √  1 d n dy
√ − = 2y +0 x = √ , [y ] = ny n−1
2 x dx dx dx 2 x dx dx
dy dy 1
− − 2y =− √
dx dx 2 x
dy 1
(−1 − 2y) =− √
dx 2 x

dy − 2√1 x −1 a
a
b
= = √ =
dx −1 − 2y 2 x(−1 − 2y) c bc
1
= √
2 x(1 + 2y)

(f ) Solution.
d d
[xy] = [5] Take the derivative of both sides
dx dx
d d
[x]y + x [y] = 0 Product Rule, Constant Rule
dx dx
dy
(1)y + x =0
dx
dy
x = −y
dx
dy −y
=
dx x
(g) Solution.
d d
[3x2 + 2xy + y 2 ] = [2] Take the derivative of both sides
dx dx
d d d
3 [x2 ] + 2 [xy] + [y 2 ] = 0
dx dx dx
 
dy dy d n dy
6x + 2 y + x + 2y =0 [y ] = ny n−1
dx dx dx dx
320 Chapter 6. Applications of the Derivative II

dy dy
6x + 2y + 2x + 2y =0
dx dx
dy
(2x + 2y) = −6x − 2y
dx
dy −6x − 2y −3x − y
= =
dx 2x + 2y x+y

(h) Solution.

d d Take the deriva-


[−5x2 + xy − y 3 ] = [1] tive of both sides
dx dx
d 2 d d
−5 [x ] + [xy] − [y 3 ] = 0
dx dx dx
d dy d n dy
−5(2x) + [xy] − 3y 2 =0 [y ] = ny n−1
dx dx dx dx
 
d d dy
−5(2x) + [x]y + x [y] − 3y 2 =0 Product Rule
dx dx dx
 
dy dy
−10x + y + x − 3y 2 =0
dx dx
dy dy
x − 3y 2 = 10x − y
dx dx
dy Factor the left
(x − 3y 2 ) = 10x − y
dx hand side
dy 10x − y
=
dx x − 3y 2

(i) Solution.
d 3 2 d Take the derivative of
[x y + y 4 ] = [x]
dx dx both sides
d 3 2 d
[x y ] + [y 4 ] = 1
dx dx
d 3 2 d d
[x ]y + x3 [y 2 ] + [y 4 ] = 1 Product Rule
dx dx dx
d 3 2 d dy d n dy
[x ]y + x3 [y 2 ] + 4y 3 =1 [y ] = ny n−1
dx dx dx dx dx
dy dy
3x2 y 2 + x3 (2y) + 4y 3 =1
dx dx
dy dy
2x3 y + 4y 3 = 1 − 3x2 y 2
dx dx
dy
(2x3 y + 4y 3 ) = 1 − 3x2 y 2 Factor the left hand side
dx
dy 1 − 3x2 y 2
=
dx 2x3 y + 4y 3
6.3. Implicit Differentiation and Related Rates 321

(j) Solution.

d x+y

d Take the deriva-
= [1] tive of both sides
dx x − y dx
d d
dx
[x + y](x − y) − (x + y) dx [x − y]
2
=0 Quotient Rule
(x − y)
d d Multiply both sides
[x + y](x − y) − (x + y) [x − y] =0
dx dx by (x − y)2
   
dy dy
1+ (x − y) − (x + y) 1 − =0
dx dx
 
dy dy dy dy
x−y+x −y − x+y−x −y =0 Distribute
dx dx dx dx
dy dy dy dy
x−y+x −y −x−y+x +y =0
dx dx dx dx
dy
−2y + 2x =0 Simplify
dx
dy
2x = 2y
dx
dy 2y y
= =
dx 2x x
(k) Solution.
d y d
[e ] = [xy] Take the derivative of both sides
dx dx
dy d d y dy
ey = [xy] [e ] = ey
dx dx dx dx
dy d d
ey = [x]y + x [y] Product Rule
dx dx dx
dy dy
ey =y+x
dx dx
dy dy
ey −x =y
dx dx
dy
(ey − x) =y Factor the left hand side
dx
dy y
= y
dx e −x

(l) Solution.
d d
[yex − y] = [x] Take the derivative of both sides
dx dx
d dy
[yex ] − =1
dx dx
322 Chapter 6. Applications of the Derivative II

d d dy
[y]ex + y [ex ] − =1 Product Rule
dx dx dx
dy x dy d x
e + yex − =1 [e ] = ex
dx dx dx
dy x dy
e − = 1 − yex
dx dx
dy
(ex − 1) = 1 − yex Factor the left hand side
dx
dy 1 − yex
=
dx ex − 1

(m) Solution.
d xy3 d Take the derivative of
[e ] = [2x + 3y + 1]
dx dx both sides
3 d d d  g(x)  d
exy [xy 3 ] = [2x + 3y + 1] e = eg(x) [g(x)]
dx dx dx dx
3 d d d d
exy [xy 3 ] = 2 [x] + 3 [y] + [1]
dx dx dx dx
3 d dy
exy [xy 3 ] = 2 + 3 + 0
dx dx
 
3 d d dy
exy [x]y 3 + x [y 3 ] = 2 + 3 + 0 Product Rule
dx dx dx
 
xy 3 3 2 dy dy
e y + 3xy =2+3
dx dx
3 dy3 dy
y 3 exy + 3xy 2 exy =2+3
dx dx
3 dy dy 3
3xy 2 exy −3 = 2 − y 3 exy
dx dx
 3
 dy 3
3xy 2 exy − 3 = 2 − y 3 exy Factor the left hand side
dx
3
dy 2 − y 3 exy
=
dx 3xy 2 exy3 − 3

(n) Solution.
d d 3 3 Take the derivative of
[x + ln y] = [x y ]
dx dx both sides
d d d 3 3 d
[x] + [ln y] = [x ]y + x3 [y 3 ]
dx dx dx dx
d dy
d [y] d 3 3 d d
[x] + dx = [x ]y + x3 [y 3 ] [ln y] = dx
dx y dx dx dx y
dy  
2 3 3 2 dy d n dy
1+ dx
= 3x y + x 3y [y ] = ny n−1
y dx dx dx
6.3. Implicit Differentiation and Related Rates 323

1 dy dy
1+ = 3x2 y 3 + 3x3 y 2
y dx dx
1 dy dy
− 3x3 y 2 = 3x2 y 3 − 1
y dx dx
 
1 dy
− 3x3 y 2 = 3x2 y 3 − 1 Factor the left hand side
y dx
1 − 3x3 y 3 dy
 
a a − bc
= 3x2 y 3 − 1 −c=
y dx b b
dy (3x2 y 3 − 1)y y
= Multiply both sides by
dx 1 − 3x3 y 3 1 − 3x3 y 3

2. [Exercise on page 47]


(a) Solution.
dy
• First, we need to find the derivative dx
using implicit differentiation.

x2 − xy − y 2 = 1
d 2 d Take the derivative of
[x − xy − y 2 ] = [1]
dx dx both sides
d 2 d d
[x ] − [xy] − [y 2 ] = 0
dx dx dx
 
dy d
2x − y + x − [y 2 ] = 0 Product Rule
dx dx
 
dy dy d n dy
2x − y + x − 2y =0 [y ] = ny n−1
dx dx dx dx
dy dy
2x − y − x − 2y =0
dx dx
dy
(−x − 2y) = −2x + y
dx
dy −2x + y
=
dx −x − 2y
dy
• To get the slope at (2, 1), we substitute x with 2 and y with 1 into dx
:

−2(2) + 1 −4 + 1 −3 3
m= = = =
−(2) − 2(1) −2 − 2 −4 4
• The equation of the tangent line to the curve x2 − xy − y 2 = 1 at (2, 1) is
3
y − 1 = (x − 2)
4
3 3 Distribute the right
y−1= x−
4 2 hand side
3 1
y = x− Add 1 to both sides
4 2
324 Chapter 6. Applications of the Derivative II

(b) Solution.
dy
• First we need to find the derivative dx
. Using implicit differentiation, we
have

(x2 + y 2 )2 = 2(x3 + y 2 )

d  2 d  3 Take the deriva-


(x + y 2 )2 = 2(x + y 2 )
 
tive of both sides
dx dx
d 2 d  3
2(x2 + y 2 ) [x + y 2 ] = 2(x + y 2 )

Chain Rule
dx dx
d d  3 Constant Multiple
2(x2 + y 2 ) [x2 + y 2 ] = 2 x + y2

dx dx Rule
   
2 2 d 2 d 2 d 3 d 2
(2x + 2y ) [x ] + [y ] = 2 [x ] + [y ]
dx dx dx dx
   
2 2 dy 2 dy d n dy
(2x + 2y ) 2x + 2y = 2 3x + 2y [y ] = ny n−1
dx dx dx dx
dy dy dy
4x3 + 4x2 y + 4y 2 x + 4y 3 = 6x2 + 4y Distribute
dx dx dx
dy dy dy
4x2 y + 4y 3 − 4y = 6x2 − 4x3 − 4y 2 x
dx dx dx
dy Factor the left
(4x2 + 4y 3 − 4y) = 6x2 − 4x3 − 4y 2 x
dx hand side
dy 6x − 4x − 4y 2 x
2 3
3x2 − 2x3 − 2y 2 x
= = Simplify
dx 4x2 + 4y 3 − 4y 2x2 + 2y 3 − 2y
• The slope at (1, 1) is

dy 3(1)2 − 2(1)3 − 2(1)2 (1) 3−2−2 −1


m= = 2 3
= =
dx 2(1) + 2(1) − 2(1) 2+2−2 2

• The equation of the tangent line to the curve (x2 + y 2 )2 = 2(x3 + y 2 ) at


(1, 1) is
1
y − 1 = − (x − 1)
2
1 1 Distribute the right
y−1=− x+
2 2 hand side
1 3
y =− x+ Add 1 to both sides
2 2
3. [Exercise on page 47]
Solution.
dy
• First, we need to find dx
.

x2 − y 2 = 4
6.3. Implicit Differentiation and Related Rates 325

d 2 d
[x − y 2 ] = [4] Take the derivative of both sides
dx dx
d 2 d
[x ] − [y 2 ] = 0
dx dx
dy d n dy
2x − 2y =0 [y ] = ny n−1
dx dx dx
dy
−2y = −2x
dx
dy −2x x
= =
dx −2y y
• We now find the second derivative.
   
00 d dy d x
y = =
dx dx dx y
d d
dx
[x]y − x dx [y]
= Quotient Rule
y2
dy
y − x dx
=
y2
 
y − x xy dy x
= =
y2 dx y
x2 y 2 −x2
y− y y y 2 − x2
= = =
y2 y2 y3
4. [Exercise on page 47]
Solution.
ˆ We first need to find the marginal cost using implicit differentiation.

C 2 − 40 = x2 + 80 x
d  2  d  2 √ 
C − 40 = x + 80 x Take the derivative of both sides
dx dx
d 2 d d 2 d √ 
[C ] − [40] = [x ] + 80 x
dx dx dx dx
dC 1 d 2 dC d √  1
2C − 0 = 2x + 80 √ [C ] = 2C , x = √
dx 2 x dx dx dx 2 x
dC 2x + √40x
=
dx 2C
√ a
2x x + 40 b ac + b a
= √ = , b =
a+
2C x c c c bc

ˆ When x = 4, the equation C 2 − 40 = x2 + 80 x becomes

C 2 = 42 + 80 4 + 40 = 16 + 160 + 40 = 216

This implies that C = 216.
326 Chapter 6. Applications of the Derivative II


ˆ Substituting x with 4 and C with 216 into dC
dx
, we get

dC 2(4) 4 + 40
= √ √ ≈ 0.95
dx 2( 216) 4
So the marginal cost at x = 4 is 0.95.
ˆ The interpretation is that at the level of production of 4 items, the cost
of producing one additional item is approximately 0.95.

5. [Exercise on page 47]

(a) Solution.

Rate of Change
ˆ If y is a function of time t, the rate of change of y with respect to t
is dy
dt
. This can also be interpreted as the speed/rate at which y is
changing.

ˆ If x is a function of t, then for any real number n we have the


following (thanks to the chain rule).

d n dx
[x ] = nxn−1
dt dt

ˆ From the problem, the rate of change of x is dxdt


= 5 units/day. And we
want to find the rate of change of revenue when x = 100 units.
ˆ The rate of change of R is
dR d dx d
= [80x − 0.25x2 ] = 80 − 0.25 [x2 ]
dt dt dt dt
 
dx dx d n dx
= 80 − 0.25 2x [x ] = nxn−1
dt dt dt dt
dx dx
= 80 − 0.5x
dt dt
ˆ Substituting dx
dt
with 5 and x with 100, we get

dR
= 80(5) − 0.5(100)(5) = 400 − 250 = $150/day.
dt
(b) Solution. The rate of change of cost with respect to time is
dC d dx d dx
= [7x + 20] = 7 + [20] = 7 + 0 = 7(5) = $35/day
dt dt dt dt dt
(c) Solution. Let P be the profit. Since P = R − C, the rate of change of P is
dP d dR dC
= [R − C] = − = 150 − 35 = $115/day
dt dt dt dt
6.3. Implicit Differentiation and Related Rates 327

6. [Exercise on page 47]


Solution.

ˆ Given: dx
dt
= 10 units/month Unknown: dC
dt
when x = 95 units.
ˆ The rate of change of C is

dC d d d
= [0.3x2 + 7000] = 0.3 [x2 ] + [7000]
dt dt dt dt
 
dx d n dx
= 0.3 2x +0 [x ] = nxn−1
dt dt dt
dx
= 0.6x
dt
dx
= 0.6(95)(10) = $570/month x = 95, = 10
dt

7. [Exercise on page 47]


Solution.

ˆ Given: dC
dt
= $10/month Unknown: dR
dt
when R = $20000.
R2
ˆ To find dR
dt
, we first differentiate both sides of the equation C = 300000
+ 8000
with respect to t.

R2
C= + 8000
300000
R2
 
dC d
= + 8000
dt dt 300000
R2
 
dC d d
= + [8000]
dt dt 300000 dt
dC 1 d 2
= [R ] + 0
dt 300000 dt
 
dC 1 dR d 2 dR
= 2R [R ] = 2R
dt 300000 dt dt dt
dC R dR
=
dt 150000 dt
Multiplying both sides of the latter equation by 150000, we get

dC dR
150000 =R
dt dt
Dividing both sides by R, we get

dR 150000 dC 150000
= = (10) = $75/month
dt R dt 20000
328 Chapter 6. Applications of the Derivative II

8. [Solution on page 48]


dp dq
Solution. Given: dt
= $21/month Unknown: dt
when q = 7 units.
Taking the derivative of both sides of the equation 3p − 14 q 2 = 62 with respect to time
t, we have
d 1 d
[3p − q 2 ] = [62]
dt 4 dt
d 1d 2
3 [p] − [q ] = 0
dt 4 dt
 
dp 1 dq d 2 dx
3 − 2q =0 [x ] = 2x
dt 4 dt dt dt
dp 1 dq
3 − q =0
dt 2 dt
dp 1 dq
3 = q
dt 2 dt
6 dp dq dq
= Solve for
q dt dt dt
dq 6
= (21) Substitute
dt 7
dq
= $18/month
dt
9. [Exercise on page 48]
Solution. We know that the surface of a balloon with radius r is given by the formula
S = 4πr2 . Taking the derivative of both sides with respect to t, we get
dS d d dr dr
= [4πr2 ] = 4π [r2 ] = 4π(2r) = 8πr
dt dt dt dt dt
10. [Exercise on page 48]
Solution.

How to solve a related rates problem?

In a related rates problem, two quantities (or variables) are involved, the rate
of change of one quantity is given, and the rate of change of the other quantity
is to be found. To solve such problems, one can go through the following steps.

Step 1. Assign a variable to each quantity. Draw a diagram when possible.

Step 2. Write the given rate of change and the rate of change that is to be
found.

Step 3. Find an equation relating the quantities involved in the problem.

Step 4. Take the derivative of both sides of the equation with respect to
time t.
6.3. Implicit Differentiation and Related Rates 329

Step 5. Solve for the unknown rate of change and substitute the given in-
formation.

ˆ Here the quantities that are functions of time are the volume and the radius.
Let V represent the volume, and let r represent the radius.
ˆ
dr dV 6
Given: = 0.5 cm/s Unknown: when r = = 3 cm.
dt dt 2
ˆ Equation that relates V and r: we know that the volume of a sphere of radius
r is given by the formula V = 34 πr3 .
ˆ Differentiating both sides of the equation V = 34 πr3 with respect to time t, we
get
 
dV d 4 3 4 d 4 dr d n dr
= πr = π [r3 ] = π(3r2 ) [r ] = nrn−1
dt dt 3 3 dt 3 dt dt dt
dr
= 4πr2 = 4π(3)2 0.5 = 18π cm3 /s
dt

11. [Exercise on page 48]


Solution. Let V be the volume of the balloon, and let D be its diameter.

dV dD
Given: = 180 cm3 /s Unknown: when R = 75 cm
dt dt
The volume is given by

4
V = πR3
3
 3
4 D D
= π R=
3 2 2
4 D3
= π
3 8
1
V = πD3
6

Differentiating both sides, we get


 
dV d 1 1 d
= πD = π [D3 ]
3
dt dt 6 6 dt
1 dD d n dD
= π(3D2 ) [D ] = nDn−1
6 dt dt dt
dV 1 dD
= πD2
dt 2 dt
330 Chapter 6. Applications of the Derivative II

dD
Solving this latter equation for dt
, we get

dD 2 dV
=
dt πD2 dt
2 360 dV
= 2
(180) = D = 2R = 2(75) = 150, = 180
π(150) 22500π dt
2
= cm/s
125π

12. [Exercise on page 48]


Solution. The quantities that depend on time are the volume V , and the height h.
(Note that the radius is constant here: r = 7 m.)

dV dh
Given: = 4 m3 /min Unknown: .
dt dt
We know that the volume of a cylinder with radius r and height h is given by the
formula V = πr2 h. Differentiating both sides with respect to time t, we get
dV d dh
= [πr2 h] = πr2
dt dt dt
dV
So dt
= πr2 dh
dt
. Solving this latter equation for dh
dt
, we get

dh 1 dV
= 2
dt πr dt
dV
Substituting r with 7 and dt
with 4, we get

dh 1 4
= 2
(4) = m/min
dt π(7) 49π

13. [Exercise on page 48]


Solution. Consider Figure 6.5.

x
drone at some instant

0.06 z

observer

Figure 6.5

dx dz
Given: = 90 mi/h Unknown: when z = 0.1 mi.
dt dt
6.4. Elasticity of Demand 331

Using the Pythagorean theorem, we have x2 + 0.0036 = z 2 . Differentiating both sides


with respect to t, we get
d 2 d d
[x ] + [0.0036] = [z 2 ]
dt dt dt
dx dz d 2 dx d 2 dz
2x + 0 = 2z [x ] = 2x , [z ] = 2z
dt dt dt dt dt dt
x dx dz
= Divide both sides by 2z
z dt dt
ˆ When z = 0.1, the equation x2 + 0.0036 = z 2 becomes x2 + 0.0036 = 0.12 , that
is, x2 + 0.0036 = 0.01. Subtracting 0.0036 from both sides, we get x2 = 0.0064.
Taking the square root of both sides, we have x = 0.08 (remember that x > 0).
ˆ Substituting x with 0.08, z with 0.1, and dx
dt
with 90 into the equation x dx
z dt
= dz
dt
,
get
dz x dx 0.08
= = (90) = 120 mi/h
dt z dt 0.06

6.4 Elasticity of Demand


1. [Exercise on page 48]

(a) Solution.

Elasticity of Demand
Suppose that the demand q and the price p are related by an equation.
The elasticity of demand, denoted E, is given by
p dq
E=−
q dp
ˆ If E < 1, demand is inelastic. This means that demand will not
change much if there is a slight price change.

ˆ If E > 1, demand is elastic. This means that demand is more


sensitive to price.

ˆ If E = 1, demand has unit elasticity. This means that the percentage


change in the price is more or less the same as the percentage change
in the quantity demanded.

We want to find the elasticity of demand for the equation q = −0.05p + 8.5 when
p = $102.
ˆ First, we need to find dq
dp
.

dq d d d
= [−0.05p + 8.5] = −0.05 [p] + [8.5]
dp dp dp dp
332 Chapter 6. Applications of the Derivative II

= −0.05(1) + 0 = −0.05

ˆ The elasticity is

p dq p 0.05p
E=− =− (−0.05) =
q dp −0.05p + 8.5 −0.05p + 8.5

ˆ When p = $102,

0.05(102) 5.1
E= = = 1.5
−0.05(102) + 8.5 3.4

ˆ Interpretation. Since E > 1, the demand is elastic.


(b) Solution. Taking the derivative of q = 10000 − 30p2 with respect to p, we get

dq d d
= [10000] − 30 [p2 ] = 0 − 30(2p) = −60p
dp dp dp

The elasticity is

p dq p
E=− =− (−60p)
q dp 10000 − 30p2
60p2 60(10)2
= = Replace p with 10
10000 − 30p2 10000 − 30(10)2
6000
= ≈ 0.857
7000

Interpretation. Since E < 1, the demand is inelastic.


(c) Solution. Taking the derivative of q = 100e−0.1p with respect to p, we get

dq d  d  −0.1p 
100e−0.1p = 100

= e Constant Multiple Rule
dp dp dp
 
−0.1p d d  g(p)  d
= 100 e [−0.1p] e = eg(p) [g(p)]
dp dp dp
= 100 e−0.1p (−0.1) = −10e−0.1p


The elasticity is

p dq p
−10e−0.1p

E=− =− −0.1p
q dp 100e
p 10
= = =1 Replace p with 10
10 10
Interpretation. Since E = 1, the demand has unit elasticity.

2. [Exercise on page 48]


6.4. Elasticity of Demand 333

dq
(a) Solution. We first need to find dp
.
     
dq d 2 d 1 1 −2
= =2 =2 − 2 = 2
dp dp p dp p p p

We now calculate E.
p2
   
p dq p −2 −2
E=− = −2 =− =1
q dp p
p2 2 p2

Interpretation. Since E = 1, the demand has unit elasticity.


dq
(b) Solution. First, we find dp
.

dq d  5−0.3 ln p  d d  g(p)  d
= e = e5−0.3 ln p [5 − 0.3 ln p] e = eg(p) [g(p)]
dp dp dp dp dp
   
d d 1 d 1
= e5−0.3 ln p [5] − 0.3 [ln p] = e5−0.3 ln p 0 − 0.3 [ln p] =
dp dp p dp p
 
1 0.3
= e5−0.3 ln p 0 − 0.3 = − e5−0.3 ln p
p p

The elasticity is
 
p dq p 0.3 5−0.3 ln p
E=− = − 5−0.3 ln p − e
q dp e p
= −(−0.3) = 0.3 Simplify

Interpretation. Since E < 1, the demand is inelastic.


dq
(c) Solution. We first need to find dp
.

dq d  d  −1.68 
15000p−1.68 = 15000

= p Constant Multiple Rule
dp dp dp
d  −1.68 ln p 
= 15000 e AB = eB ln A
dp
d d  g(p)  d
= 15000e−1.68 ln p [−1.68 ln p] e = eg(p) [g(p)]
dp dp dp
 
1 −25200 −1.68 ln p
= 15000e−1.68 ln p −1.68 = e
p p

The elasticity is
 
p dq p −25200 −1.68 ln p
E=− − e
q dp 15000e−1.68 ln p p
25200
= = 1.68 Simplify
15000
Interpretation. Since E > 1, the demand is elastic.
334 Chapter 6. Applications of the Derivative II

3. [Exercise on page 48]

(a) Solution. Since q is given implicitly in terms of p, we will use implicit differ-
dq
entiation to find dp .

p = 0.604q 2 − 20.16q + 263.067


d d  Take the derivative of
0.604q 2 − 20.16q + 263.067

[p] =
dp dp both sides
d d d
1 = 0.604 [q 2 ] − 20.16 [q] + [263.067]
dp dp dp
 
dq dq
1 = 0.604 2q − 20.16 + 0
dp dp
dq dq
1 = 1.208q − 20.16
dp dp
dq
1 = (1.208q − 20.16)
dp
dq 1 Divide both sides by
=
dp 1.208q − 20.16 1.208q − 20.16
dq
Alternate Way to Find dp

dq
If p is given explicitly in terms of q, to get dp
, one can use the fact that

dq 1
= dp
dp dq

dq
to find dp
. For the equation p = 0.604q 2 − 20.16q + 263.067,

dq 1 1
= dp = d
dp dq dq
[0.604q 2 − 20.16q + 263.067]
1
= d d d
0.604 dq [q 2 ] − 20.16 dq [q] + dq
[263.067]
1 1
= =
0.604(2q) − 20.16(1) + 0 1.208q − 20.16

The elasticity is
p dq
E=−
q dp
0.604q 2 − 20.16q + 263.067
 
1
=−
q 1.208q − 20.16
−(0.604q 2 − 20.16q + 263.067)
=
q(1.208q − 20.16)
6.4. Elasticity of Demand 335

−(0.604(11)2 − 20.16(11) + 263.067)


= Substitute q with 11
11(1.208(11) − 20.16)
−(73.084 − 221.76 + 263.067)
=
11(13.288 − 20.16)
−114.391
= ≈ 1.513
−75.592
Interpretation. Since E > 1, the demand is elastic.
(b) Solution. Using implicit differentiation, we get
p = 900e−0.6q
d d  Take the derivative of
900e−0.6q

[p] =
dp dp both sides
d  −0.6q 
1 = 900 e Constant Multiple Rule
dp
d d  g(q)  d
1 = 900e−0.6q [−0.6q] e = eg(q) [g(q)]
dp dp dp
dq
1 = 900e−0.6q (−0.6) Constant Multiple Rule
dp
dq
1 = −540e−0.6q
dp
1 dq Divide both sides by
=
−540e −0.6q dp −540e−0.6q
We now calculate the elasticity of demand.
900e−0.6q
 
p dq 1
E=− =−
q dp q −540e−0.6q
900 900 Simplify and substitute q
= =
540q 540(8) with 8
900
= ≈ 0.208
4320
Since E < 1, the demand is inelastic.

4. [Exercise on page 49]


(a) Solution.
dq d d d
= [108 − p2 ] = [108] − [p2 ] = 0 − 2p = −2p
dp dp dp dp
The elasticity is
p dq p 2p2
E=− =− (−2p) =
q dp 108 − p2 108 − p2
(b) Solution.
336 Chapter 6. Applications of the Derivative II

Finding the Intervals for the Price p where Demand is Elastic


and where Demand is Inelastic
Suppose q = f (p) is a function of p. Then the elasticity can be expressed
as a function of p: E = E(p). To find the price intervals where demand is
elastic and where demand is inelastic, one can proceed as follows.

Step 1. Solve the equation E = 1 for p. Let p0 be the solution.

Step 2. Select a test number p1 such that p1 < p0 and f (p1 ) ≥ 0, and
substitute it into the expression for E to see if E < 1 or E > 1
when p < p0 .

ˆ If E(p1 ) < 1, the demand is inelastic when p < p0 .


ˆ If E(p1 ) > 1, the demand is elastic when p < p0 .

Step 3. Select a test number p2 such that p2 > p0 and f (p2 ) ≥ 0.

ˆ If E(p2 ) < 1, the demand is inelastic when p > p0 .


ˆ If E(p2 ) > 1, the demand is elastic when p > p0 .

2p2
Consider the expression E = 108−p2
from part (a).
ˆ Solving the equation E = 1.

E=1
2p2
=1
108 − p2
2p2 = 108 − p2 Multiply both sides by 108 − p2
3p2 = 108 Add p2 to both sides
p2 = 36 Divide both sides by 3

p ± 36 = ±6 Take square root of both sides

Since the price is positive, we reject the negative value. So the solution to
the equation E = 1 is p0 = 6.
ˆ Select a test number p less than 6. Take for example p = 5. Substituting
this into E, we get
2(5)2 50
E= 2
= <1
108 − (5) 83
So the demand is inelastic when p < 6.
ˆ Select a test number p greater than 6. Take for example p = 7. Substituting
this into E, we get
2(7)2 98
E= 2
= >1
108 − (7) 59
So the demand is elastic when p > 6.
6.4. Elasticity of Demand 337

(c) Solution. We will use the following result.

Revenue and Elasticity


Let q be the demand at a price p. Then the total revenue is R = pq.

ˆ Taking the derivative of both sides of the equation R = pq with


respect to p, one can show that
dR
= q(1 − E)
dp

ˆ So, if the demand is inelastic (that is, E < 1), then 1 − E > 0, and
therefore dR
dp
> 0. This implies that total revenue increases as price
increases when E < 1.

ˆ Similarly, if the demand is elastic (that is, E > 1), total revenue
decreases as price increases.

ˆ Since the derivative dR


dp
is positive when E < 1 and negative when
E > 1 (see the figure below), it follows that the total revenue attains
its maximum at the price where E = 1.

Total revenue
+ −
0 E<1 E=1 E>1 p
dR dR
dp
>0 dp
<0

Since total revenue is maximized at the price where demand has unit elasticity,
and since E = 1 when p = 6 (see part (b)), it follows that the price p = $6
results in the maximum revenue. The demand at this price is
q = 108 − p2 = 108 − (6)2 = 72
The maximum revenue is then
R = pq = 6(72) = $432

5. [Exercise on page 49]


Solution.
(a) The derivative of q with respect to p is
dq d d d
= [16 − 4 ln p] = [16] − 4 [ln p]
dp dp dp dp
1 4 d 1
=0−4 =− [ln p] =
p p dp p
338 Chapter 6. Applications of the Derivative II

The elasticity is
 
p dq −p 4 4
E=− =− − =
q dp 16 − 4 ln p p 16 − 4lnp

(b) To find the price intervals where demand is elastic and where demand is inelastic,
we first need to solve the equation E = 1 for p.

E=1
4
=1
16 − 4 ln p
4 = 16 − 4 ln p Multiply both sides by 16 − 4 ln p.
−12 = −4 ln p Subtract 16 from both sides
3 = ln p Divide both sides by −4.
e3 = p Take the natural exponential of both sides

So p = e3 ≈ 20.09 is the solution to the equation E = 1.


ˆ Select a test number less than e3 . Take for example p = 20. Substituting
this into the expression for E, we get

4 4
E(20) = ≈ <1
16 − 4 ln 20 4.017

So the demand is inelastic when p < e3 ≈ 20.09.


ˆ Select a test number greater than e3 . Take for example p = 21. Substituting
this into the expression for E, we get

4 4
E(21) = ≈ >1
16 − 4 ln 21 3.822

So the demand is elastic when p > e3 ≈ 20.09.


(c) Total revenue is maximized when the price is p = e3 ≈ $20.09. The demand
at this price is

q = 16 − 4 ln p = 16 − 4 ln(e3 ) = 16 − 4(3) = 4

The maximum revenue is then R = pq = e3 (4) ≈ $80.34.

6.5 Differentials and Linear Approximations


1. Find the differential of each function. [Exercise on page 49]

(a) y = x2
Solution.
6.5. Differentials and Linear Approximations 339

Independent and Dependent Variables

ˆ If y = f (x) is a function, the variable x is called input or independent


variable. And y is called output or dependent variable. This means
that x is an arbitrary input and y depends on x.

ˆ It is possible that the output depends on two or more independent


variables. For example, in the expression z = 5x1 + 7x2 , the inde-
pendent variables are x1 and x2 , and the dependent variable is z. In
other words, z depends on x1 and x2 .

Differentials
Let y = f (x) be a function.

ˆ The differential of x, denoted dx, is an independent variable; that


is, dx is an arbitrary input.

ˆ Suppose that the function y = f (x) is differentiable. The differential


of y, denoted dy, is defined by

dy = f 0 (x)dx

where f 0 (x) is the derivative of f (x). In words, this says that the
differential of y is f 0 (x) multiplied by dx. The geometric meaning of
dy is shown below.
Note that dy depends on two independent variables, namely x and
dx.

Here we want to find the differential of y = x2 . Let f (x) = x2 . Then f 0 (x) = 2x


and
dy = f 0 (x)dx = 2xdx

(b) Solution. Let f (x) = x3 + 1. Then f 0 (x) = 3x2 and

dy = f 0 (x)dx = 3x2 dx

(c) Solution. Let f (x) = 4x3 − 5x2 + x − 8. To find the differential of y = f (x),
we first need to find the derivative of f .

d  3 d d d d
f 0 (x) = 4x − 5x2 + x − 8 = [4x3 ] − [5x2 ] + [x] − [8]

dx dx dx dx dx
2 2
= 4(3x ) − 5(2x) + 1 − 0 = 12x − 10x + 1

Now, the differential of y is

dy = f 0 (x)dx = (12x2 − 10x + 1)dx


340 Chapter 6. Applications of the Derivative II

5 1
(d) Solution. Let f (x) = 3x 4 + x 4 . The derivative of y = f (x) is

d h 5i d h 1i d h 5i d h 1i
f 0 (x) = 3x 4 + x4 = 3 x4 + x4
dx dx dx dx
   
5 5 −1 1 1 −1 5 1 1 3 d n
=3 x4 + x4 = 3 x 4 + x− 4 [x ] = nxn−1
4 4 4 4 dx
15 1 1 3
= x 4 + x− 4
4 4
So the differential of y is
 
0
15 1 1 3
−4
dy = f (x)dx = x + x
4 dx
4 4

2. [Exercise on page 49]



(a) Solution. Let f (x) = x + 3. Then
d d
d h√ i [x + 3] d hp i [g(x)]
f 0 (x) = x + 3 = dx√ g(x) = dxp
dx 2 x+3 dx 2 g(x)
d
dx
+ d [3]
[x] 1
= √ dx = √
2 x+1 2 x+3

The differential of y = f (x) is

1
dy = f 0 (x)dx = √ dx
2 x+3

Evaluating this at x = 1 and dx = 0.1, we get


1 1
dy = √ (0.1) = (0.1) = 0.025
2 1+3 4

(b) Solution. Let f (x) = 4x2 + 13. Then

0 d h√ 2 i d
dx
[4x2 + 13] d hp i d
dx
[g(x)]
f (x) = 4x + 13 = √ g(x) = p
dx 2 4x2 + 13 dx 2 g(x)
8x 4x
= √ =√
2
2 4x + 13 4x2 + 13
The differential is
4x
dy = f 0 (x)dx = √ dx
4x2 + 13
When x = 3 and dx = 0.04, this becomes

4(3) 12
dy = p (0.04) = (0.04) ≈ 0.0686
2
4(3) + 13 7
6.5. Differentials and Linear Approximations 341

15x+6
(c) Solution. Let f (x) = −x+2
. Then
d d
[15x + 6](−x + 2) − (15x + 6) dx [−x + 2]
f 0 (x) = dx
Quotient Rule
(−x + 2)2
15(−x + 2) − (15x + 6)(−1) −15x + 30 + 15x + 6 36
= 2
= 2
=
(−x + 2) (−x + 2) (−x + 2)2
The differential of y = f (x) is
36
dy = f 0 (x)dx = dx
(−x + 2)2
Substituting x with −2 and dx with 0.03 into this, we get
36 36 36
dy = 2
(0.03) = 2
(0.03) = (0.03) = 0.0675
(−(−2) + 2) (2 + 2) 16
x
(d) Solution. Let f (x) = e 8 . Then
d  x x d
hxi d  g(x)  d
f 0 (x) = e8 = e8 e = eg(x) [g(x)]
dx dx 8 dx dx
   
x 1 d x 1 1 x
= e8 [x] = e 8 (1) = e 8
8 dx 8 8

The differential of y = f (x) is


1 x
dy = f 0 (x)dx = e 8 dx
8
When x = 0 and dx = −0.02, this becomes
1 0 1
dy = e 8 (−0.02) = e0 (−0.02) = −0.0025
8 8
3. [Exercise on page 49]

(a) Solution.

Using Differentials to Approximate Changes

Let y = f (x) be a function.


ˆ If the variable x changes from x1 to x2 , the change in x, denoted
∆x, is the difference x2 − x1 (final value minus initial value); that is,

∆x = x2 − x1

ˆ If x changes from x1 to x2 , the quantity dx that appears in the


formula for the differential is defined to be ∆x; that is,

dx = ∆x
342 Chapter 6. Applications of the Derivative II

ˆ Let x1 and x2 be numbers in the domain of f . If x changes from


x1 to x2 , then the corresponding change in y, denoted ∆y, is the
difference f (x2 ) − f (x1 ); that is,

∆y = f (x2 ) − f (x1 )

ˆ Now, suppose that the function y = f (x) is differentiable. If x


changes from x1 to x2 and ∆x is small, then one can show that ∆y
can be approximated by the differential of y evaluated at x = x1 and
dx = ∆x; that is,
∆y ≈ f 0 (x1 )∆x (6.5.1)

Note. As x changes from x1 to x2 , the quantity ∆y is the actual change


in y, whereas dy is the approximate change in y. That is,

∆y ≈ dy

Geometrically, ∆x, ∆y, and dy are shown. (T is the tangent line to the
graph of f at P (x, f (x + ∆x)), and x = x1 , x + ∆x = x2 .)
y
y = f (x)

f (x + ∆x)

∆y
dy
P
f (x)
∆x

x x + ∆x x

To approximate ∆y, the change in y, as x changes from x1 to x2 , one


can proceed as follows.

Step 1. Find the differential of y, which is given by dy = f 0 (x)dx.

Step 2. Evaluate dy for x = x1 and dx = ∆x = x2 − x1 :

dy = f 0 (x1 )∆x

Step 3. Use the fact that ∆y ≈ dy to get an approximation of ∆y.

Here we want to approximate ∆y for y = −x3 − x2 + 150, x1 = 1, x2 = 2.


6.5. Differentials and Linear Approximations 343

ˆ First, we need to find the derivative of f .


d
f 0 (x) = [−x3 − x2 + 150] = −3x2 − 2x
dx
Using this, we find

dy = f 0 (x)dx = (−3x2 − 2x)dx

ˆ Evaluating the differential at

x = x1 = 1 and dx = ∆x = x2 − x1 = 2 − 1 = 1

we find
dy = (−3(1)2 − 2(1))(1) = (−3 − 2)(1) = −5
ˆ Since ∆y ≈ dy, it follows that ∆y ≈ −5.
Thus, as x changes from 1 to 2, the change in y is approximately −5.
(b) Solution. First, we need to find the derivative of f (x) = xe−2x .
d  −2x  d d
f 0 (x) = xe = [x]e−2x + x [e−2x ] Product Rule
dx dx dx
 
d d  g(x)  d
= (1)e−2x + x e−2x [−2x] e = eg(x) [g(x)]
dx dx dx
= e−2x + xe−2x (−2) = (1 − 2x)e−2x Factor out e−2x

Now, the differential of y is

dy = f 0 (x)dx = (1 − 2x)e−2x dx
Evaluate dy at x = x1 = 0 and
= (1 − 2(0))e−2(0) (0.2)
dx = ∆x = x2 −x1 = 0.2−0 = 0.2
= (1 − 0)e0 (0.2) = 0.2

Since ∆y ≈ dy, we have ∆y ≈ 0.2.


ln(x−2)
(c) Solution. The derivative of f (x) = x+1
is
d d
[ln(x − 2)](x + 1) − ln(x − 2) dx [x + 1]
f 0 (x) = dx
2
Quotient Rule
(x + 1)
d
[x−2] d d
dx
x−2
(x + 1) − ln(x − 2) dx [x + 1] d dx
[g(x)]
= [ln g(x)] =
(x + 1)2 dx g(x)
1 x+1
x−2
(x + 1) − ln(x − 2)(1) x−2
− ln(x − 2)
= =
(x + 1)2 (x + 1)2
Using this, we find
!
x+1
x−2
− ln(x − 2)
dy = f 0 (x)dx = dx
(x + 1)2
344 Chapter 6. Applications of the Derivative II

Evaluating dy at
x = x1 = 3 and dx = ∆x = x2 − x1 = 2.9 − 3 = −0.1
we get !
3+1
− ln(3 − 2)
 
3−2 4 − ln 1
dy = (−0.1) = = −0.025
(3 + 1)2 16
Since ∆y ≈ dy, it follows that ∆y ≈ −0.025.

4. [Exercise on page 49]

(a) Solution.

Linear Approximations
Let f be a function, and let a be a number. Suppose f is differentiable at
a. Then the equation of the tangent line to the graph of f at the point
(a, f (a)) is

y − f (a) = f 0 (a)(x − a) Slope = m = f 0 (a)


y = f (a) + f 0 (a)(x − a) Add f (a) to both sides

When x is near a, the tangent line is an approximation to the graph of f .


So, when x is near a,

f (x) ≈ f (a) + f 0 (a)(x − a)

The function L defined by

L(x) = f (a) + f 0 (a)(x − a)

is called the linear approximation of f at a.

We want to find the linear approximation of f (x) = x2 at a = 4. First, f 0 (x) =


2x. Evaluating this at 4, we get
m = f 0 (4) = 2(4) = 8
So the linear approximation of f at a = 4 is
L(x) = f (a) + f 0 (a)(x − a) = f (4) + f 0 (4)(x − 4)
42 + 8(x − 4) = 16 + 8(x − 4)

(b) Solution. The derivative of f (x) = x is f 0 (x) = 2√1 x . Substituting x with 25
into this, we get
1 1 1
f 0 (25) = √ = =
2 25 2(5) 10
So the linear approximation of f at a = 25 is
√ 1 1
L(x) = f (25) + f 0 (25)(x − 25) = 25 + (x − 25) = 5 + (x − 25)
10 10
6.5. Differentials and Linear Approximations 345

(c) Solution. The derivative of f (x) = 73x+1 is

d  3x+1  d  (3x+1) ln 7 
f 0 (x) = 7 = e AB = eB ln A
dx dx
d d  g(x)  d
= e(3x+1) ln 7 [(3x + 1) ln 7] e = eg(x) [g(x)]
dx dx dx
(3x+1) ln 7
=e (3) ln 7 = (3 ln 7)e(3x+1) ln 7

Evaluating this at a = − 13 , we get


 
1 1
0
f − = (3 ln 7)e(3(− 3 )+1) ln 7 = (3 ln 7)e(−1+1) ln 7 = (3 ln 7)e0 = 3 ln 7
3

Now, we find the linear approximation of f at a = − 13 .


    
0 1 0 1 1
L(x) = f (a) + f (a)(x − a) = f − +f − x − (− )
3 3 3
   
1 1
= 7−1+1 + (3 ln 7) x + = 1 + (3 ln 7) x +
3 3

(d) Solution. First we find the derivative of f (x) = log5 (6x + 7).
d d
0 d dx
[6x + 7] d dx
[g(x)]
f (x) = [log5 (6x + 7)] = [loga g(x)] =
dx (ln 5)(6x + 7) dx (ln a)g(x)
6
=
(ln 5)(6x + 7)

Evaluating this at 3, we get


6 6 6
f 0 (3) = = =
(ln 5)(6(3) + 7) (ln 5)(25) 25 ln 5

We can now find the linear approximation of f at a = 3.


6
L(x) = f (a) + f 0 (a)(x − a) = log5 (6(3) + 7) + (x − 3)
25 ln 5
6 6
= log5 (25) + (x − 3) = 2 + (x − 3) log5 (25) = 2
25 ln 5 25 ln5

5. [Exercise on page 49]



(a) 5
Solution.

Using Linear Approximations to estimate Numbers

Let f be a function. To approximate a number f (b), one can go through


the following steps.
346 Chapter 6. Applications of the Derivative II

Step 1. Find the closest number to b, say a, such that f (a) is exact. By
exact we mean a number
√ that has a finite √ number of decimal
places. For example, 25 is exact while 26 is not.

Step 2. Find the linear approximation, L(x), of f at a.

Step 3. Substitute x with b into L(x) to get an approximation of f (b):


f (b) ≈ L(b); that is,

f (b) ≈ f (a) + f 0 (a)(b − a)


We want to approximate 5 using linear approximations.
ˆ Finding√the function
√ and b. The natural function we can consider here is
f (x) = x. Since 5 = f (5), it follows that b = 5.
ˆ Finding a. Since the closest number to 5 such that its image under f is
exact and easy to calculate is 4, it follows that a = 4.
ˆ Now, we find the linear approximation of f at a = 4. First, f 0 (x) = 1

2 x
and

1 1 1
f 0 (4) = √ = =
2 4 2(2) 4

Using the formula for the linear approximation, we have


√ 1 1
L(x) = f (4) + f 0 (4)(x − 4) = 4 + (x − 4) = 2 + (x − 4)
4 4
ˆ √
To finish, we substitute x with b = 5 into L(x) to get an approximation of
5:
√ 1 1 9
5 ≈ L(5) = 2 + (5 − 4) = 2 + = = 2.25
4 4 4

So 5 ≈ 2.25.
(b) Solution.
√ √
ˆ For 47, f (x) = x and b = 47.
ˆ The closest number to 47 such that its image under f (x) is exact and easy
to calculate is a = 49.

ˆ The derivative of f (x) = x is 2√1 2 . Evaluating this at a = 49, we have

1 1 1
f 0 (49) = √ = =
2 49 2(7) 14

Now, the linear approximation of f at a = 49 is


√ 1 1
L(x) = f (49) + f 0 (49)(x − 49) = 49 + (x − 49) = 7 + (x − 49)
14 14
6.5. Differentials and Linear Approximations 347

ˆ Substituting x with 47 into L(x), we get an approximation of f (47):


√ 1 2 96
47 ≈ 7 + (47 − 49) = 7 − = ≈ 6.86
14 14 14

Thus, 47 ≈ 6.86.
(c) Solution.
√ √
ˆ For 3 28, f (x) = 3 x and b = 28.
ˆ The closest number to √ 28 such that its image under f (x) is exact and easy
to calculate is a = 27 ( 3 27 = 3).

ˆ The derivative of f (x) = 3 x is
d h 1 i 1 −2 1
f 0 (x) = x3 = x 3 = 2
dx 3 3x 3
Evaluating this at a = 27, we have
1 1 1
f 0 (27) = 2 = =
3(27) 3 3(9) 27

Now, the linear approximation of f at a = 27 is

L(x) = f (a) + f 0 (a)(x − a) = f (27) + f 0 (27)(x − 27)


√3 1 1
= 27 + (x − 27) = 3 + (x − 27)
27 27
ˆ Substituting x with b = 28 into L(x), we get an approximation of f (28):

3 1 1 82
28 ≈ 3 + (28 − 27) = 3 + = ≈ 3.037
27 27 27

3
Thus, 28 ≈ 3.037.
(d) Solution. Here we want to estimate e0.02 .
ˆ Let f (x) = ex and b = 0.02. Then estimating e0.02 amounts to estimating
f (b).
ˆ The closest number to 0.02 such that its image under f (x) is exact and easy
to calculate is a = 0 (e0 = 1).
ˆ The derivative of f is f 0 (x) = ex and f 0 (0) = e0 = 1.
ˆ The linear approximation of f at a = 0 is

L(x) = f (0) + f 0 (0)(x − 0) = e0 + 1(x − 0) = 1 + x

ˆ Substituting x with b = 0.02 into L(x), we get an approximation of e0.02 :

e0.02 ≈ 1 + 0.02 = 1.02

(e) Solution.
ˆ Let f (x) = ln x and b = 0.99. Then estimating ln(0.99) amounts to esti-
mating f (b).
348 Chapter 6. Applications of the Derivative II

ˆ The closest number to 0.99 such that its image under f (x) is exact and easy
to calculate is a = 1 (ln 1 = 0).
ˆ The derivative of f is f 0 (x) = x1 and f 0 (1) = 11 = 1.
ˆ The linear approximation of f at a = 1 is

L(x) = f (1) + f 0 (1)(x − 1) = ln 1 + 1(x − 1) = 0 + (x − 1) = x − 1

ˆ Substituting x with b = 0.99 into L(x), we get an approximation of ln(0.99):

ln(0.99) ≈ 0.99 − 1 = −0.01

(f ) Solution.
ˆ To estimate (−2.001)5 , we consider the function f (x) = x5 and we let b =
−2.001.
ˆ The closest number to −2.001 such that its image under f (x) is exact and
easy to calculate is a = −2 ((−2)5 = −32).
ˆ The derivative of f is f 0 (x) = dx
d
[x5 ] = 5x4 and f 0 (−2) = 5(−2)4 = 80
ˆ The linear approximation of f at a = −2 is

L(x) = f (a) + f 0 (a)(x − a) = f (−2) + f 0 (−2)(x − (−2))


= (−2)5 + 80(x + 2) = −32 + 80(x + 2)

ˆ Substituting x with b = −2.001 into L(x), we get an approximation of


(−2.001)5 :

(−2.001)5 ≈ −32 + 80(−2.001 + 2) = −32.08

6. [Exercise on page 50]


(a) Solution. First, we need to find the derivative of C(x).
d 3
C 0 (x) = [x − 2x2 + 4x + 2500] = 3x2 − 4x + 4
dx
As x changes from 8 to 9, the initial value is x1 = 8 and the final value is x2 = 9.
So

∆x = x2 − x1 = 9 − 8 = 1 and C 0 (x1 ) = C 0 (8) = 3(8)2 − 4(8) + 4 = 164

Now, using (6.5.1), we have

∆C ≈ C 0 (x1 )∆x = 164(1) = 164

So the change in cost as x changes from 8 to 9 is approximately $164.


(b) Solution. Here x1 = 29, x2 = 30, ∆x = 30 − 29 = 1, and

C 0 (x1 ) = C 0 (29) = 3(29)2 − 4(29) + 4 = 2411

Using (6.5.1), we get

∆C ≈ C 0 (x1 )∆x = 2411(1) = $2411


6.5. Differentials and Linear Approximations 349

7. [Exercise on page 50]

(a) Solution. We first need to find the profit function P (x).

P (x) = R(x) − C(x) = 25000 ln(0.03x + 2) − (478 + 89x)


= 25000 ln(0.03x + 2) − 478 − 89x

Taking the derivative of this, we have


d d d
P 0 (x) = [25000 ln(0.03x + 2)] − [478] − [89x]
dx dx dx
d
= 25000 [ln(0.03x + 2)] − 0 − 89
dx
d d
dx
[0.03x + 2] d dx
[g(x)]
= 25000 − 89 [ln g(x)] =
0.03x + 2 dx g(x)
0.03 750
= 25000 − 89 = − 89
0.03x + 2 0.03x + 2
Substituting x with x1 = 500, we find
750
P 0 (x1 ) = P 0 (500) = − 89 ≈ −44.88
0.03(500) + 2
Therefore, the differential of P is

dP = P 0 (x)dx = P 0 (x1 )∆x Evaluate dP at x = x1 and dx = ∆x


= (−44.88)(1) ∆x = x2 − x1 = 501 − 500 = 1
= −44.88

Since ∆P ≈ dP , we have ∆P ≈ −44.88.


Thus, as x changes from 500 to 501, the profit decreases by about $44.88.
(b) Solution. At the level of production of 500 units, the marginal profit is
750
P 0 (500) = − 89 ≈ −44.88
0.03(500) + 2

Note that this coincides with the answer found in part (a). This is true in general
when ∆x = 1.

Change in y when ∆x = 1

Let y = f (x) be a function. Suppose f is differentiable and x changes


from x1 to x2 . Also suppose that ∆x = x2 − x1 = 1. Then

∆y ≈ f 0 (x1 )∆x By (6.5.1)


= f 0 (x1 )(1) ∆x = 1
= f 0 (x1 )
350 Chapter 6. Applications of the Derivative II

This says that if the change in x as x changes from x1 to x2 is 1, then the


corresponding change in y is approximately f 0 (x1 ).

8. [Exercise on page 50]


Solution. We want to estimate the change in price when the quantity q changes
from q1 = 3 to q2 = 3.7 (remember that q is in hundreds of units). First we need to
find the derivative of p with respect to q.
 
0 d d 4000
p (q) = [p] =
dq dq 0.05q 2 + 3
d d
dq
[4000](0.05q 2 + 3) − 4000 dq [0.05q 2 + 3]
= Quotient Rule
(0.05q 2 + 3)2
0(0.05q 2 + 3) − 4000(0.1q) −400q
= 2 2
=
(0.05q + 3) (0.05q 2 + 3)2

Evaluating this at q1 = 3, we find


−400(3)
p0 (q1 ) = p0 (3) = ≈ −100.819
(0.05(3)2 + 3)2
Therefore, the differential of p is

dp = p0 (q)dq = p0 (q1 )∆q Evaluate dp at q = q1 and dq = ∆q


= (−100.819)(0.7) ∆q = q2 − q1 = 3.7 − 3 = 0.7
= −70.57

Since ∆p ≈ dp, it follows that ∆p ≈ −70.57. This means that when the quantity q
changes from 300 to 370 units, the price p decreases by about $70.57.
9. [Exercise on page 50]
Solution.
ˆ Let V be the volume of the balloon, and let r be its radius. We want to approx-
imate the change in V , ∆V, if the diameter increases from 10 cm to 10.3 cm (or,
if the radius increases from r1 = 5 cm to r2 = 5.15 cm).
ˆ We know that the volume of a sphere of radius r is given by the formula
4
V = V (r) = πr3
3
Taking the derivative of this with respect to r, we get
 
0 d 4 3 4 d 4
V (r) = πr = π [r3 ] = π(3r2 ) = 4πr2
dr 3 3 dr 3
Substituting r with r1 = 5 into V 0 (r), we have

V 0 (r1 ) = V 0 (5) = 4π(5)2 = 4π(25) = 100π


6.6. Newton’s Method 351

ˆ Now, the differential of V is

dV = V 0 (r)dr = V 0 (r1 )∆r Evaluate dV at r = r1 and dr = ∆r


= (100π)(0.15) ∆r = r2 − r1 = 5.15 − 5 = 0.15
≈ 47.12

ˆ Since ∆V ≈ dV , it follows that ∆V ≈ 47.12.

Thus, if the radius increases from r1 = 5 cm to r2 = 5.15 cm, the approximate change
in volume is 47.12 cm3 .
10. [Exercise on page 50]
Solution. Let A be the area of the field. We want to estimate the change in A if
x = 225 and dx = ∆x = ±0.005. The formula for the area of a square of side x is

A = x2

Taking the differential of this gives

dA = A0 (x)dx = 2xdx = 2(225)(±0.005) = ±2.25

So the error that might occur when the area of the field is calculated is about
±2.25 ft2 .

6.6 Newton’s Method


1. [Exercise on page 50]
Solution.

Newton’s Method
Let f be a function. Suppose that f is continuous on [a, b]. Also suppose
that f (a) and f (b) have opposite signs (that is, f (a)f (b) < 0). Then, by the
intermediate value theorem, there exists a number c in (a, b) such that f (c) = 0.
In other words, c is a solution to the equation f (x) = 0. Suppose that f
is differentiable on [a, b]. Newton’s method consists of producing successively
better approximations
c1 , c2 , c3 , · · · , cn , cn+1 , · · ·
to c as follows.
ˆ Let c1 be an initial guess. Of course, c1 must be chosen in the interval
[a, b].
ˆ Suppose an approximation cn to c has been found. If f 0 (cn ) 6= 0, the
number cn+1 is obtained by the formula
f (cn )
cn+1 = cn − (6.6.1)
f 0 (cn )
352 Chapter 6. Applications of the Derivative II

Usually, cn+1 is a better approximation of c than was cn .

Using (6.6.1), we have

f (c1 )
c2 = c1 −
f 0 (c1 )
f (c2 )
c3 = c2 −
f 0 (c2 )
f (c3 )
c4 = c3 −
f 0 (c3 )
···

Actually, the formula (6.6.1) is obtained by considering tangent lines and look-
ing at where they cross the x-axis. Specifically, let cn be a number in the
interval [a, b], and let Tn be the tangent line to the graph of f at (cn , f (cn )).
The equation of Tn is

y − f (cn ) = f 0 (cn )(x − cn )


y − f (cn ) = xf 0 (cn ) − cn f 0 (cn )
y = xf 0 (cn ) − cn f 0 (cn ) + f (cn )

The tangent line crosses the x-axis when y = 0. That is, when

xf 0 (cn ) − cn f 0 (cn ) + f (cn ) = 0


xf 0 (cn ) = cn f 0 (cn ) − f (cn )
cn f 0 (cn ) f (cn )
x= − Divide both sides by f 0 (cn )
f 0 (cn ) f 0 (cn )
f (cn )
x = cn − Simplify
f 0 (cn )

The number cn+1 is defined as the point of intersection between the tangent
line Tn and the x-axis. So
f (cn )
cn+1 = cn −
f 0 (cn )

As an illustration, we have the following figure showing c1 , c2 , and c.


6.6. Newton’s Method 353

a c1 c c2 b x

T1

We want to approximate a solution to the equation x2 + x − 1 = 0 in the interval


[0, 1] to the nearest hundredth. Let f (x) = x2 + x − 1. Then f 0 (x) = 2x + 1.

ˆ First, we need to make an initial guess for c. Let c1 = 0.


ˆ Using the formula (6.6.1) with n = 1, we have

f (c1 ) f (0)
c2 = c1 − 0
=0− 0
f (c1 ) f (0)
02 + 0 − 1 −1
=0− =− =1
2(0) + 1 1

ˆ Using (6.6.1) with n = 2, we have

f (c2 ) f (1)
c3 = c2 − 0
=1− 0
f (c2 ) f (1)
12 + 1 − 1 1 2
=1− = 1 − = ≈ 0.67
2(1) + 1 3 3

ˆ Similarly,

f (c3 ) f (0.67) 0.672 + 0.67 − 1


c4 = c3 − = 0.67 − = 0.67 − ≈ 0.62
f 0 (c3 ) f 0 (0.67) 2(0.67) + 1
f (c4 ) f (0.62) 0.622 + 0.62 − 1
c5 = c4 − = 0.62 − = 0.62 − ≈ 0.62
f 0 (c4 ) f 0 (0.62) 2(0.62) + 1

Since c4 = c5 , the approximation to the solution of the equation x2 + x − 1 = 0


to the nearest hundredth is 0.62.
Note. We calculated c2 through c5 to get the required degree of accuracy. If
we start with a different initial guess, the number of steps might be different,
but we will end up with the same final answer. Remember to always choose the
initial guess in the given interval.
354 Chapter 6. Applications of the Derivative II

2. [Exercise on page 50]

(a) Solution. Here the function is f (x) = 3x2 − 8x − 2, and the interval is [2, 3].
The derivative of f is f 0 (x) = 6x − 8. Let c1 = 2.

f (c1 ) f (2) 3(2)2 − 8(2) − 2 −6


c2 = c1 − 0
= 2 − 0
= 2 − =2− ≈ 3.5
f (c1 ) f (2) 6(2) − 8 4
f (c2 ) f (3.5) 3(3.5)2 − 8(3.5) − 2
c3 = c2 − = 3.5 − = 3.5 − ≈ 2.98
f 0 (c2 ) f 0 (3.5) 6(3.5) − 8
f (c3 ) f (2.98) 3(2.98)2 − 8(2.98) − 2
c4 = c3 − = 2.98 − = 2.98 − ≈ 2.90
f 0 (c3 ) f 0 (2.98) 6(2.98) − 8
f (c4 ) f (2.9) 3(2.9)2 − 8(2.9) − 2
c5 = c4 − = 2.9 − = 2.9 − ≈ 2.90
f 0 (c4 ) f 0 (2.9) 6(2.9) − 8

Since c4 = c5 , it follows that c ≈ 2.90.


(b) Solution. Let f (x) = x3 + x2 − 5, so that f 0 (x) = 3x2 + 2x. Let c1 = 1. Then

f (c1 ) 13 + 12 − 5
c2 = c1 − = 1 − ≈ 1.6
f 0 (c1 ) 3(1)2 + 2(1)
f (c2 ) (1.6)3 + (1.6)2 − 5
c3 = c2 − = 1.6 − ≈ 1.45
f 0 (c2 ) 3(1.6)2 + 2(1.6)
f (c3 ) (1.45)3 + (1.45)2 − 5
c4 = c3 − 0 = 1.45 − ≈ 1.43
f (c3 ) 3(1.45)2 + 2(1.45)
f (c4 ) (1.43)3 + (1.43)2 − 5
c5 = c4 − 0 = 1.43 − ≈ 1.43
f (c4 ) 3(1.43)2 + 2(1.43)

Since c4 = c5 , it follows that c ≈ 1.43.


(c) Solution. Let f (x) = −2x3 + 6x2 − 2x + 13, so that f 0 (x) = −6x2 + 12x − 2.
Let c1 = 3.

f (c1 ) f (3) −2(3)3 + 6(3)2 − 2(3) + 13


c2 = c1 − = 3 − = 3 − ≈ 3.35
f 0 (c1 ) f 0 (3) −6(3)2 + 12(3) − 2
f (c2 ) f (3.35)
c3 = c2 − = 3.35 −
f 0 (c2 ) f 0 (3.35)
−2(3.35)3 + 6(3.35)2 − 2(3.35) + 13
= 3.35 − ≈ 3.30
−6(3.35)2 + 12(3.35) − 2
f (c3 ) f (3.3)
c4 = c3 − 0
= 3.3 − 0
f (c3 ) f (3.3)
−2(3.3)3 + 6(3.3)2 − 2(3.3) + 13
= 3.3 − ≈ 3.30
−6(3.3)2 + 12(3.3) − 2

Since c4 = c3 , it follows that c ≈ 3.30.


6.6. Newton’s Method 355

(d) Solution. Let f (x) = x4 + 3x3 − 5x2 − 9, so that f 0 (x) = 4x3 + 9x2 − 10x. First,
let’s approximate c in the interval [−5, −4]. Let c1 = −5.

f (c1 ) (−5)4 + 3(−5)3 − 5(−5)2 − 9


c2 = c1 − 0 = −5 − ≈ −4.48
f (c1 ) 4(−5)3 + 9(−5)2 − 10(−5)
f (c2 ) (−4.48)4 + 3(−4.48)3 − 5(−4.48)2 − 9
c3 = c2 − 0 = −4.48 − ≈ −4.30
f (c2 ) 4(−4.48)3 + 9(−4.48)2 − 10(−4.48)
f (c3 ) (−4.3)4 + 3(−4.3)3 − 5(−4.3)2 − 9
c4 = c3 − = −4.3 − ≈ −4.28
f 0 (c3 ) 4(−4.3)3 + 9(−4.3)2 − 10(−4.3)
f (c4 ) (−4.28)4 + 3(−4.28)3 − 5(−4.28)2 − 9
c5 = c4 − = −4.28 − ≈ −4.28
f 0 (c4 ) 4(−4.28)3 + 9(−4.28)2 − 10(−4.28)

Since c5 = c4 , the approximation to the solution of the equation f (x) = 0, to


the nearest hundredth, in the interval [−5, −4] is −4.28.

To approximate a solution in the interval [1, 2], let c1 = 1. Using a similar


approach as above, we find that c ≈ 1.71.
1
(e) Solution. Let f (x) = 3x 5 − 2x4 + 7, so that
 
0 1 −4
f (x) = 3 x 5 − 8x3 = 0.6x−0.8 − 8x3 .
5

Let c1 = 1.

f (c1 ) 3(1)0.2 − 2(1)4 + 7


c2 = c1 − = 1 − ≈ 2.08
f 0 (c1 ) 0.6(1)−0.8 − 8(1)3
f (c2 ) 3(2.08)0.2 − 2(2.08)4 + 7
c3 = c2 − = 2.08 − ≈ 1.70
f 0 (c2 ) 0.6(2.08)−0.8 − 8(2.08)3
f (c3 ) 3(1.7)0.2 − 2(1.7)4 + 7
c4 = c3 − = 1.7 − ≈ 1.54
f 0 (c3 ) 0.6(1.7)−0.8 − 8(1.7)3
f (c4 ) 3(1.54)0.2 − 2(1.54)4 + 7
c5 = c4 − = 1.54 − ≈ 1.51
f 0 (c4 ) 0.6(1.54)−0.8 − 8(1.54)3
f (c5 ) 3(1.51)0.2 − 2(1.51)4 + 7
c6 = c5 − = 1.51 − ≈ 1.50
f 0 (c5 ) 0.6(1.51)−0.8 − 8(1.51)3
f (c6 ) 3(1.5)0.2 − 2(1.5)4 + 7
c7 = c6 − = 1.5 − ≈ 1.50
f 0 (c6 ) 0.6(1.5)−0.8 − 8(1.5)3

Since c7 = c6 , c ≈ 1.5.
(f ) Solution. Let f (x) = x3 e−2x + x2 + 1. The derivative of f is

d  3 −2x  d d
f 0 (x) = xe + [x2 ] + [1]
dx dx dx
d 3 −2x d d d
= [x ]e + x3 [e−2x ] + [x2 ] + [1] Product Rule
dx dx dx dx
356 Chapter 6. Applications of the Derivative II

= 3x2 e−2x + x3 −2e−2x + 2x + 0




= 3x2 − 2x3 e−2x + 2x




Let c1 = −1. Then


f (c1 ) (−1)3 e−2(−1) + (−1)2 + 1
c2 = c1 − 0 = −1 − ≈ −0.85
f (c1 ) (3(−1)2 − 2(−1)3 ) e−2(−1) + 2(−1)
f (c2 ) (−0.85)3 e−2(−0.85) + (−0.85)2 + 1
c3 = c2 − = −0.85 −
f 0 (c2 ) (3(−0.85)2 − 2(−0.85)3 ) e−2(−0.85) + 2(−0.85)
≈ −0.75
f (c3 ) (−0.75)3 e−2(−0.75) + (−0.75)2 + 1
c4 = c3 − = −0.75 −
f 0 (c3 ) (3(−0.75)2 − 2(−0.75)3 ) e−2(−0.75) + 2(−0.75)
≈ −0.72
f (c4 ) (−0.72)3 e−2(−0.72) + (−0.72)2 + 1
c5 = c4 − = −0.72 −
f 0 (c4 ) (3(−0.72)2 − 2(−0.72)3 ) e−2(−0.72) + 2(−0.72)
≈ −0.71
f (c5 ) (−0.71)3 e−2(−0.71) + (−0.71)2 + 1
c6 = c5 − = −0.71 −
f 0 (c5 ) (3(−0.71)2 − 2(−0.71)3 ) e−2(−0.71) + 2(−0.71)
≈ −0.71

(g) Solution. Let f (x) = 3 ln x + x − 5, so that


 
0 1
f (x) = 3 + 1 = 3x−1 + 1
x
Let c1 = 2. Then
f (c1 ) 3 ln(2) + 2 − 5
c2 = c1 − 0
=2− ≈ 2.37
f (c1 ) 3(2)−1 + 1
f (c2 ) 3 ln(2.37) + 2.37 − 5
c3 = c2 − 0
= 2.37 − ≈ 2.39
f (c2 ) 3(2.37)−1 + 1
f (c3 ) 3 ln(2.39) + 2.39 − 5
c4 = c3 − 0
= 2.39 − ≈ 2.39
f (c3 ) 3(2.39)−1 + 1

3. [Exercise on page 51]



(a) Solution. First, observe that if x = 5, then x2 = 5 or x2 − 5 = 0. So 5 is the
positive solution to the equation x2 − 5 = 0. Let f (x) = x2 − 5.
ˆ To find an interval [a, b] containing the positive solution to the equation
f (x) = 0, we find the values of f at 0, 1, 2, · · · until we find two values of
opposite signs.
f (0) = −5, f (1) = −4, f (2) = −1, f (3) = 4
6.6. Newton’s Method 357

Since f (2) and f (3) have opposite signs, and since the function f is con-
tinuous, it follows by the intermediate value theorem that there exists c in
[2, 3] such that f (c) = 0.
ˆ Let c1 = 2 be the initial guess. Then
f (c1 ) f (2) (2)2 − 5
c2 = c1 − 0 =2− 0 =2− ≈ 2.25
f (c1 ) f (2) 2(2)
f (c2 ) f (2.25) (2.25)2 − 5
c3 = c2 − = 2.25 − = 2.25 − ≈ 2.2361
f 0 (c2 ) f 0 (2.25) 2(2.25)
f (c3 ) f (2.2361) (2.2361)2 − 5
c4 = c4 − = 2.2361 − = 2.2361 − ≈ 2.2361
f 0 (c3 ) f 0 (2.2361) 2(2.2361)

Since c4 = c3 , we conclude that 5 ≈ 2.2361, to four decimal places.

(b) Solution. First, observe that 23 is the positive solution to the equation
x2 − 23 = 0. So we take f (x) = x2 − 23. Evaluating f at 0, 1, 2, · · · , we get

f (0) = −23, f (1) = −22, f (2) = −19, f (3) = −14, f (4) = −7, f (5) = 2

Since f (4) and f (5) have opposite signs, the solution to the equation f (x) = 0
lies in the interval [4, 5]. Let the initial guess be c1 = 4. Then

f (c1 ) f (4) (4)2 − 23


c2 = c1 − = 4 − = 4 − ≈ 4.875
f 0 (c1 ) f 0 (4) 2(4)
f (c2 ) f (4.875) (4.875)2 − 5
c3 = c2 − = 4.875 − = 4.875 − ≈ 4.7965
f 0 (c2 ) f 0 (4.875) 2(4.875)
f (c3 ) f (4.7965) (4.7965)2 − 5
c4 = c3 − = 4.7965 − = 4.7965 − ≈ 4.7958
f 0 (c3 ) f 0 (4.7965) 2(4.7965)
f (c4 ) f (4.7958) (4.7958)2 − 5
c5 = c4 − 0 = 4.7958 − 0 = 4.7958 − ≈ 4.7958
f (c4 ) f (4.7958) 2(4.7958)

Since c5 = c4 , it follows that 23 ≈ 4.7958, to four decimal places.

(c) Solution. First, we observe that 3 50 is the solution to the equation x3 = 50
or x3 − 50 = 0. Let f (x) = x3 − 50. Evaluating f at 0, 1, 2, · · · , we get

f (0) = −50, f (1) = −49, f (2) = −42, f (3) = −23, f (4) = 14

Since
√ f (3) and f (4) have opposite signs, the solution to the equation f (x) = 0,
3
50, is between 3 and 4. Let c1 = 3 be the initial guess. Then, since f 0 (x) = 3x2 ,
f (c1 ) f (3) (3)3 − 50
c2 = c1 − 0 =3− 0 =3− ≈ 3.8519
f (c1 ) f (3) 3(3)2
f (c2 ) f (3.8519) (3.8519)3 − 50
c3 = c2 − 0 = 3.8519 − 0 = 3.8519 − ≈ 3.6912
f (c2 ) f (3.8519) 3(3.8519)2
f (c3 ) f (3.6912) (3.6912)3 − 50
c4 = c3 − = 3.6912 − = 3.6912 − ≈ 3.6841
f 0 (c3 ) f 0 (3.6912) 3(3.6912)2
358 Chapter 6. Applications of the Derivative II

f (c4 ) f (3.6841) (3.6841)3 − 50


c5 = c4 − = 3.6841 − = 3.6841 − ≈ 3.6840
f 0 (c4 ) f 0 (3.6841) 3(3.6841)2
f (c5 ) f (3.684) (3.684)3 − 50
c6 = c5 − = 3.684 − = 3.684 − ≈ 3.6840
f 0 (c5 ) f 0 (3.684) 3(3.684)2

3
So 50 ≈ 3.6840, to four decimal places.

(d) Solution. The number 7 139 is clearly the solution to the equation x7 = 139
or x7 − 139 = 0. So we take f (x) = x7 − 139. Evaluating f at 0, 1, 2, · · · , we get

f (0) = −139, f (1) = −138, f (2) = −11, f (3) = 2048

Since f (2) and f (3) have opposite signs, the solution to the equation f (x) = 0 lies
in the interval [2, 3]. Let c1 = 2 be the initial approximation. Since f 0 (x) = 7x6 ,
we have
f (c1 ) f (2) (2)7 − 139
c2 = c1 − 0 =2− 0 =2− ≈ 2.0246
f (c1 ) f (2) 7(2)6
f (c2 ) f (2.0246) (2.0246)7 − 139
c3 = c2 − 0 = 2.0246 − 0 = 2.0246 − ≈ 2.0237
f (c2 ) f (2.0246) 7(2.0246)6
f (c3 ) f (2.0237) (2.0237)7 − 139
c4 = c3 − = 2.0237 − = 2.0237 − ≈ 2.0237
f 0 (c3 ) f 0 (2.0237) 7(2.0237)6


7
Since c4 = c3 , it follows that 139 ≈ 2.0237, to four decimal places.
4. Solution. [Exercise on page 51]
The equation x2 + 3x + 4 = 0 is a quadratic equation with a = 1, b = 3, and c = 4.
Using the quadratic formula, the solution is
√ p √
−b ± b2 − 4ac −3 ± 32 − 4(1)(4) −3 ± −7
x= = =
2a 2(1) 2
√ √
Since −7 is undefined (remember that x is defined for x ≥ 0), the equation
x2 + 3x + 4 = 0 has no solution. In other words, if f (x) = x2 + 3x + 4, there is no c
such that f (c) = 0. So Newton’s method does not work here since it is not possible
to approximate something that does not exist.
5. [Exercise on page 51]
(a) Solution. We want to solve the equation 1 + ln(x − 2) = 0. Subtracting 1 from
both sides, we get ln(x − 2) = −1. Taking the natural exponential on both sides,
we have x − 2 = e−1 . Adding 2 to both sides, we get

x = 2 + e−1 ≈ 2.37

(b) Solution. Let f (x) = 1 + ln(x − 2). Then


d
d d − 2]
[x 1
f 0 (x) = [1] + [ln(x − 2)] = 0 + dx
=
dx dx x−2 x−2
6.6. Newton’s Method 359

Let c be the solution to the equation 1 + ln(x − 2) = 0. We want to approximate


c to the nearest hundredth using Newton’s method. If c1 = 2.1, then

f (c1 ) 1 + ln(2.1 − 2)
c2 = c1 − = 2.1 − 1 ≈ 2.23
f 0 (c1 ) 2.1−2

f (c2 ) 1 + ln(2.23 − 2)
c3 = c2 − 0
= 2.23 − 1 ≈ 2.34
f (c2 ) 2.23−2

f (c3 ) 1 + ln(2.34 − 2)
c4 = c3 − 0
= 2.34 − 1 ≈ 2.37
f (c3 ) 2.34−2

f (c4 ) 1 + ln(2.37 − 2)
c5 = c4 − = 2.37 − 1 ≈ 2.37
f 0 (c4 ) 2.37−2

Since c5 = c4 , it follows that c ≈ 2.37. Note that this matches with the answer
found in part (a).
(c) Solution. Here c1 = 3.

f (c1 ) 1 + ln(3 − 2)
c2 = c1 − 0
=3− 1 =2
f (c1 ) 3−2

f (c2 ) 1 + ln(2 − 2) 1 + ln(0)


c3 = c2 − 0
=2− 1 =2− 1
f (c2 ) 2−2 0

The value c3 is undefined because ln(0) (as well as 1/0) is not defined. So
Newton’s method does not work for the equation 1 + ln(x − 2) = 0 if the initial
guess is c1 = 3.
Comment. This question is an illustration of the fact that in some situations,
if the initial guess is “not close” to the exact solution, Newton’s method may
not work.

6. [Exercise on page 51]

(a) Solution. The derivative of f (x) = x4 + x3 − 2x + 1 is

f 0 (x) = 4x3 + 3x2 − 2

To find the critical number for f (x), we need to solve the equation f 0 (x) = 0.
Since this is not easy to solve, we will use Newton’s method to approximate the
solution.
ˆ First, we need to find an interval [a, b] containing the solution. To this
end, we will use the intermediate value theorem. The idea is to find two
numbers a and b such that f (a) and f (b) have opposite signs. We will use
the trial and error method. To be more precise, we will try the numbers
0, ±1, ±2, · · · until we get two numbers whose images have opposite signs.
– Let’s begin with 0. Substituting x with 0 into f 0 (x), we get

f 0 (0) = 4(0)3 + 3(0)2 − 2 = −2


360 Chapter 6. Applications of the Derivative II

– Now, substituting x with 1 into f 0 (x), we get

f 0 (1) = 4(1)3 + 3(1)2 − 2 = 4 + 3 − 2 = 5

ˆ Since f 0 (0) and f 0 (1) have opposite signs, it follows that N = 0 is between
f 0 (0) and f 0 (1). So, since f 0 is continuous, by the intermediate value theo-
rem, there exists a number c in the interval [0, 1] such that f 0 (c) = 0.
ˆ We now use Newton’s method to approximate a solution to the equation
f 0 (x) = 0 in the interval [0, 1]. First, the derivative of f 0 (x) is
d 0 d
f 00 (x) = [f (x)] = [4x3 + 3x2 − 2] = 12x2 + 6x
dx dx
As an initial guess, let c1 = 1 (we don’t choose 0 because f 00 (0) = 0). Then
f 0 (c1 ) f 0 (1) 4(1)3 + 3(1)2 − 2
c2 = c1 − = 1 − = 1 − ≈ 0.72
f 00 (c1 ) f 00 (1) 12(1)2 + 6(1)
f 0 (c2 ) f 0 (0.72) 4(0.72)3 + 3(0.72)2 − 2
c3 = c2 − = 0.72 − = 0.72 − ≈ 0.62
f 00 (c2 ) f 00 (0.72) 12(0.72)2 + 6(0.72)
f 0 (c3 ) f 0 (0.62) 4(0.62)3 + 3(0.62)2 − 2
c4 = c3 − = 0.62 − = 0.62 − ≈ 0.61
f 00 (c3 ) f 00 (0.62) 12(0.62)2 + 6(0.62)
f 0 (c4 ) f 0 (0.61) 4(0.61)3 + 3(0.61)2 − 2
c5 = c4 − = 0.61 − = 0.61 − ≈ 0.61
f 00 (c4 ) f 00 (0.61) 12(0.61)2 + 6(0.61)
So, to the nearest hundredth, the critical number is c = 0.61.
(b) Solution. The critical number c ≈ 0.61 determines two intervals: (−∞, 0.61)
and (0.61, ∞). And the sign of the derivative in each of these intervals is shown
in Figure 6.6.

− +
−∞ 0.61 ∞
choose 0 choose 1
f 0 (0) = −2 < 0 f 0 (1) = 5 > 0

Figure 6.6: Sign of f 0 (x) = 4x3 + 3x2 − 2

From Figure 6.6, the function f is decreasing on the interval (−∞, 0.61) and
increasing on (0.61, ∞). So f has a relative minimum of

f (0.61) = x4 + x3 − 2x + 1 = (0.61)4 + (0.61)3 − 2(0.61) + 1 ≈ 0.15

when x = 0.61.
7. [Exercise on page 51]
Solution.
6.6. Newton’s Method 361

ˆ The derivative of f (x) = (1 − x)ex + x is


d d
f 0 (x) = [(1 − x)ex ] + [x]
dx dx
d d d
= [1 − x]ex + (1 − x) [ex ] + [x] Product Rule
dx dx dx
d x
= (0 − 1)ex + (1 − x)ex + 1 [e ] = ex
dx
= (0 − 1 + 1 − x)ex + 1 = −xex + 1

ˆ Set f 0 (x) = 0. Then −xex + 1 = 0. Since this latter equation is not easy to
solve, we will find an approximate solution using Newton’s method.
– First, we need to find an interval [a, b] such that f 0 (a) and f 0 (b) have opposite
signs. To do this, we calculate f 0 (0), f 0 (1), f 0 (−1), f 0 (2), f 0 (−2), · · · until we
get two values of opposite signs.
f 0 (0) = −0e0 + 1 = 1, f 0 (1) = −(1)e1 + 1 ≈ −1.718
Since f 0 (0) and f 0 (1) have opposite signs, and since f 0 is continuous, the
solution to the equation f 0 (x) = 0 lies in the interval [0, 1] (thanks to the
intermediate value theorem).
– Now, the derivative of f 0 (x) is
d d d
f 00 (x) = [−xex + 1] = [−xex ] + [1]
dx dx dx
d d
= [−x]ex − x [ex ] + 0 = (−1)ex − xex = (−1 − x)ex
dx dx
As an initial guess, let c1 = 0. Then
f 0 (c1 ) −(0)e0 + 1
c2 = c1 − = 0 − ≈1
f 00 (c1 ) (−1 − 0)e0
f 0 (c2 ) −(1)e1 + 1
c3 = c2 − = 1 − ≈ 0.68
f 00 (c2 ) (−1 − 1)e1
f 0 (c3 ) (−0.68)e0.68 + 1
c4 = c3 − = 0.68 − ≈ 0.58
f 00 (c3 ) (−1 − 0.68)e0.68
f 0 (c4 ) (−0.58)e0.58 + 1
c5 = c4 − = 0.58 − ≈ 0.57
f 00 (c4 ) (−1 − 0.58)e0.58
f 0 (c5 ) (−0.57)e0.57 + 1
c6 = c5 − 00 = 0.57 − ≈ 0.57
f (c5 ) (−1 − 0.57)e0.57
Since c6 = c5 , the critical number is c = 0.57, to the nearest hundredth.
ˆ Froxm the sign of the derivative, which is shown in Figure 6.7, the function
f (x) = (1 − x)ex + x has a relative maximum of
f (0.57) = (1 − 0.57)e0.57 + 0.57 ≈ 1.33
when x = 0.57.
362 Chapter 6. Applications of the Derivative II

+ −
−∞ 0.57 ∞
choose 0 choose 1
f 0 (0) = 1 > 0 f 0 (1) = −e + 1 < 0

Figure 6.7: Sign of f 0 (x) = −xex + 1

8. [Exercise on page 51]


Solution. Remember that the break-even quantity is the quantity found when the
revenue is equal to the cost. So, to find the break-even quantity, we need to solve the
equation R(x) = C(x) for x. This is equivalent to solving the equation R(x)−C(x) =
0, that is,
1
15(x + 12) 3 − (3x + 29) = 0
1
15(x + 12) 3 − 3x − 29 = 0
1
Let f (x) = 15(x + 12) 3 − 3x − 29, so that
d h 1
i d d
f 0 (x) = 15(x + 12) 3 − [3x] − [29]
dx dx dx
 
1 2 d 2
= 15 (x + 12)− 3 [x + 12] − 3 = 5(x + 12)− 3 − 3
3 dx
To find an interval containing the solution to the equation f (x) = 0, we calculate the
values of f at 0, 1, 2, · · · until we get two values of opposite signs. (Note that x can’t
be negative.)

f (0) = 5.34, f (1) ≈ 3.27, f (2) ≈ 1.15, f (3) ≈ −1.007

Since f (2) and f (3) have opposite signs, and since f is continuous, it follows by
the intermediate value theorem that the solution to the equation f (x) = 0 is in the
interval [2, 3]. Let the initial guess be c1 = 2. Then
1
f (c1 ) 15(14) 3 − 3(2) − 29
c2 = c1 − 0 =2− 2 ≈ 2.54
f (c1 ) 5(14)− 3 − 3
1
f (c2 ) 15(14.54) 3 − 3(2.54) − 29
c3 = c2 − 0 = 2.54 − 2 ≈ 2.54
f (c2 ) 5(14.54)− 3 − 3
Since c3 = c2 , to the nearest hundredth, the break-even quantity is 2.54.
Suppose the savings and costs in dollars of a firm are given by

S(t) = t2 + 55t + 30 and C(t) = t3 + 7t2 + 11

where t is time in years.


6.6. Newton’s Method 363

(a) Sketch the graphs of S and C on the same axes and show that the equation
S(t) = C(t) has only one solution t ≥ 0.
(b) Find the number of years, to the nearest hundredth, that the firm will realize
savings.

9. [Exercise on page 51]

(a) Solution.
ˆ Sketching the graph of S(t) = t2 + 55t + 30.
– Since t represents time, t can’t be a negative quantity. So the domain
of S is [0, ∞).
– The derivative is S 0 (t) = 2t + 55. Since 2t ≥ 0 and 55 > 0 for all t ≥ 0,
it follows that S 0 (t) is positive for every t ≥ 0. So the function S is
increasing on the interval [0, ∞).
– The second derivative is S 00 (t) = 2. Since S 00 (t) is always positive, it
follows that S is concave upward on [0, ∞).
– Putting all these information together, we get the graph of S as shown
in Figure 6.8.
ˆ Sketching the graph of C(t) = t3 + 7t2 + 11.
– As before, the domain of C is [0, ∞).
– The derivative is C 0 (t) = 3t2 + 14t = t(3t + 14). Clearly, this is positive
for all t > 0 (C 0 (t) is zero when t = 0). So the function C is increasing
on (0, ∞).
– The second derivative is C 00 (t) = 6t + 14. Since 6t ≥ 0 and 14 > 0 for
all t, it follows that 6t + 14 > 0 for every t ≥ 0. Therefore, the function
C is concave upward on [0, ∞).
– The graph of C is shown in Figure 6.8.

y
400 C S

300

200

100

2 4 τ 6 8 10 t

Figure 6.8
364 Chapter 6. Applications of the Derivative II

From Figure 6.8, there is only one point of intersection between the graphs of C
and S. This means that the equation S(t) = C(t) has a unique solution.
(b) Solution. The firm will realize savings in the time interval where S(t) ≥ C(t).
Graphically, this means that the firm will realize savings in the interval where
the graph of S is above the graph of C. From Figure 6.8, this happens when
t is between 0 and τ , where τ is the x-coordinate of the point of intersection
between the graphs. And τ is to be found.
ˆ By definition, τ is the solution to the equation S(t) = C(t) or S(t) − C(t) =
0. Let
f (t) = S(t) − C(t) = t2 + 55t + 30 − t3 + 7t2 + 11


= t2 + 55t + 30 − t3 − 7t2 − 11 = −t3 − 6t2 + 55t + 19


Then τ is the solution to the equation f (t) = 0. We will use Newton’s
method to approximate τ .
ˆ From the graphs, τ is in the interval [4, 6].
ˆ Let c1 = 6 be the initial approximation for τ . Since f 0 (t) = −3t2 − 12t + 55,
we have
f (c1 ) −(6)3 − 6(6)2 + 55(6) + 19
c2 = c1 − = 6 − ≈ 5.34
f 0 (c1 ) −3(6)2 − 12(6) + 55
f (c2 ) −(5.34)3 − 6(5.34)2 + 55(5.34) + 19
c3 = c2 − = 5.34 − ≈ 5.23
f 0 (c2 ) −3(5.34)2 − 12(5.34) + 55
f (c3 ) −(5.23)3 − 6(5.23)2 + 55(5.23) + 19
c4 = c3 − = 5.23 − ≈ 5.22
f 0 (c3 ) −3(5.23)2 − 12(5.23) + 55
f (c4 ) −(5.22)3 − 6(5.22)2 + 55(5.22) + 19
c5 = c4 − = 5.22 − ≈ 5.22
f 0 (c4 ) −3(5.22)2 − 12(5.22) + 55
So τ ≈ 5.22. Thus, to the nearest hundredth, the firm will realize savings over
the first 5.22 years.

10. [Exercise on page 52]


(a) Solution. First, the derivative of P (x) is
 
0 d 1 5 5
P (x) = − x + 2x + 3x + 5x − 4 = − x4 + 6x2 + 6x + 5
3 2
dx 7 7
Since it is not easy to find the exact solution to the equation P 0 (x) = 0, we will
use Newton’s method to approximate it.
Evaluating P 0 (x) at 0, 1, 2, · · · , we get
P 0 (0) = 5, P 0 (1) ≈ 16.29, P 0 (2) ≈ 29.57, P 0 (3) ≈ 19.14, P 0 (4) ≈ −57.86
Since P 0 (3) and P 0 (4) have opposite signs, the solution to the equation P 0 (x) = 0
is in the interval [3, 4]. Let c1 = 3 be the initial guess. Since the derivative of P 0
is  
00 d 5 4 20
P (x) = − x + 6x + 6x + 5 = − x3 + 12x + 6,
2
dx 7 7
6.6. Newton’s Method 365

we have
P 0 (c1 ) P 0 (3) − 57 (3)4 + 6(3)2 + 6(3) + 5
c2 = c1 − = 3 − = 3 − ≈ 3.54
P 00 (c1 ) P 00 (3) − 20
7
(3)3 + 12(3) + 6

P 0 (c2 ) P 0 (3.54)
c3 = c2 − = 3.54 −
P 00 (c2 ) P 00 (3.54)
− 57 (3.54)4 + 6(3.54)2 + 6(3.54) + 5
= 3.54 − ≈ 3.4
− 20
7
(3.54)3 + 12(3.54) + 6
P 0 (c3 ) P 0 (3.4)
c4 = c3 − = 3.4 −
P 00 (c3 ) P 00 (3.4)
− 57 (3.4)4 + 6(3.4)2 + 6(3.4) + 5
= 3.4 − ≈ 3.39
− 20
7
(3.4)3 + 12(3.4) + 6
P 0 (c4 ) P 0 (3.39)
c5 = c4 − = 3.39 −
P 00 (c4 ) P 00 (3.39)
− 57 (3.39)4 + 6(3.39)2 + 6(3.39) + 5
= 3.39 − ≈ 3.39
− 20
7
(3.39)3 + 12(3.39) + 6

So, to the nearest hundredth, the critical number is 3.39. This determines two
intervals: (0, 3.39) and (3.39, ∞). The sign of P 0 in these intervals is shown in
Figure 6.9.

+ −
0 3.39 ∞
choose 1 choose 4
P 0 (1) = 16.29 > 0 P 0 (4) = −57.86 < 0

Figure 6.9: Sign of P 0 (x) = − 57 x4 + 6x2 + 6x + 5

From Figure 6.9, the number of kilograms, to the nearest hundredth, that should
be sold in order to maximize the total profit is 3.39 kilograms.
(b) Solution. The maximum profit is
1
P (3.39) = − (3.39)5 + 2(3.39)3 + 3(3.39)2 + 5(3.39) − 4 = 61.3839
7
Since the profit is in thousands of dollars, the maximum profit is approximately
$61383.9.

You might also like